You are on page 1of 279

Міністерство Охорони Здоровя України

НАЦІОНАЛЬНИЙ МЕДИЧНИЙ УНІВЕРСИТЕТ


імені О.О. БОГОМОЛЬЦЯ

Кафедра біоорганічної та біологічної хімії

Збірник тестових завдань


для підготовки до Змістового модуля №2
«Загальні закономірності метаболізму
вуглеводів, ліпідів, амінокислот»
(підготовлений із Збірників тестових завдань для складання ліцензійного іспиту
«Крок 1. Загальна Лікарська підготовка» та «Крок 1. Стоматологія»)

для студентів 2 курсу медичних факультетів, медико-


психологічного, стоматологічного факультетів та факультету
підготовки збройних сил України

Київ-2018
Ферменти та коферменти. Регуляція метаболізму
В деяких речовинах відбувається Жінці 36-ти років після хірургічного
денатурація білків. Вкажіть речовину, яка втручання внутрішньовенно ввели
використовується для оборотної денатурації концентрований розчин альбуміну. Це
гемоглобіну: зумовило посилений рух води в такому
Сечовина напрямку:
Толуол З міжклітинної рідини в капіляри
С. Сірчана кислота З міжклітинної рідини в клітини
D. Азотна кислота З клітин у міжклітинну рідину
E. Гідроксид натрію З капілярів у міжклітинну рідину
Збірник тестових завдань для складання Змін руху води не буде
ліцензійного іспиту Крок-1 «Загальна Збірник тестових завдань для складання
лікарська підготовка». –2005. – № 47. ліцензійного іспиту Крок-1 «Загальна
лікарська підготовка». –2009. – № 96.
Деякі білки слини виконують захисну
функцію. Який з них захищає слизову Катіонні глікопротеїни є основними
оболонку ротової порожнини від механічних компонентами слини привушних залоз. Які
ушкоджень? амінокислоти обумовлюють їх позитивний
Муцин заряд?
Лізоцим Лізин, аргінін, гістидин
Каталаза Аспартат, глутамат, гліцин
Пероксидаза Аспартат, аргінін, глутамат
Ренін Глутамат, валін, лейцин
Збірник тестових завдань для складання Цистеїн, гліцин, пролін
ліцензійного іспиту Крок-1 «Стоматологія». Збірник тестових завдань для складання
– 2013. – № 185. ліцензійного іспиту Крок-1 «Стоматологія».
– 2015. – № 44.
Для дослідження білків сироватки можуть
бути використані різні фізичні та фізико- Пародонтит супроводжується активацією
хімічні методи. Зокрема, сироваткові протеолізу в тканинах пародонту.
альбуміни та глобуліни можуть бути Підвищення якого компоненту ротової
розділені таким способом: рідини свідчить про активацію протеолізу?
Електрофорез Амінокислоти
Полярографія Органічні кислоти
Діаліз Глюкоза
Спектрографія Бiогенні аміни
Рефрактометрія Холестерол
Збірник тестових завдань для складання Збірник тестових завдань для складання
ліцензійного іспиту Крок-1 «Загальна ліцензійного іспиту Крок-1 «Стоматологія».
лікарська підготовка». –2013. – № 112. – 2008. – № 52.

Електрофоретичне дослідження сироватки Студент використав консервовану донорську


крові хворого на пневмонію показало кров для визначення часу її зсідання. Однак,
збільшення одної з білкових фракцій. будь-якого позитивного результату він
Вкажіть її: отримати не зміг. Причиною цього є
γ-глобуліни відсутність в крові:
Альбуміни Іонізованого кальцію
α1-глобулiни Фактора Хагемана
α2-глобулiни Тромбопластину
β-глобуліни Фібриногену
Збірник тестових завдань для складання Вітаміну K
ліцензійного іспиту Крок-1 «Стоматологія». Збірник тестових завдань для складання
– 2013. – № 25. ліцензійного іспиту Крок-1 «Загальна
лікарська підготовка». –2017. – № 163.
Тривалий вплив на організм токсичних Збірник тестових завдань для складання
речовин призвів до значного зниження ліцензійного іспиту Крок-1 «Стоматологія».
синтезу білків у гепатоцитах. Які органели – 2015. – № 47.
постраждали від інтоксикації найбільше?
Гранулярна ендоплазматична сітка Хірург використав 70% розчин спирту
Мітохондрії етилового для обробки рук перед
Мікротрубочки оперативним втручанням. Який основний
Лізосоми механізм антисептичної дії препарату на
Комплекс Гольджі мікроорганізми?
Збірник тестових завдань для складання Дегідратація білків протоплазми
ліцензійного іспиту Крок-1 «Стоматологія». Блокада сульфгідрильних груп
– 2005. – № 65. ферментних систем
Окислення органічних компонентів
У дітей, хворих на квашіоркор, поряд з протоплазми
іншими ознаками виявлені порушення Взаємодія з аміногрупами білків
процесу утворення зубів. В основі цього протоплазми
явища лежить недостатнє надходження в Взаємодія з гідроксильними групами
організм: ферментів
Білків Збірник тестових завдань для складання
Жирів ліцензійного іспиту Крок-1 «Стоматологія».
Вуглеводів – 2005. – № 186.
Вітаміну С
Вітаміну В1 Яка речовина надає слинi в’язкий, слизовий
Збірник тестових завдань для складання характер, виконує захисну роль, у тому числі
ліцензійного іспиту Крок-1 «Стоматологія». від механічного пошкодження слизової рота?
– 2005. – № 62. Муцин
Глюкоза
У добовому раціоні дорослої здорової Калікреїн
людини повинні бути жири, білки, Амілаза
вуглеводи, вітаміни, мінеральні солі та вода. Лізоцим
Вкажіть добову кількість білку (г), яка Збірник тестових завдань для складання
забезпечує нормальну життєдіяльність ліцензійного іспиту Кро -1 «Стоматологія».
організму: – 2005. – № 103.
A. 100-120
B. 50-60 Тільки один фактор можев пливати на заряд
C. 10-20 амінокислотних радикалів в активному
D. 70-80 центрі ферменту. Назвіть цей фактор:
E. 40-50 pHсередовища
Збірник тестових завдань для складання Тиск
ліцензійного іспиту Крок-1 «Стоматологія». Температура
– 2016. – № 25. Наявність конкурентного інгібітора
Надлишок продукту
Хворому з печінковою недостатністю Збірник тестових завдань для складання
проведено дослідження електрофоретичного ліцензійного іспиту Крок-1 «Загальна
спектру білків сироватки крові. Які фізико- лікарська підготовка»–2005. – № 51.
хімічні властивості білкових молекул лежать
в основі цього методу? Кон’югований білок обов’язково містить
Наявність заряду спеціальний компонент в якості
Гідрофільність простетичної групи. Оберіть речовину, яка
Здатність набрякати не може виконувати цю функцію:
Оптична активність HNO3
Нездатність до діалізу АТФ
Тіамін пірофосфат
AMФ форм в крові. Аналіз на який фермент
Глюкоза вважається оптимальним для підтвердження
Збірник тестових завдань для складання або спростування діагнозу інфаркт міокарду
ліцензійного іспиту Крок-1 «Загальна на ранній стадії після появи у пацієнта
лікарська підготовка»–2005. – № 46. торакального болю?
MB ізоформи креатинкінази
60-річний чоловік звернувся до лікаря після MM ізоформи креатинкінази
появи болю в грудній клітці. В сироватці Ізоферменту ЛДГ-1
крові виявлено значне зростання активності Ізоферменту ЛДГ-5
ферментів: креатинфосфокінази та її Цитоплазматичного ізоферменту аспартат
МВ-ізоформи, амінотрансферази
аспартатамінотрансферази. Про розвиток Збірник тестових завдань для складання
патологічного процесу в якій тканині ліцензійного іспиту Крок-1 «Стоматологія».
свідчать ці зміни? – 2017. – № 105.
Сердцевий м’яз
Легені Для лiкування деяких iнфекцiйних
Скелетні м’язи захворювань, що викликаються бактерiями,
Печінка застосовуються сульфанiламiднi препарати.
Гладенькі м’язи Який механiзм дiї цих препаратiв?
Збірник тестових завдань для складання Є антивiтамiнами п-амiнобензойної кислоти.
ліцензійного іспиту Крок-1 «Стоматологія». Є алостеричними iнгiбiторами ферментiв.
– 2013. – № 34. Є алостеричними активаторами ферментів.
Беруть участь в окисно-вiдновних процесах.
Видiляють декiлька груп молекулярних Iнгiбують всмоктування фолiєвої кислоти.
механiзмiв, якi мають важливе значення в Збірник тестових завдань для складання
патогенезi ушкодження клiтин, що сприяє ліцензійного іспиту Крок-1 «Стоматологія».
розвитку патологiї. Якi процеси – 2013. – № 40.
забезпечують протеїновi механiзми
ушкодження? До вiддiлення реанiмацiї надiйшов чоловiк
Пригнiчення ферментiв 47-ми рокiв з дiагнозом iнфаркт мiокарду.
Перекисне окиснення лiпiдiв Яка з фракцiй лактатдегiдрогенази (ЛДГ)
Активацiя фосфолiпаз буде переважати в сироватцi кровi протягом
Осмотичне розтягнення мембран перших двох дiб?
Ацидоз ЛДГ 1
Збірник тестових завдань для складання ЛДГ 2
ліцензійного іспиту Крок-1 «Загальна ЛДГ 3
лікарська підготовка» – 2008. – № 123. ЛДГ 4
ЛДГ 5
Відомо, що активність навколовушних Збірник тестових завдань для складання
слинних залоз піддається віковим змінам. ліцензійного іспиту Крок-1 «Загальна
Активність якого ферменту буде різко лікарська підготовка» – 2016. – № 34.
зменшуватись в слині?
Амілази. З сироватки кровi людини видiліли п’ять
Гексокінази. iзоферментних форм лактатдегiдрогенази i
Реніну. вивчили їх властивостi. Яка властивiсть
Мальтази. доводить, що видiленi iзоферментнi форми
Фосфатази. одного i того ж ферменту?
Збірник тестових завдань для складання Каталiзують одну i ту ж реакцiю
ліцензійного іспиту Крок-1 «Стоматологія». Однакова молекулярна маса
– 2012. – № 54.

Для біохімічної діагностики інфаркту


міокарду необхідно визначити активність
декількох ферментів та їх ізоферментних
Однаковi фiзико-хiмiчнi властивостi Печiнка (можливий гепатит)
Тканинна локалiзацiя Серцевий м’яз (можливий iнфаркт
Однакова електрофоретична рухливiсть мiокарда)
Збірник тестових завдань для складання Скелетнi м’язи
ліцензійного іспиту Крок-1 «Загальна Нирки
лікарська підготовка» – 2008. – № 7. Сполучна тканина
Збірник тестових завдань для складання
Одним із шляхів регуляції активності ліцензійного іспиту Крок-1 «Загальна
ферментів в людському тілі є ковалентна лікарська підготовка» – 2011. – № 46.
модифікація. Активність
глікогенфосфорилази і глікогенсинтази У кровi хворого концентрацiя альбумiнiв
регулюється завдяки наступному типу складає 2,8 г/л, пiдвищена концентрацiя
ковалентної модифікації: лактатдегiдрогенази 5 (ЛДГ5). Про
Фосфорилювання-дефосфорилювання захворювання якого органа це свiдчить?
АДФ-рибозилювання Печiнка
Метилювання Нирка
Обмежений протеоліз Серце
Сульфування Легеня
Збірник тестових завдань для складання Селезiнка
ліцензійного іспиту Крок-1 «Стоматологія». Збірник тестових завдань для складання
– 2014. – № 98. ліцензійного іспиту Крок-1 «Загальна
лікарська підготовка» – 2005. – № 35.
При дослiдженнi кровi хворого виявлено
значне збiльшення активностi МВ-форм У крові пацієнта спостерігається підвищення
КФК (креатинфосфокiнази) та ЛДГ-1. Яку активності АсАТ, ЛДГ-1, ЛДГ-2 і КФК.
патологiю можна припустити? Патологічний процес найбільш імовірно
Iнфаркт мiокарда розвивається у:
Гепатит Серці
Ревматизм Скелетних м’язів
Панкреатит Нирках
Холецистит Печінці
Збірник тестових завдань для складання Надниркових залозах
ліцензійного іспиту Крок-1 «Загальна Збірник тестових завдань для складання
лікарська підготовка» – 2014. – № 15. ліцензійного іспиту Крок-1 «Стоматологія».
– 2017. – № 114.
У кровi хворого виявлено пiдвищення
активностi ЛДГ1, ЛДГ2, АсАТ, У плазмi кровi пацiєнта зпiдвищилась
креатинкiнази. В якому органi хворого активнiсть iзоферментiв ЛДГ1 i ЛДГ2. Про
найбiльш вiрогiдний розвиток патологiчного патологiю якого органа це свiдчить?
процесу? A. Мiокард
Серце B. Печiнка
Пiдшлункова залоза C. Нирки
Печiнка D. Мозок
Нирки E. Скелетнi м’язи
Скелетнi м’язи Збірник тестових завдань для складання
Збірник тестових завдань для складання ліцензійного іспиту Крок-1 «Стоматологія».
ліцензійного іспиту Крок-1 «Загальна – 2017. – № 154.
лікарська підготовка» – 2005. – № 52.
У хворого виявлено пiдвищення активностi
У кровi хворого виявлено пiдвищення ЛДГ1,2, АсАТ,
активностi ЛДГ4,5, АлАТ, креатинфосфокiнази. В якому органi
карбамоїлорнiтинтрансферази. В якому (органах) найбiльш вiрогiдний розвиток
органi можна передбачити розвиток патологiчного процесу?
патологiчного процесу? A. Серцевий м’яз
B. Скелетнi м’язи E. ЛДГ 5
C. Нирки та наднирковi залози Збірник тестових завдань для складання
D. Сполучна тканина ліцензійного іспиту Крок-1 «Загальна
E. Печiнка та нирки лікарська підготовка» – 2013. – № 130.
Збірник тестових завдань для складання
ліцензійного іспиту Крок-1 «Загальна В плазмі крові пацієнта виявлено виисокий
лікарська підготовка» – 2007. – № 11. рівень ізоформ
лактатдегідрогенази (ЛДГ) за рахунок
Хворий 49 років водій за професією зростания концентрації ЛДГ-1 та ЛДГ-2.
скаржиться на нестерпні стискаючі болі за Вкажіть найбільш імовірний діагноз:
грудиною, що “віддають” у ділянку шиї, які A. Інфаркт міокарду
виникли 2 години тому. Стан важкий, B. Дистрофія скелетних м’язів
блідість, тони серця послаблені. Лабораторне C. Цукровий діабет
обстеження показало високу активність D. Вірусний гепатит
креатинкинази та ЛДГ1. Для якого E. Гострий панкреатит
захворювання характерні подібні зміни? Збірник тестових завдань для складання
A. Гострий інфаркт міокарда. ліцензійного іспиту Крок-1 «Загальна
B. Гострий панкреатит. лікарська підготовка» – 2005. – № 52.
C. Стенокардія.
D. Жовчнокам'яна хвороба. Сукцинатдегідрогеназа каталізує
E. Цукровий діабет. дегідрування сукцинату. Малонова кислота
Збірник тестових завдань для складання HOOC - CH2 - COOH використовується для
ліцензійного іспиту Крок-1 «Загальна інгібування дії цього ферменту. Оберить тип
лікарська підготовка» – 2009. – № 189. її дії:
А. Конкурентна
Через 6 годин після інфаркту міокарду у Б. Алостерична
хворого в крові піднялась активність C. Неконкурентна
лактатдегідрогенази. Наявність якого D. Обмежений протеоліз
ізоферменту треба очікувати в цьому E. Дефосфорилювання
випадку? Збірник тестових завдань для складання
A. ЛДГ 1 ліцензійного іспиту Крок-1 «Загальна
B. ЛДГ 2 лікарська підготовка» – 2005. – № 53.
C. ЛДГ 3
D. ЛДГ 4
Біохімія водо – та жиророзчинних вітамінів
(водорозчинні вітаміни)
1. У пацієнта, який впродовж тривалого часу C. B2
харчувався виключно кукурудзою, виявлено D. B9
дерматит, діарею, деменцію. З недостатністю E. B8
якого вітаміну пов’язані дані порушення? Збірник тестових завдань для складання
A. Вітаміну РР. ліцензійного іспиту Крок-1 «Стоматологія».
B. ВітамінуВ2. – 2005. – № 110.
C. Вітаміну В8.
D. Вітаміну В9. 5. До лiкарнi звернувся чоловiк 50-ти рокiв з
E. Вітаміну В1. розладами пам’ятi, болiсними вiдчуттями по
Збірник тестових завдань для складання ходу нервових стовбурiв, зниженням
ліцензійного іспиту Крок-1 «Стоматологія». iнтелектуальних функцiй, порушеннями з
– 2005. – № 110. боку серцево-судинної системи i явищами
диспепсiї. В анамнезi хронiчний алкоголiзм.
2. На прийом до лiкаря звернувся хворий з Дефiцит якого вiтамiну може викликати цi
симетричним дерматитом вiдкритих дiлянок симптоми?
шкiри. З бесiди з пацiєнтом встановлено, що A. Тiамiн
вiн харчується, в основному, крупами i B. Нiацин
вживає мало м’яса, молока та яєць. Дефiцит C. Ретинол
якого вiтамiну є провiдним у цього пацiєнта? D. Кальциферол
A. Нiкотинамiд E. Рибофлавiн
B. Кальциферол Збірник тестових завдань для складання
C. Фолiєва кислота ліцензійного іспиту Крок-1 «Загальна
D. Бiотин лікарська підготовка». –2015. - № 184.
E. Токоферол
Збірник тестових завдань для складання 6. У хворого виявлена болiснiсть походу
ліцензійного іспиту Крок-1 «Загальна крупних нервових стовбурiв та пiдвищений
лікарська підготовка». –2008. - № 22. вмiст пiрувату в кровi. Нестача якого
вiтамiну може викликати такiзмiни?
3. Дитина 3-х років із симптомами A. B1
стоматиту, гінгівіту, дерматиту відкритих B. B2
ділянок шкіри була госпіталізована. При C. РР
обстеженні встановлено спадкове порушення D. Пантотенова кислота
транспорту нейтральних амінокислот у E. Бiотин
кишці. Нестачею якого вітаміну будуть Збірник тестових завдань для складання
зумовлені дані симптоми? ліцензійного іспиту Крок-1 «Стоматологія».
A. Ніацину. – 2006. – № 95
B. Пантотенової кислоти.
C. Вітаміну А. 7. У жiнки, яка тривалий час знаходилася на
D. Кобаламіну. дiєтi з використанням очищеного рису,
E. Біотину. виявлено полiневрит (хвороба Берi-Берi).
Збірник тестових завдань для складання Вiдсутнiсть якого вітаміну в рацiонi
ліцензійного іспиту Крок-1 «Загальна призводить до розвитку цього
лікарська підготовка». –2008. - № 110 . захворювання?
A. Тiамiн
4. У хворого спостерiгаються дерматит, B. Аскорбiнова кислота
дiарея, деменцiя. В анамнезу вiдомо, що C. Пiридоксаль
основним продуктом харчування хворого є D. Фолiєва кислота
кукурудза. З нестачею якого вiтамiну E. Рибофлавiн
пов’язанi цi порушення? Збірник тестових завдань для складання
A. РР ліцензійного іспиту Крок-1 «Загальна
B. B1 лікарська підготовка». – 2007. – № 33.
8. У пацієнта в кровi збiльшена концентрацiя 12. Піровиноградна кислота як проміжний
пiрувату. Значна кiлькiсть його метаболіт вуглеводного, ліпідого й
екскретується з сечею. Який авiтамiноз амінокислотного обміну підлягає окисному
спостерiгається у пацієнта? декарбоксилюванню. Причиною порушення
A. Вітаміну B1. цього процесу може бути недостатність в
B. Вітаміну B2. раціоні наступного компоненту:
C. Вітаміну B3. A. Тіамін
D. Вітаміну B6. B. Піридоксин
E. Вітаміну E. C. Аскорбінова кислота
Збірник тестових завдань для складання D. Цитрін
ліцензійного іспиту Крок-1 «Загальна E. Пангамова кислота
лікарська підготовка». –2009. - № 29. Збірник тестових завдань для складання
ліцензійного іспиту Крок-1 «Стоматологія».
9. У хворого, що харчувався винятково – 2014. – № 151.
полiрованим рисом, недостатнiсть тiамiну
стала причиною полiневриту. Екскрецiя якої 13. У пацієнта з хронічним алкоголізмом
сполуки з сечею можебути iндикатором з’явилися симптоми поліневриту і серцевої
цього авiтамiнозу? недостатності. Який вітамінний препарат
A. Пiровиноградна кислота необхідно призначити цьому пацієнту?
B. Малат A. Тіамін
C. Метил-малонова кислота B. Ергокальциферол
D. Сечова кислота C. Ретинол
E. Фенiлпiруват D. Рутін
Збірник тестових завдань для складання E. Філохінон
ліцензійного іспиту Крок-1 «Стоматологія». Збірник тестових завдань для складання
– 2011. – № 108. ліцензійного іспиту Крок-1 «Стоматологія».
– 2006. – № 191.
10. Внаслiдок дефiциту вiтамiну B1
порушується окисне декарбоксилювання α- 14. У пацієнта 32 років має місце
кетоглутарової кислоти. Синтез якого з гіповітаміноз В2. Причиною виникнення
наведених коферментiв порушується при специфічних симптомів (ураження епітелію,
цьому? слизових, шкіри, рогівки ока) найімовірніше
A. Тiамiнпiрофосфат є дефіцит:
B. Нiкотинамiдаденiндинуклеотид A. Флавінових коферментів.
C. Флавiнаденiндинуклеотид B. Цитохрому в.
D. Лiпоєва кислота C. Цитохромоксидази.
E. Коензим А D. Цитохрому с.
Збірник тестових завдань для складання E. Цитохрому а1.
ліцензійного іспиту Крок-1 «Загальна Збірник тестових завдань для складання
лікарська підготовка». –2008. - № 81. ліцензійного іспиту Крок-1 ««Загальна
лікарська підготовка». –2013. - № 175.
11. Для покращання трофiки серцевого м’яза
пацiєнту призначено препарат, до складу 15. При малярії призначають препарати -
якого входить кокарбоксилаза структурні аналоги вітаміну В2 (рибофлавін).
(тiамiндифосфат) – коферментна форма Порушення синтезуяких ферментів у
вiтамiну: плазмодію викликаютьці препарати?
A. B1 A. ФАД-залежні дегідрогенази
B. B2 B. Цитохромоксидази
C. B5 C. Пептидази
D. B6 D. НАД-залежні дегідрогенази
E. B12 E. Амінотрансферази
Збірник тестових завдань для складання Збірник тестових завдань для складання
ліцензійного іспиту Крок-1 «Стоматологія». ліцензійного іспиту Крок-1 «Загальна
– 2015. – № 110. лікарська підготовка». –2013. - № 2
16. При ентеробіозі призначають акрихін – D. Вітаміну К.
структурний аналог вітаміну В2. Порушення E. Вітаміну РР.
синтезу яких ферентів спричиняє цей Збірник тестових завдань для складання
препарат у мікроорганізмів? ліцензійного іспиту Крок-1 «Загальна
A. ФАД-залежних дегідрогеназ лікарська підготовка». –2007. - № 52.
B. Цитохромоксидаз
C. Пептидаз 20. У пацiєнта навеснi з’явилися петехiальнi
D. НАД-залежних дегідрогеназ крововиливи, розхитування зубiв, вiн
E. Амінотрансфераз вiдмiчає високу чутливiсть до простудних
Збірник тестових завдань для складання хвороб. Лiкар припустив гiповiтамiноз C.
ліцензійного іспиту Крок-1 «Загальна Чим пояснюються змiни з боку зубiв?
лікарська підготовка» – 2005. – № 43. A. Порушення структури колагену
перiодонтальних зв’язок
17. Для лікування пародонтиту в комплекс B. Змiна структури глiкозамiноглiканiв
препаратів був включений лікарський засіб із C. Пiдвищення проникностi мембран
групи водорозчинних вітамінів, похідне навколозубних тканин
біофлавоноїдів, який призначають разом із D. Механiчне ушкодження зубiв
кислотою аскорбіновою. Препарат має E. Порушення окисно-вiдновних процесiв у
антиоксидантні властивості, зменшує навколозубних тканинах
кровоточивість ясен. Який це препарат? Збірник тестових завдань для складання
A. Рутин. ліцензійного іспиту Крок-1 «Стоматологія».
B. Кальцію пантотенат. – 2011. – № 197.
C. Кальцію пангамат.
D. Кислота фолієва. 21. Бiльша частина учасникiв експедицiї
E. Ціанкобаламін. Магелана до Америки загинула від
Збірник тестових завдань для складання авiтамiнозу. Це захворювання проявлялося
ліцензійного іспиту Крок-1 «Стоматологія». загальною слабкiстю, пiдшкiрними
– 2006. – № 177 крововиливами, випадiнням зубiв,
кровотечею з ясен. Як називається цей
18. Пiд час огляду дитини, що не отримувала авiтамiноз?
впродовж зими свiжих овочiв і фруктiв, A. Скорбут (цинга)
виявленi множиннi пiдшкiрнi крововиливи, B. Пелагра
запалення ясен, карiознi порожнини в зубах. C. Рахiт
Комбiнацiю яких вiтамiнiв слiд призначити D. Полiневрит (берi-берi)
дитинi? E. Анемiя Бiрмера
A. Аскорбiнова кислота та рутин. Збірник тестових завдань для складання
B. Кальциферол та аскорбiнова кислота. ліцензійного іспиту Крок-1 «Загальна
C. Рибофлавiн і нiкотинамiд. лікарська підготовка». –2006. - № 52 .
D. Тiамiн і пiридоксин.
E. Фолiєва кислота і кобаламiн. 22. Дiвчинка 10-ти рокiв часто хворiє на
Збірник тестових завдань для складання гострi респiраторнi iнфекцiї, після яких
ліцензійного іспиту Крок-1 «Стоматологія». спостерiгаються множиннi точковi
– 2007. – № 88 крововиливи в мiсцях тертя одягу. Який
гiповiтамiноз має мiсце в дiвчинки:
19. У пацієнта з частими кровотечами у A. C
внутрішні органи та слизові оболонки B. B6
виявлено недостатність гідроксипроліну та C. B1
гідроксилізину у складі колагенових D. A
волокон. Через нестачу якого вітаміну E. B2
порушено процеси гідроксилування вказаних Збірник тестових завдань для складання
амінокислот? ліцензійного іспиту Крок-1 «Загальна
A. Вітаміну С. лікарська підготовка». –2010. - № 4.
B. Вітаміну А.
C. Вітаміну Н.
23. У хворого спостерiгається збiльшення A. Пiридоксину.
проникностi стiнок кровоносних судин iз B. Кобаламiну.
розвитком пiдвищеної кровоточивостi ясен, C. Бiотину.
виникнення дрiбнокрапчастих крововиливiв D. Тiамiну.
на шкiрi, випадiння зубiв. Яким порушенням E. Фолiєвої кислоти.
вiтамiнного обмiну пояснюються цi Збірник тестових завдань для складання
симптоми? ліцензійного іспиту Крок-1 «Стоматологія».
A. Гiповiтамiноз С – 2011. – № 111.
B. Гiпервiтамiноз D
C. Гiпервiтамiноз С 27. Який вітамін входить до складу
D. Гiповiтамiноз D декарбоксилази глютамінової кислотита
E. Гiповiтамiноз А приймає участь в утворенні ГАМК, а його
Збірник тестових завдань для складання недостатність проявляється судомами?
ліцензійного іспиту Крок-1 «Стоматологія». A. Піридоксин
– 2005. – № 109. B. Кобаламін
C. Токоферол
24. 20-річний хворой скаржиться на загальну D. Фолієва кислота
слабкість, швидку втомлюваність, E. Аскорбінова кислота
дратівливість, зниженя працездатності, Збірник тестових завдань для складання
кровоточивість ясен, петехії на шкірі. ліцензійного іспиту Крок-1 «Стоматологія».
Дефицит якого вітаміну міг викликати ці – 2014. – № 136.
зміни?
A. Аскорбінова кислота 28. Дитина 9-ми мiсяцiв харчується
B. Рибофлавін штучними сумiшами, якi не збалансованi за
C. Тіамін вмiстом вiтамiну B6. У дитини
D. Ретинол спостерiгається пелагроподiбний дерматит,
E. Фолієва кислота судоми, анемiя. Розвиток судом може бути
Збірник тестових завдань для складання пов’язаний з порушенням утворення:
ліцензійного іспиту Крок-1 «Загальна A. ГАМК
лікарська підготовка». –2013. - № 99 B. Гiстамiну
C. Серотонiну
25. Пацієнт із дiагнозом вогнищевий D. ДОФА
туберкульоз верхньої частки правої легенi в E. Дофамiну
складi комбiнованої терапiї отримує Збірник тестових завдань для складання
iзонiазид. Через деякий час він почав ліцензійного іспиту Крок-1 «Загальна
скаржитися на м’язову слабкiсть, втрату лікарська підготовка». –2009. - № 149 .
чутливостi шкіри, порушення зору та
координацiї рухiв. Який вiтамiнний препарат 29. У немовляти спостерiгаються
доцiльно використати для усунення вказаних епiлептиформнi судоми, викликанi
явищ? дефiцитом вiтамiну В6. Це спричинено
A. Вiтамiн В6. зменшенням у нервовiй тканинi гальмiвного
B. Вiтамiн А. медiатора- γ-амiномасляної кислоти.
C. Вiтамiн В12. Активнiсть якого ферменту знижена при
D. Вiтамiн C. цьому?
E. Вiтамiн D. A. Глутаматдекарбоксилаза
Збірник тестових завдань для складання B. Аланiнамiнотрансфераза
ліцензійного іспиту Крок-1 «Загальна C. Глутаматдегiдрогеназа
лікарська підготовка». – 2011. – № 5. D. Пiридоксалькiназа
E. Глутаматсинтетаза
26. 40-рiчному чоловiковi, який хворiє на Збірник тестових завдань для складання
туберкульоз легенів, призначено iзонiазид. ліцензійного іспиту Крок-1 «Загальна
Нестача якого вiтамiну може розвинутися лікарська підготовка». –2016. - № 16 .
внаслiдок тривалого вживання даного
препарату?
30. На основі клінічних симптомів чоловіку Збірник тестових завдань для складання
призначили піридоксальфосфат. Який ліцензійного іспиту Крок-1 «Загальна
процес в організмі коректує цей препарат? лікарська підготовка». –2015. - № 87.
A. Трансамінування і декарбоксилювання
амінокислот 34. Для нормального перебiгу процесу
B. Окисне декарбоксилювання кетокислот реплiкацiї потрiбнi тимiдиловi нуклеотиди,
C. Дезамінування пуринових нуклеотидів синтез яких відбувається за участю
D. Синтез пуринових і піримідинових основ тимiдилатсинтетази, в якостi коферменту
E. Синтез білків використовується:
Збірник тестових завдань для складання A. Метилентетрагiдрофолат
ліцензійного іспиту Крок-1 «Стоматологія». B. Карбоксибiотин
– 2006. – № 110. C. Тiамiндифосфат
D. Пiридоксальфосфат
31. У клiнiчнiй практицi для лiкування E. Нiкотинамiдаденiндинуклеотид
туберкульозу застосовують препарат Збірник тестових завдань для складання
iзонiазид - антивiтамiн, який здатний ліцензійного іспиту Крок-1 «Стоматологія».
проникати у туберкульозну паличку. – 2014. – № 165.
Туберкулостатичний ефект обумовлений 35. Похідні птерину (аміноптерин та
порушенням процесiв реплiкацiї, окисно- метотрексат) є інгібіторами
вiдновних реакцiй, завдякиутвореннюне дигідрофолатредуктази, отже, вони
справжнього коферменту з: гальмують відновлення тетрагідрофолієвої
A. НАД+ кислоти з дигідрофолату. Ці ліки
B. ФАД пригнічують міжмолекулярний транспорт
C. ФМН одновуглецевих фрагментів, що інгібує
D. ТДФ біосинтез наступного полімеру:
E. КоQ A. ДНК
Збірник тестових завдань для складання B. Білків
ліцензійного іспиту Крок-1 «Загальна C. Гомополісахаридів
лікарська підготовка». –2009. - № 177. D. Гангліозидів
32. Реакції міжмолекулярного транспорту E. Глікозаміногліканів
одновуглецевих радикалів є необхідними для Збірник тестових завдань для складання
синтезу білків та нуклеїнових кислот. З якого ліцензійного іспиту Крок-1 «Загальна
з перерахованих нижче попередників лікарська підготовка». –2013. - № 3.
утворюється кофермент, потрібний для
вказаних реакцій? 36. Після тривалого лікування
A. Фолієвої кислоти. сульфаніламидними препаратами у пациента
B. Рибофлавіну. розвинулась макроцитарна анемія.
C. Аскорбінової кислоти. Утворення активної форми наступного
D. Пантотенової кислоти. вітаміну було порушено в даних умовах:
E. Тіаміну. A. Фолієвої кислоти
Збірник тестових завдань для складання B. Тіаміну
ліцензійного іспиту Крок-1 «Загальна C. Рибофлавіну
лікарська підготовка». –2017. - № 172. D. Піридоксину
E. Ціанкобаламіну
33. У синтезi пуринових нуклеотидiв беруть Збірник тестових завдань для складання
участь деякi амiнокислоти, похiднi вiтамiнiв, ліцензійного іспиту Крок-1 «Загальна
фосфорнi ефiри рибози. Коферментна форма лікарська підготовка». –2016. - № 6.
якого вiтамiну є переносником
одновуглецевих фрагментiв в цьому синтезi? 37. Вiдомо, що частина диоксиду вуглецю
A. Фолiєва кислота використовується в органiзмi в бiосинтезi
B. Пантотенова кислота жирних кислот, сечовини, глюконеогенезi
C. Нiкотинова кислота тощо. Який вiтамiн утворюєCO2-
D. Рибофлавiн транспортуючу форму для цих реакцiй?
E. Пiридоксин A. Бiотин
B. Тимiн A. B12
C. Рибофлавiн B. C
D. Нiкотинамiд C. P
E. Ретинол D. B6
Збірник тестових завдань для складання E. PP
ліцензійного іспиту Крок-1 «Стоматологія». Збірник тестових завдань для складання
– 2015. – № 136. ліцензійного іспиту Крок-1 «Загальна
лікарська підготовка». –2011. - № 88.
38. У пацієнта дiагностовано себорейний
дерматит, пов’язаний iз дефiцитом вiтамiну 42. Обстежуючи ротову порожнину хворого,
H (бiотину). Порушення активностi якого з стоматолог звернув увагу на наявнiсть
перелiчених ферментiв спостерiгається у запально-дистрофiчного процесу в слизовiй
пацієнта? оболонцi (гунтеровський глосит, атрофiчний
A. Ацетил-КоА-карбоксилази. стоматит). Аналiз кровi виявив гiперхромну
B. Амiнотрансферази. анемiю. Який фактор є причиною цього
C. Алкогольдегiдрогенази. захворювання?
D. Карбамоїлфосфатсинтетази. A. Гiповiтамiноз B12
E. Пiруватдекарбоксилази. B. Гiповiтамiноз B1
Збірник тестових завдань для складання C. Гiповiтамiноз B6
ліцензійного іспиту Крок-1 «Стоматологія». D. Пiдвищення кислотностi шлункового соку
– 2010. – № 94. E. Гiповiтамiноз A
39. У пацієнта діагностована мегалобластна Збірник тестових завдань для складання
анемія. Вкажіть вітамін, дефіцит якого може ліцензійного іспиту Крок-1 «Стоматологія».
призвести до розвитку цього захворювання. – 2012. – № 67.
A. Ціанокобаламін.
B. Рутин. 43. У хворого, що перенiс 5 рокiв тому
C. Нікотинамід. субтотальну резекцію шлунка, розвинулась
D. Тіамін. В12-фолiєводефiцитна анемiя. Який
E. Холекальциферол. механізм є провiдним у розвитку такої
Збірник тестових завдань для складання анемiї?
ліцензійного іспиту Крок-1 «Загальна A. Вiдсутнiсть внутрішнього фактора Касла
лікарська підготовка». –2012. - № 154. B. Вiдсутнiсть зовнішнього фактора Касла
C. Порушення всмоктування вiтамiну В12 в
40. У 52-річної пацієнтки діагностовано тонкiй кишцi
хронічний атрофічний гастрит, D. Дефiцит фолiєвої кислоти
мегалобластну гіперхромну анемію. E. Дефiцит транскобаламiну.
Лабораторні аналізи виявляють підвищення Збірник тестових завдань для складання
виділення метилмалонової кислоти з сечею. ліцензійного іспиту Крок-1 «Загальна
Недостатнім надходженням якого вітаміну лікарська підготовка». –2016. - № 71.
обумовлене виникнення таких порушень
A. Вітаміну В12. 44. У хворого зi скаргами на бiль у шлунку
B. Вітаміну В2. встановлено зменшення його секреторної
C. Вітаміну В3. функцiї, що супроводжується анемiєю.
D. Вітаміну В5. Нестатнiсть якої речовини обумовлює
E. Вітаміну В1. розвиток у хворого гiповiтамiнозу B12 та
Збірник тестових завдань для складання виникнення анемiї?
ліцензійного іспиту Крок-1 «Стоматологія». A. Фактор Кастла
– 2005. – № 127. B. Тiамiн
C. Бiотин
41. Пiсля видалення у пацiєнта 2/3 шлунка у D. Пiридоксин
кровi зменшився вмiст гемоглобiну, кiлькiсть E. Кальциферол
еритроцитiв, збiльшилися розмiри цих клiтин Збірник тестових завдань для складання
кровi. Дефiцит якого вiтамiну призводить до ліцензійного іспиту Крок-1 «Стоматологія».
таких змiн у кровi? – 2017. – № 86.
45. Пiсля оперативного видалення частини 48. При обстеженні 50-річного пацієнта
шлунка у хворого порушилося всмоктування дантист звернув увагу на малиновий
вiтамiну B12, вiн виводиться з калом. «полірований» язик. Аналіз крові показав
Розвинулася анемiя. Який фактор зниження рівня ШОЕ і концентрації
необхiдний для всмоктування цього гемоглобіну, кольоровий показник 1,3,
вiтамiну? симптоми мегалобластичного гемопоезу,
A. Гастромукопротеїн дегенеративні зміни лейкоцитів. Яке
B. Гастрин захворювання крові виявлено у цього
C. Соляна кислота пацієнта?
D. Пепсин A. B12-фолієво-дефіцитна анемія
E. Фолiєва кислота B. Залізодефіцитна анемія
Збірник тестових завдань для складання C. Міелоидна лейкемія
ліцензійного іспиту Крок-1 «Загальна D. Апластична анемія
лікарська підготовка». –2018. - № 135. E. Гемолітична анемія
Збірник тестових завдань для складання
46. У пацієнта 65-ти років з тривалими ліцензійного іспиту Крок-1 «Стоматологія».
скаргами, характерними для хроничного – 2013. – № 37.
гастриту, в периферичній крові виявлені
мегалоцити, у кістковому мозку 49. Коензим А бере участь у багатьох
мегалобластичний еритропоез. Який діагноз важливих реакціях метаболізму. Він є
найбільш імовірний? похідним наступного вітаміну:
A. B12-фолієво дефіцитна анемія A. Пантотенова кислота
B. Апластична анемія B. Тіамін
C. Гіпопластичная анемія C. Ніацин
D. Гемолітичная анемія D. Кальциферол
E. Залізодефіцитна анемія E. Убіхінон
Збірник тестових завдань для складання Збірник тестових завдань для складання
ліцензійного іспиту Крок-1 «Загальна ліцензійного іспиту Крок-1 «Загальна
лікарська підготовка». –2017. - № 55 лікарська підготовка». –2018. - № 186

47. 13-ти річній дівчинці призначили ліки 50. Чоловiку з виразковою хворобою
для лікування мегалобластичної анемії. Ці дванадцятипалої кишки лiкар після курсу
ліки стимулюють перехід від терапiї пропонує вживання соків iз капусти
мегалобластичного гемопоезу до та картоплi. Вмiст якої речовини в цих
нормобластичного, бере участь у синтезі овочах сприяє профілактиці та загоєнню
пуринових і піримідинових основ, активує виразок?
синтез білків та метіоніну. Які ліки A. Вiтамiн U
призначили пацієнці? B. Пантотенова кислота
A. Ціанкобаламін C. Вiтамiн C
B. Еритропоетин D. Вiтамiн B1
C. Гемостимулін E. Вiтамiн K
D. Заліза сульфат Збірник тестових завдань для складання
E. --- ліцензійного іспиту Крок-1 «Загальна
Збірник тестових завдань для складання лікарська підготовка». –2010. - № 41
ліцензійного іспиту Крок-1 «Стоматологія».
– 2016. – № 150.
Біохімія водо – та жиророзчинних вітамінів
(жиророзчинні вітаміни)
1. У 2-річної дитини діагностовано Збірник тестових завдань для складання
кишковий дисбактеріоз, на тлі якого з’явився ліцензійного іспиту Крок-1 «Загальна
геморагічний синдром. Що може бути лікарська підготовка». –2009. - № 153 .
причиною такого стану?
A. Нестача вітаміну К. 5. У хворого спостерiгаються геморагiї, в
B. Гіповітаміноз РР. кровi знижена концентрацiя протромбiну.
C. Гіпокальціемія. Недостатнiсть якого вiтамiну призвела до
D. Дефіцит фібриногену. порушення синтезу цього фактору згортання
E. Активація тканинного тромбопластину. кровi?
Збірник тестових завдань для складання A. K
ліцензійного іспиту Крок-1 «Стоматологія». B. A
– 2007. – № 112. C. D
D. C
2. Оперативне втручання у пацієнта з E. E
механічною жовтяницею та порушенням Збірник тестових завдань для складання
всмоктування в кишці ускладнилося ліцензійного іспиту Крок-1 «Загальна
кровотечею. Недостатність якого вітаміну лікарська підготовка». –2011. - № 134.
призвело до цього?
A. Вітаміну К. 6. У хворого 37-ми років на фонi тривалого
B. Вітаміну В12. застосування антибіотиків спостерiгається
C. Вітаміну С. підвищена кровоточивiсть при невеликих
D. Вітаміну В6. пошкодженнях. У кровi - зниження
E. Фолієвої кислоти. активності факторiв згортання кровi,
Збірник тестових завдань для складання подовження часу згортання кровi.
ліцензійного іспиту Крок-1 «Загальна Недостатнiстю якого вiтамiну обумовленi
лікарська підготовка». –2009. - № 73. зазначенi змiни?
A. Вiтамiн К
3. При захворюваннях печiнки, що B. Вiтамiн А
супроводжуються недостатнiм C. Вiтамiн С
надходженням жовчi в кишку, D. Вiтамiн D
спостерiгається погiршення гемокоагуляцiї. E. Вiтамiн Е
Чим можна пояснити це явище? Збірник тестових завдань для складання
A. Дефiцитом вiтамiну К. ліцензійного іспиту Крок-1 «Загальна
B. Тромбоцитопенiєю. лікарська підготовка». –2016. - № 62.
C. Еритропенiєю.
D. Лейкопенiєю. 7. Плазмовi фактори згортання кровi
E. Дефiцитом залiза. зазнають посттрансляцiйної модифiкацiї за
Збірник тестових завдань для складання участю вiтамiну K. Як кофактор, вiн
ліцензійного іспиту Крок-1 «Стоматологія». потрiбен у ферментнiй системi γ-
– 2008. – № 39 карбоксилювання бiлкових факторiв
коагуляцiї кровi, завдяки збільшенню
4. У юнака 16-ти рокiв пiсля перенесеного спорiдненостi їх молекул з iонами кальцiю.
захворювання знижена функція синтезу Яка амiнокислота карбоксилюється в цих
білків-факторів згортання крові у печiнцi бiлках?
внаслiдок нестачi вiтамiну K. Це може A. Глутамiнова
призвести до порушення: B. Валiн
A. Зсiдання кровi C. Серин
B.Швидкостi осiдання еритроцитiв D. Фенiлаланiн
C. Утворення антикоагулянтiв E. Аргiнiн
D. Утворення еритропоетинiв
E. Осмотичного тиску кровi
Збірник тестових завдань для складання D. E
ліцензійного іспиту Крок-1 «Загальна E. C
лікарська підготовка». –2008. - № 188. Збірник тестових завдань для складання
ліцензійного іспиту Крок-1 «Загальна
8. В клініці обстежується чоловік 49-ти років лікарська підготовка». –2009. - № 73
з суттєвим збільшенням часу згортання
крові, шлунково-кишечними кровотечами, 12. За кілька днів до операції пацієнту
підшкірними крововиливами. Нестачею призначають вітамін K або його синтетичний
якого вітаміну можна пояснити ці симптоми? аналог вікасол. Вітамін K бере участь у
A. K наступній посттрансляційній модифікації II,
B. B1 VII, IX, X факторів згортання крові:
C. PP A. Карбоксилювання
D. H B. Декарбоксилювання
E. E C. Дезамінування
Збірник тестових завдань для складання D. Трансамінування
ліцензійного іспиту Крок-1 «Загальна E. Глікозилювання
лікарська підготовка». –2010. - № 61 Збірник тестових завдань для складання
ліцензійного іспиту Крок-1 «Стоматологія».
9. Внаслідок пост трансляційних змін деяких – 2012. – № 171.
білків, що приймають участь у згортанні
крові, зокрема протромбіну, вони набувають 13. У пацієнта, що тривалий час страждає на
здатність зв’язувати кальцій.Уцьому процесі кишковий дисбактеріоз, посилилися
приймає участь вітамін: кровотечі внаслідок порушення
A. K посттрансляційної модифікації II, VII, IХ і Х
B. C факторів згортання крові в печінці. Дефіцит
C. A якого вітаміну є причиною цього стану?
D. B1 A. К
E. B2 B. B12
Збірник тестових завдань для складання C. B9
ліцензійного іспиту Крок-1 «Загальна D. С
лікарська підготовка». –2008. - № 95 E. Р
Збірник тестових завдань для складання
10. Для попередження післяопераційної ліцензійного іспиту Крок-1 «Загальна
кровотечі 6-ти річній дитині рекомендовано лікарська підготовка». –2015. - № 139
приймати вікасол, який є синтетичним
аналогом вітаміну K. Вкажіть, які 14. У дитини 10-ти мiсяцiв спостерiгається
посттрансляційні зміни факторів згортання надмірна збудливiсть, порушення сну,
крові активуються під впливом вікасолу? знижений тонус м’язiв, запiзнiле
A. Карбоксилювання глутамінової кислоти прорiзування зубiв з недостатньо
B. Фосфорилювання радикалів серину звапнованою емаллю. Дефiцит в органiзмi
C. Частковий протеоліз якого вiтамiну зумовлює такi змiни?
D. Полімеризація A. Холекальциферолу.
E. Глікозилювання B. Ретинолу.
Збірник тестових завдань для складання C. Рибофлавiну.
ліцензійного іспиту Крок-1 «Загальна D. Тiамiну.
лікарська підготовка». –2007. - № 102 E. Нiкотинамiду.
Збірник тестових завдань для складання
11. У хворих з непрохідністю жовчовивідних ліцензійного іспиту Крок-1 «Стоматологія».
шляхів пригнічується згортання крові, – 2009. – № 163
виникають кровотечі, що є наслідком
недостатнього засвоєння такого вітаміну: 15. У пацієнта з нирковою недостатнiстю
A. К розвинулась остеодистрофiя, що
B. А супроводжується iнтенсивною
C. D демiнералiзацiєю кiсток. Порушення
утворення активної форми якого вiтамiну є 19. У жiнки 35-ти рокiв iз хронiчним
причиною даного ускладнення? захворюванням нирок розвинувся
A. Кальциферолу. остеопороз. Дефiцит якої з перерахованих
B. Ретинолу. нижче речовин є основною причиною цього
C. Тiамiну. ускладнення?
D. Нафтохiнону. A. 1,25-дигiдрокси-D3
E. Рибофлавiну. B. 25-гiдрокси-D3
Збірник тестових завдань для складання C. D3
ліцензійного іспиту Крок-1 «Стоматологія». D. D2
– 2012. – № 43. E. Холестерин
Збірник тестових завдань для складання
16. У дитини вiдзначається затримка ліцензійного іспиту Крок-1 «Стоматологія».
прорiзування зубiв, неправильне їх – 2011. – № 41.
розташування, при огляді помітна сухiсть
ротової порожнини, в кутиках рота – 20. У дитини з нирковою недостатнiстю
трiщини з нагноєнням. З нестачею якого виявлена затримка прорізування зубiв.
вiтамiну може бути пов’язаний цей стан? Порушення утворення в нирках якої
A. Вітаміну D. речовини найбiльш iмовiрно є причиною
B. Вітаміну С. цього?
C. Вітаміну E. A. 1,25 (OH)2D3
D. Вітаміну K. B. Глiкоцiамiн
E. Вітаміну А. C. Глутамат
Збірник тестових завдань для складання D. α-кетоглутарат
ліцензійного іспиту Крок-1 «Стоматологія». E. Гiдроксильований лiзин
– 2011. – № 86. Збірник тестових завдань для складання
ліцензійного іспиту Крок-1 «Стоматологія».
17. У дитини першого року життя – 2010. – № 165.
спостерiгається збiльшення розмiрiв голови
та живота, запiзнiле прорізування зубiв, 21. При обстеженнi дитини лiкар виявив
порушення структури емалi. Наслiдком ознаки рахiту. Нестача якої сполуки в
якого гiповiтамiнозу є цi змiни? органiзмi дитини сприяє розвиткуцього
A. Гiповiтамiноз D захворювання?
B. Гiповiтамiноз С A. 1,25 [ОН]-дигiдроксихолекальциферол
C. Гiповiтамiноз А B. Бiотин
D. Гiповiтамiноз В1 C. Токоферол
E. Гiповiтамiноз В2 D. Нафтохiнон
Збірник тестових завдань для складання E. Ретинол
ліцензійного іспиту Крок-1 «Стоматологія». Збірник тестових завдань для складання
– 2006. – № 90. ліцензійного іспиту Крок-1 «Загальна
лікарська підготовка». –2008. - № 115.
18. У дитини 6-ти рокiв спостерiгається
затримка росту, порушення процесiв 22. У дитини першого року життя під час
окостенiння, декальцифiкацiя зубiв. Що профiлактичного огляду виявлено
може бути причиною цього? порушення мiнералiзацiї кiсток. Нестача
A. Дефiцит вiтамiну D якого вiтамiну може бути причиною цього?
B. Зменшення продукцiї глюкагону A. Кальциферол
C. Iнсулiнова недостатнiсть B. Рибофлавiн
D. Гiпертиреоз C. Токоферол
E. Авiтамiноз С D. Фолiєва кислота
Збірник тестових завдань для складання E. Кобаламiн
ліцензійного іспиту Крок-1 «Стоматологія». Збірник тестових завдань для складання
– 2012. – № 195. ліцензійного іспиту Крок-1 «Загальна
лікарська підготовка». –2005. - № 156.
23. У хворого пiсля видалення жовчного 27. При обстеженні дитини 11-ти місяців
мiхура утрудненi процеси всмоктування педіатр виявив викривлення кісток нижніх
Ca2+через стiнку кишечнику. Призначення кінцівок і затримку мінералізації кісток
якого вiтамiну буде стимулювати цей черепа. Нестача якого вітаміну призводить
процес? до даної патології?
A. D3 A. Холекальциферол
B. РР B. Тіамін
C. С C. Пантотенова кислота
D. B12 D. Біофлавоноїди
E. К E. Рибофлавін
Збірник тестових завдань для складання Збірник тестових завдань для складання
ліцензійного іспиту Крок-1 «Загальна ліцензійного іспиту Крок-1 «Загальна
лікарська підготовка». –2006. - № 64. лікарська підготовка». –2009. - № 99

24. У пацiєнта, що страждає на хронiчну 28. Дитині з ознаками рахіту педіатр і


ниркову недостатнiсть, розвинувся дантист призначили жиророзчинний
остеопороз. Порушення синтезу в нирках вітамінний препарат. Цей препарат впливає
якого регулятора мінерального обмiну є на обмін фосфору й кальцію в організмі та
основною причиною остеопорозу? сприяє накопиченню в кістковій тканині та у
A. Утворення 1,25(OH)2D3 дентині кальцію. Якщо його вміст в
B. Гiдроксилювання пролiну організмі недостатній, порушуються процес
C. Гiдроксилювання лiзину осифікації, структура дентину й прикус.
D. Карбоксилювання глутамату Назвіть цей препарат:
E. Гiдроксилювання кортизолу A. Ергокальциферол
Збірник тестових завдань для складання B. Ретинолу ацетат
ліцензійного іспиту Крок-1 «Стоматологія». C. Токоферолу ацетат
– 2009. – № 41. D. Менадіон (Вікасол)
E. Тироїдин
25. Упацієнта спостерігається ерозія емалі. Збірник тестових завдань для складання
Який вітамін необхідно призначити для ліцензійного іспиту Крок-1 «Стоматологія».
лікування? – 2016. – № 55.
A. D3
B. C 29. Гормоноподібна форма вітаміну індукує
C. K на рівні геному синтез Ca-зв’вязуючих білків
D. B1 у ентероцитах і регулює всмоктування іонів
E. PP Ca2+ у кишечнику, від чого залежит розвиток
Збірник тестових завдань для складання тканин зуба. Який це вітамін?
ліцензійного іспиту Крок-1 «Стоматологія». A. D3
– 2013. – № 33. B. A
C. B1
26. У 5-ти річної дитини спостерігається D. E
недостатнє звапніння емалі, карієс зубів. E. K
Гіповітаміноз якого вітаміну зумовлює Збірник тестових завдань для складання
розвиток такого процесу? ліцензійного іспиту Крок-1 «Стоматологія».
A. Кальциферол – 2014. – № 36.
B. Токоферол
C. Біотин 30. 4-річна дитина зі спадковою нирковою
D. Нікотинова кислота недостатністю має ознаки рахіту;
E. Фолієва кислота концентрація вітаміну D у крові в межах
Збірник тестових завдань для складання норми. Яка найбільш імовірна причина
ліцензійного іспиту Крок-1 «Стоматологія». розвитку рахіту?
– 2017. – № 32. A. Пригнічення синтезу кальцитріолу
B. Посилення виділення кальцію
C. Гіперфункція прищитоподібних залоз
D. Гіпофункція пращитоподібних залоз C. Нiкотинова кислота
E. Недостатність кальцію в їжі D. Ергокальциферол
Збірник тестових завдань для складання E. Пантотенова кислота
ліцензійного іспиту Крок-1 «Загальна Збірник тестових завдань для складання
лікарська підготовка». –2015. - № 92. ліцензійного іспиту Крок-1 «Загальна
лікарська підготовка». –2017. - № 186.
31. Вiтамiн A у комплексi зi специфiчними
циторецепторами проникає через ядернi 35. З метою прискорення загоєння
мембрани, iндукує процеси транскрипцiї, що променевої виразки був призначений
стимулює рiст та диференцiювання клiтин. вiтамiнний препарат. Вкажiть його:
Ця бiологiчна функція реалiзується A. Ретинолу ацетат
наступною формою вiтамiну A: B. Ретаболiл
A. Транс-ретиноєва кислота C. Преднiзолон
B. Каротин D. Левамiзол
C. Цис-ретиналь E. Метилурацил
D. Ретинол Збірник тестових завдань для складання
E. Транс-ретиналь ліцензійного іспиту Крок-1 «Загальна
Збірник тестових завдань для складання лікарська підготовка». –2008. - № 186.
ліцензійного іспиту Крок-1 «Загальна
лікарська підготовка». –2017. - № 67. 36. Пiд час огляду ротової порожнини
пацiєнта лiкар-стоматолог визначив сухiсть
32. У хворого спостерiгається порушення слизової оболонки, численнi ерозiї.
зору - гемералопiя ("куряча слiпота"). Який Недостатнiсть якого вiтамiну спричинила цi
вiтамiнний препарат треба вживати хворому, явища?
щоб вiдновити зiр? A. Вiтамiн А
A. Ретинолу ацетат B. Вiтамiн К
B. Вiкасол C. Вiтамiн Р
C. Пiридоксин D. Вiтамiн Н
D. Тiамiну хлорид E. Вiтамiн РР
E. Токоферолу ацетат Збірник тестових завдань для складання
Збірник тестових завдань для складання ліцензійного іспиту Крок-1 «Стоматологія».
ліцензійного іспиту Крок-1 «Загальна – 2006. – № 175.
лікарська підготовка». –2007. - № 159.
37. З метою профiлактики запалення ясен та
33. При обстеженнi хворого окуліст виявив покращення регенерацiї епiтелiальних клiтин
збiльшення часу адаптацiї ока до темряви. пародонту до зубних паст додають один з
Нестача якого вiтамiну може бути наступних вiтамiнiв:
причиноютакого симптому? A. Ретинол
A. A B. Кальциферол
B. B6 C. Тiамiн
C. B2 D. Бiотин
D. C E. Фiлохiнон
E. K Збірник тестових завдань для складання
Збірник тестових завдань для складання ліцензійного іспиту Крок-1 «Стоматологія».
ліцензійного іспиту Крок-1 «Загальна – 2008. – № 161.
лікарська підготовка». –2014. - № 138.
38. У жінки 64-х років спостерігаються
34. Який вiтамiнний препарат доцільно порушення сутінкового зору (гемералопія).
призначити пацiєнту зi скаргами на Препарат якого вітаміну їй слід
зниження гостроти зору в сутiнках, сухість рекомендувати в першу чергу?
шкiри, частi застуднi захворювання, ламкiсть A. A
волосся? B. B2
A. Ретинолу ацетат C. E
B. Тiамiну хлорид D. C
E. B6 який впливає на процеси світлосприйняття
Зборник тестовых заданий для проведения шляхом утворення світлочутливого пігменту
лицензионного экзамена Крок-1 «Медицина». – родопсину. Який це вітамін?
–2010. - № 100 A. Ретинолу ацетат
B. Токоферолу ацетат
39 У хворого спостерігається погіршення C. Піридоксину гідрохлорид
сутінкового зору. Який з вітамінних D. Ціанкобаламін
препаратів слід призначити пацієнту? E. Тіамін
A. Ретинолу ацетат Збірник тестових завдань для складання
B. Ціанокобаламін ліцензійного іспиту Крок-1 «Загальна
C. Піридоксину гідрохлорид лікарська підготовка». –2014. - № 177
D. Кислота аскорбінова
E. Кислота нікотинова 43. При різноманітних захворюваннях рівень
Збірник тестових завдань для складання активних форм кисню різко зростає, що
ліцензійного іспиту Крок-1 «Загальна призводить до руйнування клітинних
лікарська підготовка». –2009. - № 138 мембран. Для запобігання цьому
використовують антиоксиданти.
40. При пародонтозі хворому призначили Найпотужнішим з них є:
жиророзчинний вітамінний препарат, який A. α-Токоферол.
бере активну участь в окислювально- B. Вітамін D.
відновлювальних процесах в організмі. C. Гліцерол.
Антиоксидант є фактором росту, D. Глюкоза.
антиксерофтальмічним, забезпечує E. Жирні кислоти.
нормальний зір. В стоматологічній практиці Збірник тестових завдань для складання
використовується для прискорення ліцензійного іспиту Крок-1 «Стоматологія».
епітелізації при захворюваннях слизових – 2009. – № 42.
оболонок при пародонтозі. Визначте цей
препарат: 44. Вагiтнiй жінці, в анамнезі якої кiлька
A. Ретинолу ацетат мимовiльних абортів, призначено терапiю
B. Ергокальциферол вiтамiнними препаратами. Який вiтамiн
C. Токоферолу ацетат сприяє виношуванню плода?
D. Вікасол A. Альфа-токоферол.
E. Ціанокобаламін B. Пiридоксальфосфат.
Збірник тестових завдань для складання C. Рутин.
ліцензійного іспиту Крок-1 «Стоматологія». D. Тiамiн.
– 2018. – № 55. E. Цiанокобаламiн.
Збірник тестових завдань для складання
41. У хворого відмічені такі зміни: ліцензійного іспиту Крок-1 «Загальна
порушення зорув сутінках, підсихання лікарська підготовка». –2005. - № 17.
кон’юнктивий рогової оболонки. Такі
порушення можуть бути при недостатності 45. В процесi лiкування пародонтозу
вітаміну: застосовують антиоксиданти. Вкажiть, яка з
A. Вітамін A наведених природних сполук
B. Вітамін B використовується в якостi антиоксидантного
C. Вітамін C засобу:
D. Вітамін D A. Токоферол
E. Вітамін B12 B. Тiамiн
Збірник тестових завдань для складання C. Глюконат кальцiю
ліцензійного іспиту Крок-1 D. Пiридоксин
«Стоматологія».– 2011. – № 86. E. Холiн
Збірник тестових завдань для складання
42. Пацієнт скаржиться на розлад ліцензійного іспиту Крок-1 «Стоматологія».
світлосприйняття та часті гострі вірусні – 2009. – № 42.
захворювання. Йому призначили вітамін
46. При лiкуваннi сiалоаденiтiв (запалення 49. Пiд впливом iонiзуючого опромiнення
слинних залоз) використовують препарати або при авiтамiнозi Е в клiтинi
вiтамiнiв. Який iзнаведених вiтамiнiв спостерiгається пiдвищення проникностi
вiдiграє важливу роль в антиоксидантному мембран лiзосом. До яких наслідків може
захистi? призвести така патологiя?
A. Токоферол A. Часткове чи повне руйнування клiтини
B. Тiамiн B. Iнтенсивний синтез бiлкiв
C. Рибофлавiн C. Iнтенсивний синтез енергiї
D. Пантотенова кислота D. Вiдновлення цитоплазматичної мембрани
E. Пiридоксин E. Формування веретена подiлу
Збірник тестових завдань для складання Збірник тестових завдань для складання
ліцензійного іспиту Крок-1 «Стоматологія». ліцензійного іспиту Крок-1 «Стоматологія».
– 2014. – № 119. – 2013. – № 191.

47. Хворiй, що перенесла мастектомiю в 50. У чоловіка, який тривалий час не вживав
зв’язку з раком молочної залози, був з їжею жирів, але отримував достатню
призначений курс променевої терапiї. Який з кількість вуглеводів і білків, виявлено
перерахованих вiтамiнних препаратiв має дерматит, погане загоювання ран,
виражену радiопротекторну дiю, зумовлену погіршення зору. Дефіцит яких компонентів
антиоксидантною активнiстю? харчування став причиною порушення
A. Токоферолу ацетат обміну речовин?
B. Ергокальциферол A. Лінолевої кислоти, вітамінів А,D,Е,К.
C. Тiамiну хлорид B. Вітамінів РР, Н.
D. Рибофлавiн C. Пальмітинової кислоти.
E. Фолiєва кислота D. Мінеральних солей.
Збірник тестових завдань для складання E. Олеїнової кислоти.
ліцензійного іспиту Крок-1 «Загальна Збірник тестових завдань для складання
лікарська підготовка». –2006. - № 31. ліцензійного іспиту Крок-1 «Загальна
лікарська підготовка». –2007. - № 49 .
48. При дефiцитi якого вiтамiну
спостерiгається одночасне порушення 51. Для профілактики атеросклерозу,
репродуктивної функцiї i дистрофiя ішемічної хвороби серця, порушень
скелетної мускулатури? мозкового кровообiгу рекомендується
A. Вiтамiн Е споживання їжі з високим вмiстом
B. Вiтамiн А поліненасичених жирних кислот (F). Однiєю
C. Вiтамiн К з таких жирних кислот є:
D. Вiтамiн Д A. Лiнолева
E. Вiтамiн В1 B. Олеїнова
Збірник тестових завдань для складання C. Лауринова
ліцензійного іспиту Крок-1 «Стоматологія». D. Пальмiтоолеїнова
– 2005. – № 123. E. Стеаринова
Збірник тестових завдань для складання
ліцензійного іспиту Крок-1 «Загальна
лікарська підготовка». –2016. - № 128.
Основні закономірності обміну речовин та енергії
1. Цикл Кребса вiдiграє важливу роль у Збірник тестових завдань для складання
реалiзацiї глюкопластичного ефекту ліцензійного іспиту Крок-1 «Стоматологія».
амiнокислот. Це зумовлено обов’язковим – 2008. – № 127.
перетворенням безазотистого залишку в:
A. Оксалоацетат. 5. При отруєннi цiанiдами настає миттєва
B. Малат. смерть. В чому полягає механізм дiї цiанiдiв
C. Сукцинат. на молекулярному рiвнi?
D. Фумарат. A. Iнгiбують цитохромоксидазу
E. Цитрат. B. Зв’язують субстрати ЦТК
Збірник тестових завдань для складання C. Блокують сукцинатдегiдрогеназу
ліцензійного іспиту Крок-1 «Загальна D. Iнактивують кисень
лікарська підготовка». –2017. - № 53. E. Iнгiбують цитохром b
Збірник тестових завдань для складання
2. У хворого 57 рокiв, який страждає на ліцензійного іспиту Крок-1 «Стоматологія».
цукровий дiабет, розвинувся кето-ацидоз. – 2007. – № 96.
Бiохiмiчною основою цього стану є
зменшення ступеня утилізації ацетил-КоА. 6. До лікарні доставлений хворий з
Нестачею якої сполуки в клiтинах це отруєнням інсектицидом - ротеноном. Яка
обумовлено? ділянка мітохондріального ланцюга
A. Оксалоацетат переносу електронів блокується цією
B. 2-оксоглутарат речовиною?
C. Глутамат A. НАДН-коензим Q-редуктаза.
D. Аспартат B. АТФ-синтетаза.
E. Сукцинат C. Коензим Q-цитохром С-редуктаза.
Збірник тестових завдань для складання D. Сукцинат-коензим Q-редуктаза.
ліцензійного іспиту Крок-1 «Стоматологія». E. Цитохром С-оксидаза.
– 2006. – № 94. Збірник тестових завдань для складання
ліцензійного іспиту Крок-1 «Загальна
3. При окисненні вуглеводів, ліпідів лікарська підготовка». –2010. - № 188.
утворюється велика кількість енергії,
основна частина якої утворюється завдяки 7. Вiдомо, що ротенон викликає iнгiбування
окисненню ацетил-КоА. Скільки молекул дихального ланцюгу. Який комплекс
АТФ утворюється при повному окисненні дихального ланцюга мiтохондрiй інгібується
однієї молекули ацетил-КоА? цією речовиною?
А. 12. A. НАДН-коензимQ-редуктаза
В. 8. B. Цитохромоксидаза
С. 38. C. КоензимQ-цитохромс-редуктаза
D. 24. D. Сукцинат-коензимQ-редуктаза
E. 36 E. АТФ-синтетаза
Збірник тестових завдань для складання Збірник тестових завдань для складання
ліцензійного іспиту Крок-1 «Стоматологія». ліцензійного іспиту Крок-1 «Стоматологія».
– 2014. – № 192. – 2018. – № 2.

4. Синильна кислота та цiанiди належать до 8. До реанiмацiйного вiддiлення в тяжкому


найсильнiших отрут. Залежно вiд дози станi, без свiдомостi надiйшов пацiєнт.
смерть наступає через декiлька секунд чи Дiагностовано передозування барбiтуратiв,
хвилин. Пригнiчення активностi якого якi спричинили феномен тканинної гiпоксiї.
ферменту є причиною смертi? На якому рiвнi вiдбулося блокування
A. Цитохромоксидаза. електронного транспорту?
B. Ацетилхолiнестераза. A. НАДН-коензим-Q-редуктаза
C. Каталаза. B. Цитохромоксидаза
D. Метгемоглобiнредуктаза. C. Цитохром b - цитохром c1
E. АТФ-синтетаза. D. Убiхiнон
E. АТФ-синтаза B. Пiдвищує спряження окиснення
Збірник тестових завдань для складання тафосфорилування
ліцензійного іспиту Крок -1 «Загальна C. Зменшує β-окиснення жирних кислот
лікарська підготовка». –2011. - № 189. D. Зменшує дезамiнування амiнокислот
E. Сприяє накопиченню ацетил-КоА
9. У хворих на тиреотоксикоз Збірник тестових завдань для складання
спостерiгаються гiпертермiя, булiмiя, ліцензійного іспиту Крок -1 «Загальна
зменшення маси тiла, що пов’язане з лікарська підготовка». –2005. - № 146.
порушенням:
A. Спряження окислення i фосфорилювання 13. Клiтину обробили речовиною, яка блокує
B. Розпаду АТФ фосфорилювання нуклеотидiв у
C. Синтезу жирiв мiтохондрiях. Який процес життєдiяльностi
D. Циклу лимонної кислоти клiтини буде порушений в першу чергу?
E. β-окиснення жирних кислот A. Окиснювальне фосфорилювання.
Збірник тестових завдань для складання B. Гліколіз.
ліцензійного іспиту Крок-1 «Загальна C. Iнтеграцiя функцiональних бiлкових
лікарська підготовка». –2011. - № 56. молекул.
D. Аеробне окиснення глюкози
10. При тиреотоксикозі підвищується E. Синтез мiтохондрiальних бiлкiв.
продукція тиреоїдних гормонів Т3 та Т4, Збірник тестових завдань для складання
розвивається схуднення, тахікардія, психічна ліцензійного іспиту Крок-1 «Стоматологія».
збудливість та інше. Який механізм їх дії? – 2011. – № 83.
A. Роз`єднують окиснення та окисне
фосфорилювання. 14. В присутностi 2,4-дiнитрофенолу
B. Активують субстратне фосфорилювання. окиснення субстратiв може тривати, але
C. Блокують субстратне фосфорилювання. синтез молекул АТФ неможливий. Який
D. Блокують дихальний ланцюг. механiзм його дiї?
E. Активують окисне фосфорилювання. A. Роз’єднання окиснення i фосфорилювання
Збірник тестових завдань для складання в мiтохондрiях
ліцензійного іспиту Крок-1 «Загальна B. Активацiя ферменту АТФ-ази
лікарська підготовка». –2013. - № 62. C. Перенiс субстратiв за межi мiтохондрiї
D. Стимуляцiя гiдролiзу утвореного АТФ
11. У жiнки, що знаходиться на лiкуваннi з E. Iнгiбування ферменту цитохромооксидаза
приводу тиреотоксикозу, спостерiгається Збірник тестових завдань для складання
пiдвищення температури тiла. Що лежить в ліцензійного іспиту Крок-1 «Стоматологія».
основi цього явища? – 2014. – № 103.
A. Роз’єднання окисного фосфорилювання
B. Зниження утилiзацiї глюкози тканинами 15. Дослiдженнями останнiх десятиліть
C. Зниження окислення жирiв у печiнцi встановлено, що безпосереднiми
D. Порушення дезамiнування амiнокислот ”виконавцями” апоптозу в клiтинi є особливі
E. Порушення синтезу глiкогену ферменти - каспази. В утвореннi одного з
Збірник тестових завдань для складання них бере участь цитохром С. Вкажiть його
ліцензійного іспиту Крок-1 «Стоматологія». функцiю в нормальнiй клiтинi:
– 2017. – № 92. A. Фермент дихального ланцюга переносу
електронiв
12. Жiнка 38 рокiв скаржиться на пiдвищену B. Фермент ЦТК
пiтливiсть, серцебиття, пiдвищення C. Фермент бета-окислювання жирнихкислот
температури тiла у вечiрнi години. Основний D. Компонент H+ - АТФ-азной системи
обмiн збiльшений на60%. Лiкар встановив E. Компонент пiруватдегiдрогеназної
дiагноз тиреотоксикоз. Якi властивостi системи
тироксину приводять до пiдсилення Збірник тестових завдань для складання
теплопродукцiї? ліцензійного іспиту Крок-1 «Загальна
A. Розщеплює окисне фосфорилування лікарська підготовка». –2017. - № 137.
16. Ціаніди – це отрути, що викликають 18. Ціанід калію – отрута, що надійшла в
миттєву смерть організму. Які ферменти в організм пацієнта й, через кілька хвилин
мітохондріях страждаютьвід дії ціанідів? після цього, викликала смерть. Найбільш
A. Цитохромоксидаза (aa3) імовірною причиною токсичного ефекту
B. Флавінові ферменти було порушення активності
C. Цитохром b5 A. Цитохромоксидази
D. НAД+-залежна дегідрогеназа B. Каталазы
E. Цитохром P-450 C. АТФ-синтетази
Збірник тестових завдань для складання D. НАДФ-Н-дегідрогенази
ліцензійного іспиту Крок-1 «Загальна E. Синтезу гемоглобіна
лікарська підготовка». –2014. - № 12. Збірник тестових завдань для складання
ліцензійного іспиту Крок-1 «Стоматологія».
17. Синильна кислота і ціаніди – 2009. – №78.
єнайсильнішими отрутами. В залежності від
дози смерть настає через кілька секунд або 19. В реанімаційне відділення надійшов
хвилин. Причиною смерті є інгібування пацієнт у тяжкому стані. Відомо, що він
активності наступного ферменту помилково прийняв фторид натрію, який
A. Цитохромоксидаза. блокує цитохромоксидазу. Який тип гіпоксії
B. Ацетилхолінестераза. розвинувся у пацієнта?
C. АТФ-синтетаза. A. Тканинна
D. Каталаза. B. Гемічна
E. Метгемоглобінредуктаза. C. Кардіо-васкулярна
Збірник тестових завдань для складання D. Гіпоксична
ліцензійного іспиту Крок-1 «Стоматологія». E. Респіраторна
– 2008. – № 125. Збірник тестових завдань для складання
ліцензійного іспиту Крок-1 «Загальна
лікарська підготовка». –2012. - № 14.
Обмін вуглеводів та його регуляція
1. У хворої 60-ти рокiв знижена активнiсть 5. Пiсля переходу до змiшаного харчування у
основного травного ферменту слини. В новонародженої дитини виникла диспепсiя з
цьому випадку порушується первинний дiареєю, метеоризмом, відставанням у
гiдролiз: розвитку. Бiохiмiчна основа даної патологiї
A. Вуглеводiв. полягає у недостатностi:
B. Жирiв. A. Сахарази та iзомальтази
C. Бiлкiв. B. Лактази та целобiази
D. Клiтковини. C. Трипсину та хiмотрипсину
E. Молочного цукру. D. Лiпази та креатинкiнази
Збірник тестових завдань для складання E. Целюлази
ліцензійного іспиту Крок-1 «Стоматологія». Збірник тестових завдань для складання
– 2010. – № 135. ліцензійного іспиту Крок-1 «Загальна
лікарська підготовка». –2015. - 3.
2. З вiком знижується активнiсть
навколовушних слинних залоз. Активнiсть 6. У пацієнта спостерігається підвищення
якого ферменту вуглеводного обміну буде вмісту лактату при аналізі слини. Активація
зменшуватися в слинi? якого процесу є причиною цього?
A. Амiлаза A. Гліколіз.
B. Лiзоцим B. Аеробного окиснення глюкози.
C. Фосфатаза C. Глюкозо-лактатного циклу.
D. Гексокiназа D. Розпаду глікогену.
E. Мальтаза E. Травлення вуглеводів.
Збірник тестових завдань для складання Збірник тестових завдань для складання
ліцензійного іспиту Крок-1 «Стоматологія». ліцензійного іспиту Крок-1 «Стоматологія».
– 2006. – № 168. – 2009. – № 38.

3. У новонародженої дитини після годування 7. Який тип катаболізму глюкози має місце у
молоком спостерігалися диспепсія, блювота. ракових клітинах хворого, клінічне
При годуванні розчином глюкози ці явища обстеження котрого дозволило встановити
зникали. Недостатня активність якого попередній діагноз, рак шлунку, в
ферменту призводить до таких розладів? шлунковому соку виявлено молочну
A. Лактази. кислоту.
B. Амілази. A. Гліколіз.
C. Ізомальтази. B. Аеробне окиснення глюкози.
D. Мальтази. C. Глюкозо-аланіловий цикл.
E. Сахарази. D. Глюконеогенез.
Збірник тестових завдань для складання E. Пентозофосфатний шлях.
ліцензійного іспиту Крок-1 «Загальна Збірник тестових завдань для складання
лікарська підготовка». –2009. - № 165. ліцензійного іспиту Крок-1 «Стоматологія».
– 2008. – № 131.
4. При визначеннi енерговитрат органiзму
людини встановлено, що дихальний 8. Який процес забезпечує еритроцити
коефiцiєнт дорiвнює 1,0. Цеозначає, що у необхідною кількістю енергії у вигляді АТФ
клiтинах дослiджуваного переважно для їх життєдіяльності?
окислюються: A. Гліколіз.
A. Вуглеводи B. Аеробне окислення глюкози.
B. Бiлки C. β-Окислення жирних кислот.
C. Жири D. Пентозний цикл.
D. Бiлки i вуглеводи E. Цикл трикарбонових кислот.
E. Вуглеводи та жири Збірник тестових завдань для складання
Збірник тестових завдань для складання ліцензійного іспиту Крок-1 «Загальна
ліцензійного іспиту Крок-1 «Загальна лікарська підготовка». –2013. - № 77.
лікарська підготовка». –2009. - № 103.
9. Еритроцити людини не мiстять Збірник тестових завдань для складання
мiтохондрiй. Який основний шлях утворення ліцензійного іспиту Крок-1 «Загальна
АТФ в цих клiтинах? лікарська підготовка». –2005. - № 122.
A. Глiколiз.
B. Аеробне окиснення глюкози. 13. Після відновлення кровообігу в
C. Аденiлаткiназна реакцiя. ушкодженій тканині припиняється
D. Креатинкiназна реакцiя. накопичення лактату та зменшується
E. Окиснювальне фосфорилювання. швидкість споживання глюкози. Активацією
Збірник тестових завдань для складання якого процесу зумовлені ці метаболічні
ліцензійного іспиту Крок-1 «Стоматологія». порушення?
– 2014. – № 38. A. Аеробного окиснення глюкози.
B. Гліколізу.
10. При вживаннi печива, цукерок у змiшанiй C. Біосинтезу глікогену.
слинi тимчасово зростає рiвень лактату. D. Глюконеогенезу.
Активацiя якого бiохiмiчного процесу E. Ліполізу.
призводить до цього? Збірник тестових завдань для складання
A. Анаеробний глiколiз ліцензійного іспиту Крок-1 «Стоматологія».
B. Тканинне дихання – 2009. – № 112.
C. Аеробний глiколiз
D. Глюконеогенез 14. У жiнки 32-х рокiв запалення ясен
E. Мiкросомальне окислення (гiнгiвiт) супроводжується їх гiпоксiєю.
Збірник тестових завдань для складання Утворення якого метаболiту вуглеводного
ліцензійного іспиту Крок-1 «Стоматологія». обмiну значно збiльшується при цьому в
– 2005. – № 119. тканинах пародонта?
A. Лактату.
11. Після тривалого фізичного навантаження B. Глiкогену.
під час заняття з фізичної культури у C. Глюкозо-6-фосфату.
студента розвинулася м’язова крепатура. D. НАДФ-Н.
Причиною виникнення даного стану стало E. Рибозо-5-фосфату.
накопичення в скелетних м’язах молочної Збірник тестових завдань для складання
кислоти, яка утворилася після активування в ліцензійного іспиту Крок-1 «Стоматологія».
організмі: – 2016. – № 24.
A. Гліколізу.
B. Глікогенезу. 15. Пiд час бiгу на короткi дистанцiї у
C. Ліполізу. нетренованої людини виникає м’язова
D. Пентозофосфатного шляху. гiпоксiя. До накопичення якого метаболiту
E. Глюконеогенезу. вуглеводного обміну в м’язах це
Збірник тестових завдань для складання призводить?
ліцензійного іспиту Крок-1 «Стоматологія». A. Лактату.
– 2011. – № 116. B. Кетонових тiл.
C. Глюкозо-6-фосфату.
12. Пiд час бiгу на довгi дистанцiї скелетна D. Оксалоацетату.
мускулатура тренованої людини E. Цитрату.
використовує глюкозу з метою отримання Збірник тестових завдань для складання
енергiї АТФ для м’язового скорочення. ліцензійного іспиту Крок-1 «Загальна
Вкажiть основний процес утилiзацiї глюкози лікарська підготовка». –2009. - № 107.
в цих умовах:
A. Аеробний глiколiз 16. Анаеробне розщеплення глюкозидо
B. Анаеробний глiколiз молочної кислоти регулюється вiдповiдними
C. Глiкогенолiз ферментами. Який ферментє головним
D. Глюконеогенез регулятором цього процесу?
E. Глiкогенез A. Фосфофруктокiназа
B. Глюкоза-6-фосфат-iзомераза
C. Альдолаза
D. Енолаза участю кількох функціонально зв’язаних
E. Лактатдегiдрогеназа коферментiв. Вкажiть цей комплекс:
Збірник тестових завдань для складання A. ТДФ, ФАД, КоА-SН, НАД, лiпоєва
ліцензійного іспиту Крок-1 «Стоматологія». кислота
– 2013. – № 41. B. ФАД, ТГФК, ПАЛФ, ТДФ, холiн
C. НАД, ПАЛФ, ТДФ, метилкобаламiн,
17. Велика кількість метаболітів окислення бiотин
глюкози розчинена в цитоплазмі міоцитів. D. КоА-SН, ФАД, ПАЛФ, ТГФК, карнiтин
Назвіть один з них, який безпосередньо E. Лiпоєва кислота, ТГФК, ПАЛФ,
перетворюється в лактат: метилкобаламiн
A. Піруват. Збірник тестових завдань для складання
B. Глюкозо-6-фосфат. ліцензійного іспиту Крок-1 «Стоматологія».
C. Гліцерофосфат. – 2018. – № 69.
D. Оксалоацетат.
E. Фруктозо-6-фосфат. 21. Встановлено, що до складу пестициду
Збірник тестових завдань для складання входить арсенат натрiю, який блокує лiпоєву
ліцензійного іспиту Крок-1 «Стоматологія». кислоту. Вкажiть, активнiсть яких ферментiв
– 2007. – № 110. порушується:
A. ПВК-дегiдрогеназного комплексу.
18. Фармацевтичний препарат B. Глутатiонредуктази.
кокарбоксилаза (тіамінпірофосфат) C. Глутатiонпероксидази.
використовується для забезпечення клітин D. Монооксигенази.
енергією при лікуванні багатьох E. Метгемоглобiнредуктази.
захворювань. Вкажіть, який метаболічний Збірник тестових завдань для складання
процес при цьому активується? ліцензійного іспиту Крок-1 «Стоматологія».
A. Окисне декарбоксилювання пірувату. – 2014. – № 77.
B. Дезамінування глутамату.
C. Декарбоксилювання амінокислот. 22. В еритроцитах пацiєнта, хворого на
D. Декарбоксилювання біогенних амінів. гемолітичну анемiю, була значно знижена
E. Детоксикація шкідливих речовин у активність пiруваткiнази. Який метаболічний
печінці. процес порушений за цих умов?
Збірник тестових завдань для складання A. Глiколiз
ліцензійного іспиту Крок-1 «Стоматологія». B. Глiкогенолiз
– 2010. – № 118. C. Глюконеогенез
D. Пентозофосфатний шлях окислення
19. До лікарні надійшла робітниця хімічного глюкози
підприємства з ознаками отруєння. У волоссі E. Синтез глiкогену
цієї жінки виявлено підвищену Збірник тестових завдань для складання
концентрацію арсенату, який блокує ліпоєву ліцензійного іспиту Крок-1 «Стоматологія».
кислоту. Порушення якого процесу є – 2018. – № 138.
найбільш ймовірною причиною отруєння?
A. Окиснювального декарбоксилювання 23. За умов гіпоксії спостерігається
ПВК. зниження співвідношення аденілових
B. Мікросомального окиснення. нуклеотидів АТФ/АДФ, що призводить до
C. Відновлення метгемоглобіну. посилення гліколізу в тканинах пародонту.
D. Відновлення органічних перекисів. Яка реакція при цьому активується?
E. Знешкодження супероксидних іонів. A. Фосфофруктокіназна.
Збірник тестових завдань для складання B. Альдолазна.
ліцензійного іспиту Крок-1 «Стоматологія». С. Енолазна.
– 2008. – № 131. D. Тріозофосфатізомеразна.
E. Лактатдегідрогеназна.
20. Окисне декарбоксилювання Збірник тестових завдань для складання
пiровиноградної кислоти каталiзується ліцензійного іспиту Крок-1 «Стоматологія».
складним поліферментним комплексом з – 2006. – № 97.
24. У деяких анаеробних бактерiй пiруват, Збірник тестових завдань для складання
що утворюється внаслiдок глiколiзу, ліцензійного іспиту Крок-1 «Загальна
перетворюється на етиловий спирт (спиртове лікарська підготовка». –2015. - № 44.
бродiння). У чому бiологiчний сенс цього
процесу? 28. Внаслідок довготривалого вживання
A. Поповнення фонду НАД+ сульфаніламідних препаратів у молодої
B. Утворення лактату жінки виявилися ознаки гемолітичної анемії,
C. Утворення АДФ що зумовлюється спадковим порушенням
D. Забезпечення клiтини НАДФН синтезу фермента пентозофосфатного шляху
E. Утворення АТФ глюкозо-6-фосфатдегідрогенази, який
Збірник тестових завдань для складання забезпечує утворення в організмі:
ліцензійного іспиту Крок-1 «Загальна A. НАДФН.
лікарська підготовка». –2012. - № 140. B. АТФ.
C. НАД.
25. У медичнiй практицi для профiлактики D. ФАД.
алкоголiзму широко використовують E. ФМН.
препарат, який є iнгiбiтором Збірник тестових завдань для складання
альдегiддегiдрогенази. Пiдвищення в кровi ліцензійного іспиту Крок-1 «Стоматологія».
якого метаболiту викликає вiдразу до – 2010. – № 81.
алкоголю?
A. Ацетальдегiд 29. У 22-рiчної жiнки внаслiдок
B. Етанол довготривалого вживання сульфанiламiдних
C. Малоновий альдегiд препаратiв з’явилися ознаки гемолiтичної
D. Пропiоновий альдегiд анемiї, що зумовлюється спадковим
E. Метанол порушенням синтезу ферменту
Збірник тестових завдань для складання пентозофосфатного циклу глюкозо-6-
ліцензійного іспиту Крок-1 «Загальна фосфатдегiдрогенази, який забезпечує
лікарська підготовка». –2006. - № 161. утворення в органiзмi:
А. НАДФ-Н2
26. Хворий пiд час курсу лiкування В. НАД
препаратом, який блокує С. ФАД
алкогольдегідрогеназу, вжив невелику D. ФМН
кiлькiсть алкоголю, внаслiдок чого Е. АТФ
розвинулось тяжке отруєння. Пояснiть Збірник тестових завдань для складання
причину отруєння: ліцензійного іспиту Крок-1 «Стоматологія».
A. Накопичення ацетальдегiду – 2009. – № 122.
B. Алергiчна реакцiя
C. Невралгiчнi розлади 30. Сульфаніламіди використовують у
D. Серцево-судинна недостатнiсть клінічній практиці як протимікробні засоби.
E. Порушення функцiї нирок Генетичний дефект якого ферменту
Збірник тестових завдань для складання пентозофосфатного обміну в еритроцитах
ліцензійного іспиту Крок-1 «Загальна під час лікування такими препаратами може
лікарська підготовка». –2006. - № 87. призвести до розвитку гемолітичної анемії?
А. Глюкозо-6-фосфатдегідрогеназа
27. Бiосинтез пуринового кiльця В. Гексокіназа
вiдбувається на рибозо-5-фосфатi шляхом С. Транскетолаза
поступового нарощення атомiв азоту i D. Трансальдолаза
вуглецю та замикання кiлець. Джерелом Е. Піруваткіназа
рибозо-5-фосфату є наступний процес: Збірник тестових завдань для складання
A. Пентозофосфатний цикл ліцензійного іспиту Крок-1 «Стоматологія».
B. Глiколiз – 2016. – № 116.
C. Глiконеогенез
D. Глюконеогенез
E. Глiкогенолiз
31. У хворої 38-ми рокiв пiсля прийому B. Цукровий дiабет
аспiрину i сульфанiламiдiв спостерiгається C. Лактоземiя
посилений гемолiз еритроцитів, який D. Стероїдний дiабет
викликаний недостатністю глюкозо-6- E. Фруктоземiя
фосфатдегiдрогенази. Порушенням Збірник тестових завдань для складання
утворення якого коферменту зумовлена ця ліцензійного іспиту Крок-1 «Загальна
патологiя? лікарська підготовка». –2007. - № 147.
A. НАДФ-Н
B. ФАД-Н2 35. У хворої дитини виявлена затримка
C. Пiридоксальфосфат розумового розвитку, збiльшення печiнки,
D. ФМН-Н2 погiршення зору. Лiкар пов’язує цi симптоми
E. Убiхiнон з дефiцитом в органiзмi галактозо-1-
Збірник тестових завдань для складання фосфатуридилтрансферази. Який
ліцензійного іспиту Крок-1 «Стоматологія». патологiчний процес має мiсце у дитини?
– 2010. – № 81. A. Галактоземiя
B. Фруктоземiя
32. У хворого має мiсце хронiчний запальний C. Гiперглiкемiя
процес мигдаликiв. За рахунок якого D. Гiпоглiкемiя
бiохiмiчного процесу у вогнищi запалення E. Гiперлактатацидемiя
пiдтримується концентрація НАДФН, Збірник тестових завдань для складання
необхiдного для реалiзацiї механiзму ліцензійного іспиту Крок-1 «Загальна
фагоцитозу? лікарська підготовка». –2010. - № 173.
A. Пентозо-фосфатний шлях
B. Цикл Корi 36. У хлопчика 2-х рокiв спостерiгається
C. Цикл Кребса збiльшення в розмiрах печiнки та селезiнки,
D. Орнiтиновий цикл катаракта. В кровi пiдвищена концентрацiя
E. Глiколiз цукру, але тест толерантностідо глюкози в
Збірник тестових завдань для складання нормi. Спадкове порушення обмiну якої
ліцензійного іспиту Крок-1 «Загальна речовини є причиною цьогостану?
лікарська підготовка». –2017. - № 181. A. Галактоза
B. Фруктоза
33. У дитини 7-ми років виражені ознаки C. Глюкоза
гемолітичної анемiї. При бiохiмiчному D. Мальтоза
аналiзi еритроцитів встановлено знижену E. Сахароза
концентрацію НАДФН i вiдновленого Збірник тестових завдань для складання
глутатiону. Дефiцит якого ферменту ліцензійного іспиту Крок-1 «Загальна
зумовлює у даному випадку біохімічні змiни лікарська підготовка». –2015. - № 20.
i клінічні прояви?
A. Глюкозо-6-фосфатдегiдрогеназа 37. Вкажіть, спадкова недостатність якого
B. Гексокiназа ферменту є причиною блювоти та діареї
C. Фруктокiназа після прийому фруктових соків у 9-місячної
D. Пiруваткiназа дитини, у якої навантаження фруктозою
E. Лактатдегiдрогеназа привело до гіпоглікемії?
Збірник тестових завдань для складання A. Фруктозо -1- фосфатальдолази.
ліцензійного іспиту Крок-1 «Загальна B. Фосфофруктокінази
лікарська підготовка». –2018. - № 180. C. Гексокінази.
D. Фруктозо-1.6-дифосфатази.
34. У кровi дитини виявлено високий вмiст E. Фруктокінази.
галактози, концентрацiя глюкози понижена. Збірник тестових завдань для складання
Спостерiгаються катаракта, розумова ліцензійного іспиту Крок-1 «Стоматологія».
вiдсталiсть, розвивається жирове – 2012. – № 51.
переродження печiнки. Яке захворювання
має мiсце?
A. Галактоземiя
38. У 8-мiсячної дитини спостерiгаються Збірник тестових завдань для складання
блювання та дiарея пiсля прийому ліцензійного іспиту Крок-1 «Стоматологія».
фруктових сокiв. Навантаження фруктозою – 2007. – № 105.
призвело до гiпоглiкемiї. Спадкова
недостатнiсть якого ферменту є причиною 42. Хворому на інсулінзалежний цукровий
цих порушень? діабет був введений інсулін. Через деякий
A. Фруктозо-1-фосфатальдолаза час у хворого з'явились слабкість,
B. Фруктокiназа дратівливість, посилення потовиділення.
C. Гексокiназа Який основний механізм розвитку
D. Фосфофруктокiназа гіпоглікемічної коми, що виникла?
E. Фруктозо-1,6-дифосфатаза A. Вуглеводне голодування головного
Збірник тестових завдань для складання мозку.
ліцензійного іспиту Крок-1 «Стоматологія». B. Зменшення глюконеогенезу.
– 2011. – № 38. C. Посилення глікогенолізу.
D. Посилення кетогенезу.
39. Надмiрна концентрацiя глюкози в E. Посилення ліпогенезу.
ротовiй рiдинi при цукровому дiабетi Збірник тестових завдань для складання
призводить до розвитку: ліцензійного іспиту Крок-1 «Загальна
A. Множинного карiєсу лікарська підготовка». –2005. - № 111.
B. Гiперплазiї емалi
C. Гiпоплазiї емалi 43. При обстеженні жінки,що хвора на
D. Флюорозу цукровий діабет 1-го типу, виявлено
E. Посиленої кальцифiкацiї емалi порушення білкового обміну, що при
Збірник тестових завдань для складання біохімічному дослідженні проявляється
ліцензійного іспиту Крок-1 «Стоматологія». аміноацидемією, уповільненням загоєння
– 2017. – № 31. ран і зменшенням біосинтезу антитіл. Який з
перерахованих механізмів викликає розвиток
40. При біохімічному дослідженні крові ацидемії?
хворої виявлено гіперглікемію, А. Підвищення протеолізу.
гіперкетонемію, в сечі глюкоза, кетонові В. Гіперпротеїнемія.
тіла. На електрокардіограмі дифузні зміни в С. Зменшення концентрації амінокислот у
міокарді. Вона скаржиться на сухість в роті, крові.
спрагу, часте сечовипускання, загальну D. Підвищення онкотичного тиску в плазмі
слабкість. У хворої ймовірно: крові.
A. Цукровий діабет. Е. Збільшення вмісту ліпопротеїдів високої
B. Аліментарна гіперглікемія. щільності.
C. Гострий панкреатит. Збірник тестових завдань для складання
D. Ішемічна хвороба серця. ліцензійного іспиту Крок-1 «Загальна
E. Нецукровий діабет. лікарська підготовка». –2015. - № 125.
Збірник тестових завдань для складання
ліцензійного іспиту Крок-1 «Загальна 44. Вiдомо,що при цукровому дiабетi у
лікарська підготовка». –2012. - № 2. хворих частiше зустрiчаються запальнi
процеси, знижена регенерацiя,
41. Які патологічні компоненти виявлені при уповiльнюється загоєння ран. Причиною
лабораторному дослідженні сечі пацієнтки, цього є:
що лікується в ендокринологічному A. Зниження протеосинтезу
відділенні з діагнозом цукровий діабет, B. Пiдвищення лiполiзу
скаржиться на спрагу та підвищений апетит? C. Прискорення глюконеогенезу
A. Глюкоза, кетонові тіла. D. Зниження лiполiзу
B. Білірубін, уробілін. E. Посилення катаболiзму
C. Білок, амінокислоти. Збірник тестових завдань для складання
D Білок, креатин. ліцензійного іспиту Крок-1 «Загальна
E. Кров. лікарська підготовка». –2012. - № 131.
45. У хворого на цукровий діабет 49. У хворого на цукровий дiабет після
розвинулася діабетична кома внаслідок введення iнсулiну настала непритомнiсть,
порушення кислотно-лужної рівноваги. спостерiгаються судоми. Який результат
Назвіть вид цього порушення. бiохiмiчного аналiзу кровi на вмiст цукру?
A. Метаболічний ацидоз. A. 1,5 ммоль/л
B. Респіраторний ацидоз. B. 3,3 ммоль/л
C. Негазовий алкалоз. C. 8 ммоль/л
D. Метаболічний алкалоз. D. 10 ммоль/л
E. Газовий алкалоз. E. 5,5 ммоль/л
Збірник тестових завдань для складання Збірник тестових завдань для складання
ліцензійного іспиту Крок-1 «Загальна ліцензійного іспиту Крок-1 «Загальна
лікарська підготовка». –2008. - № 2. лікарська підготовка». –2006. - № 125.

46. Рівень якого білка плазми крові дозволяє 50. Хвора 46-ти рокiв скаржиться на сухiсть
ретроспективно (за попередні 4-8 тижні до в ротi, спрагу, почащений сечопуск, загальну
обстеження) оцінити рівень глікемії, якщо слабкiсть. У кровi: гiперглiкемiя,
пацієнт хворіє на цукровий діабет, що гiперкетонемiя. У сечi: глюкоза, кетоновi
супроводжується гіперглікемією натще тiла. На ЕКГ: дифузні змiни в мiокардi. Який
понад 7,2 ммоль/л? найбiльш iмовiрний дiагноз?
A. Глікозильованого гемоглобіну. A. Цукровий дiабет
B. Альбуміну. B. Алiментарна гiперглiкемiя
C. С-Реактивного білка. C. Гострий панкреатит
D. Церулоплазміну. D. Нецукровий дiабет
E. Фібриногену. E. Iшемiчна хвороба серця
Збірник тестових завдань для складання Збірник тестових завдань для складання
ліцензійного іспиту Крок-1 «Загальна ліцензійного іспиту Крок-1 «Загальна
лікарська підготовка». –2012. - № 11. лікарська підготовка». –2012. - № 2.

47. У жiнки 62-х рокiв розвинулася катаракта 51. Хворий пiсля перенесеного епiдемiчного
(помутнiння кришталика) на фонi цукрового паротиту схуднув, постiйно вiдчуває спрагу,
дiабету. Який тип модифiкацiї бiлкiв має п’є багато води, відмічає часте
мiсце при дiабетичнiй катарактi? сечовидiлення, пiдвищений апетит, шкiрний
A. Глiкозилювання свербiж, слабкiсть, фурункульоз. У кровi:
B. Фосфорилювання глюкоза - 16 ммоль/л, кетонових тiл - 100
C. АДФ-рибозилювання мкмоль/л; глюкозурiя. Яке захворювання
D. Метилювання розвинулось у пацiєнта?
E. Обмежений протеолiз A. Iнсулiнозалежний цукровий дiабет
Збірник тестових завдань для складання B. Iнсулiнонезалежний цукровий дiабет
ліцензійного іспиту Крок-1 «Загальна C. Стероїдний дiабет
лікарська підготовка». –2009. - № 20. D. Нецукровий дiабет
E. Цукровий дiабет недостатнього хар-
48. Пiдлiток 12 рокiв протягом 3 мiсяцiв Чування
втратив 7 кг маси тiла. Вмiст глюкози у кровi Збірник тестових завдань для складання
становить 20 ммоль/л. Несподiвано ліцензійного іспиту Крок-1 «Загальна
розвинулася кома. Який вид цукрового лікарська підготовка». –2012. - № 92.
дiабету найбiльш вiрогiдний у хлопчика?
A. Iнсулiнозалежний (I тип) 52. Хвора 38-ми рокiв надiйшла до
B. Iнсулiнонезалежний (II тип) реанiмацiйного вiддiлення в несвiдомому
C. Гiпофiзарний станi. Рефлекси вiдсутнi. Цукор кровi - 2,1
D. Стероїдний ммоль/л. В анамнезi - цукровий дiабет з 18-
E. Гiпертiреоїдний ти рокiв. Яка кома має мiсце у хворої?
Збірник тестових завдань для складання A. Гiпоглiкемiчна
ліцензійного іспиту Крок-1 «Загальна B. Кетоацидотична
лікарська підготовка». –2005. - № 111. C. Лактацидемiчна
D. Гiперосмолярна C. Глікогеноліз.
E. Гiперглiкемiчна D. Реабсорбція глюкози.
Збірник тестових завдань для складання E. Транспорт глюкози в клітину.
ліцензійного іспиту Крок-1 «Загальна Збірник тестових завдань для складання
лікарська підготовка». –2012. - № 96. ліцензійного іспиту Крок-1 «Стоматологія».
– 2014. – № 115.
53. В кровi пацiєнта вмiст глюкози
натщесерце 5,6 ммоль/л, через 1 годину пiсля 57. У хворого 57 рокiв, який страждає на
цукрового навантаження - 13,8 ммоль/л, а цукровий дiабет, розвинувся кетоацидоз.
через 3 години - 9,2 ммоль/л. Для якої Бiохiмiчною основою цього стану є
патологiї характернi такi показники? зменшення ступеня утилiзацiї ацетил-КоА в
A. Прихована форма цукрового дiабету циклі трикарбонових кислот. Нестачею якої
B. Здорова людина сполуки в клiтинах це обумовлено?
C. Тиреотоксикоз A. Оксалоацетату.
D. Хвороба Iценко-Кушiнга B. Аспартату.
E. Акромегалiя C. Глутамату.
Збірник тестових завдань для складання D. 2-Оксоглутарату.
ліцензійного іспиту Крок-1 «Загальна E. Сукцинату.
лікарська підготовка». –2006. - № 189. Збірник тестових завдань для складання
ліцензійного іспиту Крок-1 «Стоматологія».
54. У хворого 15-ти рокiв концентрацiя – 2006. – № 94.
глюкози натще 4,8 ммоль/л, через годину
пiсля цукрового навантаження - 9,0 ммоль/л, 58. Хворий з дiагнозом цукровий дiабет,
через 2 години - 7,0 ммоль/л, через 3 години вранцi натще отримав призначену дозу
- 4,8 ммоль/л. Цi показники характернi для iнсулiну пролонгованої дiї. Пропустив
такого захворювання: черговий прийом їжi i невдовзi вiдчув
A. Прихований цукровий дiабет слабкiсть, бiль голови, запаморочення,
B. Цукровий дiабет I типу пiтливiсть, тремтiння тiла, судоми, вiдчуття
C. Цукровий дiабет II типу голоду, явища гiпоглiкемiї. Застосування
D. Хвороба Iценко-Кушiнга глюкози стан не полегшило. Який препарат
E. – необхiдно ввести для купiрування даного
Збірник тестових завдань для складання стану?
ліцензійного іспиту Крок-1 «Загальна A. Адреналiн
лікарська підготовка». –2014. - № 173. B. Трiамцiнолон
C. Норадреналiн
55. Який лікарський засіб необхідно ввести D. Пренiзолон
хворому на цукровий діабет для виведення E. Гiдрокортизон
його з коми при подвійному передозуванні Збірник тестових завдань для складання
інсуліну? ліцензійного іспиту Крок-1 «Загальна
A. Глюкозу. лікарська підготовка». –2017. - № 43.
B. Інсулін.
C. Лідазу. 59. Хворий протягом 5-ти рокiв страждає на
D. Соматотропін. цукровий дiабет. В результатi порушення
E. Норадреналін. дiєти у нього розвинувся коматозний стан.
Збірник тестових завдань для складання Лiкар швидкої допомоги ввiв глюкозу. Стан
ліцензійного іспиту Крок-1 «Загальна хворого покращився. Яка кома найбiльш
лікарська підготовка». –2011. - № 98. вiрогiдно була в хворого?
A. Гiпоглiкемiчна
56. Для діабету ІІ типу характерними B. Ацидотична
ознаками є гіперглікемія, гіпохлоремія. Який C. Гiперглiкемiчна
з перерахованих нижче процесів активується D. Печiнкова
в першу чергу? E. Гiпотиреоїдна
A. Глюконеогенез.
B. Гліколіз.
Збірник тестових завдань для складання ймовірні порушення в печінці. Про зміну
ліцензійного іспиту Крок-1 «Стоматологія». якої функції печінки йдеться?
– 2009. – № 146. A. Глікогендепонуючої.
B. Гліколітичну.
60. У хворого з синдромом Iценко-Кушинга C. Екскреторну.
спостерiгаються стiйка гiперглiкемiя та D. Кетогенну.
глюкозурiя. Синтез та секрецiя якого E. Холестериноутворюючу.
гормону збiльшенi у цього хворого? Збірник тестових завдань для складання
A. Кортизол ліцензійного іспиту Крок-1 «Стоматологія».
B. Адреналiн – 2006. – № 92.
C. Глюкагон
D. Тироксин 64. У дитини 2-х рокiв дiагностовано
E. Альдостерон хворобу Гiрке, яка проявляється важкою
Збірник тестових завдань для складання гiпоглiкемiєю. Причиною такого стану є
ліцензійного іспиту Крок-1 «Загальна вiдсутнiсть ферменту глюкозо-6-фосфатази.
лікарська підготовка». –2017. - № 180. З порушенням якого процесу пов’язана ця
патологiя?
61. Пiдшлункова залоза - орган змiшаної A. Мобiлiзацiя глiкогену.
секрецiї. Ендокринно продукує бета- B. Глюконеогенез.
клiтинами гормон iнсулiн, який впливає на C. Глiколiз.
обмiн вуглеводiв. Як вiн впливає на D. Кетогенез.
активнiсть глiкогенфосфорилази (ГФ) i E. Синтез глiкогену.
глiкогенсинтетази (ГС)? Збірник тестових завдань для складання
A. Пригнiчує ГФ, активує ГС ліцензійного іспиту Крок-1 «Стоматологія».
B. Активує ГФ i ГС – 2012. – № 128.
C. Пригнiчує ГФ i ГС
D. Активує ГФ, пригнiчує ГС 65. Глiкоген, що надiйшов з їжею,
E. Не впливає на активнiсть ГФ i ГС гiдролiзувався у шлунково-кишковомутрактi.
Збірник тестових завдань для складання Який кiнцевий продукт утворився в
ліцензійного іспиту Крок-1 «Загальна результатi цього процесу?
лікарська підготовка». –2015. - № 177. A. Глюкоза
B. Лактат
62. У дитяче відділення доставлена трирічна C. Лактоза
дівчинка. Дитина квола, апатична, печінка D. Галактоза
збільшена. Біопсія вказує на значний E. Фруктоза
надлишок глікогену. Концентрація глюкози Збірник тестових завдань для складання
в крові нижча від норми. Найбільш ліцензійного іспиту Крок-1 «Загальна
ймовірною причиною гіпоглікемії є: лікарська підготовка». –2014. - № 39.
A. Знижена активність
глікогенфосфорилази. 66. У рацiонi людини велика кiлькiсть
B. Знижена активність глікогенсинтази. вуглеводiв. Кiлькiсть яких структур
C. Знижена активність глюкозо-6- збiльшиться у цитоплазмi гепатоцитiв?
фосфатдегідрогенази. A. Гранули глiкогену
D. Знижена активність глюкозо-1- B. Краплини жиру
фосфатуридинтрансферази. C. Лiзосоми
E. Підвищена активність глюкокінази. D. Вiльнi рибосоми
Збірник тестових завдань для складання E. Включення лiпофусцину
ліцензійного іспиту Крок-1 «Загальна Збірник тестових завдань для складання
лікарська підготовка». –2007. - № 43. ліцензійного іспиту Крок-1 «Загальна
лікарська підготовка». –2005. - № 57.
63. Після введення адреналіну у пацієнтки з
постійною гіпоглікемією аналіз крові 67. Хвороба Андерсена належить до групи
суттєво не змінився. За таких обставин спадкових хвороб, що розвиваються
внаслідок уродженої недостатності синтезу
певних ферментів глікогенолізу. 71. Гіперглікемія розвивається при
Недостатність якого ферменту є хронічному передозуванні глюкокортикоїдів
молекулярною основою цього глікогенозу? у хворого. Вкажіть процес вуглеводного
А. Аміло (1-4→1-6) трансглікозидази. обміну, за рахунок якого зростає
В. Глікогенсинтетази. концентрація глюкози:
С. Глюкозо-6-фосфатази. A. Глюконеогенез.
D. Лізосомальної глікозидази. B. Аеробне окиснення глюкози.
E. Фосфофруктокінази. C. Глікогеноліз.
Збірник тестових завдань для складання D. Глікогенез.
ліцензійного іспиту Крок-1 «Загальна E. Пентозофосфатний цикл.
лікарська підготовка». –2014. - № 161. Збірник тестових завдань для складання
ліцензійного іспиту Крок-1 «Стоматологія».
68. Характерним ознакою глікогенезу є – 2006. – № 108.
м'язові болі при фізичній роботі, які
супроводжуються гіпоглікемію. Дефіцитом 72. Пiд час голодування нормальний рiвень
якого вродженого ферменту викликана така глюкози пiдтримується за рахунок активацiї
патологія: глюконеогенезу. Назвiть речовину, яка може
A. Глікогенфосфорилаза використовуватись як субстрат для цього
B. Глюкоза 6-фосфатдегідрогеназа процесу:
C. Альфаамілаза A. Аланiн.
D. Гамма-амілаза B. Аденiн.
Е. Лізосомальна глікозидаза C. Амонiак.
Збірник тестових завдань для складання D. Гуанiн.
ліцензійного іспиту Крок-1 «Загальна E. Сечовина.
лікарська підготовка». –2016. - № 1. Збірник тестових завдань для складання
ліцензійного іспиту Крок-1 «Стоматологія».
69. Пiд час голодування м’язовi бiлки – 2010. – № 107.
розпадаються до вiльних амiнокислот. В
якому процесi найбiльш iмовiрно будуть 73. У хворого, який проходить курс
використовуватися амiнокислоти за таких лiкувального голодування, нормальний
умов? рiвень глюкози у кровi підтримується
A. Глюконеогенез у печiнцi. головним чином за рахунок глюконеогенезу.
B. Глюконеогенез у м’язах. З якої амiнокислоти при цьому упечiнцi
C. Глiкогенолiз. людини найбiльш активно синтезується
D. Декарбоксилювання. глюкоза?
E. Синтез вищих жирних кислот. A. Аланiн
Збірник тестових завдань для складання B. Лiзин
ліцензійного іспиту Крок-1 «Загальна C. Валiн
лікарська підготовка». –2010. - № 150. D. Глутамiнова кислота
E. Лейцин
70. Відомо, що резерви вуглеводів в Збірник тестових завдань для складання
організмі людини швидко зникають ліцензійного іспиту Крок-1 «Загальна
внаслідок тривалого голодування. Який з лікарська підготовка». –2011. - № 66.
процесів метаболізму поновлює вміст
глюкози в крові? 74. Глюконеогенез активується в печінці
A. Глюконеогенез. після інтенсивних фізичних тренувань. Яка
B. Аеробне окиснення глюкози. речовина використовується в першу чергу:
C. Гліколіз. А. Лактат
D. Глікогеноліз. Б. Піруват
E. Пентозофосфатний шлях. С. Глюкоза
Збірник тестових завдань для складання D. Глутамат
ліцензійного іспиту Крок-1 «Стоматологія». Е. Аланін
– 2005. – № 128.
Збірник тестових завдань для складання Збірник тестових завдань для складання
ліцензійного іспиту Крок-1 «Загальна ліцензійного іспиту Крок-1 «Стоматологія».
лікарська підготовка». –2005. - № 56. – 2009. – № 38.

75. При напруженiй фiзичнiй роботi у 79. Відомо, пентозофосфатний шлях, що


м’язовiй тканинi накопичується молочна протікає в адипоцитах жирової тканини, діє
кислота, яка дифундує в кров i як цикл. Яка основна функція цього циклу в
підхоплюється печiнкою та серцем. Який жировій тканині?
процес забезпечує вiдновлення запасiв A. Генерація НАДФН2
глiкогену в м’язах? B. Виробництво рибозофосфату
A. Цикл Корi C. Дезінтоксикація ксенобіотиків
B. Цикл лимонної кислоти D. Виробництво енергії
C. Орнiтиновий цикл E. Окислення глюкози до кінцевих продуктів
D. Цикл трикарбонових кислот Збірник тестових завдань для складання
E. Пентозофосфатний шлях ліцензійного іспиту Крок-1 «Стоматологія».
Збірник тестових завдань для складання – 2013. – № 119.
ліцензійного іспиту Крок-1 «Загальна
лікарська підготовка». –2015. - № 145. 80. Відомо, що фруктозурія пов'язана зі
спадковим дефіцитом фруктозо-1-фосфат
76. Хворому поставлений діагноз бері-бері. альдолази. Який продукт фруктозного
Активність якого ферменту порушена у обміну накопичується в організмі, що
пацієнта? призводить до токсичної дії?
A. Піруватдегідрогенази. A. Фруктозо-1-фосфат
B. Цитратсинтази. B. Глюкозо-1-фосфат
C. Малатдегидрогенази. C. Глюкозо-6-фосфат
D. Сукцинатдегидрогенази. D. Фруктозо-1,6-біфосфат
E. Фумарази. Е. Фруктозо-6-фосфат
Збірник тестових завдань для складання Збірник тестових завдань для складання
ліцензійного іспиту Крок-1 «Стоматологія». ліцензійного іспиту Крок-1 «Загальна
2010. - № 118. лікарська підготовка». – 2016. - № 173.

77. При недостатності тіаміну - вітаміну В1 81. Генетичний дефект піруваткарбоксилази


виникає хвороба бері-бері (поліневрит) та є причиною затримки фізичного та
порушується вуглеводний обмін. Який психічного розвитку та ранньої смерті у
метаболіт при цьому накопичується в крові? дітей. Цей дефект характеризується
A. Піруват. лактемією, лактацидурією, розладом ряду
B. Лактат. метаболічних шляхів. Зокрема, інгібується
C. Сукцинат. наступний процес:
D. Цитрат. А. Цикл лимонної кислоти та глюконеогенез
E. Малат. B. Гліколіз та глікогеноліз
Збірник тестових завдань для складання С. Глікогенез та глікогеноліз
ліцензійного іспиту Крок-1 «Стоматологія». D. Ліполіз та ліпогенез
– 2009. – № 118. Е. Пентозофосфатний шлях та гліколіз
Збірник тестових завдань для складання
78. У пацієнта спостерігається підвищення ліцензійного іспиту Крок-1 «Загальна
вмісту лактату при аналізі слини. Активація лікарська підготовка». –2013. - № 143.
якого процесу є причиною цього?
A. Гліколіз. 82. Полісахарид глікоген синтезується з
B. Аеробного окиснення глюкози. активної форми глюкози. Безпосереднім
C. Глюкозо-лактатного циклу. донором залишків глюкози під час
D. Розпаду глікогену. глікогенезу є:
E. Травлення вуглеводів. A. УДФ-глюкоза
B. Глюкозо-1-фосфат
C. АДФ-глюкоза
D. Глюкозо-6-фосфат 84. У дитини в історії хвороби наявні
E. Глюкозо-3-фосфат гепатомегалія, гіпоглікемія, судоми,
Збірник тестових завдань для складання особливо натщесерце і у стресових
ліцензійного іспиту Крок-1 «Загальна ситуаціях. Дитині діагностована хвороба
лікарська підготовка». –2013. - № 57. Гірке. Це захворювання викликане
генетичним дефектом наступного ферменту:
83. Дитина з точковою мутацією, яка A. Глюкозо-6-фосфатаза
характеризується відсутність глюкозо-6- B. Амілоїд-1,6-глікозидаза
фосфатази, гіпоглікемією та гепатомегалією. C. Фосфоглюкомутаза
Для якої патології характерні дані ознаки? D. Глікогенфосфорилаза
А. Хвороба Гірке (глікогеноз типу I) Е. Глюкокіназа
B. Хвороба Корі (глікогеноз типу III) Збірник тестових завдань для складання
C. Хвороба Аддісона (первинна надниркова ліцензійного іспиту Крок-1 «Загальна
недостатність) лікарська підготовка». –2014. - № 173.
D. Хвороба Паркінсона
Е. Хвороба Мак-Ардля (глікогеноз типу V) 85. У хворих з глікогенозом, тобто хворобою
Збірник тестових завдань для складання фон Гірке, інгібується перетворення
ліцензійного іспиту Крок-1 «Стоматологія». глюкозо-6-фосфату в глюкозу, що
– 2012. – № 128. супроводжується неналежним розщепленням
глікогену в печінці. Причиною цього стану є
наступний недолік ферментів:
A. Глюкозо-6-фосфатаза
B. Глікогенфосфорилаза
C. Глюкозо-6-фосфатдегідрогеназа
D. Фосфофруктокіназа
Е. Фосфоглюкомутаза
Збірник тестових завдань для складання
ліцензійного іспиту Крок-1 «Стоматологія».
– 2014. – № 135.
Метаболізм ліпідів та його регуляція
1. При обстеженнi хворого встановлено, що Збірник тестових завдань для складання
причиною гіпоплазії зубiв є гiповiтамiноз А ліцензійного іспиту Крок-1 «Загальна
та D. Цi вiтамiни призначили перорально, лікарська підготовка». –2015. - № 60.
проте лiкувального ефекту не досягли. Яка
можлива причина порушення засвоєння 5. Хвора 65-ти рокiв страждає на
вiтамiнiв? жовчнокам’яну хворобу. Останнiм часом
A. Нестача жовчних кислот з’явилися ознаки ахолiчного синдрому
B. Нестача фосфоліпази А2 внаслiдок обтурацiї жовчних шляхiв.
C. Нестача холестеролестерази Засвоєння яких компонентів їжi буде
D. Нестача коліпази порушене найбiльше?
E. Нестача панкреатичної ліпази A. Жири
Збірник тестових завдань для складання B. Вуглеводи
ліцензійного іспиту Крок-1 «Стоматологія». C. Бiлки
–2008. – № 196. D. Нуклеїновi кислоти
E. Електролiти
2. У чоловiка, який тривалий час не вживав з Збірник тестових завдань для складання
їжею жирiв, але отримував достатню ліцензійного іспиту Крок-1 «Стоматологія».
кiлькiсть вуглеводiв i бiлкiв, виявлено –2011. – № 128.
дерматит, погане загоювання ран,
погiршення зору. Дефiцит яких компонентiв 6. Хворому призначено препарат з
є причиною порушення обмiну речовин? вираженими лiпофiльними властивостями.
A. Лiнолева кислота, вiтамiни A, D, E, K Яким буде головний механiзм його
B. Пальмiтинова кислота всмоктування?
C. Вiтамiни PP, H A. Пасивна дифузiя
D. Мiнеральнi солi B. Активний транспорт
E. Олеїнова кислота C. Зв’язування з транспортними бiлками
Збірник тестових завдань для складання D. Пiноцитоз
ліцензійного іспиту Крок-1 «Загальна E. Фiльтрацiя
лікарська підготовка». –2007. - № 49. Збірник тестових завдань для складання
ліцензійного іспиту Крок-1 «Стоматологія».
3. У людини порушено всмоктування –2008. – № 83.
продуктів гiдролiзу жирiв. Причиною цього
може бути дефіцит у порожнинi тонкої 7. Внаслідок блокування загальної жовчної
кишки: протоки, що підтверджено радіографією,
A. Жовчних кислот надходження жовчі до дванадцятипалої
B. Лiполiтичних ферментiв кишки зупинилося. Очікується ослаблення:
C. Жовчних пiгментiв A. Емульгування жирів
D. Iонiв натрiю B. Всмоктування білків
E. Жиророзчинних вiтамiнiв C. Гідролізу вуглеводів
Збірник тестових завдань для складання D. Секреції соляної кислоти
ліцензійного іспиту Крок-1 «Загальна E. Слиновиділення
лікарська підготовка». –2016. - № 69. Збірник тестових завдань для складання
ліцензійного іспиту Крок-1 «Загальна
4. У хворого нормально забарвлений кал, у лікарська підготовка». –2013. - № 174.
складi якого є велика кiлькiсть вiльних
жирних кислот. Причиною цього є 8. Пiсля вживання жирної їжi у хворого
порушення: з’являються нудота та печiя, має мiсце
A. Всмоктування жирiв стеаторея. Причиною такого стану може
B. Гiдролiзу жирiв бути:
C. Жовчовидiлення A. Нестача жовчі
D. Жовчоутворення B. Пiдвищене видiлення лiпази
E. Секрецiї лiпаз C. Порушення синтезу трипсину
D. Нестача амiлази
E. Порушення синтезу фосфолiпази Збірник тестових завдань для складання
Збірник тестових завдань для складання ліцензійного іспиту Крок-1 «Загальна
ліцензійного іспиту Крок-1 «Загальна лікарська підготовка». –2013. - № 142.
лікарська підготовка». –2008. - № 1.
13. Пацiєнтцi з високим ступенем ожирiння у
9. При копрологiчному дослiдженнi якостi харчової добавки було рекомендовано
встановлено, що кал знебарвлений, у ньому карнiтин для полiпшення "спалювання"жиру.
знайдено краплi нейтрального жиру. Яку безпосереднюу часть бере карнiтин у
Найбiльш вiрогiдною причиною цього є процесi окиснення жирiв?
порушення: A. Транспорт ВЖК з цитозоля до
A. Надходження жовчi до кишечнику мiтохондрiй
B. Кислотностi шлункового соку B. Транспорт ВЖК з жирових депо до
C. Секрецiї пiдшлункового соку тканин
D. Секрецiї кишкового соку C. Бере участь в однiй з реакцiй бета-
E. Процесiв всмоктування в кишечнику окиснення ВЖК
Збірник тестових завдань для складання D. Активацiя ВЖК
ліцензійного іспиту Крок-1 «Загальна E. Активацiя внутрiшньоклiтинного лiполiзу
лікарська підготовка». –2010. - № 56. Збірник тестових завдань для складання
ліцензійного іспиту Крок-1 «Загальна
10. Дихальний коефiцiєнт у хворого складає лікарська підготовка». –2009. - № 32.
0,7. Це свiдчить, що у клiтинах людини
переважає: 14. До клiнiки потрапила дитина 1-го року з
A. Окислення жирiв ознаками ураження м’язiв кiнцiвок та
B. Окислення вуглеводiв тулуба. Пiсля обстеження виявлений дефiцит
C. Окислення бiлкiв карнiтину в м’язах. Бiохiмiчною основою
D. Змiшане окислення жирiв та вуглеводiв цiєї патологiї є порушення процесу:
E. Змiшане окислення жирiв та бiлкiв A. Транспорту жирних кислот у мiтохондрiї
Збірник тестових завдань для складання B. Регуляцiї рiвня Ca2+ в мiтохондрiях
ліцензійного іспиту Крок-1 «Загальна C. Субстратного фосфорилювання
лікарська підготовка». –2014. - № 90. D. Утилiзацiї молочної кислоти
E. Окисного фосфорилювання
11. Для пiдвищення спортивних результатiв Збірник тестових завдань для складання
чоловiку рекомендували застосовувати ліцензійного іспиту Крок-1 «Загальна
препарат, що мiстить карнiтин. Який процес лікарська підготовка». –2010. - № 153.
у найбiльшому ступенi активiзується
карнiтином? 15. Емоційний стрес спричиняє активацію
A. Транспорт жирних кислот до мiтохондрiй гормончутливої тригліцеридліпази
B. Синтез стероїдних гормонiв адипоцитів. Який вторинний месенджер бере
C. Синтез кетонових тiл участь у цьому процесі
D. Синтез лiпiдiв A. цАМФ
E. Тканинне дихання B. цГМФ
Збірник тестових завдань для складання C. АМФ
ліцензійного іспиту Крок-1 «Загальна D. Діацилгліцерол
лікарська підготовка». –2007. - №94. E. Іони Ca2+
Збірник тестових завдань для складання
12. Одним з факторів, які спричиняють ліцензійного іспиту Крок-1 «Загальна
ожиріння, є пригнічення окислення жирних лікарська підготовка». –2013. - № 54.
кислот, що зумовлене
A. Низьким рівнем карнітину 16. У копрограмi пацієнта виявили значну
B. Ослабленням синтезу фосфоліпідів кiлькiсть неперетравлених жирiв.
C. Надмірним вживанням жирної їжі Порушення секреції яких ферментів
D. Нестачею холіну найiмовiрнiше має мiсце у даної людини?
E. Бідною на вуглеводи дієтою A. Панкреатичнi лiпази
B. Панкреатична амiлаза
C. Панкреатичнi протеази синтетичного глюкокортикоїда пов’язана з
D.Жовчнiлiпази блокуванням вивiльнення арахідонової
E.Шлунковi протеази кислоти шляхом гальмування:
Збірник тестових завдань для складання A. Фосфолiпази А2
ліцензійного іспиту Крок-1 «Загальна B. Лiпоксигенази
лікарська підготовка». –2018. - № 157. C. Фосфолiпази С
D. Пероксидази
17. Ожиріння є поширеним захворюванням, E. Циклооксигенази
лікування якого має на меті зниження вмісту Збірник тестових завдань для складання
в тілі нейтральних жирів. Який ліцензійного іспиту Крок-1 «Загальна
гормончутливий фермент є найбільш лікарська підготовка». –2016. - № 164.
важливим для внутрішньоклітинного
ліполізу 21. При лабораторному обстеженнi кровi
A. Триацилгліцеридліпаза людини, яку вкусила змiя, виявлено гемолiз
B. Протеїнкіназа еритроцитiв, гемоглобiнурiю. Дiя змiїної
C. Аденілатциклаза отрути зумовлена наявнiстю в нiй ферменту:
D. Діацилгліцеридліпаза A. Фосфолiпаза А2
E. Моноацилгліцеридліпаза B. Фосфолiпаза D
Збірник тестових завдань для складання C. Фосфолiпаза С
ліцензійного іспиту Крок-1 «Загальна D. Фосфолiпаза А1
лікарська підготовка». –2013. - № 161. E. Сфiнгомiєлiназа
Збірник тестових завдань для складання
18. Порушення процесiв розщеплення лiпiдiв ліцензійного іспиту Крок-1 «Стоматологія».
у тонкому кишечнику зумовлено –2017. – № 166.
порушенням активності лiпази. Який з
наведених чинникiв активує лiпазу? 22. Хворому iз больовим синдромом в
A. Жовчнi кислоти суглобах постiйно призначають аспiрин.
B. Соляна кислота Який з перерахованих ферментів він
C. Ентерокiназа пригнiчує?
D. Пепсин A. Циклооксигеназа
E. Солi Na+ B. Фосфолiпаза А2
Збірник тестових завдань для складання C. Фосфолiпаза D
ліцензійного іспиту Крок-1 «Стоматологія». D. Лiпооксигеназа
–2016. – № 88. E. Фосфолiпаза С
Збірник тестових завдань для складання
19. У чоловiка 35-ти рокiв феохромоцитома. ліцензійного іспиту Крок-1 «Загальна
В кровi спостерiгається пiдвищений рiвень лікарська підготовка». –2017. - № 177.
адреналiну та норадреналiну, концентрацiя
вiльних жирних кислот зросла в 11 разiв. 23. До бiорегуляторiв клiтинних функцiй
Активацiя якого ферменту пiд впливом лiпiдної природи належать тромбоксани.
адреналiну пiдвищує лiполiз? Джерелом для синтезу цих сполук є:
A. ТАГ-лiпаза A. Арахiдонова кислота
B. Лiпопротеїдлiпаза B. Стеаринова кислота
C. Фосфолiпаза А2 C. Пальмiтинова кислота
D. Фосфолiпаза С D. Фосфатидна кислота
E. Холестеролестераза E. Пальмiтоолеїнова кислота
Збірник тестових завдань для складання Збірник тестових завдань для складання
ліцензійного іспиту Крок-1 «Загальна ліцензійного іспиту Крок-1 «Загальна
лікарська підготовка». –2015. - № 12. лікарська підготовка». –2013. - № 97.

20. Пацiєнту, який знаходився в клiнiцi з 24. Метильнi групи (−CH3)


приводу пневмонiї, ускладненої плевритом, використовуються в органiзмi для синтезу
у складi комплексної терапії вводили таких важливих сполук, як креатин, холiн,
преднiзолон. Протизапальна дiя цього
адреналiн, iншi. Джерелом цих групп є одна 28. Вiдомо, що частина диоксиду вуглецю
з незамiнних амiнокислот, а саме: використовується в органiзмi в бiосинтезi
A. Метiонiн жирних кислот, сечовини, глюконеогенезi
B. Валiн тощо. Який вiтамiн утворює CO2-
C. Лейцин транспортуючу форму для цих реакцiй?
D. Iзолейцин A. Бiотин
E. Триптофан B. Тимiн
Збірник тестових завдань для складання C. Рибофлавiн
ліцензійного іспиту Крок-1 «Загальна D. Нiкотинамiд
лікарська підготовка». –2012. - № 1. E. Ретинол
Збірник тестових завдань для складання
25. При нестачi бiотину спостерiгається ліцензійного іспиту Крок-1 «Стоматологія».
порушення синтезу вищих жирних кислот. –2015. – № 136.
Утворення якого iз зазначених метаболiтiв
може бути порушено при цьому? 29. Молода людина 25-ти рокiв споживає
A. Малонiл-КоА надмiрну кiлькiсть вуглеводiв (600 г на
B. Сукцинiл-КоА добу), що перевищує її енергетичні потреби.
C. Пiруват Який процес буде активуватися в органiзмi
D. Аланiн людини у даному випадку?
E. Серотонiн A. Лiпогенез
Збірник тестових завдань для складання B. Глiколiз
ліцензійного іспиту Крок-1 «Загальна C. Лiполiз
лікарська підготовка». –2012. - № 146. D. Глюконеогенез
E. Окиснення жирних кислот
26. Недостатнiсть в органiзмi лiнолевої та Збірник тестових завдань для складання
лiноленової кислот призводить до ліцензійного іспиту Крок-1 «Загальна
ушкоджень шкiри, випадiння волосся, лікарська підготовка». –2017. - № 175.
сповiльненого загоювання ран,
тромбоцитопенiї, зниження опiрностi до 30. Порушення процесів мієлінізації
iнфекцiйних захворювань. Порушення нервових волокон призводить до
синтезу яких речовин найiмовiрнiше неврологічних розладiв i розумової
зумовлює вказанi симптоми? вiдсталостi. Такi симптоми характернi для
A. Ейкозаноїди спадкових i набутих порушень обмiну:
B. Iнтерлейкiни A. Сфiнголiпiдiв
C. Iнтерферони B. Нейтральних жирiв
D. Катехоламiни C. Вищих жирних кислот
E. Кортикостероїди D. Холестерину
Збірник тестових завдань для складання E. Фосфатидної кислоти
ліцензійного іспиту Крок-1 «Загальна Збірник тестових завдань для складання
лікарська підготовка». –2012. - № 150. ліцензійного іспиту Крок-1 «Стоматологія».
–2017. – № 135.
27. Препарат "Гептрал", який
використовують при хворобах печiнки, 31. До лiкаря звернулась мати з приводу
мiстить S-аденозилметiонiн. Ця активна поганого самопочуття дитини – вiдсутнiсть
амiнокислота бере участь у синтезi: апетиту, поганий сон, дратiвливiсть. При
A. Фосфолiпiдiв біохімічному дослiдженнi в кровi виявлено
B. Жовчних кислот вiдсутнiсть ферменту глюкоцереброзидази.
C. Триацилглiцеролiв Для якої патологiї це характерно?
D. Холестерину A. Хвороба Гоше
E. Гему B. Хвороба Тея-Сакса
Збірник тестових завдань для складання C. Хвороба Нiмана-Пiка
ліцензійного іспиту Крок-1 «Загальна D. Хвороба Гiрке
лікарська підготовка». –2014. - № 85. E. Хвороба Помпе
Збірник тестових завдань для складання D. Валiн, B3, B2
ліцензійного іспиту Крок-1 «Загальна E. Iзолейцин, B1, B2
лікарська підготовка». –2011. - № 124. Збірник тестових завдань для складання
ліцензійного іспиту Крок-1 «Загальна
32. У хворого дiагностований лікарська підготовка». –2013. - № 160.
глюкоцереброзидний ліпідом (хвороба
Гоше), що проявляється спленомегалiєю, 36. У експериментальної тварини, що
збiльшенням печiнки, ураженням кісткової знаходиться на безбiлковому рацiонi,
тканини i нейропатiями. Недостатнiсть якого розвинулася жирова інфільтрація печінки
ферменту катаболiзму складних лiпiдiв внаслiдок дефiциту метилюючих агентiв.
зумовлює це захворювання? Утворення якого метаболiту порушено у
A. Глюкоцереброзидаза пiддослiдної тварини?
B. Гексозамiнiдаза A. Холiн
C. Сфiнгомiєлiназа B. ДОФА
D. β-галактозидаза C. Холестерин
E. Гiалуронiдаза D. Ацетоацетат
Збірник тестових завдань для складання E. Лiнолева кислота
ліцензійного іспиту Крок-1 «Загальна Збірник тестових завдань для складання
лікарська підготовка». –2018. - № 171. ліцензійного іспиту Крок-1 «Стоматологія».
–2005. – № 115.
33. При утилізації арахiдонової кислоти за
циклооксигеназним шляхом утворюються 37. У культурi клiтин, отриманих вiд хворого
біологічно активнi речовини. Вкажiть їх: з лiзосомною патологiєю, виявлено
A. Простагландини накопичення значної кiлькостi лiпiдiв у
B. Тироксин лiзосомах. При якому з перелiчених
C. Бiогеннiамiни захворювань має мiсце це порушення?
D. Соматомедини A. Хвороба Тея-Сакса
E. Iнсулiноподiбнi фактори росту B. Подагра
Збірник тестових завдань для складання C. Фенiлкетонурiя
ліцензійного іспиту Крок-1 «Загальна D. Хвороба Вiльсона-Коновалова
лікарська підготовка». –2007. - № 127. E. Галактоземiя
Збірник тестових завдань для складання
34. Хворому з метою попередження жирової ліцензійного іспиту Крок-1 «Загальна
дистрофії печінки лiкар призначив лікарська підготовка». –2007. - № 188.
ліпотропний препарат – донор метильних
груп. Це iмовiрно: 38. Синтез фосфоліпідів порушується при
A. S-Аденозилметiонiн жировій інфільтрації печінки. Вкажіть, яка з
B. Холестерин наступних речовин здатна покращити процес
C. Бiлiрубiн метилювання при синтезі фосфоліпідів:
D. Валiн A. Метіонін
E. Глюкоза B. Аскорбінова кислота
Збірник тестових завдань для складання C. Глюкоза
ліцензійного іспиту Крок-1 «Загальна D. Гліцерин
лікарська підготовка». –2016. - № 99. E. Цитрат
Збірник тестових завдань для складання
35. Стеатоз виникає внаслiдок накопичення ліцензійного іспиту Крок-1 «Загальна
триацилглiцеролiв у гепатоцитах. Одним з лікарська підготовка». –2005. - № 9.
механiзмiв розвитку цього захворювання є
зменшення утилізації нейтрального жиру 39. Експериментальні тварини отримували
ЛПДНЩ. Якi лiпотропнi речовини надмірну кількість глюкози з міченими
попереджують розвиток стеатозу? атомами карбону протягом тижня. В якій
A. Метiонiн, BC, B12 сполуці можна виявити цю мітку:
B. Аргiнiн, B2, B3 A. Пальмітинова кислота
C. Аланiн, B1, PP B. Метіонін
C. Вітамін A печінки й селезінки. Діагностовано спадкове
D. Холін захворювання Німана-Піка. Який генетичний
E. Арахідонова кислота дефект став причиною даного
Збірник тестових завдань для складання захворювання?
ліцензійного іспиту Крок-1 «Загальна A. Дефіцит сфінгомієлінази
лікарська підготовка». –2007. - № 50. B. Нестача глюкозо-6-фосфатази
C. Дефіцит аміло-1,6-глюкозидази
40. 6-річна дитина доставлена у лікарню. D. Дефіцит кислої ліпази
Обстеження показало, що дитина не здатна E. Нестача ксантиноксидази
фіксувати погляд, не слідкує за іграшками, Збірник тестових завдань для складання
на очному дні відзначається симптом ліцензійного іспиту Крок-1 «Загальна
вишневої кісточки. Лабораторні аналізи лікарська підготовка». –2017. - № 112.
показали, що мозок, печінка і селезінка
містять високий рівень гангліозидів 44. У 3-річної дівчинки із затримкою
глікометидів. Яка вроджена хвороба в цієї розумового розвитку діагностовано
дитини? сфінголіпідоз (хворобу Німана-Піка). Синтез
A. Хвороба Тея-Сакса якої з наступних речовин порушено при цій
B. Хвороба Вільсона-Коновалова хворобі:
C. Хвороба Шерешевського-Тернера A. Сфінгомієлінази
D. Хвороба Німана-Піка B. Глікозилтрансферази
E. Хвороба МакАргдля C. Сфінгозіну
Збірник тестових завдань для складання D. Церамідів
ліцензійного іспиту Крок-1 «Загальна E. Гангліозидів
лікарська підготовка». –2008. - № 147. Збірник тестових завдань для складання
ліцензійного іспиту Крок-1 «Загальна
41. У робітника хімчистки виявлена жирова лікарська підготовка». –2016. - № 143.
дистрофія печінки. Порушення синтезу якої
речовини в печінці може привести до даної 45. Сучаснi антиатеросклеротичнi препарати
патології? застосовуються з метою профiлактики та
A. Фосфатидилхоліну. лiкування атеросклерозу. Такi препарати як
B. Тристеарину. гемфiброзил та фенфiбрат гальмують
C. Сечовини. бiосинтез холестерину шляхом iнгiбування
D. Фосфатидної кислоти. ферменту:
E. Холевої кислоти. A. β-ГОМК-редуктаза
Збірник тестових завдань для складання B. Гексокиназа
ліцензійного іспиту Крок-1 «Загальна C. Глюкозо-6-фосфатаза
лікарська підготовка». –2014. - № 17. D. Ацилтрансфераза
E. Ацил-КоА-холестеринацилтрансфераза
42. Ключовою реакцією синтезу жирних Збірник тестових завдань для складання
кислот є утворення малоніл-КоА. Який ліцензійного іспиту Крок-1 «Загальна
метаболіт є джерелом для синтезу малоніл- лікарська підготовка». –2015. - № 135.
КоА?
А. Ацетил-КоА 46. Лiкар дав жiнцi рекомендацiю
В. Сукциніл-КоА продовжити низькокалорiйну дiєту. Вона
С. Ацил-КоА вирiшила отримувати ту ж саму кiлькiсть
D.Малонат калорiй, але замiнити вуглеводи на жири.
E. Цитрат Рiвень якої з наведених лiпопротеїнових
Збірник тестових завдань для складання фракцiй буде пiдвищеним внаслiдок цієї
ліцензійного іспиту Крок-1 «Загальна дiєти?
лікарська підготовка». –2016. - № 92. A. Хiломiкрони
B. ЛПДНЩ
43. У 2-річної дитини спостерігається різке C. ЛПНЩ
відставання в психомоторному розвитку, D. ЛППЩ
зниження слуху та зору, різке збільшення E. ЛПВЩ
Збірник тестових завдань для складання C. Протеїназа
ліцензійного іспиту Крок-1 «Стоматологія». D. Гемсинтетаза
–2016. – № 83. E. Фенiлаланiнгiдроксилаза
Збірник тестових завдань для складання
47. Для профілактики атеросклерозу, ліцензійного іспиту Крок-1 «Стоматологія».
ішемічної хвороби серця, порушень –2007. – № 104.
мозкового кровообiгу рекомендується
споживання жирів iз високим вмiстом 51. Кролiв годували їжею з додаванням
поліненасичених жирних кислот. Однiєю з холестерину. Через 5 мiсяцiв виявленi
таких жирних кислот є: атеросклеротичнi змiни в аортi. Назвiть
A. Лiнолева головну причину атерогенезу в даному
B. Олеїнова випадку:
C. Лауринова A. Екзогенна гiперхолестеринемiя
D. Пальмiтоолеїнова B. Переїдання
E. Стеаринова C. Гiподинамiя
Збірник тестових завдань для складання D. Ендогенна гiперхолестеринемiя
ліцензійного іспиту Крок-1 «Загальна E. Cтрес
лікарська підготовка». –2016. - № 128. Збірник тестових завдань для складання
ліцензійного іспиту Крок-1 «Стоматологія».
48. Спадкова гiперлiпопротеїнемiя I типу –2007. – № 124.
обумовлена недостатнiстю
лiпопротеїнлiпази. Пiдвищення рівня яких 52. При обстеженнi у жiнки встановлена
транспортних форм ліпідів в плазмi навiть недостатнiсть активностi лiпопротеїнлiпази,
натщесерце є характерним? котра гiдролiзує триглiцериди хiломiкронiв
A. Хiломiкрони на поверхнi ендотелiю капiлярiв жирової
B. Лiпопротеїни низької густини тканини. Якi бiохiмiчнi порушення слiд
C. Лiпопротеїни дуже низької густини очiкувати?
D. Лiпопротеїни високої густини A. Гiперлiпопротеїнемiя I типу
E. Модифiкованi лiпопротеїни B. Гiперлiпопротеїнемiя II А типу
Збірник тестових завдань для складання C. Гiперлiпопротеїнемiя III типу
ліцензійного іспиту Крок-1 «Загальна D. Гiперлiпопротеїнемiя IV типу
лікарська підготовка». –2016. - № 180. E. Гiперлiпопротеїнемiя II Б типу
Збірник тестових завдань для складання
49. Пiд час дослiдження плазми кровi ліцензійного іспиту Крок-1 «Стоматологія».
пацiєнта через 4 години пiсля приймання –2009. – № 135.
ним жирної їжi встановлено, що вона є
каламутною. Найбiльш ймовiрною 53. Чоловiк 60-ти рокiв страждає на
причиною даного стану є пiдвищення атеросклероз судин головного мозку. При
концентрацiї в плазмi: обстеженнi виявлена гiперлiпiдемiя. Вмiст
A. Хiломiкронiв якого класу лiпопротеїнiв найбiльш iмовiрно
B. ЛПВГ буде пiдвищений при дослiдженнi сироватки
C. ЛПНГ кровi?
D. Холестерину A. Лiпопротеїди низької щiльностi
E. Фосфолiпiдiв B. Лiпопротеїди високої щiльностi
Збірник тестових завдань для складання C. Комплекси жирних кислот з альбумiнами
ліцензійного іспиту Крок-1 «Стоматологія». D. Хiломiкрони
–2005. – № 126. E. Холестерин
Збірник тестових завдань для складання
50. У хворої дитини в кровi встановлено ліцензійного іспиту Крок-1 «Стоматологія».
гiперлiпопротеїнемiю, що передалася у –2014. – № 100.
спадок. Генетичний дефект синтезу якого
ферменту обумовлює це явище? 54. У хворого пiсля курсу лiкування
A. Лiпопротеїнлiпаза атеросклерозу в плазмi кровi лабораторно
B. Глiкозидаза доведено збiльшення рiвня антиатерогенної
фракцiї лiпопротеїнiв. Збiльшення рiвня яких Збірник тестових завдань для складання
лiпопротеїнiв пiдтверджує ефективнiсть ліцензійного іспиту Крок-1 «Стоматологія».
терапiї захворювання? –2008. – № 48.
A. ЛПВЩ
B. ЛПДНЩ 58. При цукровому дiабетi i голодуваннi в
C. ЛППЩ кровi збiльшується вмiст ацетонових тiл, що
D. ЛПНЩ використовуються в якостi енергетичного
E. Хiломiкрони матерiалу. Назвiть речовину, з якої вони
Збірник тестових завдань для складання синтезуються:
ліцензійного іспиту Крок-1 «Стоматологія». A. Ацетiл-КоА
–2014. – № 191. B. Сукцинiл-КоА
C. Цитрат
55. При обстеженнi хворого виявлено D. Малат
пiдвищення вмiсту в сироватцi кровi E. Кетоглутарат
лiпопротеїнiв низької щiльностi. Яке Збірник тестових завдань для складання
захворювання можна передбачити у цього ліцензійного іспиту Крок-1 «Стоматологія».
хворого? –2015. – № 136.
A. Атеросклероз
B. Ураження нирок 59. 67-річний чоловік споживає яйця, сало,
C. Гострий панкреатит масло, молоко і м’ясо. Результати аналізу
D. Гастрит крові: холестерол – 12,3 ммоль/л, загальні
E. Запалення легень ліпіди – 8,2 г/л, зростання фракції
Збірник тестових завдань для складання ліпопротеїнів низької щільності (ЛПНЩ).
ліцензійного іспиту Крок-1 «Стоматологія». Який тип гіперліпопротеїнемії
–2015. – № 39. спостерігається у цього пацієнта?
A. Гіперліпопротеїнемія типу IIa
56. При обстеженнi пiдлiтка, що страждає на B. Гіперліпопротеїнемія типу I
ксантоматоз, виявлено сiмейну C. Гіперліпопротеїнемія типу IIb
гiперхолестеролемiю. Концентрацiя яких D. Гіперліпопротеїнемія типу IV
транспортних форм лiпiдiв пiдвищується при E. Холестерол, гіперліпопротеїнемія
цьому захворюваннi? Збірник тестових завдань для складання
A. Лiпопротеїди низької щiльностi ліцензійного іспиту Крок-1 «Загальна
B. Хiломiкрони лікарська підготовка». –2017. - № 151.
C. Лiпопротеїди дуже низької щiльностi
D. Лiпопротеїди високої щiльностi 60. Вміст холестеролу в сироватці крові 12-
E. ЛППЩ річного хлопця – 25 ммоль/л. В анамнезі
Збірник тестових завдань для складання спадкова гіперхолестеринемія спричинена
ліцензійного іспиту Крок-1 «Стоматологія». порушенням синтезу рецепторів для:
–2015. – № 198. A. Ліпопротеїнів низької щільності
B. Ліпопротеїнів високої щільності
57. У кровi хворих на цукровий дiабет C. Хіломікронів
спостерiгається пiдвищення вмiсту вiльних D. Ліпопротеїнів дуже низької щільності
жирних кислот (ВЖК). Причиною цього E. Ліпопротеїнів проміжної щільності
може бути: Збірник тестових завдань для складання
A. Пiдвищення активностi триглiцеридлiпази ліцензійного іспиту Крок-1 «Загальна
адипоцитiв лікарська підготовка». –2016. - № 176.
B. Накопичення в цитозолiпальмiтоїл-КоА
C. Активацiя утилiзацiї кетонових тiл 61. Підвищення рівня ЛПВЩ знижує ризик
D. Активацiя синтезу аполiпопротеїнiвА-1, атеросклерозу. Який механізм
А-2, А-4 антиатерогенної дії ЛПВЩ?
E. Зниження активностi фосфатидилхолiн- A. Вони забирають холестерол із тканин
холестеринацилтрансферази плазми кровi B. Вонипостачають тканини холестеролом
C. Вони ускладнюють розщеплення
холестеролу
D. Вони активують перетворення Збірник тестових завдань для складання
холестеролу на жовчні кислоти ліцензійного іспиту Крок-1 «Загальна
E. Вони покращують всмоктування лікарська підготовка». –2015. - № 13.
холестеролу в кишечнику
Метаболізм амінокислот та його регуляція
1. При захворюваннях пiдшлункової залози E. Стрептокiназа
порушується утворення та секреція Збірник тестових завдань для складання
трипсину. Назвiть речовини, травлення яких ліцензійного іспиту Крок-1 «Стоматологія».
буде порушене? – 2018. – № 48.
A. Бiлки
B. Лiпiди 5. Пiддослiднiй тваринi через зонд у
C. Вуглеводи порожнину шлунку ввели 150 мл м’ясного
D. Нуклеїновi кислоти бульйону. Вмiст якої речовини швидко
E. Фосфолiпiди збiльшиться у кровi?
Збірник тестових завдань для складання A. Гастрин
ліцензійного іспиту Крок-1 «Стоматологія». B. Соматостатин
–2016. – № 28. C. Iнсулiн
D. Глюкагон
2. У добовому раціоні дорослої здорової E. Нейротензин
людини повинні бути жири, білки, Збірник тестових завдань для складання
вуглеводи, вітаміни, мінеральні солі та вода. ліцензійного іспиту Крок-1 «Стоматологія».
Вкажіть кількість білку на добу, яка –2005. – № 93.
забезпечує нормальну життєдіяльність
організму. 6. У молодого чоловiка внаслiдок
A. 100-120 г подразнення сонячного сплетення запальним
B. 50-60 г процесом (солярит) пiдвищена
C. 10-20 г функцiональна активнiсть залоз шлунка, що
D. 70-80 г виражається, зокрема, у збiльшеннi
E. 40-50 г продукцiї хлоридної кислоти. Яка з вказаних
Збірник тестових завдань для складання нижче речовин викликає гiперхлоргiдрiю у
ліцензійного іспиту Крок-1 «Стоматологія». даному випадку?
–2016. – № 25. A. Гастрин
B. Гастроiнгiбуючий пептид
3. У хворого 30-ти рокiв iз C. Урогастрон
гостримзапаленням пiдшлункової залози D. Глюкагон
(панкреатитом) виявлено порушення E. Калiкреїн
порожнинного травлення бiлкiв. Це Збірник тестових завдань для складання
можебути пов’язано iз недостатнiм ліцензійного іспиту Крок-1 «Загальна
синтезомта видiленням залозоютакого лікарська підготовка». –2017. – № 170.
ферменту:
A. Трипсин 7. Хворому проведено видалення пiлоричної
B. Пепсин частини шлунка. Зменшення секрецiї якого
C. Лiпаза гормону слiд очiкувати перш за все?
D. Дипептидаза A. Гастрин
E. Амiлаза B. Гiстамiн
Збірник тестових завдань для складання C. Секретин
ліцензійного іспиту Крок -1 «Загальна D. Холецистокiнiн
лікарська підготовка». –2010. – № 48. E. Шлунковий iнгiбуючий пептид
Збірник тестових завдань для складання
4. У лікарню було привезено хворого з ліцензійного іспиту Крок -1 «Загальна
опiками шкiри. Для очищення ран вiд лікарська підготовка». –2018. – № 161.
мертвих тканин та слизу лiкар для
локального лiкування призначив 8. З метою визначення максимальної секрецiї
ферментний препарат. Назвiть його: хлористоводневої кислоти шлункового соку
A. Трипсин пацiєнту 42-х рокiв ввели розчин гiстамiну.
B. Панзинорм Це призвело до збiльшення секрецiї
C. Аспарагiназа пiдшлунковою залозою такого компоненту
D. Пепсин соку:
A. Бiкарбонати 12. По клінічним показам хворому
B. Трипсиноген призначений піридоксальфосфат для
C. Лiпаза корекції процесів:
D. Амiлаза А. Трансамінування і декарбоксилування
E. Слиз амінокислот
Збірник тестових завдань для складання B. Окисного декарбоксилування кетокислот
ліцензійного іспиту Крок-1 «Стоматологія». С. Дезамінування пуринових нуклеотидів
– 2015. – № 119. D. Синтезу пуринових і піримідинових основ
E. Синтезу білка
9. В гострому дослiдi тваринi в порожнину Збірник тестових завдань для складання
12-ти палої кишки ввели слабкий розчин ліцензійного іспиту Крок-1 «Стоматологія».
хлористоводневої кислоти. До збiльшення – 2015. – № 36.
секрецiї якого гастроiнтестинального
гормону це призведе? 13. До лiкарнi швидкої допомоги доставили
A. Секретин дитину 7 рокiв у станi алергічного шоку,
B. Гастрин який розвинувся пiсля того, як її вжалила
C. Мотилiн оса. У кровi пiдвищена концентрацiя
D. Нейротензин гiстамiну. В результатi якої реакцiї
E. Гiстамiн утворився цей амiн?
Збірник тестових завдань для складання A. Декарбоксилювання
ліцензійного іспиту Крок-1 «Стоматологія». B. Гiдрооксилювання
– 2018. – № 22. C. Дегiдрування
D. Дезамiнування
10. Перетравлення білків у шлунку є E. Вiдновлення
початковою стадією розщеплення білків у Збірник тестових завдань для складання
травному каналі людини. Назвіть ферменти, ліцензійного іспиту Крок-1 «Загальна
які беруть участь в перетравлені білків у лікарська підготовка». –2005. – № 68.
шлунку:
A. Пепсин та гастриксин 14. При тестуваннi на гiперчутливiсть
B. Трипсин та катепсини пацiєнту під шкiру ввели алерген, після чого
C. Хімотрипсин та лізоцим спостерігалось почервонiння, набряк, бiль
D. Ентеропептидаза та еластаза внаслiдок дії гiстамiну. В результатi якого
E. Карбоксипептидаза та амінопептидаза перетворення амінокислоти гiстидину
Збірник тестових завдань для складання утворюється цей біогенний амiн?
ліцензійного іспиту Крок-1 «Стоматологія». A. Декарбоксилювання
–2013. – № 37. B. Метилювання
C. Фосфорилювання
11. У новонародженої дитини у шлунку D. Iзомеризацiя
відбувається “згурджування” молока, тобто E. Дезамiнування
перетворення розчинних білків молока Збірник тестових завдань для складання
казеїнів у нерозчинні – параказеїни за ліцензійного іспиту Крок-1 «Загальна
участю іонів кальцію і ферменту: лікарська підготовка». –2018. – № 133.
A. Ренніну
B. Пепсину 15. Депресії, емоційні розлади є наслідком
C. Гастрину нестачі у головному мозку дофаміну,
D. Секретину норадреналіну, серотоніну та інших
E. Ліпази біогенних амінів. Збільшення їх вмісту у
Збірник тестових завдань для складання синапсах можна досягти за рахунок
ліцензійного іспиту Крок -1 «Загальна антидепресантів, які гальмують фермент:
лікарська підготовка». –2010. – № 49. A Моноамінооксидазу
B Диамінооксидазу
C Оксидазу L-амінокислот
D Оксидазу D-амінокислот
E Фенілаланін-4-монооксигеназу
Збірник тестових завдань для складання Збірник тестових завдань для складання
ліцензійного іспиту Крок-1 «Загальна ліцензійного іспиту Крок-1 «Загальна
лікарська підготовка». –2018. – № 129. лікарська підготовка». –2016. – № 16.

16. Вiдомо, що в метаболiзмi 20. Дитина 9-ми мiсяцiв харчується


катехоламiнових медiаторiв особлива роль штучними сумiшами, якi не збалансованi за
належить ферменту моноамiноксидазi вмiстом вiтамiну B6. У дитини
(МАО). Яким шляхом цей фермент iнактивує спостерiгається пелагроподiбний дерматит,
медiатори (норадреналiн, адреналiн, судоми, анемiя. Розвиток судом може бути
дофамiн)? пов’язаний з порушенням утворення:
A. Окисне дезамiнування A. ГАМК
B. Приєднання амiногрупи B. Гiстамiну
C. Видалення метальної групи C. Серотонiну
D. Карбоксилювання D. ДОФА
E. Гiдролiз E. Дофамiну
Збірник тестових завдань для складання Збірник тестових завдань для складання
ліцензійного іспиту Крок-1 «Стоматологія». ліцензійного іспиту Крок-1 «Загальна
– 2018. – № 72. лікарська підготовка». –2009. – № 149.

17. В ході катаболізму гістидину 21. Який нейромедiатор у тканинi мозку


утворюється біогенний амін, що має може бути синтезований з продукту
потужну судинорозширюючу дію. Назвіть переамiнування альфа-кетоглутарової
його. кислоти?
A. Гістамін. A. ГАМК
B. ДОФА. B. Триптамiн
C. Дофамін. C. Дофамiн
D. Серотонін. D. Серотонiн
E. Норадреналін. E. Норадреналiн
Збірник тестових завдань для складання Збірник тестових завдань для складання
ліцензійного іспиту Крок-1 «Стоматологія». ліцензійного іспиту Крок-1 «Стоматологія».
– 2013. – № 45. – 2005. – № 14.

18. При декарбоксилюванні глутамату в 22. У хворого з дiагнозом злоякiсного


ЦНС утворюється медіатор гальмування. карцiноїду рiзко пiдвищений вмiст
Назвіть його. серотонiну в кровi. Оберiть амiнокислоту, з
A. ГАМК. якої утворюється ця сполука:
B. Глутатіон. A. Триптофан
C. Гістамін. B. Аланiн
D. Серотонін. C. Лейцин
E. Аспарагін. D. Треонiн
Збірник тестових завдань для складання E. Метiонiн
ліцензійного іспиту Крок-1 «Загальна Збірник тестових завдань для складання
лікарська підготовка». –2007. – № 125. ліцензійного іспиту Крок-1 «Загальна
лікарська підготовка». –2009. – № 149.
19. Біогенні аміни використовують в
психіатрії для лікування ряду захворювань 23. Госпiталiзовано хворого з дiагнозом
ЦНС. Вкажіть препарат цієї групи, який карциноїд кишечнику. Аналiз виявив
являється медіатором гальмування: пiдвищену продукцiю серотонiну. Вiдомо,
A. γ-Аміномасляна кислота. що ця речовина утворюється з амiнокислоти
B. Дофамін. триптофану. Який бiохiмiчний механiзм
C. Гістамін. лежить в основi даного процесу?
D. Серотонін. A. Декарбоксилювання
E. Таурин. B. Дезамiнування
C. Мiкросомальне окиснення
D. Трансамiнування D. Гістидин.
E. Утворення парних сполук E. Аланін.
Збірник тестових завдань для складання Збірник тестових завдань для складання
ліцензійного іспиту Крок-1 «Загальна ліцензійного іспиту Крок-1 «Стоматологія».
лікарська підготовка». –2007. – № 142. – 2016. – № 27.

24. Метильнi групи (−CH3) 28. Відомо, що нагромадження аміаку є


використовуються в органiзмi для синтезу основною причиною мозкової коми при
таких важливих сполук, як креатин, холiн, печінковій недостатності. Яка вільна
адреналiн, iншi. Джерелом цих груп є одна з амінокислота відіграє першочергову роль в
незамiнних амiнокислот, а саме: утилізації цієї токсичної речовини в мозку?
A. Метiонiн A. Глутамінова кислота.
B. Валiн B. Аланін.
C. Лейцин C. Гістидин.
D. Iзолейцин D. Триптофан.
E. Триптофан E. Цистеїн.
Збірник тестових завдань для складання Збірник тестових завдань для складання
ліцензійного іспиту Крок-1 «Загальна ліцензійного іспиту Крок-1 «Стоматологія».
лікарська підготовка». –2012. – № 1. – 2005. – № 31.

25. Катiоннi глiкопротеїни є основними 29. Після травми мозку у пацієнта


компонентами слини привушних залоз. Якi спостерігається підвищене утворення аміаку.
амiнокислоти обумовлюють їх позитивний Яка амінокислота бере участь в видаленні
заряд? аміаку з цієї тканини?
A. Лiзин, аргiнiн, гiстидин A. Глутамінова.
B. Аспартат, глутамат, глiцин B. Валін.
C. Аспартат, аргiнiн, глутамат C. Лізин.
D. Глутамат, валiн, лейцин D. Тирозин.
E. Цистеїн, глiцин, пролін E. Триптофан.
Збірник тестових завдань для складання Збірник тестових завдань для складання
ліцензійного іспиту Крок-1 «Загальна ліцензійного іспиту Крок-1 «Загальна
лікарська підготовка». –2013. – № 90. лікарська підготовка». –2006. – № 130.

26. Є декілька шляхів знешкодження аміаку 30. Хворий поступив у клiнiку зi струсом
в організмі, але для окремих органів є мозку. На фонi неврологiчних симптомів у
специфічні. Який шлях знешкодження цієї кровi збiльшується концентрацiя амiаку. Яку
токсичної речовини характерний для клітин речовину слiд призначити для знешкодження
головного мозку? цiєї речовини у мозковiй тканини?
A. Утворення глутаміну. A. Глутамiнова кислота
B. Утворення NH4+ . B. Гiстамiн
C. Утворення аспарагіну. C. Аскорбiнова кислота
D. Утворення креатину. D. Серотонiн
E. Утворення сечовини. E. Нiкотинова кислота
Збірник тестових завдань для складання Збірник тестових завдань для складання
ліцензійного іспиту Крок-1 «Загальна ліцензійного іспиту Крок-1 «Стоматологія».
лікарська підготовка». – 2008. – № 65. – 2017. – № 72.

27. Аміак є дуже отруйною речовиною, 31. Пiсля операцiї на кишечнику у хворого
особливо для нервової системи. Яка з’явились симптоми отруєння амiаком за
речовина бере особливо активну участь у типом печiнкової коми. Який механiзм дiї
знешкодженні аміаку в тканинах мозку? амiаку на енергозабезпечення ЦНС?
A. Глутамінова кислота. A. Гальмування ЦТК в результатi
B. Лізин. зв’язування альфа-кетоглутарату
C. Пролін. B. Гальмування глiколiзу
C. Гальмування бета-окиснення жирних Збірник тестових завдань для складання
кислот ліцензійного іспиту Крок-1 «Загальна
D. Iнактивацiя ферментiв дихального лікарська підготовка». –2009. – № 63.
ланцюга
E. Роз’єднування окисного фосфорилювання 35. У дитини 2-хроків спостерiгається
Збірник тестових завдань для складання відставання в розумовому розвитку,
ліцензійного іспиту Крок-1 «Загальна непереносимість білкової їжi, важка
лікарська підготовка». –2017. – № 140. гiперамонiємiя на тлi зниженого вмiсту
сечовини в плазмi кровi, що пов’язано з
32. У новонародженої дитини вродженим дефіцитом такого ферменту
спостерігається зниження інтенсивності мiтохондрiй:
смоктання, часте блювання, гіпотонія. У сечі A. Карбомоїлфосфатсинтетаза
та крові значно підвищена концентрація B. Цитратсинтаза
цитруліну. Який метаболічний процес C. Сукцинатдегiдрогеназа
порушений? D. Малатдегiдрогеназа
А. Орнітиновий цикл E. Моноамiнооксидаза
В. Гліколіз Збірник тестових завдань для складання
С. Цикл Корі ліцензійного іспиту Крок-1 «Стоматологія».
D. ЦТК – 2016. – № 172.
Е. Глюконеогенез
Збірник тестових завдань для складання 36. При біосинтезі сечовини в печінці
ліцензійного іспиту Крок-1 «Стоматологія». вібдувається утворення орнітину та
– 2016. – № 91. сечовини. Яка амінокислота є проміжним
продуктом цього синтезу?
33. У дитини 3-х рокiв пiсля перенесеної A Аргінін.
важкої вiрусної iнфекцiї вiдзначається B Лейцин.
повторне блювання, непритомнiсть, судоми. C Цитрат.
При дослiдженнi виявлена гiперамонiємiя. З D Валін.
чим може бути пов’язана змiна бiохiмiчних E Триптофан.
показникiв кровi у цiєї дитини? Збірник тестових завдань для складання
A. Порушення знешкодження амiаку в ліцензійного іспиту Крок-1 «Загальна
орнiтиновому циклi лікарська підготовка». –2007. – № 125
B. Активацiя процесiв декарбоксилювання
амiнокислот 37. В сечі новонародженого визначається
C. Порушення знешкодження біогенних цитрулін та високий рівень аміаку. Вкажіть,
амiнiв утворення якої речовини найімовірніше
D. Посилення гниття бiлкiв у кишечнику порушене у малюка ?
E. Пригнiчення активностi ферментів A. Сечовини.
трансамінування B. Білірубіну.
Збірник тестових завдань для складання C. Креатину.
ліцензійного іспиту Крок-1 «Загальна D. Креатиніну.
лікарська підготовка». –2006. – № 78. E. Сечової кислоти.
Збірник тестових завдань для складання
34. Основна маса азоту з органiзму ліцензійного іспиту Крок-1 «Стоматологія».
виводиться у виглядi сечовини. Зниження – 2010. – № 49.
активностi якого ферменту в печiнцi
призводить до гальмування синтезу 38. У хворого iз зниженою видiльною
сечовини i нагромадження амонiаку в кровi i функцiєю нирок вiдзначається неприємний
тканинах? запах з рота. Збiльшення екскрецiї слинними
A. Карбамоїлфосфатсинтаза залозами якої речовини є причиною цього?
B. Аспартатамiнотрансфераза A. Сечовини.
C. Уреаза B. α-амiлази.
D. Амiлаза C. Муцину.
E. Пепсин D. Лiзоциму.
E. Фосфатази. 42. При зниженнi активностi ферментів
Збірник тестових завдань для складання антиоксидантного захисту посилюються
ліцензійного іспиту Крок-1 «Стоматологія». процеси перекисного окиснення лiпiдiв
– 2006. – № 67 клiтинних мембран. При нестачi якого
мiкроелементу знижується активнiсть
39. Хворий у непритомному станi глутатiонпероксидази?
доставлений бригадою швидкої допомоги до A. Селен
лiкарнi. Об’єктивно: рефлекси вiдсутнi, B. Молiбден
перiодично з’являються судоми, дихання C. Кобальт
нерiвномiрне. Пiсля лабораторного D. Марганець
обстеження було дiагностовано печiнкову E. Мiдь
кому. Нагромадження якого метаболiту у Збірник тестових завдань для складання
крові є суттєвим для появи розладiв ліцензійного іспиту Крок-1 «Загальна
центральної нервової системи? лікарська підготовка». –2015. – № 120.
A. Амонiаку.
B. Бiлiрубiну. 43. Ендотелiй судин відзначається високою
C. Гiстамiну. метаболічною активністю i синтезує вазо
D. Глутамiну. активні речовини, серед яких потужним
E. Сечовини. вазодилататором, що синтезується iз L-
Збірник тестових завдань для складання аргiнiну є:
ліцензійного іспиту Крок-1 «Стоматологія». A. Оксид азоту
– 2013. – № 110. B. Гiстамiн
C. Брадикiнiн
40. У хлопчика 4-х рокiв пiсля перенесеного D. Ацетилхолiн
важкого вiрусного гепатиту спостерiгаються E. Адреналiн
блювання, епiзоди непритомностi, судоми. У Збірник тестових завдань для складання
кровi - гiперамонiємiя. Порушення якого ліцензійного іспиту Крок-1 «Загальна
бiохiмiчного процесу в печiнцi викликало лікарська підготовка». –2018. – № 184.
такий стан хворого?
A. Знешкодження амонiаку. 44. Оксид азоту вiдiграє важливу роль у
B. Декарбоксилювання амiнокислот. релаксацiї гладеньких м’язiв судин i
C. Знешкодження бiогенних амiнiв. зниженнi артерiального тиску, розширеннi
D. Синтезу бiлкiв. коронарних артерiй. NO в органiзмi може
E. Глюконеогенезу. утворюватись з:
Збірник тестових завдань для складання A. Аргiнiну
ліцензійного іспиту Крок-1 «Стоматологія». B. Пролiну
– 2015. – № 46. C. Лiзину
D. Метiонiну
41. У 2-рiчної дитини з нирковою E. Глутамiну
недостатнiстю виявили гiпероксалурiю, Збірник тестових завдань для складання
оксалатний уролiтiаз, що призвело до ліцензійного іспиту Крок-1 «Стоматологія».
вiдкладання оксалату кальцiю в нирках. – 2017. – № 174.
Порушення обмiну якої амiнокислоти
призвело до такого стану? 45. Хвора 46-ти рокiв довгий час страждає на
A. Глiцин прогресуючу м’язову дистрофiю( Дюшена).
B. Лiзин Змiни рiвня якого ферменту кровi є
C. Метiонiн дiагностичним тестом вданому випадку?
D. Аргiнiн A. Креатинфосфокiназа
E. Гiстидин B. Лактатдегiдрогеназа
Збірник тестових завдань для складання C. Пiруватдегiдрогеназа
ліцензійного іспиту Крок-1 «Стоматологія». D. Глутаматдегiдрогеназа
– 2011. – № 114. E. Аденiлаткiназа
Збірник тестових завдань для складання рацiонi якої амiнокислоти призводить до
ліцензійного іспиту Крок-1 «Загальна даної патологiї?
лікарська підготовка». –2010. – № 3. A. Триптофан
B. Iзолейцин
46. До лiкаря звернувся пацiєнт зiскаргами C. Фенiлаланiн
на запаморочення, погiршення пам’ятi, D. Метiонiн
перiодичнi судоми. Встановлено, що E. Гiстидин
причиною таких змiн є продукт Збірник тестових завдань для складання
декарбоксилювання глутамінової кислоти. ліцензійного іспиту Крок-1 «Загальна
Назвiть його: лікарська підготовка». – 2011. – № 82.
A. ГАМК
B. ПАЛФ 50. Людина захворiла на пелагру. При
C. ТДФ опитуваннi стало вiдомо, що впродовж
D. АТФ тривалого часу вона харчувалася переважно
E. ТГФК кукурудзою, мало вживала м’яса. Дефiцит
Збірник тестових завдань для складання якої речовини у кукурудзi спричинив
ліцензійного іспиту Крок-1 «Стоматологія». розвиток хвороби?
– 2008. – № 112. A. Триптофан
B. Тирозин
47. Хворому 24-х років для лiкування C. Пролiн
епілепсії ввели глутамінову кислоту. D. Аланiн
Лiкувальний ефект при даному захворюваннi E. Гiстидин
обумовлений не самим глутаматом, а таким Збірник тестових завдань для складання
продуктом його декарбоксилювання: ліцензійного іспиту Крок-1 «Загальна
A. γ-амiномасляна кислота лікарська підготовка». –2009. – № 30.
B. Гiстамiн-4-монооксигенази
C. Серотонiн 51. У хворого, що страждає на спадкову
D. Дофамiн хворобу Хартнупа, спостерiгаються
E. Таурин пелагроподiбнi ураження шкiри, порушення
Збірник тестових завдань для складання розумового розвитку врезультатi нестачi
ліцензійного іспиту Крок-1 «Стоматологія». нiкотинової кислоти. Причиною цього
– 2018. – № 28. захворювання є порушення такого процесу:
A. Всмоктування i реабсорбцiя в нирках
48. У немовляти спостерiгаються триптофану
епiлептиформнi судоми, викликанi B. Трансамiнування фенiлаланiну
дефiцитом вiтамiну В6. Це спричинено C. Декарбоксилювання триптофану
зменшенням у нервовiй тканинi гальмiвного D. Всмоктування i реабсорбцiя в нирках
медiатора- γ-амiномасляної кислоти. метiонiну
Активнiсть якого ферменту знижена при E. Всмоктування i реабсорбцiя цистеїну
цьому? Збірник тестових завдань для складання
A. Глутаматдекарбоксилаза ліцензійного іспиту Крок-1 «Загальна
B. Аланiнамiнотрансфераза лікарська підготовка». –2014. – № 188.
C. Глутаматдегiдрогеназа
D. Пiридоксалькiназа 52. У жiнки, яка вiдпочивала на дачi, вiдразу
E. Глутаматсинтетаза після укусу оси виник бiль, через кiлька
Збірник тестових завдань для складання хвилин на шкiрi в мiсцi укусу з’явився
ліцензійного іспиту Крок-1 «Загальна пухир, еритема i сильне свербiння, а ще
лікарська підготовка». –2016. – № 16. через деякий час - кропив’янка, експіраторна
задишка. Внаслiдок дії яких факторiв у
49. Причиною захворювання на пелагру хворої розвинулась експiраторна задишка?
може бути переважне харчування A. Гiстамiн
кукурудзою i зниження у рацiонi продуктiв B. Фактор Хагемана
тваринного походження. Вiдсутнiсть у C. Лiзосомальнi ферменти
D. Норадреналiн
E. Адреналiн речовини викликало почервонiння,
Збірник тестових завдань для складання набряклiсть та болючiсть ураженої дiлянки
ліцензійного іспиту Крок-1 «Загальна шкiри?
лікарська підготовка». –2018. – № 163. A. Гiстамiн
B. Тiамiн
53. У пацієнта через 30 хвилин після C. Глутамiн
лiкування у стоматолога з’явилися червонi D. Лiзин
плями на шкiрi обличчя i слизовiй рота, що E. Галактозамiн
сверблять. Був встановлений дiагноз: Збірник тестових завдань для складання
кропивниця. Яка з біологічно активних ліцензійного іспиту Крок-1 «Загальна
речовин, що викликають розширення судин, лікарська підготовка». –2017. – № 60.
появу свербежу, виділяється при цьому типi
алергічної реакцiї? 57. Для лiкування хвороби Паркiнсона
A. Гiстамiн застосовують попередник дофамiну - ДОФА.
B. ПростагландинЕ2 З якої амінокислоти утворюється ця активна
C. Лейкотрiєн В4 речовина?
D. Iнтерлейкiн-1 A. Тирозин
E. Брадикiнiн B. Аланiн
Збірник тестових завдань для складання C. Цистеїн
ліцензійного іспиту Крок-1 «Стоматологія». D. Гiстидин
– 2018. – № 120. E. Триптофан
Збірник тестових завдань для складання
54. У хворого спостерігається алергічна ліцензійного іспиту Крок-1 «Стоматологія».
реакція, яка супроводжується свербінням, – 2018. – № 154.
набряками та почервонінням шкіри.
Концентрація якого біогенного аміну 58. У хворого спостерiгається тремтіння рук,
підвищилась у тканинах? що пов’язане з хворобою Паркiнсона.
A. Гістаміну. Дефiцит якого медiатора в стрiопалiдарних
B. Гама-аміномасляної кислоти. структурах призводить до таких симптомiв?
C. Дофаміну. A. Дофамiн
D. Серотоніну. B. ГАМК
E. Триптаміну. C. Субстанцiя Р
Збірник тестових завдань для складання D. Норадреналiн
ліцензійного іспиту Крок-1 «Стоматологія». E. Серотонiн
– 2008. – № 130. Збірник тестових завдань для складання
ліцензійного іспиту Крок-1 «Загальна
55. У хворого початкова стадiя гiнгiвiту. лікарська підготовка». –2017. – № 160.
Спостерiгається гiперемiя ясен у
пришийкових областях зубiв внаслiдок 59. Хворий 84-х рокiв страждає на
розширення судин мiкроциркуляторного паркiнсонiзм, одним з патогенетичних ланок
русла, що приносять кров. Яка речовина якого є дефiцит медiатора в окремих
тучних клiтин забезпечила вказанi змiни? структурах мозку. Якого медiатора
A. Гiстамiн насамперед?
B. Адреналiн A. Дофамiн
C. Субстанцiя Р B. Адреналiн
D. Ендорфiни C. Норадреналiн
E. Ацетилхолiн D. Гiстамiн
Збірник тестових завдань для складання E. Ацетилхолiн
ліцензійного іспиту Крок-1 «Стоматологія». Збірник тестових завдань для складання
– 2017. – № 137. ліцензійного іспиту Крок-1 «Стоматологія».
– 2018. – № 155.

56. Кухар в результатi необачностi обпiк 60. При обстеженнi в клiнiцi у чоловiка
руку парою. Пiдвищення концентрації якої дiагностували гостру променеву хворобу.
Лабораторно встановлено рiзке зниження C. Цистіт.
серотонiну в тромбоцитах. Порушення D. Фенілкетонурія.
метаболiзму якої речовини є можливою E. Хвороба Хартнупа.
причиною такого стану? Збірник тестових завдань для складання
A. 5-окситриптофану. ліцензійного іспиту Крок-1 «Загальна
B. Тирозину. лікарська підготовка». –2010. – № 42.
C. Гiстидину.
D. Фенiлаланiну. 64. Батьки дитини 3-х рокiв звернули увагу
E. Серину. на потемнiння кольору його сечі при
Збірник тестових завдань для складання вiдстоюваннi. Об’єктивно: температура у
ліцензійного іспиту Крок-1 «Стоматологія». нормi, шкiрнi покриви чистi, рожевi, печiнка
– 2011. – № 126. не збiльшена. Назвiть iмовiрну причину
даного стану:
61. Хвора 20-ти років звернулася до лiкаря зi A. Алкаптонурiя
скаргами на загальне схуднення, зниження B. Гемолiз
апетиту, слабкiсть, появу незвичайного C. Синдром Iценка-Кушiнга
кольору шкiри, що нагадує пiвденну D. Фенiлкетонурiя
”бронзову засмагу”. При обстеженнi у E. Подагра
клiнiцi, окрiм гiперпiгментацiї, виявлений Збірник тестових завдань для складання
двобічний туберкульоз наднирникiв. ліцензійного іспиту Крок-1 «Загальна
Надлишкове накопичення якої речовини лікарська підготовка». –2012. – № 130.
зумовило гiперпiгментацiю шкiри?
A. Меланiн 65. Який найбільш імовірний діагноз у
B. Бiлiрубiн дитини грудного віку, у якої спостерігається
C. Гемомеланiн потемніння склер, слизових оболонок,
D. Лiпофусцин вушних раковин, виділена сеча темніє на
E. Адренохром повітрі, у крові та сечі виявлено
Збірник тестових завдань для складання гомогентизинову кислоту?
ліцензійного іспиту Крок-1 «Стоматологія». A. Алкаптонурія.
– 2018. – № 131. B. Альбінізм.
C. Гемолітична анемія.
62. Людина впродовж тривалого D. Порфірія.
часувживала їжу, бiдну на метiонiн, E. Цистинурія.
внаслiдок чого у неї спостерiгалися розлади Збірник тестових завдань для складання
функцiї нервової та ендокринної систем. Це ліцензійного іспиту Крок-1 «Стоматологія».
може бути наслiдком порушення синтезу: – 2005. – № 112.
A. Адреналiну
B. Пiрувату 66. У грудної дитини спостерiгається
C. Тиронiну забарвлення склер, слизових оболонок.
D. Жирних кислот Видiляється сеча, яка темнiє на повiтрi.В
E. Глюкагону кровi та сечi виявлено гомогентизинову
Збірник тестових завдань для складання кислоту. Що може бути причиною даного
ліцензійного іспиту Крок-1 «Загальна стану?
лікарська підготовка». –2011. – № 158. A. Алкаптонурiя
B. Альбiнiзм
63. У 12-річного хлопчика в сечі виявлено C. Галактоземiя
високий вміст усіх амінокислот D. Цистинурiя
аліфатичного ряду. При цьому відмічена E. Гiстидинемiя
найбільш висока екскреція цистіну та Збірник тестових завдань для складання
цистеїну. Крім того, УЗД нирок показало ліцензійного іспиту Крок-1 «Загальна
наявність каменів у них. Виберіть можливу лікарська підготовка». –2006. – № 74.
патологію.
A. Цистинурія.
B. Алкаптонурія.
67. У новонародженої дитини на пелюшках 71. До лiкарнi надiйшов 9-рiчний хлопчик
виявленi темнi плями, що свiдчать про розумово i фiзично вiдсталий. При
утворення гомогентизинової кислоти. З бiохiмiчному дослiдженнi кровi:пiдвищена
порушенням обмiну якої речовини це кiлькiсть фенiлаланiну. Блокування якого
пов’язане? ферменту може призвести до такого стану?
A. Тирозин A. Фенiлаланiн-4-монооксигеназа
B. Галактоза B. Оксидаза гомогентизинової кислоти
C. Метiонiн C. Глутамiнтрансамiназа
D. Холестерин D. Аспартатамiнотрансфераза
E. Триптофан E. Глутаматдекарбоксилаза
Збірник тестових завдань для складання Збірник тестових завдань для складання
ліцензійного іспиту Крок-1 «Загальна ліцензійного іспиту Крок-1 «Загальна
лікарська підготовка». –2007. – № 30. лікарська підготовка». –2012. – № 38.

68. При алкаптонурiї вiдбувається надмiрне 72. У хворої дитини у сечі виявили
видiлення гомогентизинової кислоти iз пiдвищений рівень фенiлпiрувату (в нормі
сечею. С порушенням метаболiзму якої практично вiдсутнiй). Вмiст фенiлаланiну в
амiнокислоти пов’язано виникнення цього кровi становить 350 мг/л (норма приблизно
захворювання? 15мг/л). Вкажiть, для якого захворювання
A. Тирозин характернi наведенi симптоми:
B. Фенiлаланiн A. Фенiлкетонурiя
C. Аланiн B. Альбiнiзм
D. Метiонiн C. Тирозиноз
E. Аспарагiн D. Алкаптонурiя
Збірник тестових завдань для складання E. Подагра
ліцензійного іспиту Крок-1 «Загальна Збірник тестових завдань для складання
лікарська підготовка». –2013. – № 128. ліцензійного іспиту Крок-1 «Загальна
лікарська підготовка». –2018. – № 149.
69. Мати зауважила занадто темну сечу у її
5-рiчної дитини. Дитина скарг не висловлює. 73. Одна з форм вродженої патологiї
Жовчних пігментів у сечі не виявлено. супроводжується гальмуванням
Поставлено діагноз алкаптонурiя. Дефiцит перетворення фенiлаланiну в тирозин.
якого ферменту має мiсце у дитини? Бiохiмiчною ознакою хвороби є накопичення
A. Оксидаза гомогентизинової кислоти в органiзмi деяких органiчних кислот,
B. Фенiлаланiнгiдроксилаза зокрема:
C. Тирозиназа A. Фенiлпiровиноградна
D. Оксидаза оксифенiлпiрувату B. Лимонна
E. Декарбоксилаза фенiлпiрувату C. Пiровиноградна
Збірник тестових завдань для складання D. Молочна
ліцензійного іспиту Крок-1 «Загальна E. Глутамiнова
лікарська підготовка». –2016. – № 30. Збірник тестових завдань для складання
ліцензійного іспиту Крок-1 «Загальна
70. Який вид лікування потрібен дитині при лікарська підготовка». –2013. – № 58.
фенілкетонурії, в крові якої спостерігається
підвищена кількість фенілпіровиноградної 74. У дитини 6 мiсяцiв спостерiгається рiзке
кислоти? вiдставання в психомоторному розвитку,
A. Дієтотерапія. напади судом, блiда шкiра з екзематозними
B. Антибактеріальна терапія. змiнами, бiляве волосся, блакитнi очi. У цiєї
C. Вітамінотерапія. дитини найбiльш вiрогiдно дозволить
D. Гормонотерапія. встановити діагноз визначення концентрацiї
E. Ферментотерапія. у кровi та сечi:
Збірник тестових завдань для складання A. Фенiлпiрувату
ліцензійного іспиту Крок-1 «Загальна B. Триптофану
лікарська підготовка». – ». –2015. – № 118. C. Гiстидину
D. Лейцину 78. До лікаря звернувся пацієнт із скаргами
E. Валiнду на непереносимість сонячної радіації. Мають
Збірник тестових завдань для складання місце опіки шкіри і порушення зору.
ліцензійного іспиту Крок-1 «Загальна Попередній діагноз альбінізм. Порушення
лікарська підготовка». –2005. – № 70. обміну якої амінокислоти відзначається у
даного пацієнта?
75. У немовляти на 6-й день життя в сечi A. Тирозину.
виявлено надлишок фенiлпiрувату та B. Проліну.
фенiлацетату. Обмiн якої амінокислоти C. Лізину.
порушено в органiзмi дитини? D. Аланіну.
A. Фенiлаланiн E. Триптофану.
B. Триптофан Збірник тестових завдань для складання
C. Метiонiн ліцензійного іспиту Крок-1 «Загальна
D. Гiстидин лікарська підготовка». –2005. – № 67.
E. Аргiнiн
Збірник тестових завдань для складання 79. Альбiноси погано переносять вплив
ліцензійного іспиту Крок-1 «Стоматологія». сонця - засмага не розвивається, а
– 2007. – № 93. з’являються опiки. Порушення метаболiзму
якої амiнокислоти лежить в основi цього
76. У дитини 1,5 рокiв спостерiгається явища?
вiдставання в розумовому i фізичному A. Фенiлаланiн
розвитку, посвiтлiння шкiри i волосся, B. Метiонiн
зниження вмiсту в кровi катехоламiнiв. При C. Триптофан
додаваннi до свiжої сечi декількох крапель D. Глутамiнова
5% розчину три хлороцтового залiза E. Гiстидин
з’являється оливково-зелене забарвлення. Збірник тестових завдань для складання
Для якої патологiї обмiну амiнокислот ліцензійного іспиту Крок-1 «Загальна
характернi данi змiни? лікарська підготовка». –2006. – № 80.
A. Фенiлкетонурiя
B. Алкаптонурiя 80. Хворому 12 років, у нього
C. Тирозиноз спостерігається загальна слабість,
D. Альбiнiзм запаморочення, втомлюваність, відставання
E. Ксантонурiя у розумовому розвитку. При лабораторному
Збірник тестових завдань для складання обстеженні виявлено високу концентрацію
ліцензійного іспиту Крок-1 «Загальна валіну, ізолейцину, лейцину в крові та сечі.
лікарська підготовка». –2009. – № 57. Сеча специфічного запаху. При якій хворобі
таке відбувається?
77. До лiкарнi доставлено дитину 2-х рокiв з A. Хворобі кленового сиропу.
уповiльненим розумовим i фiзичним B. Базедовій хворобі.
розвитком, що страждає на частi блювання C. Гістидинемії.
пiсля прийому їжi. У сечi визначена D. Тирозинозі.
фенiлпiровиноградна кислота. Наслiдком E. Хворобі Аддісона.
якого порушення є дана патологiя? Збірник тестових завдань для складання
A. Обмiн амiнокислот ліцензійного іспиту Крок-1 «Стоматологія».
B. Лiпiдний обмiн – 2010. – № 44.
C. Вуглеводний обмiн
D. Водно-сольовий обмiн 81. Хлопчик 13 рокiв скаржиться на загальну
E. Фосфорно-кальцiєвий обмiн слабкiсть, запаморочення, втомлюванiсть.
Збірник тестових завдань для складання Спостерiгається вiдставання у розумовому
ліцензійного іспиту Крок-1 «Загальна розвитку. При обстеженнi виявлено високу
лікарська підготовка». –2010. – № 33. концентрацію валiну, iзолейцину, лейцину в
кровi тасечi. Сеча специфiчного запаху.
Який найбiльш вiрогiдний дiагноз?
A. Хвороба "кленового сиропу"
B. Хвороба Аддiсона Збірник тестових завдань для складання
C. Тирозиноз ліцензійного іспиту Крок-1 «Загальна
D. Гiстидинемiя лікарська підготовка». –2016. – № 107.
E. Базедова хвороба
Збірник тестових завдань для складання 85. До лiкарнi надiйшов хворий зi скаргами
ліцензійного іспиту Крок-1 «Загальна на здуття живота, дiарею, метеоризм пiсля
лікарська підготовка». –2005. – № 44. вживання бiлкової їжi, що свiдчить про
порушення травлення бiлкiв та їх посиленого
82. Сеча дитини мала характерний запах гниття. Яка з перерахованих речовин є
кленового сиропу. При лабораторному продуктом цього процесу в кишечнику?
дослідженні виявлено підвищений вміст у A. Iндол
крові та сечі лейцину, валіну, ізолейцину та B. Бiлiрубiн
їх кетопохідних. Недостатність якого C. Кадаверин
ферменту характерна для цього D. Агматин
захворювання? E. Путресцин
A. Дегідрогенази розгалужених Збірник тестових завдань для складання
амінокислот. ліцензійного іспиту Крок-1 «Стоматологія».
B. Амінотрансферази. – 2008. – № 113.
C. Глюкозо-6-фосфатази.
D. Фосфофруктокінази. 86. У чоловiка 60-ти рокiв, який страждає на
E. Фосфофруктомутази. хронiчну кишкову непрохiднiсть,
Збірник тестових завдань для складання посилюється гниття бiлкiв у товстому
ліцензійного іспиту Крок-1 «Стоматологія». кишечнику. Пiдтвердженням цього процесу
– 2008. – № 118. є:
A. Iндиканурiя
83. Немовля вiдмовляється вiд годування B. Бiлiрубiнурiя
груддю, збудливе, дихання неритмiчне, сеча C. Гiперурiкурiя
має специфiчний запах "пивної закваски"або D. Креатинурiя
"кленового сиропу". Вроджений дефект E. Глюкозурiя
якого ферменту викликав дану патологiю? Збірник тестових завдань для складання
A. Дегiдрогеназа розгалужених альфа- ліцензійного іспиту Крок-1 «Стоматологія».
кетокислот – 2012. – № 111.
B. Глюкозо-6-фосфатдегiдрогеназа
C. Глiцеролкiназа 87. У хворого, прооперованого з приводу
D. Аспартатамiнотрансфераза "гострого живота", сеча коричневого
E. УДФ-глюкуронiлтрансфераза кольору, кiлькiсть iндикану в сечі вище 93
Збірник тестових завдань для складання ммоль/добу. Про що це свiдчить?
ліцензійного іспиту Крок-1 «Загальна A. Збiльшення iнтенсивностi гниття бiлкiв у
лікарська підготовка». –2009. – № 8. кишечнику
B. Зниження активностi ферментів
84. При лабораторному дослiдженнi дитини орнiтинового циклу
виявлено підвищений вмiст у кровi та сечі C. Збiльшення швидкостi окисного
лейцину, валiну, iзолейцину та їх дезамiнування ароматичних амiнокислот
кетопохiдних. Сеча має характерний запах D. Порушення фiльтрацiйної здатності нирок
кленового сиропу. Недостатнiсть якого E. Зниження iнтенсивностi знезараження
ферменту характерно для цього амонiаку
захворювання? Збірник тестових завдань для складання
A. Дегiдрогеназа розгалужених амiнокислот ліцензійного іспиту Крок-1 «Загальна
B. Амiнотрансфераза лікарська підготовка». –2012. – № 115.
C. Глюкозо-6-фосфатаза
D. Фосфофруктокiназа 88. У хворої 43 рокiв, яка була прооперована
E. Фосфофруктомутаза з приводу "гострого живота", сеча набула
коричневого кольору, кiлькiсть iндикану в
кровi рiзко пiдвищилася. Про що може 89. Для визначення функцiонального стану
свiдчити цей показник? печінки у хворого дослiджували екскрецію
A. Посилення гниття бiлкiв в кишечнику тваринного iндикану у сечi, який
B. Посилення дезамiнування амiнокислот утворюється при детоксикацiї продуктiв
C. Зниження швидкостi клубочкової гниття амінокислоти в товстiй кишцi.
фiльтрацiї у нирках Назвiть цю амiнокислоту:
D. Зниження iнтенсивностi орнiтинового A. Триптофан
циклу B. Валiн
E. Iнгiбування глюконеогенезу C. Глiцин
Збірник тестових завдань для складання D. Серин
ліцензійного іспиту Крок-1 «Стоматологія». E. Цистеїн
– 2005. – № 17. Збірник тестових завдань для складання
ліцензійного іспиту Крок-1 «Стоматологія».
– 2018. – № 139.
Міністерство Охорони Здоровя України
НАЦІОНАЛЬНИЙ МЕДИЧНИЙ УНІВЕРСИТЕТ
імені О.О. БОГОМОЛЬЦЯ

Кафедра біоорганічної та біологічної хімії

Збірник тестових завдань


для підготовки до Змістового модуля №3
«Біохімія тканин та фізіологічних функцій»
(підготовлений із Збірників тестових завдань для складання ліцензійного іспиту
«Крок 1. Загальна Лікарська підготовка» та «Крок 1. Стоматологія»)

для студентів 2 курсу медичних факультетів, медико-


психологічного, стоматологічного факультетів та факультету
підготовки збройних сил України

Київ-2019
1
Зміст

Основи молекулярної біології та генетики .......................................... 5


Біохімія міжклітинних комунікацій:
гормони білково-пептидної природи .............................. 17
гормони стероїдної та тіреоїдної природи .................. 26
Функціональна та клінічна біохімія органів і тканин:
Біохімія та патобіохімія крові ............................................................. 34
Біохімія імунної системи ....................................................................... 56
Біохімічні функції печінки ..................................................................... 63
Патологічні компоненти сечі .............................................................. 74
Біохімія нервової тканини ..................................................................... 82
Біохімія м’язової тканини ..................................................................... 86
Біохімія сполучної тканини .................................................................. 90
Біохімія зуба та слини (для стомат. фак.-ту).................................. 93

2
Основи молекулярної біології та генетики
1. Хворому на подагру призначили 5. У чоловiка 42 рокiв, який страждає на
алопуринол, що гальмує синтез сечової подагру, в кровi пiдвищена концентрацiя
кислоти шляхом інактивації такого сечової кислоти. Для зниження рiвня сечової
ферменту: кислоти йому призначено алопуринол.
A. Ксантиноксидаза Конкурентним iнгiбiтором якого ферменту є
B. Дезаміназа алопуринол?
C. Гиалуронидаза A. Ксантиноксидаза
D. Лактатдегидрогеназа B. Аденозиндезамiназа
E. Трансаміназа C. Аденiнфосфорибозилтрансфераза
Сборник тестовых заданий для D. Гiпоксантинфосфорибозилтрансфераза
лицензионного экзамена Крок-1 «Медицина». E. Гуанiндезамiназа
- 2012. - № 88. Збірник тестових завдань для складання
ліцензійного іспиту Крок -1 «Загальна
2. На основi лабораторного аналiзу у хворого лікарська підготовка». – 2005. - № 43.
пiдтверджено дiагноз - подагра. Для
встановлення дiагнозу було проведено 6. Хворому на сечокам’яну хворобу пiсля
визначення вмiсту: обстеження призначили алопуринол -
A. Сечової кислоти в кровi та сечi конкурентний iнгiбiтор ксантиноксидази.
B. Креатинiну в сечi Пiдставою для цього був хiмiчний аналiз
C. Залишкового азоту в кровi ниркових каменiв, якi складалися переважно
D. Сечовини в кровi та сечi з:
E.Амiаку в сечi A. Урату натрiю
Збірник тестових завдань для складання B. Дигiдрату оксалату кальцiю
ліцензійного іспиту Крок -1 «Загальна C. Моногiдрату оксалату кальцiю
лікарська підготовка». – 2013. - № 24. D. Фосфату кальцiю
E. Сульфату кальцію
3. У чоловiка 53-х рокiв дiагностовано Збірник тестових завдань для складання
сечокам’яну хворобу з утворенням уратiв. ліцензійного іспиту Крок -1 «Загальна
Цьому пацiєнту призначено аллопурiнол, лікарська підготовка». – 2006. - № 55.
який є конкурентним iнгiбiтором ферменту:
A. Ксантиноксидаза 7. Пацiєнт 46-ти рокiв звернувся до лiкаря зi
B. Уреаза скаргами на болi в дрiбних суглобах ніг та
C. Уратоксидаза рук. Суглоби збiльшенi, мають вигляд
D. Дигiдроурацилдегiдрогеназа потовщених вузлiв. У сироватцi встановлено
E. Уридiлтрансфераза пiдвищений вмiст уратiв. Це може бути
Збірник тестових завдань для складання спричинене:
ліцензійного іспиту Крок -1 «Загальна A. Порушенням обмiну пуринiв
лікарська підготовка». – 2014. - № 21. B. Порушенням обмiну вуглеводiв
C. Порушенням обмiну лiпiдiв
4. У крові 12-ти річного хлопчика виявлено D. Порушенням обмiну пiримiдинiв
зниження концентрації сечової кислоти і E. Порушенням обмiну амiнокислот
накопичення ксантину та гіпоксантину. Збірник тестових завдань для складання
Генетичний дефект якого ферменту має ліцензійного іспиту Крок -1 «Загальна
місце у дитини? лікарська підготовка». – 2015. - № 28.
A. Ксантиноксидаза
B. Аргіназа 8. 46-річний пацієнт звернувся до лікаря зі
C. Уреаза скаргами на біль у суглобах, яка стає
D. Орнітінкарбамоілтрансфераза сильнішою напередодні змін погоди.
E. Гліцеролкіназа Обстеження крові виявило підвищену
Сборник тестовых заданий для концентрацію сечової кислоти. Найбільш
лицензионного экзамена Крок-1 «Медицина». вірогідною причиною захворювання є
- 2010. - № 4. посилення розпаду наступної речовини:
А. Аденозинмонофосфат
3
B. Цитидинмонофосфат 12. У пацiєнта встановлено гіповітаміноз
C. Уридинтрифосфат фолiєвої кислоти, що може призвести до
D. Уридинмонофосфат порушення синтезу:
E. Тимідинмонофосфат A. Пуринових та тимiдилових нуклеотидiв
Test items for licensing examination Krok 1 B. Пуринових нуклеотидiв та холестерину
«Medicine». – 2011. - № 19. C. Тимiдилових нуклеотидiв та жирних
кислот
9. У хворого в кровi пiдвищений вмiст D. Гема та креатину
сечової кислоти, що клiнiчно проявляється E. Цитрату та кетонових тiл
больовим синдромом внаслiдок вiдкладення Збірник тестових завдань для складання
уратiв у суглобах. У результатi якого ліцензійного іспиту Крок -1 «Загальна
процесу утворюється ця кислота? лікарська підготовка». – 2015. - № 181.
A. Розпад пуринових нуклеотидiв
B. Розпад пiримiдинових нуклеотидiв 13. Бiосинтез пуринового кiльця
C. Катаболiзм гему вiдбувається на рибозо-5-фосфатi шляхом
D. Розщеплення бiлкiв поступового нарощення атомiв азоту i
E. Реутилiзацiя пуринових основ вуглецю та замикання кiлець. Джерелом
Збірник тестових завдань для складання рибозофосфату є наступний процес:
ліцензійного іспиту Крок -1 A. Пентозофосфатний цикл
«Стоматологія». – 2007. – № 107. B. Глiколiз
C. Глiконеогенез
10. Чоловiк 65-ти рокiв, який страждає на D. Глюконеогенез
подагру, скаржиться на бiль в дiлянцi нирок. E. Глiкогенолiз
Приультразвуковому обстеженнi Збірник тестових завдань для складання
встановлена наявнiсть ниркових каменiв. ліцензійного іспиту Крок -1 «Загальна
Пiдвищення концентрацiї якої речовини є лікарська підготовка». – 2015. - № 44.
найбiльш вiрогiдною причиною утворення
каменiв у даному випадку? 14. У хлопчика 8-ми років хвороба Леш-
A. Сечова кислота Ніхана. У крові збільшена концентрація
B. Холестерин сечової кислоти. Вкажіть, порушення якого
C. Бiлiрубiн процесу є причиною цього спадкового
D. Сечовина захворювання?
E. Цистин A. Розпад пуринових нуклеотидів
Збірник тестових завдань для складання B. Синтез пуринових нуклеотидів
ліцензійного іспиту Крок -1 «Загальна C. Синтез піримідинових нуклеотидів
лікарська підготовка». – 2008. - № 37. D. Розпад піримідинових нуклеотидів
E. Утворення дезоксирибонуклеотидів
11. У синтезi пуринових нуклеотидiв беруть Сборник тестовых заданий для
участь деякi амiнокислоти, похiднi вiтамiнiв, лицензионного экзамена Крок-1
фосфорнi ефiри рибози. Коферментна форма «Стоматология». - 2011. - № 48.
якого вiтамiну є переносником
одновуглецевих фрагментiв в синтезi 15. У дітей з синдромом Леша-Ніхана
пуринових нуклеотидiв? спостерігається важка форма гіперурикемії,
A. Фолiєва кислота що супроводжується появою тофусів,
B. Пантотенова кислота уратних каменів у сечовивідних шляхах і
C. Нiкотинова кислота важкими нервово-психічними порушеннями.
D. Рибофлавiн Зниження активності якого ферменту є
E. Пiридоксин причиною цього захворювання?
Збірник тестових завдань для складання A.Гіпоксантін-
ліцензійного іспиту Крок -1 «Загальна гуанінфосфорибозилтрансфераза
лікарська підготовка». – 2013. - № 107. B. Ксантиноксидаза
C. Гідрофолатредуктаза
D. Тимідилатсинтаза
E. Карбомоілфосфатсинтетаза
4
Сборник тестовых заданий для E. Цитрату та кетонових тiл
лицензионного экзамена Крок-1 «Медицина». Збірник тестових завдань для складання
- 2013. - № 160. ліцензійного іспиту Крок -1 «Загальна
лікарська підготовка». – 2015. - № 181.
16. У 19-мiсячної дитини iз затримкою
розвитку та проявами самоагресiї, вмiст 20. Похiднi птерину (амiноптерин i
сечової кислоти в кровi - 1,96 ммоль/л. При метотрексат) - є конкурентними iнгiбiторами
якому метаболічному порушеннi це дигiдрофолатредуктази, внаслiдок чого вони
спостерiгається? пригнiчують регенерацiю те-
A. Синдром Леша-Нiхана трагiдрофолiєвої кислоти з дигiдрофолату.
B. Подагра Цi лiкарськi засоби призводять до
C. Синдром набутого iмунодефiциту гальмування мiжмолекулярного транспорту
D. Хвороба Гiрке одновуглецевих груп. Бiосинтез якого
E. Хвороба Iценко-Кушiнга полiмеру при цьому пригнiчується?
Збірник тестових завдань для складання A. ДНК
ліцензійного іспиту Крок -1 «Загальна B. Бiлок
лікарська підготовка». – 2014. - № 100. C. Гомополiсахариди
D. Ганглiозиди
17. В сечі новонародженої дитини, яка E. Глiкозамiноглiкани
погано набирає вагу, виявлено підвищений Збірник тестових завдань для складання
вміст оротової кислоти, що свідчить про ліцензійного іспиту Крок -1 «Загальна
порушення синтезу піримідинових лікарська підготовка». – 2013. – №3.
нуклеотидів Який метаболіт необхідно
використати для нормалізації метаболізму? 21. Для лікування злоякісної пухлини
A. Уридин. призначають метотрексат (структурний
B. Гістидин. аналог фолієвої кислоти, що є конкурентним
C. Гуанозин. інгібітором дигідрофолатредуктази). На
D. Тимідин. якому рівні метотрексат перешкоджає
E. Аденозин. синтезу нуклеїнових кислот?
Збірник тестових завдань для складання A. Мононуклеотидний синтез
ліцензійного іспиту Крок -1 B. Реплікація
«Стоматологія». – 2008. – № 164. C. Транскрипція
D. Репарація
18. При оротацидурії виділення оротової E. Процесінг
кислоти в багато разів перевищує норму. Test items for licensing examination Krok 1
Синтез яких речовин буде порушений при «Medicine». – 2005. - № 16.
цій патології?
A. Піримідинових нуклеотидів. 22. Онкологiчному хворому призначили
B. Біогенних амінів. препарат метотрексат, до якого з часом
C. Пуринових нуклеотидів. клiтини-мiшенi пухлини втратили
D. Сечовини. чутливiсть. Експресiя гену якого ферменту
E. Сечової кислоти. при цьому змiнюється?
Збірник тестових завдань для складання A. Дегiдрофолатредуктаза
ліцензійного іспиту Крок -1 B. Тимiназа
«Стоматологія». – 2012. – № 56. C. Дезамiназа
D. Фолатоксидаза
19. У пацiєнта встановлено гіповітаміноз E. Фолатдекарбоксилаза
фолiєвої кислоти, що може призвести до Збірник тестових завдань для складання
порушення синтезу: ліцензійного іспиту Крок -1 «Загальна
A. Пуринових та тимiдилових нуклеотидiв лікарська підготовка». – 2009. – №195.
B. Пуринових нуклеотидiв та холестерину
C. Тимiдилових нуклеотидiв та жирних 23. Лейкозы лікують препаратом меторексат.
кислот Якій вітамін є його антагоністом?
D. Гема та креатину А. Фолієва кислота
5
Б. Ціанокобаламін десятків, сотень і тисяч мономерів.
C. Філохінон Молекула здатна самовідтворюватися і бути
Д. Пірідоксин носієм інформації. За допомогою
Е. Рутин рентгеноструктурного аналізу виявлено, що
Test items for licensing examination Krok 1 молекула складається з двох спірально
«Stomatology». – 2012. - № 78. закручених ниток. Вкажіть цю сполуку:
A. ДНК
24. Для нормального перебiгу процесу B. РНК
реплiкацiї потрiбнi тимiдиловi нуклеотиди, C. Целюлоза
синтез яких відбувається за участю D. Вуглевод
тимiдилатсинтетази, в якостi коферменту E. Гормон
використовується: Test items for licensing examination Krok 1
A. Метилентетрагiдрофолат «Stomatology». – 2018. - № 76.
B. Карбоксибiотин
C. Тiамiндифосфат 28. Прокарiотичнi та еукарiотичнi клiтини
D. Пiридоксальфосфат характеризуються здатнiстю до подiлу.
E. Нiкотинамiдаденiндинуклеотид Подiл прокарiотичних клiтин вiдрiзняється
Збірник тестових завдань для складання вiд подiлу еукарiотичних, але iснує
ліцензійного іспиту Крок -1 молекулярний процес, який лежить в основi
«Стоматологія». – 2014. – № 165. цих подiлiв. Який це процес?
A. Реплiкацiя ДНК
25. Чоловiковi 58-ми рокiв зроблено B. Транскрипцiя
операцiю з приводу раку простати. Через 3 C. Репарацiя
мiсяцi йому проведено курс променевої та D. Трансляцiя
хiмiотерапiї. До комплексу лiкарських E. Амплiфiкацiя генiв
препаратiв входив 5-фтордезоксиуридин- Збірник тестових завдань для складання
iнгiбiтор тимiдилатсинтази. Синтез якої ліцензійного іспиту Крок -1 «Загальна
речовини блокується цим препаратом? лікарська підготовка». – 2017. – №173.
A. ДНК
B. i-РНК 29. В ходi регенерацiї епiтелiю слизової
C. р-РНК оболонки порожнини рота (розмноження
D. т-РНК клiтин) вiдбулася реплiкацiя
E. – (авторепродукцiя) ДНК за
Збірник тестових завдань для складання напiвконсервативним механiзмом. При
ліцензійного іспиту Крок -1 «Загальна цьому нуклеотиди нової нитки ДНК є
лікарська підготовка». – 2014. - № 149 комплементарними до:
A. Материнської нитки
26. Відповідно до моделі подвійної спіралі B. Змiстовних кодонiв
ДНК, запропонованої Уотсоном і Криком, C. Ферменту ДНК-полiмерази
було встановлено, що один з ланцюгів D. Iнтронних дiлянок гену
зберігається при реплікації, а інший E. Ферменту РНК-полiмерази
синтезується комплементарно першому. Як Збірник тестових завдань для складання
називається цей спосіб реплікації? ліцензійного іспиту Крок -1 «Загальна
A. Напівконсервативний лікарська підготовка». – 2015. – №115.
B. Аналогічний
C. Ідентичний 30. Під час поділу клітини для реплікації
D. Дисперсний ДНК надходить сигнал з цитоплазми, і певна
E. Консервативний ділянка спіралі ДНК розкручується та
Сборник тестовых заданий для розділяється на два ланцюги. За допомогою
лицензионного экзамена Крок-1 «Медицина». якого ферменту це здійснюється?
- 2010. - № 40. A. Геліказа
B. РНК-полімераза
27. Серед органічних речовин клітини C. Лігаза
виявлений полімер, який складається з D. Рестриктаза
6
E. ДНК-полімераза Збірник тестових завдань для складання
Test items for licensing examination Krok 1 ліцензійного іспиту Крок -1 «Загальна
«Medicine». – 2015. - № 120. лікарська підготовка». – 2009. – №102

31. У лабораторiї група дослідників 35. Було доведено, що молекула незрiлої i-


експериментально отримала без’ядерцевi РНК (про-i-РНК) мiстить бiльше триплетiв,
мутантнi клiтини. Синтез яких сполук буде в чим знайдено амiнокислот у синтезованому
них порушений у першу чергу? бiлку. Це пояснюється тим, що трансляцiї у
A. Рибосомна РНК нормi передує:
B. Транспортна РНК A. Процесiнг
C. Лiпiди B. Iнiцiацiя
D. Моносахариди C. Репарацiя
E. Полiсахариди D. Мутацiя
Збірник тестових завдань для складання E. Реплiкацiя
ліцензійного іспиту Крок -1 Збірник тестових завдань для складання
«Стоматологія». – 2008. – №174. ліцензійного іспиту Крок -1 «Загальна
лікарська підготовка». – 2008. – №8.
32. В ядерцевих організаторах хромосом 13-
15, 21, 22 людини знаходяться біля 200 36. Сьогодні в структурі t-РНК знайдено
кластерних генів, що синтезують РНК. близько 50 специфічних основ, крім
Інформацію про який тип РНК несуть ці основних чотирьох азотистих основ.
ділянки хромосом? Виберіть специфічну азотситу основу:
A. рРНК А. Дигідроурацил
B. тРНК В. Урацил
C. мРНК C. Цистеїн
D. мяРНК D. Aденін
E. тРНК + рРНК E. Цитозин
Test items for licensing examination Krok 1 Test items for licensing examination Krok 1
«Stomatology». – 2013. - № 152. «Medicine». – 2005. - № 49.

33. Амінокислоти з'єднують один з одним в 37. У клiтинi людини в гранулярну


рибосомах гранулярного ендоплазматичного ендоплазматичну сiтку до рибосом
ретикулума. Знаючи послідовність доставлена i-РНК, що мiстить як екзоннi, так
амінокислот і застосовуючи генетичний код, i iнтроннi дiлянки. Який процес НЕ
можна визначити послідовність нуклеоїдів в: вiдбувається?
А. мРНК A. Процесiнг
В. Інтрони B. Реплiкацiя
C. Білки C. Транскрипцiя
D. Вуглеводи D. Трансляцiя
E. рРНК E. Пролонгацiя
Test items for licensing examination Krok 1 Збірник тестових завдань для складання
«Stomatology». – 2017. - № 180. ліцензійного іспиту Крок -1
«Стоматологія». – 2010. – №5.
34. Вивчається робота оперону бактерiї.
Вiдбулося звiльнення гена-оператора вiд 38. Синтез i-РНК проходить на матрицi ДНК
бiлка репресора. Безпосередньо пiсля цього в з урахуванням принципу комплементарностi.
клiтинi почнеться: Якщо триплети у ДНК наступнi - АТГ-ЦГТ,
A. Транскрипцiя то вiдповiднi кодони i-РНК будуть:
B. Трансляцiя A. УАЦ-ГЦА
C. Реплiкацiя B. АУГ-ЦГУ
D. Процесiнг C. АТГ-ЦГТ
E. Репресiя D. УАГ-ЦГУ
E. ТАГ-УГУ

7
Збірник тестових завдань для складання Фермент зворотної транскрипції отримав
ліцензійного іспиту Крок -1 «Загальна назву:
лікарська підготовка». – 2013. – №164. A. Ревертаза
B. ДНК-полімераза
39. У загальному виглядi генетичний апарат C. лігаза
еукарiот є таким: екзон–iнтрон–екзон. Така D. праймаза
структурно-функцiональна органiзацiя гена E. Топоізомераза
зумовлює особливостi транскрипцiї. Якою Сборник тестовых заданий для
буде про-i-РНК вiдповiдно до згаданої лицензионного экзамена Крок-1
схеми? «Стоматология». - 2014. - № 140.
A. Екзон-iнтрон-екзон
B. Екзон-екзон-iнтрон 43. У клiтинi людини вiдбувається
C. Екзон-екзон транскрипцiя. Фермент РНК-полiмераза,
D. Iнтрон-екзон пересуваючись вздовж молекули ДНК, досяг
E. Екзон-iнтрон певної послiдовностi нуклеотидiв. Пiсля
Збірник тестових завдань для складання цього транскрипцiя припинилась. Ця дiлянка
ліцензійного іспиту Крок -1 «Загальна ДНК має назву:
лікарська підготовка». – 2005. – №51. A. Термiнатор
B. Промотор
40. В ядрі клітин у еукаріот спочатку C. Репресор
синтезується молекула про-і-РНК, яка D. Оператор
комплементарна екзонам та інтронам E. Регулятор
структурного гена. Але до рибосом Збірник тестових завдань для складання
надходить така і-РНК, яка комплементарна ліцензійного іспиту Крок -1
тільки екзонам. Це свідчить про те, що в ядрі «Стоматологія». – 2011. – №5.
має місце:
A. Процесінг 44. Пiд час пресинтетичного перiоду
B. Транскрипція мiтотичного циклу у клiтинi було порушено
C. Репарація синтез ферменту ДНК-залежної-ДНК-
D. Реплікація полiмерази. До яких наслiдкiв це може
E. Зворотня транскрипція призвести?
Сборник тестовых заданий для A. Порушення реплiкацiї ДНК
лицензионного экзамена Крок-1 «Медицина». B. Порушення формування веретена подiлу
- 2011. - № 175. C. Порушення цитокiнезу
D. Скорочення тривалостi мiтозу
41. В результатi iнтоксикацiї в епiтелiальнiй E. -
клiтинi слизової оболонки порожнини рота Збірник тестових завдань для складання
не синтезуються ферменти, що забезпечують ліцензійного іспиту Крок -1
сплайсинг. Яка причина припинення «Стоматологія». – 2014. – №181.
бiосинтезу бiлку у цьому випадку?
A. Не утворюється зрiла i-РНК 45. РНК-полімераза B (II) блокується
B. Не синтезується АТФ внаслідок отруєння аманітином (смертельна
C. Не утворюється р-РНК отрута). Це заважає:
D. Не активуються амiнокислоти A. Синтез m-РНК
E. Порушено транспорт амiнокислот B. Синтез t-РНК
Збірник тестових завдань для складання С. Зворотна транскрипція
ліцензійного іспиту Крок -1 D. Синтез праймерів
«Стоматологія». – 2005. – №16. E. Дозрівання m-РНК
Test items for licensing examination Krok 1
42. При репродукції деяких РНК-вірусів, що «Medicine». – 2006. - № 11.
викликають пухлини у тварин, генетична
інформація може передаватися в зворотному 46. РНК, що мiстить вiрус iмунодефiциту
напрямку від РНК в ДНК - за допомогою людини, проникла всередину лейкоцита i за
особливого, специфічного ферменту. допомогою ферменту ревертази змусила
8
клiтину синтезувати вiрусну ДНК. В основi 50. У клiтинi в гранулярнiй ЕПС
цього явища лежить: вiдбувається етап трансляцiї, при якому
A. Зворотня транскрипцiя спостерiгається просування i-РНК щодо
B. Репресiя оперона рибосоми. Амiнокислоти з’єднуються
C. Зворотня трансляцiя пептидними зв’язками в певнiй
D. Дерепресiя оперона послiдовностi - вiдбувається бiосинтез
E. Конварiантна реплiкацiя полiпептиду. Послiдовнiсть амiнокислот у
Збірник тестових завдань для складання полiпептидi буде вiдповiдати послiдовностi:
ліцензійного іспиту Крок -1 «Загальна A. Кодонiв i-РНК
лікарська підготовка». – 2007. – №51. B. Нуклеотидiв т-РНК
C. Антикодонiв т-РНК
47. Встановлено ураження вiрусом ВIЛ Т- D. Нуклеотидiв р-РНК
лiмфоцитiв. При цьому фермент вiрусу E. Антикодонiв р-РНК
зворотня траскриптаза (РНК-залежна ДНК- Збірник тестових завдань для складання
полiмераза) каталiзує синтез: ліцензійного іспиту Крок -1 «Загальна
A. ДНК на матрицi вiрусної i-РНК лікарська підготовка». – 2013. – №180.
B. Вiрусної i-РНК на матрицi ДНК
C. ДНК на вiруснiй р-РНК 51. Студенти пiд час вивчення особливостей
D. Вiрусної ДНК на матрицi ДНК генетичного коду з’ясували, що є
E. i-РНК на матрицi вiрусного бiлка амiнокислоти, яким вiдповiдають по 6
Збірник тестових завдань для складання кодонiв, 5 амiнокислот - 4 рiзнi кодони. Iншi
ліцензійного іспиту Крок -1 «Загальна амiнокислоти кодуються трьома або двома
лікарська підготовка». – 2010. – №62. кодонами i тiльки двi амiнокислоти - одним
кодоном. Вкажiть, яку властивiсть
48. Для лiкування урогенiтальних iнфекцiй генетичного коду виявили студенти?
використовують хiнолони - iнгiбiтори A. Надлишковiсть
ферменту ДНК-гiрази. Який процес B. Унiверсальнiсть
порушується пiд дiєю хiнолонiв у першу C. Колiнеарнiсть
чергу? D. Однонаправленiсть
A. Реплiкацiя ДНК E. Триплетнiсть
B. Репарацiя ДНК Збірник тестових завдань для складання
C. Амплiфiкацiя генiв ліцензійного іспиту Крок -1
D. Рекомбiнацiя генiв «Стоматологія». – 2006. – №9.
E. Зворотна транскрипцiя
Збірник тестових завдань для складання 52. Мутація структурного гена не привела до
ліцензійного іспиту Крок -1 «Загальна заміщення амінокислот в молекулs білка. У
лікарська підготовка». – 2015. – 6. цьому проявилась наступна властивість
генетичного коду:
49. Встановлено, що деякi сполуки, A. Виродженiсть
наприклад, токсини грибiв та деякi Б. Мутабельність
антибiотики, можуть пригнiчувати C. Коллінеарність
активнiсть РНК-полiмерази. Порушення D. Недостатність
якого процесу вiдбувається в клiтинi у E. Універсальність
випадку пригнiчування даного ферменту? Сборник тестовых заданий для
A. Транскрипцiя лицензионного экзамена Крок-1
B. Процесiнг «Стоматология». – 2011. – № 5.
C. Реплiкацiя
D. Трансляцiя 53. Вiдомо, що iнформацiю про
E. Репарацiя послiдовнiсть амiнокислот у молекулi білка
Збірник тестових завдань для складання записано у виглядi послiдовностi чотирьох
ліцензійного іспиту Крок -1 «Загальна видiв нуклеотидiв у молекулi ДНК, причому
лікарська підготовка». – 2010. – №131. рiзнi амiнокислоти кодуються рiзною
кiлькiстю триплетiв - вiд одного до шести.

9
Як називається така особливiсть генетичного 57. У 36-річного пацієнта в стоматологічній
коду? клініці був видалений зуб. Через два тижні
A. Виродженiсть на місці видалення відновлюється
B. Унiверсальнiсть багатошаровий плоский епітелій. Які
C. Неперекривнiсть органели беруть участь у відновленні
D. Триплетнiсть слизової оболонки?
E. Специфiчнiсть А. Рибосоми
Збірник тестових завдань для складання Б. Центросоми
ліцензійного іспиту Крок -1 C. Лізосоми
«Стоматологія». – 2007. – №14. D. Гладкий ЕПР
Е. Мітохондрії
53. Експериментально (дiєю мутагенних Test items for licensing examination Krok 1
факторiв) у клiтинi порушено формування «Stomatology». – 2016. - № 162.
субодиниць рибосом. На якому
метаболiчному процесi це позначиться? 58. Генетична інформація зберігається в
A. Бiосинтез бiлка ДНК, але не бере безпосередньої участі в
B. Бiосинтез вуглеводiв синтезі білка в клітинах ДНК. Який процес
C. Синтез АТФ забезпечує передачу генетичної інформації в
D. Фотосинтез поліпептидний ланцюг?
E. Бiологiчне окиснення А. Трансляція
Збірник тестових завдань для складання B. Синтез рРНК
ліцензійного іспиту Крок -1 C. Синтез РНК
«Стоматологія». – 2009. – №6. D. Синтез РНК
Е. Реплікація
55. Амінокислоти з'єднуються один з одним Test items for licensing examination Krok 1
в рибосомах гранульованого «Stomatology». – 2016. - № 49.
ендоплазматичного ретикулума. Знаючи
послідовність аміно кислот і застосування 59. У хворого виявлено зниження вмiсту
генетичного коду, можна визначити iонiв магнiю, якi потрiбнi для прикрiплення
послідовність нуклеотідів у: рибосом до гранулярної ендоплазматичної
А. мРНК сiтки. Вiдомо, що це призводить до
В. Інтронах порушення бiосинтезу бiлка. Який саме етап
C. Білках бiосинтезу білка буде порушено?
D. Вуглеводах A. Трансляцiя
E. рРНК B. Транскрипцiя
Test items for licensing examination Krok 1 C. Реплiкацiя
«Stomatology». – 2017. - № 180. D. Активацiя амiнокислот
E. Термiнацiя
56. У клiтинах усiх органiзмiв присутнi Збірник тестових завдань для складання
безмембраннi органели, що складаються з ліцензійного іспиту Крок -1 «Загальна
двох неоднакових за розміром частинок. лікарська підготовка». – 2009. – №14.
Вони мають мiкроскопiчний
розмiр та беруть участь у синтезi бiлкiв. Як 60. Для утворення транспортної форми
називаються цi органели? амiнокислот для синтезу бiлка необхiдно:
A. Рибосоми A. Амiноацил-тРНК-синтетаза
B. Лiзосоми B. ГТФ
C. Комплекс Гольджi C. м-РНК
D. Клiтинний центр D. Рибосома
E. Мiтохондрiї E. Ревертаза
Збірник тестових завдань для складання Збірник тестових завдань для складання
ліцензійного іспиту Крок -1 ліцензійного іспиту Крок -1 «Загальна
«Стоматологія». – 2007. – №15. лікарська підготовка». – 2013. – №54.

10
61. Під час дослідження клітин було 65. Для вивчення локалiзацiї бiосинтезу
встановлено в їх цитоплазмі високий вміст бiлка в клiтинах, мишi ввели мiченi
ферменту аміноацил-тРНК- синтетаза. Цей амiнокислоти аланiн та триптофан. Бiля яких
фермент забезпечує в клітині такий процес: органел буде спостерiгатися накопичення
A. Активація амінокислот мiчених амiнокислот?
B. Репарація A. Рибосоми
C. Елонгація B. Гладенька ЕПС
D. Транскрипція C. Клiтинний центр
E. Реплікація D. Лiзосоми
Сборник тестовых заданий для E. Апарат Гольджi
лицензионного экзамена Крок-1 Збірник тестових завдань для складання
«Стоматология».– 2018. –№ 9. ліцензійного іспиту Крок -1 «Загальна
лікарська підготовка». – 2005. – №27.
63. В клiтинi вiдбувається процес трансляцiї.
Коли рибосома доходить до кодонiв УАА, 66. Хворому призначили антибіотик
УАГ або УГА, синтез полiпептидного хлорамфенiкол (левомецитин), який порушує
ланцюга закiнчується. Цi кодони у процесi у мiкроорганiзмiв синтез білку шляхом
бiосинтезу полiпептиду не розпiзнаються гальмування процесу:
жодною т-РНК i тому є сигналом: A. Елонгацiя трансляцiї
A. Термiнацiї B. Утворення полiрибосом
B. Посттрансляцiйної модифiкацiї C. Транскрипцiя
C. Початку транскрипцiї D. Процесiнг
D. Елонгацiї E. Амплiфiкацiя генів
E. Iнiцiацiї Збірник тестових завдань для складання
Збірник тестових завдань для складання ліцензійного іспиту Крок -1 «Загальна
ліцензійного іспиту Крок -1 лікарська підготовка». – 2017. – №166.
«Стоматологія». – 2013. – №167.
67. Для лікування інфекційних захворювань
63. Одним з етапів синтезу білка є використовують антибіотики (стрептоміцин,
розпізнавання. Перший триплет i-РНК еритроміцин, хлорамфенікол). Який етап
починається з триплету UAU. Який синтезу білків вони інгібують?
додатковий триплет є в тРНК? A. Трансляція
A. AUA B. Транскрипція
B. AAA C. Реплікація
C. GUG D. Процесінг
D. UGU E. Сплайсінг
Е. CUC Test items for licensing examination Krok 1
Test items for licensing examination Krok 1 «Medicine». – 2014. - № 123.
«Stomatology». – 2011. - № 9.
68. Хворому 28-ми рокiв на бактерiальну
64. При цитологiчних дослiдженнях було пневмонiю призначили курс лiкування
виявлено велику кiлькiсть рiзних молекул т- еритромiцином. Вiдомо, що його
РНК, якi доставляють амiнокислоти до антибактерiальнi властивостi зумовленi
рибосоми. Кiлькiсть рiзних типiв т-РНК у здатнiстю цього середника сполучатися з
клiтинi буде дорiвнювати кiлькостi: вiльною 50S-субодиницею рибосоми. Синтез
A. Триплетiв, що кодують амiнокислоти яких речовин блокує цей антибiотик у
B. Нуклеотидiв бактерiальних клiтинах?
C. Амiнокислот A. Бiлки
D. Бiлкiв, синтезованих у клiтинi B. РНК
E. Рiзних типiв i-РНК C. ДНК
Збірник тестових завдань для складання D. Жири
ліцензійного іспиту Крок -1 E. Полiсахариди
«Стоматологія». – 2016. – №1.

11
Збірник тестових завдань для складання дiлянка лактозного оперону стає
ліцензійного іспиту Крок -1 розблокованою вiд репресору за цих умов?
«Стоматологія». – 2014. – №73. A. Оператор
B. Промотор
69. Стрептоміцин та інші аміноглікозиди C. Структурний ген
запобігають приєднанню D. Регуляторний ген
формілметіонилової т-РНК шляхом E. Праймер
зв'язування з 30S рибосомальною Збірник тестових завдань для складання
субодиницею. Цей ефект призводить до ліцензійного іспиту Крок -1 «Загальна
порушення наступного процесу: лікарська підготовка». – 2015. – №103
А. Ініціація трансляции в прокаріотах
B. Ініціація трансляции в еукаріотах 73. Вiдомо, що ген, вiдповiдальний за
С. Ініціація транскрипції в прокаріотах розвиток груп кровi системи MN, має два
D. Ініціація транскрипції в еукаріотах алельних стани. Якщо ген М вважати
E. Ініціація реплікації в прокаріотах вихiдним, то поява алельного йому гена N
Test items for licensing examination Krok 1 вiдбулася внаслiдок:
«Medicine». – 2017. - № 148. A. Мутацiї
B. Комбiнацiї генiв
70. У генетичнiй лабораторiї пiд час роботи з C. Репарацiї ДНК
молекулами ДНК бiлих щурів лiнiї Вiстар D. Реплiкацiї ДНК
замiнили один нуклеотид на iнший. При E. Кросинговеру
цьому отримали замiну лише однiєї Збірник тестових завдань для складання
амiнокислоти у пептидi. Такий результат ліцензійного іспиту Крок -1
буде наслiдком наступної мутацiї: «Стоматологія». – 2013. – №5.
A. Трансверсiя
B. Делецiя 74. Обробка вірусної РНК азотистою
C. Дуплiкацiя кислотою привела до змін кодону УЦА на
D. Змiщення рамки зчитування кодон УГА. Якого типу відбулася мутація?
E. Транслокацiя A. Транзиція
Збірник тестових завдань для складання Б. Делеція нуклеотида
ліцензійного іспиту Крок – 1 C. Місенс
«Стоматологія». – 2007. – №10. D. Вставка нуклеотида
E. Інверсія
71. Хвороба Хартнепа зумовлена точковою Test items for licensing examination Krok 1
мутацiєю лише одного гена, наслiдком чого є «Stomatology». – 2015. - № 99.
порушення всмоктування амiнокислоти
триптофану в кишечнику та реабсорбцiї її в 75. У клiтинi вiдбулася мутація в першому
ниркових канальцях. Це призводить до екзонi структурного гена. В ньому
одночасних розладiв у травнiй i зменшилась кiлькiсть пар нуклеотидiв –
сечовидiльнiй системах. Яке генетичне замість 290 пар стало 250. Визначте тип
явище спостерiгається в цьому випадку? мутацiї:
A. Плейотропiя A. Делецiя
B. Комплементарна взаємодiя B. Iнверсiя
C. Полiмерiя C. Дуплiкацiя
D. Кодомiнування D. Транслокацiя
E. Неповне домінування E. Нулiсомiя
Збірник тестових завдань для складання Збірник тестових завдань для складання
ліцензійного іспиту Крок -1 «Загальна ліцензійного іспиту Крок -1 «Загальна
лікарська підготовка». – 2008. – №185. лікарська підготовка». – 2018. – №90.

72. В ходi експерименту було 76. При обстеженнi 2-х мiсячної дитини
продемонстровано пiдвищення активностi β- педiатр звернула увагу, що плач дитини
галактозидази пiсля внесення лактози до нагадує котячий крик. Дiагностованi
культурального середовища з E.coli. Яка мiкроцефалiя i вада серця. За допомогою
12
цитогенетичного метода з’ясований карiотип Збірник тестових завдань для складання
дитини 46, XX, 5р-. Дане захворювання є ліцензійного іспиту Крок -1
наслiдком такого процесу: «Стоматологія». – 2017. – №119.
A. Делецiя
B. Дуплiкацiя 80. Юнак 15-ти рокiв скаржиться на загальну
C. Iнверсiя слабкiсть, запаморочення, швидку
D. Транслокацiя стомлюванiсть. При обстеженнi виявлено
E. Плейотропiя еритроцити змiненої форми, кiлькiсть їх
Збірник тестових завдань для складання знижена. Попереднiй дiагноз: серпоподiбно-
ліцензійного іспиту Крок -1 «Загальна клiтинна анемiя. Який тип мутацiї зумовлює
лікарська підготовка». – 2014. – №104. розвиток цього патологiчного стану?
A. Точкова мутацiя
77. При дослiдженнi карiотипу п’ятирiчного B. Мутацiя зсуву рамки зчитування
хлопчика виявлено 46 хромосом. Одна з C. Делецiя
хромосом 15-ої пари довша вiд звичайної, D. Iнверсiя
тому що до неї приєднана дiлянка E. Хромосомна аберація
хромосоми з 21-ої пари. Вкажiть вид мутацiї, Збірник тестових завдань для складання
що має мiсце в цього хлопчика: ліцензійного іспиту Крок -1 «Загальна
A. Транслокацiя лікарська підготовка». – 2017. – №176.
B. Дуплiкацiя
C. Делецiя 81. Внаслiдок впливу γ-випромiнювання
D. Iнверсiя дiлянка ланцюга ДНК повернулася на 180
E. Полiплоїдiя градусiв. Яка з перелiчених видiв мутацiй
Збірник тестових завдань для складання вiдбулася в ланцюзi ДНК?
ліцензійного іспиту Крок -1 A. Iнверсiя
«Стоматологія». – 2016. – №114. B. Делецiя
C. Дуплiкацiя
78. У районах Пiвденної Африки у людей D. Транслокацiя
розповсюджена серпоподiбно-клiтинна E. Реплiкацiя
анемiя, при якiй еритроцити мають форму Збірник тестових завдань для складання
серпа внаслiдок замiни в молекулi ліцензійного іспиту Крок -1 «Загальна
гемоглобiну амiнокислоти глутамiну на лікарська підготовка». – 2006. – №108.
валiн. Чим викликана ця хвороба?
A. Генна мутацiя 82. В експериментi було показано, що
B. Порушення механiзмiв реалiзацiї опромiненi ультрафiолетом клітини шкiри
генетичної iнформацiї хворих на пiгментну ксеродерму, через
C. Кросинговер дефект ферменту репарацiї, повiльнiше
D. Геномнi мутацiї вiдновлюють нативну структуру ДНК, нiж
E. Трансдукцiя клiтини здорових людей. За допомогою
Збірник тестових завдань для складання якого ферменту вiдбувається цей процес?
ліцензійного іспиту Крок -1 «Загальна A. Ендонуклеаза
лікарська підготовка». – 2007. – №15. B. РНК-лiгаза
C. Праймаза
79. Серпоподiбноклiтинна анемiя у людини D. ДНК-полiмераза III
супроводжується появою в кровi E. ДНК-гiраза
аномального гемоглобiну, змiною форми Збірник тестових завдань для складання
еритроцитiв, розвитком анемiї. Дане ліцензійного іспиту Крок -1 «Загальна
захворювання є результатом: лікарська підготовка». – 2006. – №10.
A. Генної мутацiї
B. Полiтенiї 83. У процесі еволюції виникли молекулярні
C. Хромосомної аберацiї механізми виправлення пошкоджених
D. Полiплоїдiї молекул ДНК. Цей процес називається:
E. Мiтохондрiальної мутацiї A. Репарація
B. Транскрипція
13
C. Трансляція 87. Хворi на пiгментну ксеродерму
D. Реплікація характеризуються аномально високою
E. Процессінг чутливiстю до ультрафiолетових променiв,
Test items for licensing examination Krok 1 результатом чого є рак шкіри внаслiдок
«Stomatology». – 2016. - № 116. нездатностi ферментних систем
вiдновлювати ушкодження спадкового
84. У клiтинах людини пiд дiєю апарату клiтин. З порушенням якого процесу
ультрафiолетового випромiнювання пов’язана ця патологiя?
вiдбулося пошкодження молекули ДНК. A. Репарацiя ДНК
Реалiзувалася система вiдновлення B. Генна конверсiя
пошкодженої дiлянки молекули ДНК по C. Рекомбiнацiя ДНК
непошкодженому ланцюгу за допомогою D. Генна комплементацiя
специфiчного ферменту. Як називається це E. Редуплiкацiя ДНК
явище? Збірник тестових завдань для складання
A. Репарацiя ліцензійного іспиту Крок -1
B. Дуплiкацiя «Стоматологія». – 2006. – №109.
C. Реплiкацiя
D. Iнiцiацiя 88. Робiтниця хiмiчного підприємства
E. Термiнацiя внаслiдок порушення правил безпечної
Збірник тестових завдань для складання роботи зазнала токсичної дiї азотистої
ліцензійного іспиту Крок -1 кислоти та нiтритiв, якi викликають
«Стоматологія». – 2005. – №183. дезамiнування цитозину в молекулi ДНК.
Який фермент iнiцiює ланцюг репарацiйних
85. У клітинах людини, що працює в процесiв?
Чорнобильській зоні відчуження, відбулася A. Уридин-ДНК-глiкозидаза
мутація в молекулі ДНК. Проте з часом B. Цитидинтрифосфатсинтетаза
пошкоджена ділянка молекули ДНК C. Оротидилмонофосфатдекарбоксилаза
відновилася до початкової структури за D. ДНК-залежна-РНК-полiмераза
допомогою специфічного ферменту. В цьому E. Тимiдилатсинтаза
випадку сталося наступне: Збірник тестових завдань для складання
A. Репарація ліцензійного іспиту Крок -1 «Загальна
B. Реплікація лікарська підготовка». – 2011. – №170.
C. Транскрипція
D. Реверс транскрипції 89. Вживання тетрациклiнiв в першiй
E. Трансляція половинi вагiтностi призводить до
Test items for licensing examination Krok 1 виникнення аномалiй органiв i систем плода,
«Medicine». – 2016. - № 117. в тому числi до гiпоплазiї зубiв, змiни їх
кольору. До якого виду мiнливостi належить
86. Під впливом фізичних факторів може захворювання дитини?
розвинутися дефект молекули ДНК. A. Модифiкацiйна
Наприклад, ультрафіолетове B. Комбiнативна
випромінювання може викликати розвиток C. Мутацiйна
димерів. Димери є два сусідніх D. Спадкова
піримідинових нуклеотиди, з'єднаних разом. E. Рекомбiнантна
Назвіть ці димери: Збірник тестових завдань для складання
А. Тимин і цитозин ліцензійного іспиту Крок -1
B. Аденін і тимін «Стоматологія». – 2008. – №4.
C. Гуанін і цитозин
D. Аденін і гуанін 90. Жiнка вживала антибiотики в першiй
Е. Гуанін і тимін половинi вагiтностi. Це призвело до
Test items for licensing examination Krok 1 гiпоплазiї зубiв i змiни їх кольору у дитини.
«Stomatology». – 2016. - № 135. Генотип не змiнився. Встановити вид
мiнливостi, яка лежить в основi
захворювання:
14
A. Модифiкацiйна реакції для ідентифікації особистості
B. Комбiнативна заснований на наступному:
C. Мутацiйна А. Генна ампліфікація
D. Спiввiдносна B. Делеция нуклеотидів
E. Рекомбiнативна C. Генетична рекомбінація
Збірник тестових завдань для складання D. Міссенс мутація
ліцензійного іспиту Крок -1 E. Трансдукція
«Стоматологія». – 2014. – №155. Test items for licensing examination Krok 1
«Medicine». – 2017. - № 188.
91. Вади розвитку плоду можуть спричинити
такi хвороби матерi як краснуха, сифiлiс, 95. До лікаря звернувся 30-річний чоловік.
токсоплазмоз, цитомегалiя, герпес, Існує ймовірність того, що пацієнт ВІЛ-
хламiдiоз. До якої форми мiнливостi інфікований. Для уточнення діагнозу лікар
вiдносять такi вади розвитку? запропонував провести полімеразну
A. Модифiкацiйна ланцюгову реакцію. В основі цього методу
B. Мутацiйна лежить:
C. Комбiнативна А. Генна ампліфікація
D. Геномного iмпринтингу Б. Транскрипція
E. Епiмутацiйна C. Генетична рекомбінація
Збірник тестових завдань для складання D. Геномна мутація
ліцензійного іспиту Крок -1 E. Хромосомна мутація
«Стоматологія». – 2018. – №149. Test items for licensing examination Krok 1
«Medicine». – 2015. - № 112.
92. У мешканцiв Закарпаття внаслiдок
дефiциту йоду в харчових продуктах часто 96. Відомо, що ген, який відповідає за
зустрiчається ендемiчний зоб. Який вид розвиток групи крові за системою AB0, має
мiнливостi спричиняє це захворювання? три аллельних стани. Появу в людині IV
A. Модифiкацiйна групи крові можна пояснити такою формою
B. Мутацiйна мінливості:
C. Комбiнативна А. Комбінативна
D. Онтогенетична Б. Мутаційна
E. Спiввiдносна C. Фенотипична
Збірник тестових завдань для складання D. Генокопія
ліцензійного іспиту Крок -1 Е. Фенокопія
«Стоматологія». – 2010. – №2. Test items for licensing examination Krok 1
«Medicine». – 2015. - № 48.
93. Лiмфоцит уражений ретровiрусом ВIЛ
(СНIД). В цьому випадку напрямок потоку 97. Мати під час вагітності вживала
iнформацiї в клiтинi буде: синтетичні гормони. У новонародженої
A. РНК → ДНК → i-РНК → полiпептид дівчинки спостерігалося надлишкове
B. ДНК → i-РНК → полiпептид →ДНК оволосіння, що мало зовнішню подібність до
C. ДНК → полiпептид → i-РНК андрогенітального синдрому. Як називається
D. i-РНК → полiпептид → ДНК такий прояв мінливості?
E. Полiпептид → РНК → ДНК →i-РНК А. Фенокопії
Збірник тестових завдань для складання B. Мутація
ліцензійного іспиту Крок -1 «Загальна C. Рекомбінация
лікарська підготовка». – 2014. – №71. Д. Гетерезис
Е. Реплікація
94. Молода сім'я прийшла на генетичне Test items for licensing examination Krok 1
консультування, щоб визначити батька своєї «Medicine». – 2014 - № 146.
дитини. Чоловік наполягає на тому, що
дитина зовсім не схожа на нього і не може 98. Лiкар виявив у дитини рахiт, зумовлений
бути його. Метод полімеразної ланцюгової нестачею вiтамiну D, але за своїм проявом
подiбний до спадкового
15
вiтамiностiйкого рахiту (викривлення C. Моносомiї
трубчастих кiсток, деформацiя суглобiв D. Трисомiї
нижнiх кiнцiвок, зубнi абсцеси). Як E. Геннi хвороби
називаються вади розвитку, якi нагадують Збірник тестових завдань для складання
спадковi, але не успадковуються? ліцензійного іспиту Крок -1
A. Фенокопiї «Стоматологія». – 2013. – №194.
B. Генокопiї

16
Біохімія міжклітинних комунікацій: гормони білково-пептидної
природи
1. Розпад глiкогену в печiнцi стимулюється E. Ca2+
глюкагоном. Який вторинний месенджер Test items for licensing examination Krok 1
(посередник) утворюється при цьому в «Medicine». – 2005. - № 57.
гепатоцитах?
A. ц-АМФ 5. Хворий з дiагнозом цукровий дiабет,
B. ц-ГМФ вранцi натще отримав призначену дозу
C. СО iнсулiну пролонгованої дiї. Пропустив
D. NO черговий прийом їжi i невдовзi вiдчув
E. Триацилглiцерол слабкiсть, бiль голови, запаморочення,
Збірник тестових завдань для складання пiтливiсть, тремтiння тiла, судоми, вiдчуття
ліцензійного іспиту Крок -1 голоду, явища гiпоглiкемiї. Застосування
«Стоматологія». – 2011. – №91. глюкози стан не полегшило. Який
гормональний препарат необхiдно ввести
2. В регуляцiї фiзiологiчних функцій беруть для купiрування даного стану?
участь iони металiв. Один iз них отримав A. Адреналiн
назву ”король месенджерiв”. Таким B. Трiамцiнолон
бiоелементом посередником є: C. Норадреналiн
A. Ca++ D. Пренiзолон
B. Na+ E. Гiдрокортизон
C. K+ Збірник тестових завдань для складання
D. Fe+++ ліцензійного іспиту Крок -1 «Загальна
E. Zn++ лікарська підготовка». -2017. -№43.
Збірник тестових завдань для складання
ліцензійного іспиту Крок -1 6. У хворого на цукровий діабет після
«Стоматологія». – 2017. – №109. iн’єкцiї iнсулiну стан погiршився: з’явився
неспокiй, холодний пiт, тремор кiнцiвок,
3. Iнозитолтрифосфати в тканинах органiзму загальна слабкiсть, запаморочення. За
утворюються в результатi гiдролiзу допомогою якого гормонального препарату
фосфатидилiнозитолдифосфатiв i вiдiграють знiмаються цi симптоми?
роль вторинних посередникiв (месенджерiв) A. Адреналiн
в механiзмi дії гормонiв. На що направлена B. Бутамiд
їх дiя у клiтинi? C. Кофеїн
A. Вивiльнення iонiв кальцiю з клiтинних D. Норадреналiн
депо E. Глiбутид
B. Активацiя аденiлатциклази Збірник тестових завдань для складання
C. Активацiя протеїнкiнази А ліцензійного іспиту Крок -1 «Загальна
D. Гальмування фосфодiестерази лікарська підготовка». -2018. -№78.
E. Гальмування протеїнкiнази С
Збірник тестових завдань для складання 7. У стоматологiчнiй практицi широко
ліцензійного іспиту Крок -1 використовується мiсцеве знеболювання,
«Стоматологія». – 2008. – №124. коли до розчину новокаїну додають 0,1%
розчин адреналiну. Адреналiн, що додається,
4. Формування вторинного медіатора є викликає:
обов'язковим у мембранно- A. Мiсцеве звуження судин
внутрішньоклітинному механізмі дії B. Мiсцеве розширення судин
гормонів. Вкажіть речовину, яка не може C. Зниження артерiального тиску
бути вторинним посередником: D. Зниження опору судин
A. Гліцерин E. Пiдвищення артерiального тиску
B. Диацилглицерин Збірник тестових завдань для складання
C. Інозитол-3,4,5-трифосфат ліцензійного іспиту Крок -1
D. цAMФ «Стоматологія». – 2011. – №28.

17
12. У хворого на цукровий дiабет після
8. 50-річний чоловік відмовився від анестезії iн’єкцiї iнсулiну настала втрата свiдомостi,
під час стоматологічних маніпуляцій. Через судоми. Який результат дав бiохiмiчний
сильний біль розвинулася анурія, що аналiз кровi на вмiст глюкози?
виникла через різке збільшення секреції: A. 2,5 ммоль/л
А. Адреналін B. 3,3 ммоль/л
B. Ренін C. 8,0 ммоль/л
C. Тимозин D. 10 ммоль/л
D. Тироксин E. 5,5 ммоль/л
Е. Глюкагон Збірник тестових завдань для складання
Test items for licensing examination Krok 1 ліцензійного іспиту Крок -1
«Stomatology». – 2018. - № 116. «Стоматологія». – 2007– №90.

9. У хворої жінки після парентерального 13. До шпиталю було доставлено юнака 16-
введення гормону відбулося підвищення ти рокiв, хворого на iнсулiнозалежний
артеріального тиску, а також підвищилися цукровий дiабет. Рiвень глюкози у кровi
рівні глюкози і ліпідів в крові. Який гормон пацiєнта складав 18 ммоль/л. Хворому було
було введено? введено iнсулiн. Двi години потому рiвень
A. Адреналін глюкози зменшився до 8,2 ммоль/л, тому що
B. Глюкагон iнсулiн:
C. Інсулін A. Стимулює перетворення глюкози в
D. Прогестерон печiнцi у глiкоген та ТАГ
E. Фолікулін B. Стимулює транспорт глюкози через
Сборник тестовых заданий для плазматичнi мембрани в головному мозку та
лицензионного экзамена Крок-1 «Медицина». печiнцi
– 2006. – № 69. C. Гальмує синтез кетонових тiл iз глюкози
D. Стимулює розщеплення глiкогену в
10. Пацієнт, який страждає печiнцi
феохромоцитомою, скаржиться на спрагу, E. Стимулює розщеплення глiкогену у
сухість у роті, голод. Аналіз крові на цукор м’язах
виявив гіперглікемію. Який тип гіперглікемії Збірник тестових завдань для складання
це? ліцензійного іспиту Крок -1 «Загальна
А. Надниркова лікарська підготовка». -2014. -№191.
B. Гіперкортикоидна
C. Аліментарна 14. Хворому на iнсулiнзалежний цукровий
Д. Соматотропна дiабет було введено iнсулiн. Через деякий
E. Гіпоінсулінемічна час у хворого з’явилися слабкiсть,
Test items for licensing examination Krok 1 дратiвливiсть, посилення потовидiлення.
«Medicine». – 2010. - № 120. Який основний патогенетичний механiзм
розвитку гiпоглiкемiчної коми, що виникла?
11. У хворого на цукровий дiабет після A. Вуглеводне голодування головного мозку
введення iнсулiну настала непритомнiсть, B. Посилення глiкогенолiзу
спостерiгаються судоми. Який результат C. Посилення кетогенезу
бiохiмiчного аналiзу кровi на вмiст цукру? D. Посилення лiпогенезу
A. 1,5 ммоль/л E. Зменшення глюконеогенезу
B. 3,3 ммоль/л Збірник тестових завдань для складання
C. 8 ммоль/л ліцензійного іспиту Крок -1
D. 10 ммоль/л «Стоматологія». – 2005. – №179.
E. 5,5 ммоль/л
Збірник тестових завдань для складання 15. Перш ніж клітини можуть
ліцензійного іспиту Крок -1 «Загальна використовувати глюкозу, вона спочатку
лікарська підготовка». -2006. -№125. транспортується з позаклітинного простору
через плазматичну мембрану всередину

18
клітини. Цей процес стимулюється D. Тироксином
наступним гормоном: E. Кортизолом
А. Інсулін Test items for licensing examination Krok 1
Б. Глюкагон «Stomatology». – 2017. - № 66.
C. Тироксин
Д. Альдостерон 20. При оглядi пацiєнта виявлене надмiрне
Е. Адреналін розростання кiсток i м’яких тканин обличчя,
Test items for licensing examination Krok 1 збiльшенi розмiри язика, розширенi мiжзубнi
«Medicine». – 2012. - № 42. проміжки в збiльшенiй зубнiй дузi. Якi змiни
секрецiї гормонiв у нього найбiльш
16. Після ін'єкції інсуліну рівень глюкози вiрогiднi?
знижується протягом декількох секунд. Це A. Збiльшена секрецiя соматотропного
відбувається завдяки активації такого гормону
процесу: B. Зменшена секрецiя соматотропного
A. Транспорт глюкози в клітини гормону
B. Гліколіз C. Збiльшена секрецiя iнсулiну
C. Синтез глікогену D. Зменшена секрецiя тироксину
D. Синтез ліпідів E. Зменшена секрецiя iнсулiну
E. Пентозофосфатний цикл Збірник тестових завдань для складання
Сборник тестовых заданий для ліцензійного іспиту Крок -1 «Загальна
лицензионного экзамена Крок-1 «Медицина». лікарська підготовка». -2009. -№124.
– 2012. – № 138.
21. У хворого 49-ти років виявлено
17. Перед утилізацією глюкози в клітинах непропорцiйне збільшення кистей рук, стоп,
вона транспортується всередину клітин з носа, вух, надбрівних дуг i виличних кiсток.
позаклітинного простору через плазматичну У кровi - гiперглiкемiя, порушення тесту
мембрану. Цей процес стимулюється толерантності до глюкози. Причиною
наступним гормоном: розвитку даної патологiї найбiльш імовірно
А. Інсулін є:
Б. Глюкагон A. Гiперсекрецiя соматотропного гормону
C. Тироксин B. Гiперсекрецiя гормонів нейрогiпофiзу
Д. Альдостерон C. Гiпосекрецiя iнсулiну
Е. Адреналін D. Гiпосекрецiя вазопресину
Test items for licensing examination Krok 1 E. Гiперсекрецiяглюкокортикоїдiв
«Medicine». – 2015. - № 19. Збірник тестових завдань для складання
ліцензійного іспиту Крок -1
18. Пацієнт перебуває в стані гіпоглікемічної «Стоматологія». – 2018. – №33.
коми. Який гормон може викликати це стан
при передозуванні? 22. У жiнки пiсля пологiв зменшилася маса
А. Інсулін тiла на 20 кг, випадають зуби та волосся,
B. Прогестерон спостерiгається атрофiя м’язiв (гiпофiзарна
C. Кортизол кахексiя). З порушенням синтезу якого
D. Соматотропін гормону гiпофiза це пов’язано?
E. Кортикотропін A. Соматотропний
Test items for licensing examination Krok 1 B. Кортикотропний
«Stomatology». – 2017. - № 22. C. Тиреотропний
D. Гонадотропний
19. У хворого під час голодування E. Пролактин
розвинувся кетоацидоз внаслідок посилення Збірник тестових завдань для складання
розкладання жирних кислот. Цей розпад ліцензійного іспиту Крок -1
може бути загальмовано: «Стоматологія». – 2006. – №118.
А. Інсуліном
Б. Глюкагоном 23. Зріст дорослої людини 100 см при
C. Адреналіном пропорційній будові тіла і нормальному
19
розумовому розвитку. Недостатня продукція Збірник тестових завдань для складання
якого гормону в дитячому віці є причиною ліцензійного іспиту Крок -1 «Загальна
цього? лікарська підготовка». -2016. -№124.
A. Соматотропний
B. Гонадотропні 27. У результатi побутової травми у пацiєнта
C. Адренокортикотропний виникла значна крововтрата, що
D. тиреотропного супроводжувалося зниженням артерiального
E. Пролактин тиску. Дiя яких гормонів забезпечує швидке
Сборник тестовых заданий для вiдновлення кров’яного тиску, викликаного
лицензионного экзамена Крок-1 крововтратою?
«Стоматология». – 2011. – № 41. A. Адреналiн, вазопресин
B. Кортизол
24. Обстеження хворого виявлено C. Статевi
розширення деяких частин тіла (щелепи, ніс, D. Окситоцин
вуха, ноги, руки), але пропорції тіла були E. Альдостерон
збережені. Це може бути викликано шляхом Збірник тестових завдань для складання
інтенсифікації секреції наступного гормон: ліцензійного іспиту Крок -1 «Загальна
A. Соматотропін лікарська підготовка». -2013. -№190.
B. Соматостатин
C. Тетрайодотіронін 28. Тяжка травма у 36-річного пацієнта
D. Трийодтиронін призвела до значної втрати крові, що
E. Кортизол супроводжувалося падінням артеріального
Test items for licensing examination Krok 1 тиску. Які гормони забезпечують швидке
«Stomatology». – 2008. - № 40. відновлення артеріального тиску після
крововтрати?
25. У студента, який складає iспит, вмiст А. Адреналін, вазопресин
глюкози у плазмi кровi складає 8 ммоль/л. B. Кортизол
Збiльшена секрецiя якого з наведених C. Статеві гормони
гормонiв сприяє розвитку гiперглiкемiї у D. Окситоцин
студента? Е. Альдостерон
A. Глюкагон Test items for licensing examination Krok 1
B. Iнсулiн «Stomatology». – 2014. - № 180.
C. Тироксин
D. Трийодтиронiн 29. У людини збiльшений об’єм
E. Альдостерон циркулюючої кровi та зменшений
Збірник тестових завдань для складання осмотичний тиск плазми. Це
ліцензійного іспиту Крок -1 «Загальна супроводжується збiльшенням дiурезу, перш
лікарська підготовка». -2012. -№111. за все, внаслiдок зменшеної секрецiї такого
гормону:
26. Стресовий стан i больове вiдчуття у A. Вазопресин
пацієнта перед вiзитом до стоматолога B. Альдостерон
супроводжуються анурією (вiдсутнiстю C. Адреналiн
сечовидiлення). Це явище зумовлене D. Ренiн
збiльшенням: E. Натрiйуретичний
A. Секрецiї вазопресину та адреналiну Збірник тестових завдань для складання
B. Активностi парасимпатичної нервової ліцензійного іспиту Крок -1
системи «Стоматологія». – 2014. – №106.
C. Активностi антиноцiцептивної системи
D. Секрецiї вазопресину та зменшенням 30. Хворий 20 років скаржиться на
адреналiну виснажливу спрагу та рясне сечовиділення
E. Секрецiї адреналiну та зменшенням до 10 л на добу. Рівень глюкози в крові
вазопресину нормальний, глюкози в сечі немає. Який
гормональний дефіцит може викликати такі
зміни?
20
A. Вазопресин D. Тиреокальцитонiн
B. Окситоцин E. Глюкагон
C. Інсулін Збірник тестових завдань для складання
D. Трийодтиронін ліцензійного іспиту Крок -1 «Загальна
E. Кортизол лікарська підготовка». -2009. -№130.
Test items for licensing examination Krok 1
«Stomatology». – 2006. - № 65. 35. Жінка 25-ти років через місяць після
пологів звернулась до лікаря зі скаргою на
31. Пацієнт з пухлиною гіпофіза скаржиться зниження утворення молока. Дефіцит якого
на підвищений добовий діурез (поліурія). гормону призвів до такого стану?
Концентрація глюкози в плазмі крові A. Пролактин
дорівнює 4,8 ммоль / л. Який гормон може B. Соматостатин
бути причиною цього, якщо його секреція C. Адренокортикотропний гормон
порушена? D. Інсулін
A. Вазопресин E. Глюкагон
Б. Альдостерон Сборник тестовых заданий для
C. Натрійуретичний гормон лицензионного экзамена Крок-1 «Медицина».
D. Інсулін – 2012. – № 53.
E. Ангіотензин I
Test items for licensing examination Krok 1 36. Продуктами гiдролiзу та модифiкацiї
«Stomatology». – 2015. - № 133. деяких бiлкiв є бiологiчно активнi речовини -
гормони. З якого iз наведених бiлкiв у
32. Обмеження споживання води призвело гiпофiзi утворюються лiпотропiн,
до зневоднення органiзму. Який механізм кортикотропiн, меланотропiн та ендорфiни?
активується за цих умов для збереження A. Проопiомеланокортин (ПОМК)
води в органiзмi? B. Нейроальбумiн
A. Збiльшення секреції вазопресину C. Нейростромiн
B. Збiльшення секреції соматостатину D. Нейроглобулiн
C. Зменшеннясекрецiї вазопресину E. Тиреоглобулiн
D. Збiльшення секреції альдостерону Збірник тестових завдань для складання
E. Зменшеннясекрецiї альдостерону ліцензійного іспиту Крок -1 «Загальна
Збірник тестових завдань для складання лікарська підготовка». -2009. -№72.
ліцензійного іспиту Крок -1
«Стоматологія». – 2016. – №180. 37. Пiсля перенесеного сепсису у хворої 27-
ми рокiв з’явився бронзовий колiр шкiри,
33. У тварини була сильна втрата крові, що характерний для аддiсонової хвороби.
призвело до зниження реабсорбції Na+ і води Механiзм гiперпiгментацiї полягає в
в ниркових канальцях. Це можна пояснити пiдвищеннi секрецiї такого гормону:
впливом наступних факторів на нирки: A. Меланоцитстимулюючий
А. Натрійуретичний гормон B. Соматотропний
Б. Альдостерон C. Гонадотропний
С. Ренин D. B-лiпотропний
D. Ангіотензин E. Тиреотропний
Е. Вазопрессин Збірник тестових завдань для складання
Test items for licensing examination Krok 1 ліцензійного іспиту Крок -1 «Загальна
«Medicine». – 2018. - № 81 лікарська підготовка». -2018. -№3.
38. Хвора 20-ти років звернулася до лiкаря зi
34. До лiкаря звернулася жiнка 32-х рокiв зi скаргами на загальне схуднення, зниження
скаргами на вiдсутнiсть лактацiї пiсля апетиту, слабкiсть, появу незвичайного
народження дитини. Дефiцитом якого кольору шкiри, що нагадує пiвденну
гормону можна пояснити дане порушення? ”бронзову засмагу”. При обстеженнi у
A. Пролактин клiнiцi, окрiм гiперпiгментацiї, виявлений
B. Соматотропiн двобічний туберкульоз наднирникiв.
C. Вазопресин
21
Надлишкове накопичення якої речовини Шийка матки розкрита, але скорочення
зумовило гiперпiгментацiю шкiри? матки вiдсутнє. Лiкар дав засiб гормональної
A. Меланiн природи для посилення пологової дiяльностi.
B. Бiлiрубiн Назвiть гормональний засiб:
C. Гемомеланiн A. Окситоцин
D. Лiпофусцин B. Гiдрокортизон
E. Адренохром C. Естрон
Збірник тестових завдань для складання D. Тестостерон
ліцензійного іспиту Крок -1 E. АКТГ
«Стоматологія». – 2018. – №129. Збірник тестових завдань для складання
ліцензійного іспиту Крок -1 «Загальна
39. Хворому з гіперсекрецією шлункового лікарська підготовка». -2013. -№169.
соку лiкар рекомендував виключити з дiєти 43. Породіллі для стимуляції родової
насиченi бульйони i овочевi вiдвари, тому діяльності призначений гормон задньої долі
що вони стимулюють шлункову секрецiю гіпофіза, чутливість до якого зростає з
переважно через активацiю: терміном вагітності, що не впливає на
A. Вироблення гастрину артеріальний тиск. Визначте препарат:
B. Смакових рецепторiв A. Окситоцин
C. Механорецепторiв ротової порожнини B. Динопростон
D. Механорецепторiв шлунка C. Динопрост
E. Вироблення секретину D. Питуїтрин
Збірник тестових завдань для складання E. Ерготал
ліцензійного іспиту Крок -1 «Загальна Test items for licensing examination Krok 1
лікарська підготовка». -2011. -№3. «Medicine». – 2006. - № 165.

40. В гострому дослiдi тваринi в порожнину 44. 40-річний хворий скаржиться на


12-ти палої кишки ввели слабкий розчин інтенсивне серцебиття, пітливість, нудоту,
хлористоводневої кислоти. До збiльшення зорові порушення, тремор рук, гіпертонію. З
секрецiї якого гастроiнтестинального його анамнезу: 2 роки тому було
гормону це призведе? діагностовано феохромоцитому.
A. Секретин Гіперпродукція яких гормонів викликає дану
B. Гастрин патологію?
C. Мотилiн А. Катехоламіни
D. Нейротензин Б. Альдостерон
E. Гiстамiн C. Глюкокортикоїди
Збірник тестових завдань для складання D. АКТГ
ліцензійного іспиту Крок -1 E. Тиреоїдні гормони
«Стоматологія». – 2011. – №29. Test items for licensing examination Krok 1
«Medicine». – 2012. - № 60.
41. До пологового вiддiлення надiйшла
вагiтна жiнка зi слабкою пологовою 45. Спецiальний режим харчування призвiв
дiяльнiстю. Призначте засiб гормональної до зменшення iонiв Ca2+ в кровi. До
природи для посилення пологової дiяльностi: збiльшення секрецiї якого гормону це
A. Окситоцин призведе?
B. Прогестерон A. Паратгормон
C. Метандростенолон B. Тирокальцитонiн
D. Гiдрокортизон C. Вазопресин
E. АКТГ D. Соматотропiн
Збірник тестових завдань для складання E. Тироксин
ліцензійного іспиту Крок -1 Збірник тестових завдань для складання
«Стоматологія». – 2006. – №88. ліцензійного іспиту Крок -1 «Загальна
лікарська підготовка». -2011. -№185.
42. Жiнка 26-ти рокiв поступила у пологове
вiддiлення в термiнi вагітності 40 тижнiв.
22
46. У хворого рiзко знизився вмiст Ca2+ у пошкоджена паращитовидна залоза. Через
кровi. До збiльшення секрецiї якого гормону місяць після операції у пацієнтки з'явилися
це призведе? ознаки гипопаратиреоза: часті судоми,
A. Паратгормон гіперрефлексія, спазм гортані. Що є
B. Тирокальцитонiн найбільш вірогідною причиною стану жінки?
C. Альдостерон A. Гіпокальциемія
D. Вазопресин B. Гіпонатриемия
E. Соматотропний C. Гіперхлоргідрія
Збірник тестових завдань для складання D. Гіпофосфатемія
ліцензійного іспиту Крок -1 E. Гіперкаліємія
«Стоматологія». – 2006. – №76. Test items for licensing examination Krok 1
«Medicine». – 2017. - № 42.
47. У людини збiльшений вмiст іонів
кальцiю в плазмi кровi, зменшений – у 51. У дитини наявне порушення формування
кiстках. Надмiрна секрецiя якого гормону емалi та дентину зубiв через знижений вмiст
може спричинити такi змiни? iонiв кальцiю в кровi. Дефiцит якого гормону
A. Паратгормон може спричинити такi порушення?
B. Тироксин A. Паратгормон
C. Трийодтиронiн B. Тирокальцитонiн
D. Тиреокальцитонiн C. Тироксин
E. Альдостерон D. Соматотропний гормон
Збірник тестових завдань для складання E. Трийодтиронiн
ліцензійного іспиту Крок -1 «Загальна Збірник тестових завдань для складання
лікарська підготовка». -2012. -№110. ліцензійного іспиту Крок -1
«Стоматологія». – 2010. – №37.
48. У нирках дослiджуваного збiльшена
реабсорбцiя iонiв кальцiю i зменшена - 52. Внаслiдок вираженого зниження
фосфатних iонiв. Впливом якого гормону це концентрацiї кальцiю в плазмi кровi у
зумовлено? дитини 2-х рокiв виникли тетанiчнi
A. Паратгормон скорочення дихальних i глоткових м’язiв.
B. Тирокальцитонiн Зниження секрецiї якого гормону може бути
C. Гормональна форма вiтамiну D3 причиною цього?
D. Альдостерон A. Паратгормон
E. Вазопресин B. Тиреокальцитонiн
Збірник тестових завдань для складання C. Альдостерон
ліцензійного іспиту Крок -1 «Загальна D. Соматотропiн
лікарська підготовка». -2006. -№170. E. Кортизол
Збірник тестових завдань для складання
49. У жінки 47-ми років після операції на ліцензійного іспиту Крок -1 «Загальна
щитоподібній залозі незабаром з'явилися лікарська підготовка». -2011. -№138.
фібрилярні посмикування м'язів рук, ніг,
обличчя. Ці порушення можна усунути 53. У дитини 2-х років виникли судоми
шляхом введення такого гормону: внаслідок зниження концентрації іонів
A. Паратгормон кальцію в плазмі крові. Функція якого
B. Трийодтиронин ендокринного органу знижена?
C. Тиреотропін A. Прищитоподібні залози
D. Тироксин B. Гіпофіз
E. Тиреотропний гормон C. Кора наднирників
Сборник тестовых заданий для D. Шишковидна заліза
лицензионного экзамена Крок-1 E. Тимус
«Стоматология». – 2014. – № 67. Test items for licensing examination Krok 1
«Medicine». – 2005. - № 17.
50. У 47-річної жінки при видаленні
щитовидної залози що гіпоплазована була
23
54. У хворого спостерігається гіпокальціємія. Збірник тестових завдань для складання
Дефіцит якого гормону може бути причиною ліцензійного іспиту Крок -1
такого стану? «Стоматологія». – 2011. – №175.
А. Паратормон
B. Тирокальцитонін 58. Періодичні напади ниркових колік
С. Альдостерон спостерігаються у жінки з первинним
D. Кортикотропін гіперпаратиреоїзмом. Ультразвукове
E. Кортиколіберин дослідження виявило дрібні камені в нирках.
Test items for licensing examination Krok 1 Що є найбільш вірогідною причиною
«Stomatology». – 2007. - № 88. утворення каменів?
А. Гіперкальціємія
55. У хворого спостерiгається остеопороз B. Гіперфосфатемія
кiсток, в кровi - гiперкальцiємiя, C. Гіперхолестеринемія
гiпофосфатемiя. Яка причина такого стану? D. Гіперурикемія
A. Посилена секреція паратгормону E. Гіперкаліємія
B. Посилена секреція тироксину Test items for licensing examination Krok 1
C. Пригнiчення секреції паратгормону «Medicine». – 2005. - № 177.
D. Посилена секреція кортикостероїдiв
E.Пригнiчення секреції кортикостероїдiв 59. У чоловіка 56-ти років виявлена пухлина
Збірник тестових завдань для складання паращитовидной залози. Спостерігаються
ліцензійного іспиту Крок -1 м'язова слабкість, остеопороз, деформація
«Стоматологія». – 2018. – №65. кісток, утворення ниркових каменів, що
складаються з оксалатів і фосфатів.
56. Хлопчик 5-ти мiсяцiв госпiталiзований з Причиною такого стану є:
приводу тонiчних судом. Хворiє з A. Збільшення секреції паратгормону
народження. Об’єктивно: волосся жорстке, B. Зменшення секреції паратгормону
нiгтi витонченi та ламкi, шкiрнi покриви C. Збільшення секреції кальцитоніну
блiдi та сухi. В бiохiмiчному аналiзi кровi: D. Зменшення секреції кальцитриолу
кальцiй - 0,5 ммоль/л (норма - 0,75-2,5 E. Збільшення секреції тироксину
ммоль/л), фосфор - 1,9 ммоль/л (норма - Сборник тестовых заданий для
0,646-1,292 ммоль/л). З чим пов’язанi цi лицензионного экзамена Крок-1 «Медицина».
змiни? – 2017. – № 113.
A. Гiпопаратиреоз
B. Гiперпаратиреоз 60. Внаслідок травмування у хворого
C. Гiперальдостеронiзм видалили прищитоподібні залози, що
D. Гiпоальдостеронiзм супроводжувалося млявістю, спрагою,
E. Гiпотиреоз різким підвищенням нервово-м'язової
Збірник тестових завдань для складання збудливості. З порушенням обміну якої
ліцензійного іспиту Крок -1 «Загальна речовини це пов'язано?
лікарська підготовка». -2010. -№192. A. Кальцій
B. Марганець
57. Хлопчик 5-ти мiсяцiв госпiталiзований з C. Хлор
приводу тонiчних судом. Хворiє з D. Молібден
народження. При оглядi волосся жорстке, E. Цинк
нiгтi витонченi та ламкi, шкiрнi покриви Сборник тестовых заданий для
блiдi та сухi. У кровi знижений вмiст iонiв лицензионного экзамена Крок-1 «Медицина».
кальцiю. З чим пов’язанi цi змiни? – 2017. – № 65.
A. Гiпопаратиреоз
B. Гiперпаратиреоз 61. В ендокринологічному відділенні
C. Гiперальдостеронiзм знаходиться хлопчик 9 років, у якого вже
D. Гiпоальдостеронiзм кілька разів були переломи кінцівок,
E. Гiпотиреоз пов'язані з крихкістю кісток. Функція яких
ендокринних залоз (залози) порушена?
A. Прищитоподібні
24
B. Щитоподібна B. Тироксин
C. Тимус C. Трийодтиронін
D. Надниркові D. Тиреотропний гормон
E. Епіфіз E. Соматотропін
Test items for licensing examination Krok 1 Test items for licensing examination Krok 1
«Stomatology». – 2007. - № 17. «Stomatology». – 2015. - № 194.

62. Хворий з ознаками остеопорозу та 66. Чоловiку 46-ти рокiв, що хворiє на


сечокам'яної хвороби поступив у дифузний токсичний зоб, була проведена
ендокринологічне відділення. Аналіз крові операцiя резекцiї щитоподiбної залози. Пiсля
виявив гіперкальціємію і гіпофосфатемію. Ці операцiї вiдмiчаються вiдсутнiсть апетиту,
зміни пов'язані з порушенням синтезом диспепсiя, пiдвищена нервово-м’язова
наступного гормону: збудливiсть. Маса тiла не збiльшилася.
A. Паратиреоїдний гормон Температура тiла у нормi. Чим, iз нижче
В. Кальцитонін перелiченого, обумовлений стан хворого?
C. Кортизол A. Зниженням продукцiї паратгормону
Д. Алдостерон B. Зниженням продукцiї тироксину
E. Кальцитріол C. Пiдвищенням продукцiї кальцитонiну
Test items for licensing examination Krok 1 D. Пiдвищенням продукцiї тиреолiберину
«Medicine». – 2013. - № 180. E. Пiдвищенням продукцiї тироксину
Збірник тестових завдань для складання
63. Основні симптоми первинного ліцензійного іспиту Крок -1 «Загальна
гіперпаратиреозу - остеопороз і ураження лікарська підготовка». -2009. -№51.
нирокразом з розвитком сечокамʼяної
хвороби. Які речовини складають основу цих 67. При операцiї на щитоподiбнiй залозi з
каменів при даному захворюванні? приводу захворювання на Базедову хворобу
A. Фосфат кальцію помилково були видаленi паращитоподiбнi
B. Сечова кислота залози. Виникли судоми, тетанiя. Обмiн
C. Цистин якого бiоелемента було порушено?
Д. Білірубін A. Кальцiй
E. Холестерин B. Магнiй
Test items for licensing examination Krok 1 C. Калiй
«Medicine». – 2010. - № 172. D. Залiзо
E. Натрiй
64. При лікуванні пародонтиту Збірник тестових завдань для складання
використовують препарати кальцію і гормон, ліцензійного іспиту Крок -1
що має здатність стимулювати мінералізацію «Стоматологія». – 2017. – №24.
зубів і гальмувати резорбцію кісткової
тканини, а саме: 68. Солдати, які отримали поранення в
A. Кальцитонін розпал битви, можуть не відчувати біль до її
B. Паратгормон завершення. Які гормони опіатної
C. Адреналін антиноцицептивної системи зменшують
D. Альдостерон відчуття болю?
E. Тироксин A. Ендорфіни
Test items for licensing examination Krok 1 B. Серотонін
«Stomatology». – 2010. - № 51. C. Вазопресин
D. Альдостерон
65. Пацієнт з гіпопаратиреозом має E. Окситоцин
множинні каріозні ураження зубів. Ця Сборник тестовых заданий для
патологія обумовлена недостатністю лицензионного экзамена Крок-1
наступного гормону: «Стоматология». – 2017. – № 106.
А. Кальцитонін

25
Біохімія міжклітинних комунікацій: гормони стероїдної та
тіреоїдної природи
1. У дiвчинки дiагностований Збірник тестових завдань для складання
адреногенiтальний синдром ліцензійного іспиту Крок -1 «Загальна
(псевдогермафродитизм). Надмiрна секрецiя лікарська підготовка». -2005. -№83.
якого гормону наднирникiв обумовила дану
патологiю? 5. 30-річна жінка має ознаки вірилізму
A. Андроген (зростання волосся на тілі, облисіння,
B. Естроген порушення менструального циклу). Це умова
C. Альдостерон може бути викликано перевиробництвом
D. Кортизол наступного гормону:
E. Адреналiн A. Тестостерон
Збірник тестових завдань для складання B. Осетріол
ліцензійного іспиту Крок -1 «Загальна С. Релаксин
лікарська підготовка». -2008. -№58. D. Окситоцин
E. Пролактин
2. Секрецiя яких гормонiв гiпофiзу Test items for licensing examination Krok 1
гальмується пiсля прийому оральних «Medicine». – 2013. - № 182.
контрацептивiв, якi мiстять статевi гормони?
A. Гонадотропнi 6. До лiкаря звернулися батьки хлопчика 10-
B. Вазопресин ти рокiв, у якого вiдзначалося збiльшення
C. Тиреотропнi волосяного покриву на тiлi, рiст бороди i вус,
D. Соматотропний низький голос. Збiльшення секрецiї якого
E. Окситоцин гормону можна припустити?
Збірник тестових завдань для складання A. Тестостерон
ліцензійного іспиту Крок -1 «Загальна B. Соматотропiн
лікарська підготовка». -2010. -№59. C. Естроген
D. Прогестерон
3. У жінки виникла загроза передчасного E. Кортизол
переривання вагітності. Це, найімовірніше, Збірник тестових завдань для складання
пов'язано з недостатньою секрецією такого ліцензійного іспиту Крок -1 «Загальна
гормону: лікарська підготовка». -2007. -№63.
A. Прогестерон
B. Естрадіол 7. У хворої внаслiдок запалення порушена
C. Окситоцин ендокринна функцiя фолiкулярних клiтин
D. Тестостерон фолiкулiв яєчника. Синтез яких гормонiв
E. Альдостерон буде пригнiчений?
Сборник тестовых заданий для A. Естрогени
лицензионного экзамена Крок-1 «Медицина». B. Прогестерон
- 2011. - № 200. C. Лютропiн
D. Фолiкулостимулюючий гормон
4. Тестостерон та його аналоги збiльшують E. Фолiстатин
масу скелетних м’язiв, що дозволяє Збірник тестових завдань для складання
використовувати їх для лiкування дистрофiй. ліцензійного іспиту Крок -1 «Загальна
Взаємодiєю з яким клiтинним субстратом лікарська підготовка». -2016. -№95.
зумовлена ця дiя?
A. Ядернi рецептори 8. Прийом пероральних контрацептивів, що
B. Мембраннi рецептори містять статеві гормони, пригнічує секрецію
C. Рибосоми гормонів гіпофізу. Секреція якого із
D. Хроматин зазначених гормонів гальмується при
E. Бiлки-активатори транскрипції прийомі оральних контрацептивів із
статевими гормонами?
А. Фолікулостимулюючий

26
B. Вазопресин B. Звикання до препарату
C. Тиротропний C. Сенсибiлiзацiя
Д. Соматотропний D. Гiперпродукцiя кортикотропного гормону
Е. Окситоцин E. Кумуляцiя
Test items for licensing examination Krok 1 Збірник тестових завдань для складання
«Medicine». – 2006. - № 20. ліцензійного іспиту Крок -1
«Стоматологія». – 2005. – №94.
9. Кортикостероїдні гормони регулюють
адаптаційні процеси організму вцілому до 13. При хронiчному передозуваннi
екологічних змін і забезпечують підтримку глюкокортикоїдiв у хворого розвивається
внутрішнього гомеостазу. Який гормон гiперглiкемiя. Який процес вуглеводного
активує гіпоталамо-залежні надниркові обмiну зумовлює це?
залози? A. Глюконеогенез
A. Кортиколіберин B. Глiкогенолiз
Б. Соматоліберин C. Аеробний глiколiз
C. Соматостатин D. Пентозофосфатний цикл
Д. Кортикостатин E. Глiкогенез
E. Тіреоліберин Збірник тестових завдань для складання
Test items for licensing examination Krok 1 ліцензійного іспиту Крок -1
«Medicine». – 2017. - № 175. «Стоматологія». – 2006. – №108.

10. Людина за призначенням лiкаря тривалий 14. У жiнки 40-ка років хвороба Iценка-
час приймала препарат з групи Кушинга – стероїдний дiабет. При
глюкокортикоїдних гормонiв. Секрецiя якого бiохiмiчному обстеженнi: гiперглiкемiя,
(яких) з наведених гормонiв буде пригнiчена гiпохлоремiя. Який з перерахованих нижче
внаслiдок цього? процесів активується в першу чергу у таких
A. Кортикотропний хворих?
B. Соматотропний A. Глюконеогенез
C. Тиротропний B. Глiкогенолiз
D. Статевi C. Реабсорбцiя глюкози
E. Мiнералокортикоїди D. Транспорт глюкози в клiтину
Збірник тестових завдань для складання E. Глiколiз
ліцензійного іспиту Крок -1 «Загальна Збірник тестових завдань для складання
лікарська підготовка». -2005. -№197. ліцензійного іспиту Крок -1 «Загальна
лікарська підготовка». -2018. -№103.
11. Видiлення гормонiв кори наднирникiв
регулюється АКТГ аденогiпофiзу. Якi 15. У пацiєнта 28-ми рокiв iз синдромом
гормони видiляються наднирниками при дiї Iценка-Кушинга виявленi гiперглiкемiя,
останнього? глюкозурiя. Основним механiзмом
A. Глюкокортикоїди гiперглiкемiї у даного хворого є стимуляцiя:
B. Андрогени A. Глюконеогенезу
C. Катехоламiни B. Глiкогенолiзу в печiнцi
D. Мiнералокортикоїди C. Глiкогенолiзу в м’язах
E. Простагландини D. Всмоктування глюкози в кишечнику
Збірник тестових завдань для складання E. Синтезу глiкогену
ліцензійного іспиту Крок -1 Збірник тестових завдань для складання
«Стоматологія». – 2014. – №101. ліцензійного іспиту Крок -1
«Стоматологія». – 2014. – №115.
12. У хворого, який протягом тривалого часу
приймав глюкокортикоїди, в результатi 16. У хворого з синдромом Iценко-Кушiнга
вiдмiни препарату виникло загострення спостерiгаються стiйка гiперглiкемiя та
хвороби, зниження артерiального тиску, глюкозурiя, гiпертензiя, остеопороз,
слабкiсть. З чим можна пов’язати цi явища? ожирiння. Синтез та секрецiя якого гормону
A. Недостатнiсть кори наднирникiв збiльшується в даному випадку?
27
A. Кортизол збiльшення маси тiла. Об’єктивно: обличчя
B. Адреналiн мiсяцеподiбне, гiрсутизм, АТ- 165/100 мм
C. Глюкагон рт.ст., зрiст - 164 см, вага - 103 кг; переважно
D. Тироксин накопичення жиру на шиї, верхньому
E. Альдостерон плечовому поясi, животi. Що є основним
Збірник тестових завдань для складання патогенетичним механiзмом ожирiння у
ліцензійного іспиту Крок -1 жiнки?
«Стоматологія». – 2008. – №175. A. Пiдвищення продукцiї глюкокортикоїдiв
B. Зниження продукцiї тиреоїдних гормонiв
17. Пiд час огляду хворого лiкар запiдозрив C. Пiдвищення продукцiї iнсулiну
синдром Iценка-Кушинга. Визначення якої D. Зниження продукцiї глюкагону
речовини в кровi хворого пiдтвердить E. Пiдвищення продукцiї
припущення лiкаря? мiнералокортикоїдів
A. Кортизол Збірник тестових завдань для складання
B. Токоферол ліцензійного іспиту Крок -1 «Загальна
C. Ретинол лікарська підготовка». -2006. -№77.
D. Адреналiн
E. Холестерин 21. Пацiєнт 16-ти рокiв, що страждає на
Збірник тестових завдань для складання хворобу Iценко-Кушiнга, консультований з
ліцензійного іспиту Крок -1 «Загальна приводу надмiрної ваги тiла. При опитуваннi
лікарська підготовка». -2005. -№69. з’ясувалося, що енергетична цiннiсть
спожитої їжi складає 1700-1900 ккал/добу.
18. У хворого з дiагнозом хвороба Iценка- Яка провiдна причина ожирiння у даному
Кушинга (гiперпродукцiя гормонiв корою випадку?
наднирникiв) в кровi визначено пiдвищену A. Надлишок глюкокортикоїдiв
концентрацiю глюкози, кетонових тiл, B. Нестача iнсулiну
натрiю. Який бiохiмiчний механiзм є C. Надлишок iнсулiну
провiдним у виникненнi гiперглiкемiї? D. Нестача глюкокортикоїдiв
A. Глюконеогенез E. Гiподинамiя
B. Глiкогенез Збірник тестових завдань для складання
C. Глiкогенолiз ліцензійного іспиту Крок -1 «Загальна
D. Глiколiз лікарська підготовка». -2014. -№54.
E. Аеробний гліколіз
Збірник тестових завдань для складання 22. У хворої жiнки 29-ти рокiв мiсяцеподiбне
ліцензійного іспиту Крок -1 «Загальна обличчя, ожирiння верхньої частини тулуба,
лікарська підготовка». -2006. -№192. стриї на переднiй черевнiй стiнцi, гiрсутизм,
у сечi підвищено рiвень 17-
19. У хворого виявлено ожирiння, гiрсутизм, оксикетостероїдiв. Вищезазначенi прояви
”мiсяцеподiбне” обличчя, рубцi багряного характернi для захворювання:
кольору на шкiрi стегон. Артерiальний тиск - A. Синдром Iценко-Кушiнга
180/110 мм рт.ст., глюкоза кровi - 17,2 B. Феохромоцитома
ммоль/л. При якiй змiнi продукцiї гормонiв C. Синдром Кона
наднирникiв можлива така картина? D. Первинний гiпоальдостеронiзм
A. Гiперпродукцiя глюкокортикоїдiв E. Вторинний гiперальдостеронiзм
B. Гiпопродукцiя глюкокортикоїдiв Збірник тестових завдань для складання
C. Гiперпродукцiя мiнералокортикоїдiв ліцензійного іспиту Крок -1
D. Гiпопродукцiя мiнералокортикоїдiв «Стоматологія». – 2012. – №69.
E. Гiпопродукцiя адреналiну
Збірник тестових завдань для складання 23. До лікаря звернувся хворий зі скаргами
ліцензійного іспиту Крок -1 «Загальна на постійну спрагу. Виявлено гіперглікемія,
лікарська підготовка». -2017. -№10. поліурія та підвищений вміст 17-
кетостероїдів в сечі. Яке захворювання
20. Жiнка 44-х рокiв скаржиться на загальну найбільш ймовірно?
слабкiсть, бiль у дiлянцi серця, значне A. Стероїдний діабет
28
B. Інсулінозалежний діабет Сборник тестовых заданий для
C. Мікседема лицензионного экзамена Крок-1 «Медицина».
D. Глікогеноз I типу - 2011. - № 188.
E. Аддисонова хвороба
Test items for licensing examination Krok 1 27. Прийом аналогів кортикостероїдів
«Medicine». – 2011. - № 93. викликає розпад м'язових білків до вільних
амінокислот. В якому процесі будуть задіяні
24. У хворого 42-х років виявили пухлину амінокислоти в таких умовах?
аденогипофізу. Об'єктивно: вага 117 кг, A. Глюконеогенез в печінці
обличчя місяцеподібне, гиперемьоване, на B. Гліколіз в м'язах
шкірі живота синебагрові смуги C. Синтез вищих жирних кислот
розтягування. Остеопороз, дистрофія м'язів. D. Глікогеноліз
АТ- 210 / 140мм рт.ст. У хворого найбільш E. Декарбоксилювання
імовірно: Сборник тестовых заданий для
A. Хвороба Іценко-Кушинга лицензионного экзамена Крок-1 «Медицина».
B. Синдром Іценко-Кушинга - 2016. - № 179.
C. Хвороба Конна
D. Цукровий діабет 28. В нефрологiчнiй клініці у юнака 19-ти
E. Гіпертонічна хвороба рокiв була виявлена підвищена кiлькiсть
Test items for licensing examination Krok 1 калiю у вториннiй сечi. Пiдвищення рiвня
«Medicine». – 2012. - № 159. якого гормону, iмовiрно, могло викликати
такi змiни?
25. Жінка 38-ми років скаржиться на A. Альдостерон
загальну слабкість, біль в області серця, B. Окситоцин
підвищення апетиту, відсутність C. Адреналiн
менструацій. Об'єктивно: зріст 166 см., Вага D. Глюкагон
108 кг, обличчя місяцеподібне, відкладення E. Тестостерон
підшкірної клітковини переважно в області Збірник тестових завдань для складання
верхнього плечового пояса, тулуба, стегон, ліцензійного іспиту Крок -1 «Загальна
живота; наявність криваво-червоних смуг. лікарська підготовка». -2018. -№158.
Ps- 62 / хв., АТ- 160/105 мм рт.ст. Для якого з
перерахованих нижче станів найбільш 29. Тваринi внутрiшньовенно ввели
характерний даний тип ожиріння? концентрований розчин хлориду натрiю, що
A. Хвороба Іценко-Кушинга зумовило зниження реабсорбцiї iонiв натрiю
B. Аліментарне ожиріння у канальцях нирок. Внаслiдок яких змiн
C. Мікседема секрецiї гормонiв це вiдбувається?
D. Інсулінома A. Зменшення альдостерону
E. Синдром Бабинського-Фреліха B. Збiльшення альдостерону
Test items for licensing examination Krok 1 C. Зменшення вазопресину
«Medicine». – 2013. - № 30. D. Збiльшення вазопресину
E. Зменшення натрiйуретичного фактора
26. Обстеження пацієнта з високим Збірник тестових завдань для складання
артеріальним тиском виявило у нього ліцензійного іспиту Крок -1 «Загальна
вторинну артеріальну гіпертензію. З'ясовано, лікарська підготовка». -2005. -№120.
що причиною такого стану пацієнта є
гормонально активна пухлина кори 30. У хворого на аденому клубочкової зони
надниркових залоз. Гіперпродукція якого кори наднирникiв (хвороба Конна)
гормону є причиною вторинної артеріальної спостерiгаються артерiальна гiпертензiя,
гіпертензії у хворого? напади судом, полiурiя.Що є головною
A. Кортизол ланкою в патогенезi цих порушень?
B. Адреналін A. Гiперсекрецiя альдостерону
C. Тироксин B. Гiпосекрецiя альдостерону
D. Інсулін C. Гiперсекрецiя катехоламiнiв
E. Глюкагон D. Гiперсекрецiя глюкокортикоїдiв
29
E. Гiпосекрецiя глюкокортикоїдiв Сборник тестовых заданий для
Збірник тестових завдань для складання лицензионного экзамена Крок-1 «Медицина».
ліцензійного іспиту Крок -1 - 2005. - № 32.
«Стоматологія». – 2005 – №146.
35. Пiддослiдному собацi ввели гормон, що
31. Чоловiк в результатi ДТП втратив багато призвело до збiльшення швидкості
кровi, свiдомiсть затьмарена, низький клубочкової фiльтрацiї за рахунок
кров’яний тиск. При цьому у нього розширення приносної артерiоли i
компенсаторно активується ренiн- зменшення реабсорбцiї iонiв натрiю i води в
ангiотензинова система, що призводить до: канальцях нефрона. Який гормон було
A. Гiперпродукцiї альдостерону введено?
B. Пiдвищення згортання кровi A. Передсердний натрiйуретичний
C. Посилення еритропоезу B. Адреналiн
D. Гiперпродукцiї вазопресину C. Тироксин
E. Посилення серцевих скорочень D. Тестостерон
Збірник тестових завдань для складання E. Окситоцин
ліцензійного іспиту Крок -1 Збірник тестових завдань для складання
«Стоматологія». – 2018. – №165. ліцензійного іспиту Крок -1
«Стоматологія». – 2015. – №194.
32. У хворого виявлена гіперкаліємія і
гіпонатріємія. Знижена секреція якого 36. У хворого має місце недостатнє
гормону може зумовити такі зміни? кровопостачання нирок, що призвело до
A. Альдостерон розвитку прессорной реакції внаслідок
B. Вазопресин звуження артеріальних судин опору. Це є
C. Кортизол наслідком посиленого впливу на судини
D. Паратгормон такого речовини:
E. Натрійуретичний A. Антіотензин II
Test items for licensing examination Krok 1 B. Ангіотензиноген
«Stomatology». – 2007. - № 83. C. Ренін
D. Катехоламіни
33. У людей, пристосованих до високих E. Норадреналін
зовнішніх температур, рясне потовиділення Test items for licensing examination Krok 1
не супроводжується втратою великих обсягів «Medicine». – 2015. - № 79.
хлориду натрію. Це пов'язано з дією
наступного гормону на потові залози: 37. Обстеження пацiєнта з високим
А. Альдостерон артерiальним тиском показало в нього
B. Вазопресин вторинну артерiальну гiпертензiю.
C. Кортизол Причиною такого стану є ренiн-продукуюча
D. Tіроксин пухлина нирки. Що є головною ланкою в
E. Натрійуретичний патогенезi вторинної артерiальної гiпертензiї
Test items for licensing examination Krok 1 в хворого?
«Medicine». – 2010. - № 73. A. Гiперпродукцiя ангiотензину 2,
альдостерону
34. У людини зменшений діурез, B. Гiперпродукцiя кортизолу
гіпернатріємія, гіпокаліємія. Гиперсекреция C. Гiперпродукцiя iнсулiну
якого гормону може бути причиною таких D. Недостатня продукцiя вазопресину
змін? E. Недостатня продукцiя катехоламiнiв
A. Альдостерон Збірник тестових завдань для складання
B. Вазопресин ліцензійного іспиту Крок -1 «Загальна
C. Передсердний натрійуретичний фактор лікарська підготовка». -2014. -№193.
D. Адреналін
E. Паратгормон 38. У хворого iз цирозом печiнки
вiдмiчається стiйка артерiальна гiпотензiя.
(АТ- 90/50 мм рт.ст.). Чим обумовлено
30
зниження артерiального тиску при такiй
патологiї печiнки? 42. У жiнки обмежений кровотiк у нирках,
A. Зниження синтезу ангiотензиногену пiдвищений артеріальний тиск.
B. Збiльшення синтезу Na-уретичного Гiперсекрецiя якого гормону зумовила
гормону пiдвищення тиску?
C. Надмiрна iнактивацiя вазопресину A. Ренiн
D. Посилення рефлекторного впливу iз B. Адреналiн
рецепторної зони дуги аорти C. Норадреналiн
E. Активацiя калiкреїн-кiнiнової системи D. Еритропоетин
Збірник тестових завдань для складання E. Вазопресин
ліцензійного іспиту Крок -1 Збірник тестових завдань для складання
«Стоматологія». – 2015. – №104. ліцензійного іспиту Крок -1 «Загальна
лікарська підготовка». -2016. -№48.
39. У людини з захворюванням нирок
збiльшений артерiальний тиск, особливо 43. У жiнки 40-ка років при обстеженнi
дiастолiчний. Посилена секрецiя якої виявлений підвищений основний обмiн.
бiологiчно активної речовини спричиняє Надлишок якого з наведених гормонів
пiдвищення тиску у хворого? зумовлює цей стан?
A. Ренiн A. Трийодтиронiн
B. Адреналiн B. Тиреокальцитонiн
C. Норадреналiн C. Глюкагон
D. Вазопресин D. Альдостерон
E. Катехоламiни E. Соматостатин
Збірник тестових завдань для складання Збірник тестових завдань для складання
ліцензійного іспиту Крок -1 ліцензійного іспиту Крок -1 «Загальна
«Стоматологія». – 2006. – №68. лікарська підготовка». -2018. -№1.

40. В умовах експерименту у кролика 44. У чоловiка 30-ти рокiв методом непрямої
перев’язали ниркову артерiю, що через 2 калориметрiї встановлено зменшення
тижнi призвело до суттєвого збiльшення основного обмiну на 30%. Зниження
артерiального тиску. У результатi концентрацiї яких гормонiв у плазмi кровi
збiльшення секрецiї якої бiологiчно активної може бути причиною цього?
речовини це вiдбулося? A. Трийодтиронiн, тетрайодтиронiн
A. Ренiн B. Тиреокальцитонiн, паратгормон
B. Адреналiн C. Глюкокортикоїди
C. Вазопресин D. Катехоламiни
D. Норадреналiн E. Соматолiберин, соматостатин
E. Натрiйуретичний гормон Збірник тестових завдань для складання
Збірник тестових завдань для складання ліцензійного іспиту Крок -1
ліцензійного іспиту Крок -1 «Загальна «Стоматологія». – 2011. – №34.
лікарська підготовка». -2005. -№94.
45. Хворий 37-ми рокiв за останнi три мiсяцi
41. У пацієнта що тривалий час вживав схуд на 5 кг, скаржиться на тремор рук,
препарати, що блокують вироблення пiдвищене потовидiлення, екзофтальм,
ангіотензину II, виникли брадикардія, тахiкардiю. Збiльшення секрецiї якого
порушення серцевого ритму. Можливою гормону може бути причиною цього?
причиною цих розладів є: A. Тироксин
A. Гіперкаліємія B. Кортизол
B. Гіпокаліємія C. Iнсулiн
C. Гіпернатріємія D. Глюкагон
D. Гіпокальціємія E. Тиреокальцитонiн
E. Гіперкальціємія Збірник тестових завдань для складання
Test items for licensing examination Krok 1 ліцензійного іспиту Крок -1
«Medicine». – 2013. - № 185. «Стоматологія». – 2011. – №120.
31
46. Жiнка 38 рокiв скаржиться на пiдвищену 49. Хвора 56-ти рокiв тривалий часхворiє на
пiтливiсть, серцебиття, пiдвищення тиреотоксикоз. Який тип гiпоксiї може
температури тiла у вечiрнi години. Основний розвинутися у цiєї хворої?
обмiн збiльшений на 60%. Лiкар встановив A. Тканинна
дiагноз тиреотоксикоз. Якi властивостi B. Гемiчна
тироксину приводять до пiдсилення C. Циркуляторна
теплопродукцiї? D. Дихальна
A. Розщеплює окисне фосфорилування E. Змiшана
B. Пiдвищує спряження окиснення та Збірник тестових завдань для складання
фосфорилування ліцензійного іспиту Крок -1 «Загальна
C. Зменшує β-окиснення жирних кислот лікарська підготовка». -2006. -№115.
D. Зменшує дезамiнування амiнокислот
E. Сприяє накопиченню ацетил-КоА 50. Хворий помилково прийняв надмiрну
Збірник тестових завдань для складання дозу тироксину. До яких змiн секрецiї
ліцензійного іспиту Крок -1 «Загальна тиреолiберину та тиреотропiну це призведе?
лікарська підготовка». -2005. -№146. A. Секрецiя гормонiв зменшиться
B. Секрецiя гормонiв збiльшиться
47. У жінки 40-ка років з дифузним C. Змiн секрецiї гормонiв не буде
токсичним зобом спостерігається постійне D. Секрецiя тиреолiберину збiльшиться,
підвищення температури тіла. Який механізм тиреотропiну - зменшиться
це обумовлює? E. Секрецiя тиреотропiну збiльшиться,
A. Роз'єднання окислення і фосфорилювання тиреолiберину – зменшиться
в мітохондріях клітин Збірник тестових завдань для складання
B. Посилення розпаду глікогену в клітинах ліцензійного іспиту Крок -1 «Загальна
печінки лікарська підготовка». -2014. -№119.
C. Посилення катаболізму білка в клітинах
D. Підвищення збудливості клітин нервової 51. 19-річна жінка страждає на тахікардію в
системи стані спокою, втрату ваги, надмірне
E. Підвищення чутливості клітин до потовиділення, екзофтальм і дратівливість.
катехоламінів Підвищення вмісту якого гормону очікується
Test items for licensing examination Krok 1 знайти в сироватці крові?
«Stomatology». – 2016. - № 92. A. Тироксин
B. Кортизол
48. Хворий знаходиться на облiку в C. Мінералокортикоїди
ендокринологiчному диспансерi з приводу D. АКТГ
гiпертиреозу. До схуднення, тахiкардiї, E. Інсулін
тремтiння пальцiв рук, приєдналися Test items for licensing examination Krok 1
симптоми гiпоксiї – головний бiль, «Medicine». – 2005. - № 145.
втомлюванiсть, мерехтiння "мушок"перед
очима. Який механiзм дії тиреоїдних 52. Дитина має симптоми затримки
гормонiв лежить в основi розвитку гiпоксiї? психічного та фізичного розвитку
A. Роз’єднання окиснення та (кретинізм). Зазвичай це пов'язано з
фосфорилування дефіцитом наступного гормону:
B. Гальмування синтезу дихальних A. Тироксин
ферментiв Б. Соматотропний
C. Конкурентне гальмування дихальних С. Кальцитонін
ферментiв D. Інсулін
D. Посилення синтезу дихальних ферментiв E. Тестостерон
E. Специфiчне зв’язування активних центрiв Test items for licensing examination Krok 1
дихальних ферментів «Stomatology». – 2011. - № 58.
Збірник тестових завдань для складання
ліцензійного іспиту Крок -1 «Загальна 53. Лікар констатував у дитини значне
лікарська підготовка». -2007. -№84. відставання в рості, непропорційне будова
32
тіла, розумову відсталість. Що найбільш фосфолiпази А2, що необхiдна для синтезу
імовірно могло зумовити таку патологію? простагландинiв. Яка речовина є
A. Гіпотиреоз попередником цих медiаторiв запалення?
B. Незадовільне харчування A. Арахiдонова кислота
C. Гіпертиреоз B. Холестерол
D. Генетичні вади C. Тирозин
E. Гипопітуїтаризм D. Проопiомеланокортин
Test items for licensing examination Krok 1 E. Пальмiтинова кислота
«Stomatology». – 2017. - № 53. Збірник тестових завдань для складання
ліцензійного іспиту Крок -1
54. У дитини 12-ти років низький зріст при «Стоматологія». – 2013. – №198.
непропорційної будові тіла і розумової
відсталості. Недостатня секреція якого 58. Пацiєнту, який знаходився в клiнiцi з
гормону може бути причиною цього? приводу пневмонiї, ускладненої плевритом,
A. Тироксин у складi комплексної терапії вводили
B. Інсулін преднiзолон. Протизапальна дiя цього
C. Кортизол синтетичного глюкокортикоїда пов’язана з
D. Соматотропін блокуванням вивiльнення арахідонової
E. Глюкагон кислоти шляхом гальмування:
Test items for licensing examination Krok 1 A. Фосфолiпази А2
«Stomatology». – 2014. - № 84. B. Циклооксигенази
C. Фосфолiпази С
55. Жителі територій з холодним кліматом D. Лiпоксигенази
мають в крові збільшений вміст гормону, що E. Пероксидази
є пристосуванням терморегуляторного Збірник тестових завдань для складання
значення. Про який гормон йдеться? ліцензійного іспиту Крок -1 «Загальна
A. Тироксин лікарська підготовка». -2016. -№164.
B. Інсулін
C. Глюкагон 59. Перетворення арахідонової кислоти за
D. Соматотропін циклооксигеназним шляхом призводить до
E. Кортизол утворення деяких біоактивних речовин.
Test items for licensing examination Krok 1 Назвіть їх:
«Medicine». – 2007. - № 122. A. Простагландини
B. Тироксин
56. В ході клінічного обстеження пацієнта C. Біогенні аміни
виявлено збільшення щитовидної залози D. Соматомедін
(зоб), підвищення основного обміну, втрата E. Інсуліноподібні фактори росту
маси тіла, порушення теплового балансу, Test items for licensing examination Krok 1
збільшення апетиту, підвищення збудливості «Medicine». – 2007. - № 127.
і дратівливості, екзофтальм і тахікардія. Яке
ендокринне порушення призводить до появи 60. Експериментальні дослідження показали,
даних симптомів? що стероїдні гормони впливають на
A. Гіперфункція щитовидної залози протеосинтез. Вони впливають на синтез
B. Гіпофункція паращитовидних залоз наступних речовин:
C. Гіперфункція гіпофіза A. Специфічний месенджер на РНК
D. Гіпофункція епіфіза B. Аденозинтрифосфат
E. Гіпофункція щитовидної залози C. Специфічна т-РНК
Сборник тестовых заданий для D. Гуанозинтрифосфат
лицензионного экзамена Крок-1 E. Специфічна р-РНК
«Стоматологія». - 2015. - № 99. Test items for licensing examination Krok 1
«Stomatology». – 2015. - № 29.
57. Вiдомо, що стероїднi протизапальнi
препарати гальмують активнiсть

33
Біохімія та патобіохімія крові
1. У дiвчинки 7-ми рокiв ознаки Збірник тестових завдань для складання
анемiї.Лабораторно встановлений дефiцит ліцензійного іспиту Крок-1 «Загальна
пiруваткiнази в еритроцитах. лікарська підготовка». –2008. - № 3.
Порушенняякого процесу грає головну роль
в розвитку анемiї у дiвчинки? 5. У пацієнтів зі спадковим дефектом
A. Анаеробний глiколiз глюкозо-6-фосфатдегідрогенази має місце
B. Окислювальне фосфорилювання підвищена схильність еритроцитів до
C. Тканинне дихання гемолізу. Який метаболічний процес
D. Розклад пероксидiв порушений при цих умовах?
E. Дезамiнування амiнокислот A. Пентозофосфатний шлях окислення
Збірник тестових завдань для складання глюкози
ліцензійного іспиту Крок-1 «Загальна B. Глюконеогенез
лікарська підготовка». –2006. - № 70. C. Аеробне окислення глюкози
D. Синтез глікогену
2. Еритроцити людини не мiстять E. -
мiтохондрiй. Який основний шлях утворення Test items for licensing examination Krok 1
АТФ в цих клiтинах? «Medicine». – 2008. - № 3.
A. Анаеробний глiколiз
B. Аеробний глiколiз 6. Сульфаніламіди використовують в
C. Окислювальне фосфорилювання клінічній практиці як протимікробні засоби.
D. Креатинкiназна реакцiя При генетичному дефекті якого ферменту
E. Аденiлаткiназна реакцiя пентозофосфатного обміну в еритроцитах
Збірник тестових завдань для складання лікування цими препаратами може призвести
ліцензійного іспиту Крок-1 «Загальна до гемолітичної анемії?
лікарська підготовка». –2013. - № 77. A. Глюкозо-6-фосфатдегідрогенази
B. Гексокіназа
3. В еритроцитах пацiєнта, хворого на C. Транскетолази
гемолітичну анемiю, була значно знижена D. Трансальдолаза
активність пiруваткiнази. Який метаболічний E. Піруваткіназа
процес порушений за цих умов? Сборник тестовых заданий для
A. Глiколiз лицензионного экзамена Крок-1
B. Глiкогенолiз «Стоматология». - 2012. - № 177.
C. Глюконеогенез
D. Пентозо-фосфатний шлях окислення 7. При біохімічному аналiзi еритроцитiв
глюкози немовляти встановлено виражену
E. Синтез глiкогену недостатнiсть глутатiонпероксидази i
Збірник тестових завдань для складання низький рівень відновленого глутатiону. Яка
ліцензійного іспиту Крок -1 анемія може розвинутись у цієї дитини?
«Стоматологія». – 2018. – №138. A. Гемолiтична
B. Пернiцiозна
4. У трирiчної дитини з підвищеною C. Мегалобластна
температурою тiла пiсля прийому аспiрину D. Серпоподiбно-клiтинна
спостерiгається посилений гемолiз E. Залiзодефiцитна
еритроцитiв. Вроджена недостатнiсть якого Збірник тестових завдань для складання
ферменту могла викликати у дитини ліцензійного іспиту Крок -1
гемолiтичну анемiю? «Стоматологія». – 2016. – №175.
A. Глюкозо-6-фосфатдегiдрогеназа
B. Глюкозо-6-фосфатаза 8. Хворий 20-ти рокiв скаржитьсяна загальну
C. Глiкогенфосфорилаза слабкiсть, запаморочення,швидку
D. Глiцеролфосфатдегiдрогеназа втомлюванiсть. У кровi: Hb- 80г/л.
E. γ-глутамiлтрансфераза Мiкроскопiчно: еритроцити змiненої форми.
Причиною цього стану може бути:
A. Серпоподiбноклiтинна анемiя
34
B. Паренхiматозна жовтяниця D. HbA2
C. Гостра перемiжна порфiрiя E. HbO2
D. Обтурацiйна жовтяниця Сборник тестовых заданий для
E. Хвороба Аддiсона лицензионного экзамена Крок-1 «Медицина».
Збірник тестових завдань для складання - 2016. - № 74.
ліцензійного іспиту Крок-1 «Загальна
лікарська підготовка». –2008. - № 17. 13. Після тривалого лікування
сульфаніламидними препаратами у пациента
9. При рядi гемоглобiнопатiй вiдбуваються розвинулась макроцитарна анемія.
амiнокислотнi замiни у α- i β-ланцюгах Утворення активної форми наступного
гемоглобiну. Яка з них характерна для вітаміну було порушено в даних умовах:
серпоподiбноклiтинної анемiї? A. Фолієвої кислоти
A. Глутамат-валiн B. Тіаміну
B. Аспартат-лiзин C. Рибофлавіну
C. Аланiн-серин D. Піридоксину
D. Метiонiн-гiстидин E. Ціанкобаламіну
E. Глiцин-серин Збірник тестових завдань для складання
Збірник тестових завдань для складання ліцензійного іспиту Крок-1 «Загальна
ліцензійного іспиту Крок -1 лікарська підготовка». –2016. - № 6.
«Стоматологія». – 2013. – №39.
14. У пацiєнта 50-ти рокiв, що звернувся до
10. Заміна глутамінової кислоти на валін стоматолога, був виявлений малиновий
була виявлена при вивченні вихідної "лакований" язик. У кровi: знижена кiлькiсть
молекулярної структури. Для якої спадкової еритроцитiв i концентрацiї гемоглобiну,
патології характерний цей симптом? наявнi ознаки мегалобластичного типу
А. Серпоподібноклітинна анемія кровотворення, дегенеративнi змiни бiлої
Б. Таласемія кровi. Яке захворювання кровi було виявлене
C. Хвороба Мінковського-Шоффара у хворого?
Д. Фавізм A. B12-фолiєводефiцитна анемiя
E. Гемоглобіноз B. Залiзодефiцитна анемiя
Test items for licensing examination Krok 1 C. Мiєлоїдний лейкоз
«Medicine». - 2005. - № 117. D. Апластична анемiя
E. Гемолiтична анемія
11. У хворого має місце мутація гена, що Збірник тестових завдань для складання
відповідає за синтез гемоглобіну. Це ліцензійного іспиту Крок -1
призвело до розвитку захворювання - «Стоматологія». – 2013. – №47.
серпоподібноклітинна анемія. Як
називається патологічний гемоглобін, який 15. Пiсля видалення у пацiєнта 2/3 шлунка у
спостерігається при цьому захворюванні? кровi зменшився вмiст гемоглобiну, кiлькiсть
A. HbS еритроцитiв, збiльшилися розмiри цих клiтин
B. HbA кровi. Дефiцит якого вiтамiну призводить до
C. HbF таких змiн у кровi?
D. HbA1 A. B12
E. Bart-Hb B. C
Test items for licensing examination Krok 1 C. P
«Stomatology». - 2016. - № 29. D. B6
E. PP
12. Поряд з нормальними типами Збірник тестових завдань для складання
гемоглобіну в організмі дорослої людини ліцензійного іспиту Крок-1 «Загальна
можуть бути присутніми патологічні. лікарська підготовка». –2011. - № 88.
Вкажіть один з них:
A. HbS 16. У хворого 43 рокiв з хронічним
B. HbF атрофiчним гастритом i мегалобластною
C. HbA1 гiперхромною анемiєю пiдвищене видiлення
35
метилмалонової кислоти з сечею. 20. У хворого, що переніс 5 рокiв тому
Недостатнiстю якого вiтамiну обумовлене субтотальну резекцію шлунка, розвинулась
виникнення зазначеного В12-фолiєводефiцитна анемiя. Який
симптомокомплексу? механізм є провiдним у розвитку такої
A. В12 анемiї?
B. В2 A. Вiдсутнiсть внутрішнього фактора Касла
C. В3 B. Вiдсутнiсть зовнішнього фактора Касла
D. В5 C. Порушення всмоктування вiтамiну В12 в
E. В6 тонкiй кишцi
Збірник тестових завдань для складання D. Дефiцит фолiєвої кислоти
ліцензійного іспиту Крок -1 E. Дефiцит транскобаламiну.
«Стоматологія». – 2005. – №127. Збірник тестових завдань для складання
ліцензійного іспиту Крок-1 «Загальна
17. До лiкаря акушера-гiнеколога звернулась лікарська підготовка». –2016. - № 71.
вагiтна жiнка, у якої дiагностували
мегалобластну анемiю. Який з нижче 21. При обстеженнi хворого на атрофiчний
наведених засобiв доцiльно призначити? гастрит виявлено мегалобластну анемiю.
A. Цiанокобаламiн Дефiцит якої речовини є причиною
B. Пентоксил виникнення анемiї у цього хворого?
C. Метилурацил A. Гастромукопротеїд
D. Глауцин B. Вiтамiн B6
E. Стрептокiназа C. Вiтамiн B1
Збірник тестових завдань для складання D. Залiзо
ліцензійного іспиту Крок-1 «Загальна E. Еритропоетини
лікарська підготовка». –2012. - № 154. Збірник тестових завдань для складання
ліцензійного іспиту Крок -1
18. У пацієнта діагностована мегалобластна «Стоматологія». – 2010. – №124.
анемія. Вкажіть вітамін, дефіцит якого може
призвести до розвитку цього захворювання. 22. Хірургічне видалення частини шлунка
A. Ціанокобаламін. призвело до порушення всмоктування
B. Рутин. вітаміну В12, він виділяється з калом. У
C. Нікотинамід. хворого була діагностована анемія. Який
D. Тіамін. фактор необхідний для засвоєння цього
E. Холекальциферол. вітаміну?
Збірник тестових завдань для складання А. Гастромукопротеїн
ліцензійного іспиту Крок-1 «Загальна B. Гастрін
лікарська підготовка». –2012. - № 154. C. Соляна кислота
D. Пепсин
19. У пацієнта 65-ти років з тривалими E. Фолієва кислота
скаргами, характерними для хроничного Test items for licensing examination Krok 1
гастриту, в периферичній крові виявлені «Medicine». - 2008. - № 133.
мегалоцити, у кістковому мозку
мегалобластичний еритропоез. Який діагноз 23. У хворого з гемолiтичною анемією
найбільш імовірний? виявлено дефiцит пiруваткiнази в
A. B12-фолієво дефіцитна анемія еритроцитах. За цих умов причиною
B. Апластична анемія розвитку гемолiзу еритроцитiв є:
C. Гіпопластичная анемія A. Зменшення активностi Na+, К+ -АТФ-ази
D. Гемолітичная анемія B. Нестача Na+ в еритроцитах
E. Залізодефіцитна анемія C. Надлишок K+ в еритроцитах
Збірник тестових завдань для складання D. Генетичнi дефекти глiкофорину А
ліцензійного іспиту Крок-1 «Загальна E. Дефiцит спектрину
лікарська підготовка». –2017. - № 55 Збірник тестових завдань для складання
ліцензійного іспиту Крок-1 «Загальна
лікарська підготовка». –2017. - № 183.
36
B. Гексокiназа
24. При лабораторному обстеженнi кровi C. Фруктокiназа
людини, яку вкусила змiя, виявлено гемолiз D. Пiруваткiназа
еритроцитiв, гемоглобiнурiю. Дiя зміїної E. Лактатдегiдрогеназа
отрути зумовлена наявнiстю в нiй ферменту: Збірник тестових завдань для складання
A. Фосфолiпаза А2 ліцензійного іспиту Крок-1 «Загальна
B. Фосфолiпаза А1 лікарська підготовка». –2018. - № 180.
C. Фосфолiпаза С
D. Фосфолiпаза D 28. У хворої 38-ми рокiв пiсля прийому
E. Сфiнгомiєлiназа аспiрину i сульфанiламiдiв спостерiгається
Збірник тестових завдань для складання посилений гемолiз еритроцитiв, який
ліцензійного іспиту Крок -1 викликаний недостатнiстю глюкозо-6-
«Стоматологія». – 2018. – №156. фосфатдегiдрогенази. Порушенням
утворення якого коферменту зумовлена ця
25. У людей, якi постiйно проживають в патологiя?
гiрськiй мiсцевостi, адаптацiя до "кисневого A. НАДФ-Н
голодування" здійснюється шляхом B. ФАД-Н2
полегшеної вiддачi кисню гемоглобiном C. Пiридоксальфосфат
внаслiдок: D. ФМН-Н2
A. Пiдвищеного утворення 2,3- E. Убiхiнон
дифосфоглiцерату в еритроцитах Збірник тестових завдань для складання
B. Зниженого утворення 2,3- ліцензійного іспиту Крок -1
дифосфоглiцерату в еритроцитах «Стоматологія». – 2010. – №81.
C. Зростання парцiального тиску CO2
D. Пiдвищення pH кровi 29. У 22-річної жінки внаслідок тривалого
E. Зниження температури кровi вживання сульфаніламідних препаратів
Збірник тестових завдань для складання з'явилися ознаки гемолітичної анемії, що
ліцензійного іспиту Крок-1 «Загальна обумовлено спадковим порушенням синтезу
лікарська підготовка». –2012. - № 178. ферменту пентозофосфатного циклу
глюкозо-6-фосфатдегідрогенази, який
26. Після аварії на хімічному виробництві забезпечує утворення в організмі:
відбулося забруднення навколишнього A. НАДФ- Н2
середовища нітроcполуками. У людей, що B. НАД
проживають в цій місцевості, з'явилися різка C. ФАД
слабкість, головний біль, задишка, D. ФМН
запаморочення. У чому причина розвитку E. АТФ
гіпоксії? Test items for licensing examination Krok 1
A. Утворення метгемоглобіну «Stomatology». - 2009. - № 121.
B. Пригнічення дегидрогеназ
C. Утворення карбоксигемоглобіну 30. У хворого має мiсце хронiчний запальний
D. Зниження функції флавинових ферментів процес мигдаликiв. За рахунок якого
E. Інактивація цитохромоксидази бiохiмiчного процесу у вогнищi запалення
Сборник тестовых заданий для пiдтримується концентрація НАДФН,
лицензионного экзамена Крок-1 необхiдного для реалiзацiї механiзму
«Стоматология». – 2011. – № 108. фагоцитозу?
A. Пентозо-фосфатний шлях
27. У дитини 7-ми років виражені ознаки B. Цикл Корi
гемолітичної анемiї. При бiохiмiчному C. Цикл Кребса
аналiзi еритроцитів встановлено знижену D. Орнiтиновий цикл
концентрацію НАДФН i вiдновленого E. Глiколiз
глутатiону. Дефiцит якого ферменту Збірник тестових завдань для складання
зумовлює у даному випадку біохімічні змiни ліцензійного іспиту Крок-1 «Загальна
i клінічні прояви? лікарська підготовка». –2017. - № 181.
A. Глюкозо-6-фосфатдегiдрогеназа
37
31. У процесi метаболiзму в органiзмi E. Сахараза, протромбін
людини утворюються активнi форми кисню, Сборник тестовых заданий для
у тому числi супероксидний анiон-радикал. лицензионного экзамена Крок-1
За допомогою якого ферменту iнактивується «Стоматология». – 2013. – № 113.
цей анiон?
A. Супероксиддисмутаза 35. Пародонтит викликає розвиток
B. Каталаза перекисного окислення ліпідів в тканинах
C. Пероксидаза пародонта, а також збільшення концентрації
D. Глутатiонпероксидаза малонового діальдегіду і перекису водню в
E. Глутатiонредуктаза порожнині рота. Який з наступних ферментів
Збірник тестових завдань для складання забезпечує антиоксидантний захист?
ліцензійного іспиту Крок-1 «Загальна А. Каталаза
лікарська підготовка». –2005. - № 23. B. Амілаза
C. Мальтаза
32. При зниженнi активностi ферментів D. Лактаза
антиоксидантного захисту посилюються E. Інвертаза
процеси перекисного окиснення лiпiдiв Test items for licensing examination Krok 1
клiтинних мембран. При нестачi якого «Stomatology». - 2013. - № 116.
мiкроелементу знижується активнiсть
глутатiонпероксидази? 36. Тi органiзми, якi в процесi еволюціїне
A. Селен створили захисту вiд H2O2, можуть жити
B. Молiбден лише в анаеробних умовах. Якi з
C. Кобальт перелiчених ферментiв можуть руйнувати
D. Марганець пероксид водню?
E. Мiдь A. Пероксидаза та каталаза
Збірник тестових завдань для складання B. Оксигенази та гiдроксилази
ліцензійного іспиту Крок-1 «Загальна C. Цитохромоксидаза, цитохром B5
лікарська підготовка». –2015. – № 120. D. Оксигеназа та каталаза
E. Флавiнзалежнi оксидази
33. При патологічних процесах, якi Збірник тестових завдань для складання
супроводжуються гiпоксiєю, вiдбувається ліцензійного іспиту Крок -1
неповне вiдновлення молекули кисню в «Стоматологія». – 2013. – №46.
дихальному ланцюзi i накопичення
пероксиду водню. Вкажiть фермент, який 37. Хворий з абсцесом розсіченої рани
забезпечує його руйнування: звернувся до травматологічного відділення.
A. Каталаза Щоб очистити рану від гною, лікар промив її
B. Цитохромоксидаза 3% перекисом водню. Піна була відсутня.
C. Сукцинатдегiдрогеназа Чим обумовлено відсутність активності
D. Кетоглутаратдегiдрогеназа препарату?
E. Аконiтаза А. Спадкова недостатність каталази
Збірник тестових завдань для складання B. Низька концентрація H2O2
ліцензійного іспиту Крок-1 «Загальна C. Спадкова недостатність
лікарська підготовка». –2016. - № 21. фосфатдегідрогенази еритроцитів
D. Поверхневість рани
34. При пародонтитах розвивається ліпідна E. Гній в рані
пероксидація в тканинах пародонта, в Test items for licensing examination Krok 1
ротовій порожнині наростає вміст «Medicine». - 2006. - № 159.
малонового діальдегіду, пероксиду водню.
Які з перелічених ферментів здійснюють 38. В результатi виснажуючої м’язової
антіоксідантний захист? роботи у робочого значно зменшилась
A. Супероксиддисмутаза, каталаза буферна ємнiсть кровi. Надходженням якої
B. Амілаза, трипсин речовини у кров можна пояснити це явище?
C. Мальтаза, хімотрипсин A. Лактат
D. Лактаза, лізоцим B. Пiруват
38
C. 1,3-бiсфосфоглiцерат Test items for licensing examination Krok 1
D. Альфа-кетоглутарат «Medicine». – 2007. - № 84.
E. 3-фосфоглицерат
Збірник тестових завдань для складання 42. Пiд час бiгу на короткi дистанцiї у
ліцензійного іспиту Крок-1 «Загальна нетренованої людини виникає м’язова
лікарська підготовка». –2017. - № 40. гiпоксiя. До накопичення якого метаболiту в
м’язах це призводить?
39. У дитини на протязi перших трьох A. Лактат
мiсяцiв пiсля народження розвинулася важка B. Кетоновi тiла
форма гiпоксiї, що проявлялася задухою та C. Ацетил-КоА
синюшнiстю шкiри. Причиною цього є D. Глюкозо-6-фосфат
порушеня замiни фетального гемоглобiну на: E. Оксалоацетат
A. Гемоглобiн А Збірник тестових завдань для складання
B. Гемоглобiн S ліцензійного іспиту Крок-1 «Загальна
C. Глiкозильований гемоглобiн лікарська підготовка». –2009. - № 107.
D. Метгемоглобiн
E. Гемоглобiн М 43. У жiнки 32-х рокiв запалення ясен
Збірник тестових завдань для складання (гiнгiвiт) супроводжується їх гiпоксiєю.
ліцензійного іспиту Крок -1 Утворення якого метаболiту вуглеводного
«Стоматологія». – 2005. – №122. обмiну значно збiльшується при цьому в
тканинах пародонта?
40. При захворюваннях дихальної системи, A. Лактат
розладах кровообігу порушується транспорт B. Рибозо-5-фосфат
кисню, що супроводжується гіпоксією. При C. Глiкоген
цих умовах енергетичний обмін D. Глюкозо-6-фосфат
здійснюється за рахунок анаеробного E. НАДФ-Н
гліколізу, що призводить до утворення та Збірник тестових завдань для складання
накопичення в крові: ліцензійного іспиту Крок -1
A. Молочна кислота «Стоматологія». – 2011. – №36.
B. Піровиноградна кислота
C. Глутамінова кислота 44. 29-річний пацієнт був доставлений в
D. Лимонна кислота лікарню через отруєння чадним газом.
E. Фумарова кислота Об'єктивно: у пацієнта спостерігаються
Test items for licensing examination Krok 1 симптоми важкої гіпоксії - задишка, ціаноз,
«Medicine». – 2013. - № 124. тахікардія. Яка сполука утворюється в
результаті інтоксикації монооксидом
41. Хворий перебуває на обліку в вуглецю?
ендокринологічному диспансері з приводу А. Карбоксигемоглобін
гіпертиреозу. До схуднення, тахікардії, Б. Метгемоглобін
тремтіння пальців рук приєдналися C. Карбгемоглобін
симптоми гіпоксії - головний біль, D. Сульфгемоглобін
стомлюваність, мерехтіння "мушок" перед Е. Оксигемоглобін
очима. Який механізм дії тиреоїдних Test items for licensing examination Krok 1
гормонів лежить в основі розвитку гіпоксії? «Stomatology». – 2011. - № 85.
A. Розщеплення окислення і
фосфорилювання 45. Чоловік втратив свідомість в салоні
B. Гальмування синтезу дихальних автомобіля, де тривалий час чекав приятеля
ферментів при включеному двигуні. Яку сполуку
C. Конкурентне гальмування дихальних гемоглобіну можна виявити в крові
ферментів потерпілого?
D. Посилення синтезу дихальних ферментів A. Карбоксигемоглобін
E. Специфічне зв'язування активних центрів B. Дезоксигемоглобін
дихальних ферментів C. Карбгемоглобін
D. Метгемолобін
39
E. Оксигемоглобін B. Респiраторний алкалоз
Test items for licensing examination Krok 1 C. Метаболiчний ацидоз
«Stomatology». – 2007. - № 77 D. Метаболiчний алкалоз
E. КОР незмiниться
46. У хворого з дихальною недостатністю рН Збірник тестових завдань для складання
крові 7,35. Визначення pCO2 показало ліцензійного іспиту Крок-1 «Загальна
наявність гіперкапнії. При дослідженні рН лікарська підготовка». –2018. - № 13.
сечі відзначається підвищення її кислотності.
Яка форма порушення кислотно-основного 50. При цукровому дiабетi внаслiдок
стану в даному випадку? активацiї процесів окислення жирних кислот
A. Ацидоз газовий, компенсований виникає кетоз. До яких порушень кислотно-
B. Ацидоз метаболічний, компенсований лужної рiвноваги може призвести надмiрне
C. Ацидоз метаболічний, декомпенсований накопичення кетонових тiл в кровi?
D. Алкалоз газовий, компенсований A. Метаболiчний ацидоз
E. Алкалоз газовий, декомпенсований B. Метаболiчний алкалоз
Test items for licensing examination Krok 1 C. Змiни не вiдбуваються
«Medicine». - 2013. - № 36. D. Дихальний ацидоз
E. Дихальний алкалоз
47. У немовляти внаслiдок неправильного Збірник тестових завдань для складання
годування виникла виражена дiарея. Одним з ліцензійного іспиту Крок-1 «Загальна
основних наслiдкiв дiареї є екскрецiя великої лікарська підготовка». –2018. - № 124.
кiлькостi бiкарбонату натрiю. Яка форма
порушення кислотно-лужного балансу має 51. Кетоз розвивається у хворих на цукровий
мiсце вцьому випадку? діабет в результаті активації процесів
A. Метаболiчний ацидоз окислення жирних кислот. Який кислотно-
B. Метаболiчний алкалоз основний дисбаланс може виникнути в
C. Респiраторний ацидоз результаті накопичення в крові надлишку
D. Респiраторний алкалоз кетонових тіл?
E. Не буде порушень кислотно- А. Метаболічний ацидоз
лужногобалансу Б. Метаболічний алкалоз
Збірник тестових завдань для складання C. Дисбаланс не виникає
ліцензійного іспиту Крок-1 «Загальна D. Респіраторний ацидоз
лікарська підготовка». –2008. - № 2. E. Респіраторний алкалоз
Test items for licensing examination Krok 1
48. Чоловік 30-ти років з цукровим діабетом «Medicine». - 2018. - № 119.
I типу був госпіталізований в стані коми.
Лабораторні дослідження виявили 52. Хворий проходив чергове обстеження, в
гіперглікемію, кетонемії. Яке з наведених результатi якого у нього виявлено
метаболічних розладів може бути виявлено у гiперглiкемiю, кетонурiю, полiурiю,
цього пацієнта? глюкозурiю. Яка форма КОС маємiсце за
A. Метаболічний ацидоз наявностi цих явищ?
B. Метаболічний алкалоз A. Метаболiчний ацидоз
C. Респіраторний ацидоз B. Газовий ацидоз
D. Респіраторний алкалоз C. Негазовий ацидоз
E. Нормальний кислотно-лужний баланс D. Газовий алкалоз
Test items for licensing examination Krok 1 E. Метаболiчний алкалоз
«Medicine». – 2015. - № 133. Збірник тестових завдань для складання
ліцензійного іспиту Крок -1
49. У хворого виявлено порушення «Стоматологія». – 2008. – №152.
прохідності дихальних шляхів на рiвнi
дрiбних i середніх бронхiв. Якi змiни 53. У хворого на цукровий дiабет
кислотно-основної рiвноваги можуть розвинулася дiабетична кома внаслiдок
розвинутись у пацiєнта? порушення кислотно-основного стану. Який
A. Респiраторний ацидоз
40
вид порушення кислотно-основного стану 57. При лабораторному дослідженні
виник при цьому? дихальної функції крові встановлено, що має
A. Метаболiчний ацидоз місце погіршення транспорту нею CO2. З
B. Метаболiчний алкалоз дефіцитом якого ферменту в еритроцитах це
C. Респiраторний ацидоз може бути пов'язано?
D. Газовий алкалоз A. Карбоангідраза
E. Негазовий алкалоз B. 2,3-дифосфоглицерат
Збірник тестових завдань для складання C. Аденилатциклаза
ліцензійного іспиту Крок -1 D. Протеїнкіназа
«Стоматологія». – 2011. – №80. E. Фосфорилаза
Сборник тестовых заданий для
54. Основним ускладненням цукрового лицензионного экзамена Крок-1
діабету є розвиток кетоацидозу за рахунок «Стоматология». - 2011. - № 150.
накопичення кетонових тіл у сироватці
крові. Яка форма порушення 58. В процесі катаболізму гемоглобіну
кислотноосновної рівноваги при цьому має звільняється залізо, яке в складі спеціального
місце? транспортного білка надходить в кістковий
A. Метаболічний ацидоз мозок і знову використовується для синтезу
B. Метаболічний алкалоз гемоглобіну. Цим транспортним білком є:
C. Респіраторний ацидоз A. Трансферрин (сідерофілін)
D. Респіраторний алкалоз B. Транскобаламін
Е. - C. Гаптоглобін
Test items for licensing examination Krok 1 D. Церулоплазмін
«Stomatology». - 2016. - № 137. E. Альбумін
Test items for licensing examination Krok 1
55. У пацiєнта у результатi тривалого «Medicine». - 2018. - № 8.
блювання вiдбувається значна втрата
шлункового соку, що є причиною порушення 59. Катаболізм гемоглобіну призводить до
кислотно-лужного стану в органiзмi. Яка з вивільнення заліза, яке транспортується в
перерахованих форм порушення кислотно- кістковий мозок певним транспортним
лужного стану має мiсце? білком і знову використовується для синтезу
A. Негазовий алкалоз гемоглобіну. Вкажіть цей транспортний
B. Газовий ацидоз білок:
C. Негазовий ацидоз А. Трансферин (сідерофілін)
D. Газовий алкалоз B. Транскобаламін
E. Метаболiчний ацидоз C. Гаптоглобін
Збірник тестових завдань для складання D. Церулоплазмін
ліцензійного іспиту Крок-1 «Загальна E. Альбумін
лікарська підготовка». –2014. - № 80. Test items for licensing examination Krok 1
«Medicine». - 2013. - № 5.
56. У альпiнiста, що пiднявся на висоту
5200м, розвинувся газовий алкалоз. Що є 60. У печінки хворого, який страждає на
причиною його розвитку? залізодефіцитну анемію, виявлено
A. Гiпервентиляцiя легенiв порушення синтезу залізовмісного білка,
B. Гiповентиляцiя легенiв який є джерелом заліза для синтезу гема. Як
C. Гiпероксемiя називається цей білок?
D. Гiпоксемiя A. Феритин
E. Зниження температури навколишнього B. Трансферин
середовища C. Гемосидерин
Збірник тестових завдань для складання D. Церулоплазмін
ліцензійного іспиту Крок-1 «Загальна E. Гемоглобін
лікарська підготовка». –2016. - № 175. Сборник тестовых заданий для
лицензионного экзамена Крок-1
«Стоматология». - 2015. - № 98.
41
зміни можуть бути викликані дефіцитом
61. У чоловiка 40-ка рокiв внаслiдок таких поживних речовин:
посиленого гемолiзу еритроцитiв А. Білки
пiдвищився вмiст залiза в плазмi кровi. Який B. Вітаміни
бiлок забезпечує його депонування в C. Жири
тканинах? D. Вуглеводи
A. Феритин Е. Мікронутрієнти
B. Гаптоглобiн Test items for licensing examination Krok 1
C. Трансферин «Stomatology». - 2014. - № 122.
D. Транскортин
E. Альбумiн 65. При токсичному ушкодженнi клітин
Збірник тестових завдань для складання печiнки з порушенням її функцiй у хворого
ліцензійного іспиту Крок-1 «Загальна з’явилися набряки. Якi змiни складу плазми
лікарська підготовка» – 2014. – № 152. кровi є провiдною причиною розвитку
набрякiв?
62. Пацiєнт звернувся до лiкаря зi скаргами A. Зниження вмiсту альбумiнiв
на задишку, що виникала після фiзичного B. Збiльшення вмiсту глобулiнiв
навантаження. Клiнiчне обстеження виявило C. Зменшення вмiсту фiбриногену
анемію та наявнiсть парапротеїну в зонi D. Збiльшення вмiсту альбумiнiв
гамма-глобулiнiв. Який показник у сечi E. Зменшення вмiсту глобулiнiв
необхiдно визначити для пiдтвердження Збірник тестових завдань для складання
дiагнозу мiєломи? ліцензійного іспиту Крок-1 «Загальна
A. Бiлок Бенс-Джонса лікарська підготовка» – 2012. – № 16.
B. Бiлiрубiн
C. Гемоглобiн 66. При токсичному ушкодженнi гепатоцитiв
D. Церулоплазмiн з порушенням їх бiлковосинтезуючої функцiї
E. Антитрипсин у пацiєнта рiзко знизився вмiст альбумiнiв у
Збірник тестових завдань для складання плазмi кровi та онкотичний тиск плазми. Що
ліцензійного іспиту Крок-1 «Загальна буде наслiдком цих змiн?
лікарська підготовка» – 2009. – № 38. A. Поява набрякiв
B. Зменшення дiурезу
63. У дiвчинки 16-ти рокiв, яка тривалий час C. Зменшення ШОЕ
намагалась знизити масу свого тiла D. Збiльшення об’єму циркулюючої кровi
голодуванням, виник набряк. Яка головна E. Збiльшення густини кровi
причина цього явища? Збірник тестових завдань для складання
A. Гiпопротеїнемiя, зумовлена порушенням ліцензійного іспиту Крок -1
синтезу бiлкiв «Стоматологія». – 2011. – №138.
B. Гiпоглiкемiя, зумовлена порушенням
синтезу глiкогену 67. За умови токсичного ураження печінки
C. Венозний застiй i підвищення венозного порушується її білоксинтезуюча функція.
тиску Який вид диспротеїнемії спостерігається при
D. Зменшення швидкості клубочкової цьому?
фiльтрацiї A. Абсолютна гіпопротеїнемія
E. Зменшення вироблення вазопресину в B. Відносна гіпопротеїнемія
гiпоталамусi C. Абсолютна гіперпротеїнемія
Збірник тестових завдань для складання D. Відносна гіперпротеїнемія
ліцензійного іспиту Крок -1 E. Парапротеїнемія
«Стоматологія». – 2018. – №136. Test items for licensing examination Krok 1
«Medicine». - 2012. - № 176.
64. У 36-річної пацієнтки, яка обмежувала
кількість продуктів в своєму раціоні 68. Концентрація альбуміну в зразку крові
протягом 3 місяців, спостерігається людини нижче норми. Це призводить до
зниження маси тіла, погіршення фізичного і набряку тканин. Яка функція крові
психічного здоров'я, набряки обличчя. Ці пошкоджена?
42
А. Підтримка онкотического кров'яного A. Крiоглобулiн
тиску B. Гаптоглобiн
B. Підтримання рівня рН C. Церулоплазмiн
C. Підтримання температури тіла D. С-реактивний бiлок
D. Підтримка системи седиментації крові E. α2-макроглобiн
E. Всі відповіді вірні Збірник тестових завдань для складання
Test items for licensing examination Krok 1 ліцензійного іспиту Крок-1 «Загальна
«Medicine». - 2005. - № 146. лікарська підготовка» – 2012. – № 171.

69. В клініку поступила дитина 4-х років з 73. У хворого 49-ти рокiв на гострий
ознаками тривалого білкового голодування: панкреатит виникала загроза некрозу
затримка росту, анемія, набряки, розумова пiдшлункової залози, що супроводжувалось
відсталість. Причиною розвитку набряків у надходженням у кров i тканини активних
цієї дитини є зниження синтезу: панкреатичних протеїназ i розщеплення
A. Альбуміну тканинних бiлкiв. Якi захиснi фактори
B. Глобуліну органiзму можуть iнгiбувати цi процеси?
C. Гемоглобіну A. α2-макроглобулiн, α1-антитрипсин
D. Ліпопротеїнів B. Iмуноглобулiни
E. Глікопротеїну C. Крiоглобулiн, iнтерферон
Test items for licensing examination Krok 1 D. Церулоплазмiн, трансферин
«Medicine». - 2007. - № 6. E. Гемоплексин, гаптоглобiн
Збірник тестових завдань для складання
70. Пролонгована дія ряду антибіотиків і ліцензійного іспиту Крок-1 «Загальна
сульфаніламідів обумовлена тим, що вони лікарська підготовка» – 2013. – № 113.
циркулюють в крові тривалий час в
комплексі з: 74. При хворобi Вiльсона-Коновалова
A. Альбуміном порушується транспорт мiдi, що призводить
B. Трансферином до накопичення цього металу в клiтинах
C. Гемоглобіном мозку та печiнки. З порушенням синтезу
D. Гаптоглобіном якого бiлку це пов’язано?
E. Гемопексином A. Церулоплазмiн
Сборник тестовых заданий для B. Металотiонеїн
лицензионного экзамена Крок-1 C. Транскобаламiн
«Стоматология». - 2005. - № 52. D. Гаптоглобiн
E. Сидерофiлiн
71. Електрофоретичне дослiдження Збірник тестових завдань для складання
сироватки кровi хворого на пневмонію ліцензійного іспиту Крок-1 «Загальна
показало збiльшення однiєї з білкових лікарська підготовка» – 2012. – № 144.
фракцiй. Вкажiть її:
A. γ-глобулiни 75. У хворого 27-ми рокiв виявлено
B. Альбумiни патологiчнi змiни печiнки i головного мозку.
C. α1-глобулiни У плазмi кровi виявлено різке зниження, а в
D. α2-глобулiни сечi - пiдвищення вмiсту мiдi. Встановлено
E. β-глобулiни дiагноз – хвороба Вiльсона. Активнiсть якого
Збірник тестових завдань для складання ферменту в сироватцi кровi необхiдно
ліцензійного іспиту Крок-1 «Загальна дослiдити для пiдтвердження дiагнозу?
лікарська підготовка» – 2012. – № 31. A. Церулоплазмiн
B. Карбоангiдраза
72. При активацiї запального процесу, C. Ксантиноксидаза
деяких аутоiмунних та iнфекцiйних D. Лейцинамiнопептидаза
захворюваннях у плазмi кровi рiзко зростає E. Алкогольдегiдрогеназа
рiвень бiлкiв гострої фази. Який iз наведених Збірник тестових завдань для складання
нижче бiлкiв здатний утворювати гель при ліцензійного іспиту Крок-1 «Загальна
охолодженнi сироватки? лікарська підготовка» – 2009. – № 37.
43
D. Креатинiн
76. Бiохiмiчний аналiз сироватки кровi E. Трансферин
пацiєнта з гепатолентикулярною Збірник тестових завдань для складання
дегенерацiєю (хвороба Вiльсона- ліцензійного іспиту Крок-1 «Загальна
Коновалова) виявив зниження вмiсту лікарська підготовка» – 2008. – № 24.
церулоплазмiну. У цього пацiєнта в
сироватцi кровi буде пiдвищена 80. У плазмi кровi здорової людини
концентрацiя таких iонiв: знаходиться декiлька десяткiв бiлкiв. При
A. Мiдь захворюваннi органiзму з’являються новi
B. Кальцiй бiлки, зокрема "бiлок гострої фази". Таким
C. Фосфор бiлком є:
D. Калiй A. С-реактивний бiлок
E. Натрiй B. Протромбiн
Збірник тестових завдань для складання C. Фiбриноген
ліцензійного іспиту Крок-1 «Загальна D. Iмуноглобулiн G
лікарська підготовка» – 2013. – № 5. E. Iмуноглобулiн А
Збірник тестових завдань для складання
77. У хворого на гепатоцеребральну ліцензійного іспиту Крок-1 «Загальна
дистрофiю в сироватцi кровi знижений вмiст лікарська підготовка» – 2010. – № 124.
церулоплазмiну. Накопичення якого
елемента в печiнцi, мозку та нирках 81. Чоловiк 60-ти рокiв скаржиться на бiль у
спостерiгається у хворого? суглобах. У сироватцi кровi пацiєнта
A. Мiдь виявлено пiдвищення концентрацiї С-
B. Кальцiй реактивного бiлку та оксипролiну.Для якого
C. Натрiй захворювання характернi ці симптоми?
D. Калiй A. Ревматизм
E. Залiзо B. Подагра
Збірник тестових завдань для складання C. Гепатит
ліцензійного іспиту Крок -1 D.Жовтяниця
«Стоматологія». – 2009. – №107. E. Цукровий діабет
Збірник тестових завдань для складання
78. Жінка 33 років страждає на ліцензійного іспиту Крок-1 «Загальна
гепатоцеребральну дистрофію (хвороба лікарська підготовка» – 2012. – № 119.
Вільсона). У крові - знижений вміст
церулоплазміну. У сечі - різко підвищений 82. У 6-мiсячної дитини спостерiгалися частi
вміст амінокислот. Посиленням якого та iнтенсивнi пiдшкiрнi крововиливи.
процесу обумовлені ці зміни? Призначення синтетичногоаналога вiтамiну
A. Комплексоутворення амінокислот з міддю K (вiкасолу) дало позитивний ефект. У γ-
B. Синтез сечовини карбоксилюваннi глутамiнової кислоти якого
C. Переамінування амінокислот бiлка зсiдальної системи кровi бере участь
D. Розпад тканинних білків цей вiтамiн?
E. Глюконеогенез A. Протромбiн
Сборник тестовых заданий для B. Фiбриноген
лицензионного экзамена Крок-1 «Медицина». C. Фактор Хагемана
- 2005. - № 74. D. Антигемофiльний глобулiн A
E. Фактор Розенталя
79. У хворої 38-ми рокiв ревматизм в Збірник тестових завдань для складання
активнiй фазi. Визначення якого ліцензійного іспиту Крок-1 «Загальна
лабораторного показника сироватки кровi лікарська підготовка» – 2005. – № 29.
має дiагностичне значення при данiй
патологiї? 83. Молодий чоловiк пiсля імплантації
A. С-реактивний бiлок серцевого клапана систематично отримує
B. Сечова кислота непрямi антикоагулянти. Його стан
C. Сечовина
44
ускладнився кровотечею. Iз зменшенням у тiла. Яка бiологiчно активна речовина
кровi якої речовини це пов’язане? вiдiграє провiдну роль у виникненні цього
A. Протромбiн прояву?
B. Гаптоглобiн A. Iнтерлейкiн-I
C. Гепарин B. Гiстамiн
D. Креатин C. Брадикiнiн
E. Церулоплазмiн D. Серотонiн
Збірник тестових завдань для складання E. Лейкотрiєни
ліцензійного іспиту Крок -1 Збірник тестових завдань для складання
«Стоматологія». – 2006. – №105. ліцензійного іспиту Крок-1 «Загальна
лікарська підготовка» – 2007. – № 53.
84. Пацієнт скаржиться на часті кровотечі з
ясен. У крові виявлено дефіцит II фактора 88. У пацiєнта пiсля переливання 200мл
згортання крові (протромбіну). Яка фаза кровi пiдвищилася температура тiла до
згортання крові порушена у людини, перш за 37,9oC. Яка з наведених речовин найбiльш
все? вiрогiдно призвела до пiдвищення
A. Утворення тромбіну температури?
B. Утворення протромбінази A. Iнтерлейкiн-1
C. Утворення фібрину B. Iнтерлейкiн-2
D. Фібриноліз C. Фактор некрозу пухлин
E. Ретракция згустку D. Iнтерлейкiн-3
Сборник тестовых заданий для E. Iнтерлейкiн-4
лицензионного экзамена Крок-1 Збірник тестових завдань для складання
«Стоматология». - 2007. - № 76. ліцензійного іспиту Крок-1 «Загальна
лікарська підготовка» – 2009. – № 81.
85. При запальних процесах в органiзмi
починається синтез бiлкiв ”гострої фази”. 89. Для розвитку гарячкових станів
Якi речовини є стимуляторами їх синтезу? характерним є зростання рiвня білків
A. Iнтерлейкiн-1 «гострої фази» церулоплазмiну,
B. Iмуноглобулiни фiбриногену, С-реактивного протеїну.
C. Iнтерферони Вкажiть можливий механiзм цього явища:
D. Бiогеннi амiни A. Стимулюючий вплив IЛ-1 на гепатоцити
E. Ангiотензини B. Руйнiвна дiя пiдвищеної температурина
Збірник тестових завдань для складання клiтини органiзму
ліцензійного іспиту Крок -1 C. Пролiферативна дiя IЛ-2 на Т-лiмфоцити
«Стоматологія». – 2017. – №39. D. Дегрануляцiя тканинних базофiлiв
E. –
86. В експериментi на кролику введення Збірник тестових завдань для складання
пiрогеналу призвело до пiдвищення у ліцензійного іспиту Крок-1 «Загальна
тварини температури тiла. Яка з лікарська підготовка» – 2006. – № 105.
перерахованих речовин відіграє роль
вторинного пiрогену, що бере участь у 90. Студент використав консервовану
механiзмi виникнення лихоманкової реакцiї? донорську кров для визначення часу її
A. Iнтерлейкiн-1 зсідання. Однак, будь-якого позитивного
B. Пiромен результату він отримати не зміг. Причиною
C. Гiстамiн цього є відсутність в крові:
D. Брадикiнiн A. Іонізованого кальцію
E. Iмуноглобулiн B. Фактора Хагемана
Збірник тестових завдань для складання C. Тромбопластину
ліцензійного іспиту Крок-1 «Загальна D. Фібриногену
лікарська підготовка» – 2012. – № 165. E. Вітаміну K
Збірник тестових завдань для складання
87. У хворого iз запаленням легень ліцензійного іспиту Крок-1 «Загальна
спостерiгається пiдвищення температури лікарська підготовка». –2017. – № 163.
45
Яка амiнокислота карбоксилюється в цих
91. У юнака 16-ти рокiв пiсля перенесеного бiлках?
захворювання знижена функція синтезу A. Глутамiнова
білків-факторів згортання крові у печiнцi B. Валiн
внаслiдок нестачi вiтамiну K. Це може C. Серин
призвести до порушення: D. Фенiлаланiн
A. Зсiдання кровi E. Аргiнiн
B.Швидкостi осiдання еритроцитiв Збірник тестових завдань для складання
C. Утворення антикоагулянтiв ліцензійного іспиту Крок-1 «Загальна
D. Утворення еритропоетинiв лікарська підготовка». –2008. - № 188.
E. Осмотичного тиску кровi
Збірник тестових завдань для складання 95. Внаслідок посттрансляційних змін
ліцензійного іспиту Крок-1 «Загальна деяких білків, що приймають участь у
лікарська підготовка». –2009. - № 153 . згортанні крові, зокрема протромбіну, вони
набувають здатність зв’язувати
92. У хворого спостерiгаються геморагiї, в кальцій.Уцьому процесі приймає участь
кровi знижена концентрацiя протромбiну. вітамін:
Недостатнiсть якого вiтамiну призвела до A. K
порушення синтезу цього фактору згортання B. C
кровi? C. A
A. K D. B1
B. A E. B2
C. D Збірник тестових завдань для складання
D. C ліцензійного іспиту Крок-1 «Загальна
E. E лікарська підготовка». –2008. - № 95.
Збірник тестових завдань для складання
ліцензійного іспиту Крок-1 «Загальна 96. Для попередження післяопераційної
лікарська підготовка». –2011. - № 134. кровотечі 6-ти річній дитині рекомендовано
приймати вікасол, який є синтетичним
93. У хворого 37-ми років на фонi тривалого аналогом вітаміну K. Вкажіть, які
застосування антибіотиків спостерiгається посттрансляційні зміни факторів згортання
підвищена кровоточивiсть при невеликих крові активуються під впливом вікасолу?
пошкодженнях. У кровi - зниження A. Карбоксилювання глутамінової кислоти
активності факторiв згортання кровi, B. Фосфорилювання радикалів серину
подовження часу згортання кровi. C. Частковий протеоліз
Недостатнiстю якого вiтамiну обумовленi D. Полімеризація
зазначенi змiни? E. Глікозилювання
A. Вiтамiн К Збірник тестових завдань для складання
B. Вiтамiн А ліцензійного іспиту Крок-1 «Загальна
C. Вiтамiн С лікарська підготовка». –2007. - № 102
D. Вiтамiн D
E. Вiтамiн Е 97. За кілька днів до операції пацієнту
Збірник тестових завдань для складання призначають вітамін K або його синтетичний
ліцензійного іспиту Крок-1 «Загальна аналог вікасол. Вітамін K бере участь у
лікарська підготовка». –2016. - № 62. наступній посттрансляційній модифікації II,
VII, IX, X факторів згортання крові:
94. Плазмовi фактори згортання кровi A. Карбоксилювання
зазнають посттрансляцiйної модифiкацiї за B. Декарбоксилювання
участю вiтамiну K. Як кофактор, вiн C. Дезамінування
потрiбен у ферментнiй системi γ- D. Трансамінування
карбоксилювання бiлкових факторiв E. Глікозилювання
коагуляцiї кровi, завдяки збільшенню Збірник тестових завдань для складання
спорiдненостi їх молекул з iонами кальцiю. ліцензійного іспиту Крок-1 «Стоматологія».
– 2012. – № 171.
46
Збірник тестових завдань для складання
98. У пацієнта, що тривалий час страждає на ліцензійного іспиту Крок-1 «Загальна
кишковий дисбактеріоз, посилилися лікарська підготовка». –2010. - № 3
кровотечі внаслідок порушення
посттрансляційної модифікації II, VII, IХ і Х 102. Хлопчику 15-ти років встановлено
факторів згортання крові в печінці. Дефіцит діагноз: гострий вірусний гепатит.
якого вітаміну є причиною цього стану? Визначення якого показника крові необхідно
A. К провести для підтвердження гострого
B. B12 ураження печінкових клітин?
C. B9 A. Активність амінотрансфераз (АлАТ і
D. С АсАТ)
E. Р B. Вміст вільного і зв'язаного білірубіну
Збірник тестових завдань для складання C. Швидкість зсідання еритроцитів (ШЗЕ)
ліцензійного іспиту Крок-1 «Загальна D. Вміст холестерину
лікарська підготовка». –2015. - № 139. E. Вміст білкових фракцій
Test items for licensing examination Krok 1
99. Новонароджена дитина має численні «Medicine». - 2016. - № 11.
крововиливи. Дослідження згортання крові
показують підвищений протромбіновий час. 103. У хворого через 12 годин після гострого
Аномалія яких біохімічних процесів нападу загрудинного болю знайдено різке
найбільш ймовірно має місце у дитини? підвищення активності АсАТ в сироватці
A. Формування гамакарбоксіглутамату крові. Вкажіть патологію, для якої
B. Конверсія гомоцистеїну в метіонін характерно це зміщення:
C. Конверсія метілмалоніл КоА в сукцинил A. Інфаркт міокарда
КоА B. Вірусний гепатит
D. Деградація глутатіону C. Колагенози
E. Гідроксилювання пролину D. Цукровий діабет
Test items for licensing examination Krok 1 E. Нецукровий діабет
«Medicine». - 2014. - № 134. Test items for licensing examination Krok 1
«Medicine». - 2007. - № 22.
100. Активація ряду факторів системи
гемостазу здійснюється шляхом приєднання 104. Чоловік 43-х років після вживання
до них іонів кальцію. Наявність якого жирної їжі та алкоголю скаржиться на
структурного компонента в їх складі сильний біль в животі. У сироватці крові
забезпечує приєднання іонів кальцію? вміст трипсину становить 850 ммоль / (год •
A. γ-карбоксіглутаміновая кислота л) (норма 60-240 ммоль / (год • л)). Для якої
B. γ-аміномасляна кислота патології системи травлення це найбільш
C. γ-оксимасляная кислота характерно?
D. Гідроксипролін A. Гострий панкреатит
E. Моноамінодикарбонові кислоти B. Динамічна кишкова непрохідність
Test items for licensing examination Krok 1 C. Механічна кишкова непрохідність
«Medicine». - 2016. - № 160. D. Виразка шлунка
E. Гепатит
101. Хвора 46-ти рокiв довгий час страждає Сборник тестовых заданий для
на прогресуючу м’язову дистрофію лицензионного экзамена Крок-1 «Медицина».
(Дюшена). Змiни рiвня якого ферменту кровi - 2011. - № 138.
є дiагностичним тестом вданому випадку?
A. Креатинфосфокiназа 105. Для біохімічної діагностики інфаркту
B. Лактатдегiдрогеназа міокарду необхідно визначити активність
C. Пiруватдегiдрогеназа декількох ферментів та їх ізоферментних
D. Глутаматдегiдрогеназа форм в крові. Аналіз на який фермент
E. Аденiлаткiназа вважається оптимальним для підтвердження
або спростування діагнозу інфаркт міокарду

47
на ранній стадії після появи у пацієнта 109. У сироватці крові пацієнта підвищена
торакального болю? активність гіалуронідази. Визначення якого
A. MB ізоформи креатинкінази біохімічного показника сироватки крові
B. MM ізоформи креатинкінази дозволить підтвердити припущення про
C. Ізоферменту ЛДГ-1 патологію сполучної тканини?
D. Ізоферменту ЛДГ-5 A. Сіалові кислоти
E. Цитоплазматичного ізоферменту B. Білірубін
аспартат-амінотрансферази C. Сечова кислота
Збірник тестових завдань для складання D. Глюкоза
ліцензійного іспиту Крок-1 «Стоматологія». E. Галактоза
– 2017. – № 105. Сборник тестовых заданий для
лицензионного экзамена Крок-1
106. У кровi хворого виявлено пiдвищення «Стоматология». - 2011. - № 116.
активностi ЛДГ4,5, АлАТ,
карбамоїлорнiтинтрансферази. В якому 110. При дослiдженнi кровi хворого
органi можна передбачити розвиток виявлено значне збiльшення активностi МВ-
патологiчного процесу? форм КФК (креатинфосфокiнази) та ЛДГ-1.
A. Печiнка (можливий гепатит) Яку патологiю можна припустити?
B. Серцевий м’яз (можливий iнфаркт A. Iнфаркт мiокарда
мiокарда) B. Гепатит
C. Скелетнi м’язи C. Ревматизм
D. Нирки D. Панкреатит
E. Сполучна тканина E. Холецистит
Збірник тестових завдань для складання Збірник тестових завдань для складання
ліцензійного іспиту Крок-1 «Загальна ліцензійного іспиту Крок-1 «Загальна
лікарська підготовка» – 2011. – № 46. лікарська підготовка» – 2014. – № 15.

107. У кровi хворого концентрацiя 111. У кровi хворого виявлено пiдвищення


альбумiнiв складає 2,8 г/л, пiдвищена активностi ЛДГ1, ЛДГ2, АсАТ,
концентрацiя лактатдегiдрогенази 5 (ЛДГ5). креатинкiнази. В якому органi хворого
Про захворювання якого органа це свiдчить? найбiльш вiрогiдний розвиток патологiчного
A. Печiнка процесу?
B. Нирка A. Серце
C. Серце B. Пiдшлункова залоза
D. Легеня C. Печiнка
E. Селезiнка D. Нирки
Збірник тестових завдань для складання E. Скелетнi м’язи
ліцензійного іспиту Крок-1 «Загальна Збірник тестових завдань для складання
лікарська підготовка» – 2005. – № 35. ліцензійного іспиту Крок-1 «Загальна
лікарська підготовка» – 2005. – № 52.
108. При аналізі крові у хворого виявлено,
що концентрація альбуміну становить 20 г/л, 112. У крові пацієнта спостерігається
підвищена активність ізоферменту підвищення активності АсАТ, ЛДГ-1, ЛДГ-2
лактатдегідрогенази 5 (ЛДГ 5). Про і КФК. Патологічний процес найбільш
захворювання якого органу свідчить цей імовірно розвивається у:
результат? A. Серці
A. Печінка B. Скелетних м’язів
B. Нирки C. Нирках
C. Серце D. Печінці
D. Легені E. Надниркових залозах
E. Селезінка Збірник тестових завдань для складання
Test items for licensing examination Krok 1 ліцензійного іспиту Крок-1 «Стоматологія».
«Medicine». - 2017. - № 27. – 2017. – № 114.

48
113. У плазмi кровi пацiєнта зпiдвищилась Збірник тестових завдань для складання
активнiсть iзоферментiв ЛДГ1 i ЛДГ2. Про ліцензійного іспиту Крок-1 «Загальна
патологiю якого органа це свiдчить? лікарська підготовка» – 2013. – № 130.
A. Мiокард
B. Печiнка 117. До вiддiлення реанiмацiї надiйшов
C. Нирки чоловiк 47-ми рокiв з дiагнозом iнфаркт
D. Мозок мiокарду. Яка з фракцiй лактатдегiдрогенази
E. Скелетнi м’язи (ЛДГ) буде переважати в сироватцi кровi
Збірник тестових завдань для складання протягом перших двох дiб?
ліцензійного іспиту Крок-1 «Стоматологія». A. ЛДГ 1
– 2017. – № 154. B. ЛДГ 2
C. ЛДГ 3
114. У хворого виявлено пiдвищення D. ЛДГ 4
активностi ЛДГ1,2, АсАТ, E. ЛДГ 5
креатинфосфокiнази. В якому органi Збірник тестових завдань для складання
(органах) найбiльш вiрогiдний розвиток ліцензійного іспиту Крок-1 «Загальна
патологiчного процесу? лікарська підготовка» – 2016. – № 34.
A. Серцевий м’яз
B. Скелетнi м’язи 118. В плазмі крові пацієнта виявлено
C. Нирки та наднирковi залози високий рівень ізоформ лактатдегідрогенази
D. Сполучна тканина (ЛДГ) за рахунок зростания концентрації
E. Печiнка та нирки ЛДГ-1 та ЛДГ-2. Вкажіть найбільш
Збірник тестових завдань для складання імовірний діагноз:
ліцензійного іспиту Крок-1 «Загальна A. Інфаркт міокарду
лікарська підготовка» – 2007. – № 11. B. Дистрофія скелетних м’язів
C. Цукровий діабет
115. Хворий 49 років водій за професією D. Вірусний гепатит
скаржиться на нестерпні стискаючі болі за E. Гострий панкреатит
грудиною, що “віддають” у ділянку шиї, які Збірник тестових завдань для складання
виникли 2 години тому. Стан важкий, ліцензійного іспиту Крок-1 «Загальна
блідість, тони серця послаблені. Лабораторне лікарська підготовка» – 2005. – № 52.
обстеження показало високу активність
креатинкинази та ЛДГ1. Для якого 119. 60-річний чоловік звернувся до лікаря
захворювання характерні подібні зміни? після появи болю в грудній клітці. В
A. Гострий інфаркт міокарда. сироватці крові виявлено значне зростання
B. Гострий панкреатит. активності ферментів: креатинфосфокінази
C. Стенокардія. та її МВ-ізоформи,
D. Жовчнокам'яна хвороба. аспартатамінотрансферази. Про розвиток
E. Цукровий діабет. патологічного процесу в якій тканині
Збірник тестових завдань для складання свідчать ці зміни?
ліцензійного іспиту Крок-1 «Загальна A. Сердцевий м’яз
лікарська підготовка» – 2009. – № 189. B. Легені
C. Скелетні м’язи
116. Через 6 годин після інфаркту міокарду у D. Печінка
хворого в крові піднялась активність E. Гладенькі м’язи
лактатдегідрогенази. Наявність якого Збірник тестових завдань для складання
ізоферменту треба очікувати в цьому ліцензійного іспиту Крок-1 «Стоматологія».
випадку? – 2013. – № 34.
A. ЛДГ 1
B. ЛДГ 2 120. Спадкова гiперлiпопротеїнемiя I типу
C. ЛДГ 3 обумовлена недостатнiстю
D. ЛДГ 4 лiпопротеїнлiпази. Пiдвищення рівня яких
E. ЛДГ 5 транспортних форм ліпідів в плазмi навiть
натщесерце є характерним?
49
A. Хiломiкрони головну причину атерогенезу в даному
B. Лiпопротеїни низької густини випадку:
C. Лiпопротеїни дуже низької густини A. Екзогенна гiперхолестеринемiя
D. Лiпопротеїни високої густини B. Переїдання
E. Модифiкованi лiпопротеїни C. Гiподинамiя
Збірник тестових завдань для складання D. Ендогенна гiперхолестеринемiя
ліцензійного іспиту Крок-1 «Загальна E. Cтрес
лікарська підготовка». –2016. - № 180. Збірник тестових завдань для складання
ліцензійного іспиту Крок-1 «Стоматологія».
121. Вміст холестеролу в сироватці крові 12- –2007. – № 124.
річного хлопця – 25 ммоль/л. В анамнезі
спадкова гіперхолестеринемія спричинена 125. У хворої дитини в кровi встановлено
порушенням синтезу рецепторів для: гiперлiпопротеїнемiю, що передалася у
A. Ліпопротеїнів низької щільності спадок. Генетичний дефект синтезу якого
B. Ліпопротеїнів високої щільності ферменту обумовлює це явище?
C. Хіломікронів A. Лiпопротеїнлiпаза
D. Ліпопротеїнів дуже низької щільності B. Глiкозидаза
E. Ліпопротеїнів проміжної щільності C. Протеїназа
Збірник тестових завдань для складання D. Гемсинтетаза
ліцензійного іспиту Крок-1 «Загальна E. Фенiлаланiнгiдроксилаза
лікарська підготовка». –2016. - № 176. Збірник тестових завдань для складання
ліцензійного іспиту Крок-1 «Стоматологія».
122. При обстеженнi хворого виявлено –2007. – № 104.
пiдвищення вмiсту в сироватцi кровi
лiпопротеїнiв низької щiльностi. Яке 126. Пiд час дослiдження плазми кровi
захворювання можна передбачити у цього пацiєнта через 4 години пiсля приймання
хворого? ним жирної їжi встановлено, що вона є
A. Атеросклероз каламутною. Найбiльш ймовiрною
B. Ураження нирок причиною даного стану є пiдвищення
C. Гострий панкреатит концентрацiї в плазмi:
D. Гастрит A. Хiломiкронiв
E. Запалення легень B. ЛПВГ
Збірник тестових завдань для складання C. ЛПНГ
ліцензійного іспиту Крок-1 «Стоматологія». D. Холестерину
–2015. – № 39. E. Фосфолiпiдiв
Збірник тестових завдань для складання
123. Чоловiк 60-ти рокiв страждає на ліцензійного іспиту Крок-1 «Стоматологія».
атеросклероз судин головного мозку. При –2005. – № 126.
обстеженнi виявлена гiперлiпiдемiя. Вмiст
якого класу лiпопротеїнiв найбiльш iмовiрно 127. Сироватка крові хворого має молочний
буде пiдвищений при дослiдженнi сироватки вигляд. Біохімічний аналіз виявив високий
кровi? вміст триацилгліцеролів і хіломікронів. Цей
A. Лiпопротеїди низької щiльностi стан викликано спадковим дефектом
B. Лiпопротеїди високої щiльностi наступного ферменту:
C. Комплекси жирних кислот з альбумiнами А. Ліпопротеїнліпаза
D. Хiломiкрони Б. Фосфоліпаза
E. Холестерин C. Панкреатична ліпаза
Збірник тестових завдань для складання D. Гормон-чутлива ліпаза жирової тканини
ліцензійного іспиту Крок-1 «Стоматологія». E. Фосфодіестераза
–2014. – № 100. Test items for licensing examination Krok 1
«Stomatology». - 2018. - № 149.
124. Кролiв годували їжею з додаванням
холестерину. Через 5 мiсяцiв виявленi 128. Пiд час обстеження пiдлiтка,
атеросклеротичнi змiни в аортi. Назвiть щостраждає на ксантоматоз, виявлена
50
сiмейна гiперхолестеринемiя. Концентрацiя Збірник тестових завдань для складання
яких лiпопротеїнiв значно пiдвищена в кровi ліцензійного іспиту Крок-1 «Загальна
при цiй патологiї? лікарська підготовка». –2005. - № 11.
A. ЛПНГ
B. Хiломiкрони 132. До лікаря звернулася жінка 42-х років,
C. ЛПДНГ що тривалий час дотримується
D. ЛПВГ вегетаріанства. При обстеженні виявили
E. НЕЖК негативний баланс азоту. Який фактор,
Збірник тестових завдань для складання найбільш ймовірно, привів до такого стану?
ліцензійного іспиту Крок-1 «Загальна A. Недостатня кількість білків в раціоны
лікарська підготовка». –2006. - № 56. B. Недостатнє вживання клітковини
C. Надмірна кількість жиру в раціоні
129. Рівень якого білка плазми крові D. Недостатня кількість жиру в раціоні
дозволяє ретроспективно (за попередні 4-8 E. Зниження інтенсивності метаболічних
тижні до обстеження) оцінити рівень процесів в організмі
глікемії, якщо пацієнт хворіє на цукровий Test items for licensing examination Krok 1
діабет, що супроводжується гіперглікемією «Stomatology». - 2017. - № 110.
натще понад 7,2 ммоль/л?
A. Глікозильованого гемоглобіну. 133. Пiд час обстеження чоловiка 45 рокiв,
B. Альбуміну. який перебуває довгий час на вегетерiанськiй
C. С-Реактивного білка. рослиннiй дiєтi, виявленонегативний
D. Церулоплазміну. азотистий баланс. Яка особливiсть рацiону
E. Фібриногену. стала причиною цього?
Збірник тестових завдань для складання A. Недостатня кiлькiсть бiлкiв
ліцензійного іспиту Крок-1 «Загальна B. Недостатня кiлькiсть жирiв
лікарська підготовка». –2012. - № 11. C. Надлишкова кiлькiсть води
D. Надлишкова кiлькiсть вуглеводiв
130. У пацієнта діагностовано E. Недостатня кiлькiсть вітамінів
панкреатичний діабет, який супроводжується Збірник тестових завдань для складання
гіперглікемією. Рівень глікемії можна ліцензійного іспиту Крок-1 «Загальна
оцінити ретроспективно (за 4-8 тижнів до лікарська підготовка». –2005. - № 134.
обстеження) шляхом вимірювання
концентрації наступного білка плазми крові: 134. У дитини 14-ти років був виявлений
А. Глікозильований гемоглобін позитивний азотистий баланс. Що з
B. Альбумін наведеного може бути причиною цього?
C. Фібриноген A. Ріст організму
D. С-реактивний білок B. Голодування
Е. Церулоплазмін C. Зниження вмісту білка в їжі
Test items for licensing examination Krok 1 D. Значні фізичні навантаження
«Stomatology». - 2015. - № 142. E. Емоційне напруження
Test items for licensing examination Krok 1
131. Людина хворiє на цукровий дiабет, що «Medicine». - 2011. - № 82.
супроводжується гiперглiкемiєю натще
понад 7,2 ммоль/л. Рiвень якого бiлка плазми 135. Через місяць після серйозної операції у
кровi дозволяє ретроспективно (за попереднi 38-річного пацієнта виявлений позитивний
4-8 тижнi до обстеження) оцiнити рiвень азотистий баланс. У сечі цього пацієнта
глiкемiї? можна виявити низьку концентрацію
A. Глiкозильований гемоглобiн наступної азотовмісного речовини:
B. Альбумiн А. Сечовина
C. Фiбриноген B. Лактат
D. С-реактивний бiлок C. Стеркобіліноген
E. Церулоплазмiн D. Галактоза
Е. 17-кетостероїди

51
Test items for licensing examination Krok 1
«Stomatology». - 2011. - № 68. 140. На основi лабораторного аналiзу у
хворого пiдтверджено дiагноз - подагра. Для
136. У дитини встановлена гостра ниркова встановлення дiагнозу було проведено
недостатнiсть. Якими бiохiмiчними визначення вмiсту:
показниками слини це можна пiдтвердити? A. Сечової кислоти в кровi та сечi
A. Пiдвищення рiвня залишкового азоту B. Креатинiну в сечi
B. Збiльшення iмуноглобулiну А C. Залишкового азоту в кровi
C. Зниження лужної фосфатази D. Сечовини в кровi та сечi
D. Збiльшення альфа-амiлази E. Амiаку в сечі
E. Зменшення рiвня фосфату Збірник тестових завдань для складання
Збірник тестових завдань для складання ліцензійного іспиту Крок-1 «Загальна
ліцензійного іспиту Крок-1 «Стоматологія». лікарська підготовка». –2013. - № 24.
–2007. – № 101.
141. У юнака 18-ти рокiв діагностована
137. У хворого, який виходить зi стану м’язова дистрофiя. Пiдвищення в сироватцi
тривалого голодування, визначили обмiн кровi вмiсту якої речовини найбiльш
азоту. Який результат можна очiкувати? iмовiрне при цiй патологiї?
A. Зниження видiлення азоту A. Креатин
B. Збiльшення видiлення азоту B. Мiоглобiн
C. Азотна рiвновага C. Мiозин
D. Азотний баланс не змiнився D. Лактат
E. Кетонемiя E. Аланiн
Збірник тестових завдань для складання Збірник тестових завдань для складання
ліцензійного іспиту Крок-1 «Стоматологія». ліцензійного іспиту Крок-1 «Загальна
–2014. – № 164. лікарська підготовка». –2011. - № 62.

138. У хворого на первинний нефротичний 142. Хвора 46-ти рокiв скаржиться на сухiсть
синдром встановлений вмiст загального в ротi, спрагу, почащений сечопуск, загальну
бiлку кровi 40 г/л. Яка причина обумовила слабкiсть. У кровi: гiперглiкемiя,
гiпопротеїнемiю? гiперкетонемiя. У сечi: глюкоза, кетоновi
A. Протеїнурiя тiла. На ЕКГ: дифузні змiни в мiокардi. Який
B. Вихiд бiлку з судин у тканини найбiльш iмовiрний дiагноз?
C. Зниження синтезу бiлку в печiнцi A. Цукровий дiабет
D. Пiдвищений протеолiз B. Алiментарна гiперглiкемiя
E. Порушення всмоктування бiлку в C. Гострий панкреатит
кишечнику D. Нецукровий дiабет
Збірник тестових завдань для складання E. Iшемiчна хвороба серця
ліцензійного іспиту Крок-1 «Стоматологія». Збірник тестових завдань для складання
–2007. – № 125. ліцензійного іспиту Крок-1 «Загальна
лікарська підготовка». –2012. - № 2.
139. У хворого, що страждає на хронiчну
ниркову недостатнiсть, виявлено у кровi 143. Хворий пiсля перенесеного
пiдвищення рiвня залишкового азоту до 35 епiдемiчного паротиту схуднув, постiйно
ммоль/л, бiльше половини якого складає вiдчуває спрагу, п’є багато води, відмічає
сечовина. Виявлена гiперазотемiя є: часте сечовидiлення, пiдвищений апетит,
A. Ретенцiйною шкiрний свербiж, слабкiсть, фурункульоз. У
B. Печiнковою кровi: глюкоза - 16 ммоль/л, кетонових тiл -
C. Продукцiйною 100 мкмоль/л; глюкозурiя. Яке захворювання
D. Резидуальною розвинулось у пацiєнта?
E. Змiшаною A. Iнсулiнозалежний цукровий дiабет
Збірник тестових завдань для складання B. Iнсулiнонезалежний цукровий дiабет
ліцензійного іспиту Крок-1 «Стоматологія». C. Стероїдний дiабет
–2010. – № 140. D. Нецукровий дiабет
52
E. Цукровий дiабет недостатнього 147. Хворий 48-ми років звернувся до лікаря
харчування зі скаргами на сильні болі, припухлість,
Збірник тестових завдань для складання почервоніння шкіри над суглобами,
o
ліцензійного іспиту Крок-1 «Загальна підвищення температури до 38 C.В крові
лікарська підготовка». –2012. - № 92. виявлено високий вміст уратів. Ймовірною
причиною такого стану може бути
144. Хвора доставлена бригадою швидкої порушення обміну:
допомоги. Об’єктивно: стан важкий, A. Пуринів
свiдомiсть вiдсутня, адинамiя. Шкiрнi B. Колагену
покриви сухi, запалi очi, ціаноз обличчя, C. Холестерину
тахiкардiя, запах ацетону з рота. Результати D. Піримідинів
аналiзiв: глюкоза кровi -20,1 ммоль/л (у E. Вуглеводів
нормi - 3,3-5,5 ммоль/л), у сечi - 3,5% (у Test items for licensing examination Krok 1
нормi - 0). Який найбiльш вiрогiдний «Medicine». - 2010. - № 22.
дiагноз?
A. Гiперглiкемiчна кома 148. У хворого в кровi пiдвищений вмiст
B. Гiпоглiкемiчна кома сечової кислоти, що клiнiчно проявляється
C. Гостра серцева недостатнiсть больовим синдромом внаслiдок вiдкладення
D. Гостре алкогольне отруєння уратiв у суглобах. У результатi якого
E. Анафiлактичний шок процесу утворюється ця кислота?
Збірник тестових завдань для складання A. Розпад пуринових нуклеотидiв
ліцензійного іспиту Крок-1 «Загальна B. Розпад пiримiдинових нуклеотидiв
лікарська підготовка». –2007. - № 28. C. Катаболiзм гему
D. Розщеплення бiлкiв
145. Хвора 38-ми рокiв надiйшла до E. Реутилiзацiя пуринових основ
реанiмацiйного вiддiлення в несвiдомому Збірник тестових завдань для складання
станi. Рефлекси вiдсутнi. Цукор кровi - 2,1 ліцензійного іспиту Крок-1 «Стоматологія».
ммоль/л. В анамнезi - цукровий дiабет з 18- –2007. – № 107.
ти рокiв. Яка кома маємiсце у хворої?
A. Гiпоглiкемiчна 149. У 19-місячної дитини з затримкою
B. Кетоацидотична розвитку та проявами самоагресії вміст
C. Лактацидемiчна сечової кислоти в крові - 1,96 ммоль / л. При
D. Гiперосмолярна якому метаболічному порушенні це
E. Гiперглiкемiчна спостерігається?
Збірник тестових завдань для складання A. Синдром Леша-Ніхана
ліцензійного іспиту Крок-1 «Загальна B. Подагра
лікарська підготовка». –2012. - № 96. C. Синдром набутого імунодефіциту
D. Хвороба Гірке
146. У хворого з дiагнозом хвороба Iценка- E. Хвороба Іценко-Кушинга
Кушинга (гiперпродукцiя гормонiв корою Test items for licensing examination Krok 1
наднирникiв) в кровi визначено пiдвищену «Stomatology». - 2011. - № 157.
концентрацiю глюкози, кетонових тiл,
натрiю. Який бiохiмiчний механiзм є 150. У пана С. представлені всі ознаки
провiдним у виникненнi гiперглiкемiї? печінкової коми: втрата свідомості,
A. Глюконеогенез відсутність рефлексів, кольки, судоми,
B. Глiкогенез порушення серцевої діяльності,
C. Глiкогенолiз переривчасте (періодичне) дихання. Яка
D. Глiколiз церебротоксична речовина накопичується в
E. Аеробний гліколіз крові при печінковій недостатності?
Збірник тестових завдань для складання А. Аміак
ліцензійного іспиту Крок-1 «Загальна В. IL-1
лікарська підготовка». –2006. - № 192. C. Аутоантитіла
D. Некрозогенні речовини
E. Кетонові тіла
53
Test items for licensing examination Krok 1 154. Азот виводиться з організму в
«Medicine». - 2005. - № 143. основному у вигляді сечовини. Коли
активність певного ферменту в печінці
151. У дитини 3-х рокiв пiсля перенесеної низька, це призводить до пригнічення
важкої вiрусної iнфекцiї вiдзначаються синтезу сечовини і нагромадження азоту в
повторне блювання, непритомнiсть, судоми. крові і тканинах. Назвіть цей фермент:
При дослiдженнi виявлена гiперамонiємiя. З А. Карбамоілфосфатсінтетаза
чим може бути пов’язана змiна бiохiмiчних B. Аспартатамінотрансфераза
показникiв кровi у цiєї дитини? C. Уреаза
A. Порушення знешкодження амiаку в Д. Амілаза
орнiтиновому циклi Е. Пепсин
B. Активацiя процесiв декарбоксилювання Test items for licensing examination Krok 1
амiнокислот «Medicine». - 2017. - № 51.
C. Порушення знешкодження бiогенних
амiнiв 155. У хворого знижений вміст індікана в
D. Посилення гниття бiлкiв у кишечнику сироватці крові, а також знижена його
E. Пригнiчення активностi ферментiв добова екскреція з сечею. Порушення
трансамiнування функції якого органу є причиною цього?
Збірник тестових завдань для складання A. Печінка
ліцензійного іспиту Крок-1 «Загальна B. Нирки
лікарська підготовка». –2006. - № 178. C. Серце
D. Легкі
152. У дитини 2-х років спостерiгається E. Підшлункова залоза
відставання в розумовому розвитку, Сборник тестовых заданий для
непереносимість білкової їжi, важка лицензионного экзамена Крок-1 «Медицина».
гiперамонiємiя на тлi зниженого вмiсту - 2011. - № 169.
сечовини в плазмi кровi, що пов’язано з
вродженим дефіцитом такого ферменту 156. У дитини при лабораторному
мiтохондрiй: дослідженні виявлено підвищений вміст в
A. Карбомоїлфосфатсинтетаза крові і сечі лейцину, валіну, ізолейцину та їх
B. Цитратсинтаза кетопохідних. Сеча мала характерний запах
C. Сукцинатдегiдрогеназа кленового сиропу. Недостатність якого
D. Малатдегiдрогеназа ферменту характерна для цього
E. Моноамiнооксидаза захворювання?
Збірник тестових завдань для складання A. Дегідрогеназа розгалужених амінокислот
ліцензійного іспиту Крок-1 «Стоматологія». B. Амінотрансфераза
–2016. – № 172. C. Глюкозо-6-фосфатаза
D. Фосфофруктокиназа
153. У новонародженої дитини E. Фосфофруктомутаза
спостерiгаються зниження iнтенсивностi Test items for licensing examination Krok 1
смоктання, часте блювання, гiпотонiя. У сечi «Medicine». - 2010. - № 117.
та кровi значно пiдвищена концентрацiя
цитрулiну. Який метаболiчний процес 157. У хворої дитини в сечі виявили
порушений? підвищений рівень фенілпірувата (в нормі
A. Орнiтиновий цикл практично відсутній). Зміст фенілаланіну в
B. ЦТК крові становить 350мг/л (норма приблизно
C. Глiколiз 15 мг/л). Вкажіть, для якого захворювання
D. Глюконеогенез характерні перераховані симптоми:
E. Цикл Корi A. Фенілкетонурія
Збірник тестових завдань для складання B. Альбінізм
ліцензійного іспиту Крок-1 «Загальна C. Тирозиноз
лікарська підготовка». –2009. - № 178. D. Алкаптонурія
E. Подагра

54
Test items for licensing examination Krok 1
«Medicine». - 2018. - № 143. 159. У крові хворих на цукровий діабет
спостерігається підвищення вмісту вільних
158. До лiкарнi надiйшов 9-рiчний хлопчик з жирних кислот (НЕЖК). Причиною цього
вiдставанням у розумовому i фiзичному може бути:
розвитку. Пiд час бiохiмiчного аналiзу кровi A. Підвищення активності трігліцерідліпази
виявлено пiдвищену кiлькiсть фенiлаланiну. адипоцитів
Блокування якого фермента може призвести B. Накопичення в цитозолі пальмітоїлКоА
до такого стану? C. Активація утилізації кетонових тіл.
A. Фенiлаланiн-4-монооксигеназа D. Активація синтезу аполіпопротеинів А-1,
B. Оксидаза гомогентизинової кислоти А-2, А-4
C. Глутамiнтрансамiназа E. Зниження активності фосфатидилхолін-
D. Аспартатамiнотрансфераза холестеринацилтрансферази плазми крові
E. Глутаматдекарбоксилаза Test items for licensing examination Krok 1
Збірник тестових завдань для складання «Medicine». - 2018. - № 59.
ліцензійного іспиту Крок-1 «Стоматологія».
–2005. – № 113.

55
Біохімія імунної системи
1. Диференцiювання В-лiмфоцитiв в 4. 13-річний хлопчик з екзематозними
плазматичні клітини призводить до висипаннями на гомілках і тулубі. Анамнез
вироблення iмуноглобулiнiв, якi показує на отит, пневмонію і фурункульоз у
вiдповiдають за специфічну імунну вiдповiдь пацієнта. Аналіз крові: тромбоцити - 70 • 109
органiзму. У якому органi імунної системи / л, низька активність Т-хелперів і Т-
відбувається диференціювання В- супресорних клітин, низький IgM, з
лiмфоцитiв? нормальними IgA і IgG. Яке імунодефіцитне
A. Мигдалики захворювання у цього хлопчика?
B. Червоний кістковий мозок А. Синдром Віскотта-Олдріча
C. Печiнка B. Синдром Луї-Бар (Ataxiatelangiectasia)
D. Тимус C. Важкий комбінований імунодефіцит
E. Щитоподiбна залоза (швейцарський тип)
Збірник тестових завдань для складання D. Синдром Ді-Джоржа
ліцензійного іспиту Крок -1 Е. Синдром Чедіака-Хигаси
«Стоматологія». – 2018. – №14. Test items for licensing examination Krok 1
«Medicine». – 2017. – № 196.
2. Серед лiмфоцитiв розрiзняють популяцiю
клiтин, що мають мембраннi рецептори до 5. Лiквiдатору наслiдкiв аварiї на
IgM, вони активуються пiд впливом Чорнобильськiй АЕС, що отримав велику
специфiчних антигенiв, мiтотично дозу опромiнення, проведено
розмножуються, диференцiюються у трансплантацiю кiсткового мозку. Через
плазматичнi клiтини, що виробляють деякий час після проведеної операцiї у
антитiла (iмуноглобулiни). Як називаються пацiєнта дiагностовано розвиток реакцiї
цi клiтини? трансплантат проти хазяїна. Якi антигени
A. В-лiмфоцити послужили пусковим механiзмом
B. Т-лiмфоцити пам’ятi виникнення цiєї реакцiї?
C. Т-лiмфоцити-кiлери A. Антигени системи HLA клiтин органiзму
D. Т-лiмфоцити-супресори лiквiдатора
E. – B. Антигени системи Rh еритроцитів
Збірник тестових завдань для складання лiквiдатора
ліцензійного іспиту Крок -1 C. Антигени HBs, HBc, Hbe
«Стоматологія». – 2013. – №101. D. Антигенами системи ABO еритроцитів
лiквiдатора
3. Хлопчик на другому роцi життя став часто E. Антигени системи HLA клiтин органiзму
хворiти на респiраторнi захворювання, донора
стоматити, гнійничкові ураження шкiри. Збірник тестових завдань для складання
Навiть невеликi пошкодження ясен i ліцензійного іспиту Крок -1 «Загальна
слизової ускладнюються запаленням, що лікарська підготовка». -2017. -№79.
протiкає тривало. Встановлено, що у кровi
дитини практично вiдсутнi імуноглобуліни 6. У чоловiка 36-ти рокiв пiсля перенесеної
усiх класiв. Зниження функціональної стрептококової iнфекцiї дiагностовано
активностi якої клiтинної популяцiї лежить в гострий гломерулонефрит. Найбiльш
основi описаного синдрому? iмовiрно, що ураження базальної мембрани
A. В-лiмфоцити ниркових тiлець виникає внаслiдок
B. Т-лiмфоцити алергiчної реакцiї такого типу:
C. Нейтрофiли A. Iмунокомплексна
D. Макрофаги B. Анафiлактична
E. NK-лiмфоцити C. Цитотоксична
Збірник тестових завдань для складання D. Сповiльнена
ліцензійного іспиту Крок -1 «Загальна E. Стимулююча
лікарська підготовка». -2011. -№192. Збірник тестових завдань для складання
ліцензійного іспиту Крок -1 «Загальна
лікарська підготовка». -2013. -№114.
56
основнi компоненти визначають цю реакцiю
7. У дитини 5-ти рокiв діагностовано органiзму?
хворобу Брутона, яка проявляється у A. Мононуклеари, Т-лiмфоцити i лiмфокiни
важкому перебiгу бактерiальних iнфекцiй, B. Гранулоцити, Т-лiмфоцити i IgG
вiдсутностi В-лiмфоцитiв та плазматичних C. Плазматичнi клiтини, Т-лiмфоцити i
клiтин. Якi змiни вмiсту iмуноглобулiнiв лiмфокiни
будуть спостерiгатися в сироватцi кровi цiєї D. В-лiмфоцити, IgМ
дитини? E. Макрофаги, В-лiмфоцити i моноцити
A. Зменшення IgA, IgM Збірник тестових завдань для складання
B. Збiльшення IgA, IgM ліцензійного іспиту Крок -1 «Загальна
C. Зменшення IgD, IgE лікарська підготовка». -2012. -№5.
D. Збiльшення IgD, IgE
E. Змiн не буде 1. Хвора 27-ми рокiв закрапала в очі краплi,
Збірник тестових завдань для складання до складу яких входить пенiцилiн. Через
ліцензійного іспиту Крок -1 «Загальна декiлька хвилин з’явився свербiж та печiння
лікарська підготовка». -2006. -№195. тiла, набряк губ та повiк, свистячий кашель;
став падати артерiальний тиск. Якi
8. Обстеження дитини, який часто страждає імуноглобуліни беруть участь в розвитку
інфекційними захворюваннями, виявило, що даної алергiчної реакцiї?
концентрація IgG в сироватці крові була в 10 A. IgE та IgG
разів нижча за норму, а також концентрація B. IgM та IgG
IgA і IgM була значно знижена. Аналіз C. IgA та IgM
показав також відсутність B-лімфоцитів і D. IgM та IgD
плазматичних клітин. Для якого E. IgG та IgD
захворювання характерні ці симптоми? Збірник тестових завдань для складання
А. Хвороба Брутона ліцензійного іспиту Крок -1 «Загальна
Б. Агаммаглобулінемія швейцарського типу лікарська підготовка». -2007. -№54.
C. Дисімуноглобулінемія
D. Синдром Луї-Бар 12. Дитина 3-х рокiв з множинними
Е. Синдром Ді Джорджа порушеннями розвитку кiсток лицевого
Test items for licensing examination Krok 1 вiддiлу черепа померла. Причина смертi -
«Medicine». – 2012. – № 158. сепсис, який розвинувся на фонi
бронхопневмонiї. В кровi вмiст
9. Батьки 5-річного хлопчика на прийомі у iмуноглобулiнiв в межах фiзiологiчної
лікаря відзначають неодноразові простудні норми. На розтинi встановлена вiдсутнiсть
захворювання у дитини, які закінчуються тимусу. Назвiть головну причину хвороби
розвитком пневмонії; є гнійні висипання на дитини:
шкірі. При лабораторному дослідженні A. Синдром недостатностi клітинного
виявлено: відсутність всіх типів iмунiтету
імуноглобулінів і відсутність В-лімфоцитів в B. Синдром комбiнованого iмунодефiциту
пунктаті лімфовузлів. Яке порушення C. Вторинний iмунодефiцитний синдром
імунної системи має місце? D. Гострий лiмфолейкоз
A. Гіпогаммаглобулінемія Брутона E. Синдром хронiчної інтоксикації
B. Імунодефіцит швейцарського типу Збірник тестових завдань для складання
С. Гіпопластична анемія ліцензійного іспиту Крок -1 «Загальна
D. Агранулоцитоз лікарська підготовка». – 2011. – №25.
E. Синдром Луї Барра
Test items for licensing examination Krok 1 13. Пiд час обстеження хворого була
«Medicine». – 2014. – № 85. виявлена недостатня кiлькiсть
iмуноглобулiнiв. Якi з клiтин iмунної
10. При пiдозрi на туберкульоз хворiй дитинi системи їх продукують?
зробили пробу Манту. Через 24 години у A. Плазматичнi
мiсцi введення алергену з’явились B. Т-хелпери
припухлiсть, гiперемiя i болiснiсть. Якi C. Т-кiлери
57
D. Т-супресори B. Ig A
E. Плазмобласти C. Ig D
Збірник тестових завдань для складання D. Ig E
ліцензійного іспиту Крок -1 «Загальна E. Ig M
лікарська підготовка». – 2005. – №73. Збірник тестових завдань для складання
ліцензійного іспиту Крок -1 «Загальна
14. Пiсля опромiнювання у людини з’явилася лікарська підготовка». – 2014. – №190.
велика кiлькiсть мутантних клiтин. Через
деякий час бiльшiсть iз них були розпiзнанi i 18. Профiлактичнi щеплення проти
знищенi клiтинами iмунної системи, а саме: полiомiєлiту здiйснили iнактивованою
A. T-лiмфоцитами-кiлерами вакциною, яку ввели парентерально. Якi
B. Плазмобластами імуноглобуліни вiдповiдають в цьому
C. T-лiмфоцитами-супресорами випадку за створення поствакцинального
D. B-лiмфоцитами iмунiтету?
E. Стовбуровими клiтинами A. IgM, IgG
Збірник тестових завдань для складання B. IgG, секреторнi IgA
ліцензійного іспиту Крок -1 «Загальна C. IgM, секреторнi IgA
лікарська підготовка». – 2009. – №175. D. Сироватковi IgА, IgM
E. Ig Е, IgM
15. У кровi дiвчини 16-ти рокiв, котра Збірник тестових завдань для складання
страждає на аутоiмунне запалення ліцензійного іспиту Крок -1 «Загальна
щитоподiбної залози, виявлено численні лікарська підготовка». – 2018. – №191.
плазматичнi клiтини. З проліферацією та
диференцiюванням яких клiтин кровi 19. Чоловiку 37-ми років при лiкуваннi
пов’язано збiльшення кiлькостi плазмоцитiв? гострого пульпiту було введено розчин
A. В-лiмфоцитiв новокаїну. Через кілька хвилин у пацiєнта
B. Т-хелперiв розвинувся анафiлактичний шок. З яким
C. Тканинних базофiлiв імуноглобуліном головним чином взаємодіє
D. Т-кiлерiв в органiзмi антиген при данiй алергiчнiй
E. Т-супресорiв реакцiї?
Збірник тестових завдань для складання A. IgE
ліцензійного іспиту Крок -1 B. IgM
«Стоматологія». – 2010. – №23. C. IgA
D. IgD
16. У хворого дiагностовано ГРВI. У E. IgG
сироватцi кровi знайдено iмуноглобулiни Збірник тестових завдань для складання
класу М. Який перiод інфекційного процесу ліцензійного іспиту Крок -1
в даному випадку? «Стоматологія». – 2016. – №30.
A. Гострий
B. Продромальний 20. Пiсля введення лiдокаїну в хворого 25-ти
C. Iнкубацiйний рокiв з’явились задишка, бронхоспазм, рiзко
D. Реконвалесценцiя знизився кров’яний тиск, що вимагало вiд
E. Мiкробоносiйство лiкарястоматолога застосування засобiв
Збірник тестових завдань для складання негайної допомоги. Який механiзм лежить в
ліцензійного іспиту Крок -1 «Загальна основi таких явищ?
лікарська підготовка». – 2013. – №187. A. Алергiчнi реакцiї за участю IgE
B. Алергiчнi цитотоксичнi реакцiї
17. У хворого 34-х рокiв пiсля перенесеної C. Явище iдiосинкразiї
кишкової iнфекцiї, викликаної D. Гiперчутливiсть, зумовлена Т-
сальмонелами, стали згасати симптоми лiмфоцитами
захворювання. Iмуноглобулiни якого класу E. Ефекти, зумовленi Т-кiлерами
будуть виявленi в кровi хворого в перiод Збірник тестових завдань для складання
реконвалесценцiї? ліцензійного іспиту Крок -1
A. Ig G «Стоматологія». – 2014. – №58.
58
D. Фiбриноген
21. У пацiєнта з бронхiальною астмою за E. Альбумiн
допомогою шкiрних алергiчних проб Збірник тестових завдань для складання
встановлено сенсибiлiзацiю алергеном ліцензійного іспиту Крок -1
тополиного пуху. Який фактор імунної «Стоматологія». – 2015. – №45.
системи вiдiграє вирiшальну роль в розвитку
цього iмунопатологiчного стану? 25. У мiсцевому iмунiтетi порожнини рота
A. IgE важливу роль вiдiграють рiзноманiтнi
B. IgD клiтини слизової оболонки i антимiкробнi
C. IgM речовини, якi синтезуються ними. Якi з
D. Т-лiмфоцити перерахованих факторiв мають вирiшальну
E. IgG роль у забезпеченнi локального iмунiтету?
Збірник тестових завдань для складання A. Секреторний IgA
ліцензійного іспиту Крок -1 «Загальна B. В-лiмфоцити
лікарська підготовка». – 2011. – №128. C. IgG
D. Макрофаги
2. У пiдлiтка було видалено зуб iз E. Еозинофiли
застосуванням новокаїну. Через 10 хвилин у Збірник тестових завдань для складання
нього з’явилися блiдiсть шкiрних покривiв, ліцензійного іспиту Крок -1
задишка, гiпотензiя. При розвитку цiєї «Стоматологія». – 2010. – №157.
реакцiї алерген на тканинних базофiлах
реагує з: 26. У нашій країні планові профілактичні
A. IgE щеплення проти поліомієліту здійснюються
B. IgA живою вакциною, яку вводять перорально.
C. IgD Які імуноглобуліни відповідають в цьому
D. IgM випадку за створення поствакцинального
E. Т-лiмфоцитами місцевого імунітету?
Збірник тестових завдань для складання A. Секреторні IgA
ліцензійного іспиту Крок -1 B. IgМ
«Стоматологія». – 2010. – №54. C. IgG
D. Сироваткові IgA
23. У хворого 34-х рокiв пiсля перенесеної E. IgЕ
кишкової iнфекцiї, викликаної Test items for licensing examination Krok 1
сальмонелами, стали згасати симптоми «Medicine». – 2013. – № 152.
захворювання. Iмуноглобулiни якого класу
будуть виявленi в кровi хворого в перiод 27. У жiнки 37-ми рокiв протягом року
реконвалесценцiї? перiодично виникали iнфекцiйнi
A. Ig G захворювання бактерiального генезу, їх
B. Ig A перебiг був вкрай тривалим, ремiсiї -
C. Ig D короткочасними. При обстеженнi виявлена
D. Ig E гiпогамаглобулiнемiя. Порушення функцiї
E. Ig M яких клiтин може бути прямою її причиною?
Збірник тестових завдань для складання A. Плазматичнi клiтини
ліцензійного іспиту Крок -1 «Загальна B. Фагоцити
лікарська підготовка». – 2014. – №190. C. Нейтрофiли
D. Макрофаги
24. Однiєю iз функцiй слини є захисна, яка E. Лiмфоцити
реалiзується, зокрема, формуванням Збірник тестових завдань для складання
мiсцевого iмунiтету слизової оболонки за ліцензійного іспиту Крок -1 «Загальна
рахунок видiлення привушними залозами лікарська підготовка». – 2015. – №53.
такого бiлку:
A. Секреторний iмуноглобулiн A 28. Пацiєнт госпіталізований з попереднiм
B. Колаген дiагнозом: гепатит В. Для дiагностики
C. Еластин захворювання здійснено постановку
59
серологічної реакцiї, яка базується на 32. У хворих iз синдромом набутого
взаємодії антигену з антитiлом, хімічно iмунодефiциту (СНIД) рiзко знижується
зв’язаним з пероксидазою або лужною iмунологiчна реактивнiсть, що проявляється
фосфатазою. Яку назву має використана розвитком хронiчних запальних процесiв,
серологiчна реакцiя? iнфекцiйних захворювань, пухлинного росту.
A. Iмуноферментний аналiз Клiтини якого типу ушкоджує ВIЛ-iнфекцiя,
B. Радiоiмунологiчний метод внаслiдок чого знижується iмунний захист?
C. Реакцiя iмунофлюоресценцiї A. Т4-хелпери
D. Реакцiя зв’язування комплементу B. Природнi кiлери (NK)
E. Реакцiя iмобiлiзацiї C. Т-супресори
Збірник тестових завдань для складання D. Т8-ефектори
ліцензійного іспиту Крок -1 E. В-лiмфоцити
«Стоматологія». – 2016. – №50. Збірник тестових завдань для складання
ліцензійного іспиту Крок -1 «Загальна
29. Пiсля обстеження хворого на лікарська підготовка». – 2013. – №104.
рецидивуючий афтозний стоматит з
супутнiм кандидозом лiкар вирiшив 3. У пацiєнтки 20-ти рокiв встановлено
виключити ВIЛ-iнфекцiю. Яке дослiдження дiагноз - СНIД. Якi популяцiї клiтин
допоможе прояснити ситуацiю та встановити найбiльш чутливi до вiрусу iмунодефiциту
попереднiй дiагноз? людини?
A. Iмуноферментний аналiз A. Т-хелпери
B. Реакцiя преципiтацiї у гелi B. Гепатоцити
C. Реакцiя гальмування гемаглютинацiї C. Ендотелiоцити
D. Реакцiя гемаглютинацiї D. Епiтелiоцити
E. Фазово-контрастна мiкроскопiя E. В-лiмфоцити
Збірник тестових завдань для складання Збірник тестових завдань для складання
ліцензійного іспиту Крок -1 ліцензійного іспиту Крок -1 «Загальна
«Стоматологія». – 2007. – №167. лікарська підготовка». – 2014. – №4.

30. Встановлено ураження вiрусом ВIЛ Т- 34. Дитина 1 року часто хворiє на вiрусно-
лiмфоцитiв. При цьому фермент вiрусу бактерiальнi iнфекцiї, якi погано пiддаються
зворотня траскриптаза (РНК-залежна ДНК- терапiї. Пiд час проведення дослiдження
полiмераза) каталiзує синтез: iмунологiчного статусу виявлено вiдсутнiсть
A. ДНК на матрицi вiрусної i-РНК у кровi лiмфоцитiв, що забезпечують
B. Вiрусна i-РНК на матрицi ДНК клiтинний iмунiтет. Який iмунодефiцит i за
C. ДНК на вiруснiй р-РНК якою системою виявлений у цiєї дитини?
D. Вiрусна ДНК на матрицi ДНК A. Первинний за Т-системою
E. i-РНК на матрицi вiрусного білка B. Вторинний за Т-системою
Збірник тестових завдань для складання C. Первинний за В-системою
ліцензійного іспиту Крок -1 «Загальна D. Первинний за макрофагальною системою
лікарська підготовка». – 2008. – №47. E. Первинний за мiкрофагами
Збірник тестових завдань для складання
31. Лiмфоцит уражений ретровiрусом ВIЛ ліцензійного іспиту Крок -1
(СНIД). В цьому випадку напрямок потоку «Стоматологія». – 2005. – №177.
iнформацiї в клiтинi буде:
A. РНК → ДНК → i-РНК → полiпептид 35. У сироватцi кровi новонародженого
B. ДНК → i-РНК → полiпептид →ДНК виявлено антитiла довiрусу кору. Про
C. ДНК → полiпептид → i-РНК наявнiсть якого iмунiтету це може свiдчити?
D. i-РНК → полiпептид → ДНК A. Природний пасивний
E. Полiпептид → РНК → ДНК →i-РНК B. Природний активний
Збірник тестових завдань для складання C. Штучний пасивний
ліцензійного іспиту Крок -1 «Загальна D. Штучний активний
лікарська підготовка». – 2014. – №71. E. Спадковий, видовий

60
Збірник тестових завдань для складання D. Iнтерлейкiн-3
ліцензійного іспиту Крок -1 «Загальна E. Iнтерлейкiн-4
лікарська підготовка». – 2016. – №171. Збірник тестових завдань для складання
ліцензійного іспиту Крок -1 «Загальна
36. Дитина народилася з вовчою пащею. При лікарська підготовка». – 2009. – №81.
обстеженнi виявлено вади аорти, в кровi -
зменшення Т-лiмфоцитiв. Який 40. У хворого з грибковим ураженням шкіри
iмунодефiцитний синдром у виявлено порушення клітинного імунітету.
новонародженого? Зниження якого показника найбільш
A. Дi Джорджi характерно при цьому?
B. Вiскотта-Олдрiча A. T-лімфоцити
C. Чедiака-Хiгасi B. Імуноглобуліни G
D. Луї-Бар C. Імуноглобуліни E
E. Швейцарський тип D. B-лімфоцити
Збірник тестових завдань для складання E. Плазмоцити
ліцензійного іспиту Крок -1 «Загальна Test items for licensing examination Krok 1
лікарська підготовка». -2008. -№124. «Medicine».-2010.- №145

37. У хворої після трансплантації печінки 41. При запальних процесах в органiзмi
через 1,5 місяці погіршився стан внаслідок починається синтез бiлкiв ”гострої фази”.
реакції відторгнення трансплантата. Який Якi речовини є стимуляторами їх синтезу?
фактор імунної системи відіграє вирішальну A. Iнтерлейкiн-1
роль в цій реакції? B. Iмуноглобулiни
A. Т-кілери C. Iнтерферони
B. Интерлейкин-1 D. Бiогеннi амiни
C. Натуральні кілери E. Ангiотензини
D. В-лімфоцити Збірник тестових завдань для складання
E. Т-хелпери ліцензійного іспиту Крок -1
Test items for licensing examination Krok 1 «Стоматологія». – 2017. – №39.
«Medicine». – 2010. – № 153.
42. Хворому з великими опiками зробили
38. До лікаря-імунолога на прийом звернувся пересадку донорської шкiри. На 8 -му добу
хворий зі скаргами на диарею, втрату ваги за трансплантат набряк, змiнився його колiр; на
кілька місяців, субфібрилітет, збільшення 11 добу почав вiдторгатися. Якi клiтини
лімфоузлів. Лікар запідозрив у паціента ВІЛ- беруть у цьому участь?
інфекцію. Вміст яких імунокомпетентних A. Т-лiмфоцити
клітин необхідно досліджувати у хворого в B. Еритроцити
першу чергу? C. Базофiли
A. Т-лімфоцити-хелпери D. Еозинофiли
B. Т-лімфоцити-супресори E. В-лiмфоцити
C. В-лімфоцити Збірник тестових завдань для складання
D. Моноцити ліцензійного іспиту Крок -1 «Загальна
E. Плазматичні клітини лікарська підготовка». – 2006. – №49.
Test items for licensing examination Krok 1
«Stomatology». – 2015. - № 146 43. До дерматолога звернулася пацiєнтка iз
скаргами на екзематозне ураження шкiри
39. У пацiєнта пiсля переливання 200 мл рук, що з’являється пiсля контакту з миючим
кровi пiдвищилася температура тiла до 37, засобом ”Лотос”. Використання гумових
9°C Яка з наведених речовин найбiльш рукавичок запобiгає цьому. Патологiчна
вiрогiдно призвела до пiдвищення реакцiя шкiри зумовлена активацiєю:
температури? A. Т-лiмфоцитiв
A. Iнтерлейкiн-1 B. В-лiмфоцитiв
B. Iнтерлейкiн-2 C. Моноцитiв
C. Фактор некрозу пухлин D. Нейтрофiлiв
61
E. Базофiлiв Збірник тестових завдань для складання
Збірник тестових завдань для складання ліцензійного іспиту Крок -1 «Загальна
ліцензійного іспиту Крок -1 «Загальна лікарська підготовка». – 2006. – №105.
лікарська підготовка». – 2017. – №32.
45. У вагітної жінки взяли кров для
4. Для розвитку гарячкових станiв підтвердження клінічного діагнозу
характерним є зростання рiвня бiлкiв "токсоплазмоз". Яка з перерахованих
"гострої фази церулоплазмiну, фiбриногену, серологічних реакцій має діагностичне
С-реактивного протеїну. Вкажiть можливий значення?
механiзм цього явища: A. Реакція зв'язування комплементу
A. Стимулюючий вплив IЛ-1 на гепатоцити B. Реакція нейтралізації
B. Руйнiвна дiя пiдвищеної температури на C. Реакція гемадсорбції
клiтини органiзму D. Реакція аглютинації
C. Пролiферативна дiя IЛ-2 на Т-лiмфоцити E. Реакція гальмування гемаглютинації
D. Дегрануляцiя тканинних базофiлiв Збірник тестових завдань для складання
E. - ліцензійного іспиту Крок -1 «Загальна
лікарська підготовка». – 2011. – №104.

62
Біохімічні функції печінки
1. Порушення процесiв розщеплення лiпiдiв Збірник тестових завдань для складання
у тонкому кишечнику зумовлено ліцензійного іспиту Крок-1 «Загальна
порушенням активності лiпази. Який з лікарська підготовка». –2008. - № 1.
наведених чинникiв активує лiпазу?
A. Жовчнi кислоти 5. Внаслідок блокування загальної жовчної
B. Соляна кислота протоки, що підтверджено радіографією,
C. Ентерокiназа надходження жовчі до дванадцятипалої
D. Пепсин кишки зупинилося. Очікується ослаблення:
E. Солi Na+ A. Емульгування жирів
Збірник тестових завдань для складання B. Всмоктування білків
ліцензійного іспиту Крок-1 «Стоматологія». C. Гідролізу вуглеводів
–2016. – № 88. D. Секреції соляної кислоти
E. Слиновиділення
2. При обстеженнi хворого встановлено, що Збірник тестових завдань для складання
причиною гіпоплазії зубiв є гiповiтамiноз А ліцензійного іспиту Крок-1 «Загальна
та D. Цi вiтамiни призначили перорально, лікарська підготовка». –2013. - № 174.
проте лiкувального ефекту не досягли. Яка
можлива причина порушення засвоєння 6. Хвора 65-ти рокiв страждає на
вiтамiнiв? жовчнокам’яну хворобу. Останнiм часом
A. Нестача жовчних кислот з’явилися ознаки ахолiчного синдрому
B. Нестача фосфоліпази А2 внаслiдок обтурацiї жовчних шляхiв.
C. Нестача холестеролестерази Засвоєння яких компонентів їжi буде
D. Нестача коліпази порушене найбiльше?
E. Нестача панкреатичної ліпази A. Жири
Збірник тестових завдань для складання B. Вуглеводи
ліцензійного іспиту Крок-1 «Стоматологія». C. Бiлки
–2008. – № 196. D. Нуклеїновi кислоти
E. Електролiти
3. При копрологiчному дослiдженнi Збірник тестових завдань для складання
встановлено, що кал знебарвлений, у ньому ліцензійного іспиту Крок-1 «Стоматологія».
знайдено краплi нейтрального жиру. –2011. – № 128.
Найбiльш вiрогiдною причиною цього є
порушення: 7. Препарат "Гептрал", який використовують
A. Надходження жовчi до кишечнику при хворобах печiнки, мiстить S-
B. Кислотностi шлункового соку аденозилметiонiн. Ця активна амiнокислота
C. Секрецiї пiдшлункового соку бере участь у синтезi:
D. Секрецiї кишкового соку A. Фосфолiпiдiв
E. Процесiв всмоктування в кишечнику B. Жовчних кислот
Збірник тестових завдань для складання C. Триацилглiцеролiв
ліцензійного іспиту Крок-1 «Загальна D. Холестерину
лікарська підготовка». –2010. - № 56. E. Гему
Збірник тестових завдань для складання
4. Пiсля вживання жирної їжi у хворого ліцензійного іспиту Крок-1 «Загальна
з’являються нудота та печiя, має мiсце лікарська підготовка». –2014. - № 85.
стеаторея. Причиною такого стану може
бути: 8. Хворому з метою попередження жирової
A. Нестача жовчі дистрофії печінки лiкар призначив
B. Пiдвищене видiлення лiпази ліпотропний препарат – донор метильних
C. Порушення синтезу трипсину груп. Це iмовiрно:
D. Нестача амiлази A. S-Аденозилметiонiн
E. Порушення синтезу фосфолiпази B. Холестерин
C. Бiлiрубiн
D. Валiн
63
E. Глюкоза C. Сечовини.
Збірник тестових завдань для складання D. Фосфатидної кислоти.
ліцензійного іспиту Крок-1 «Загальна E. Холевої кислоти.
лікарська підготовка». –2016. - № 99. Збірник тестових завдань для складання
ліцензійного іспиту Крок-1 «Загальна
9. Синтез фосфоліпідів порушується при лікарська підготовка». –2014. - № 17.
жировій інфільтрації печінки. Вкажіть, яка з
наступних речовин здатна покращити процес 13. У хлопчика 2-х рокiв спостерiгається
метилювання при синтезі фосфоліпідів: збiльшення в розмiрах печiнки та селезiнки,
A. Метіонін катаракта. В кровi пiдвищена концентрацiя
B. Аскорбінова кислота цукру, але тест толерантностідо глюкози в
C. Глюкоза нормi. Спадкове порушення обмiну якої
D. Гліцерин речовини є причиною цьогостану?
E. Цитрат A. Галактоза
Збірник тестових завдань для складання B. Фруктоза
ліцензійного іспиту Крок-1 «Загальна C. Глюкоза
лікарська підготовка». –2005. - № 9. D. Мальтоза
E. Сахароза
10. Стеатоз виникає внаслiдок накопичення Збірник тестових завдань для складання
триацилглiцеролiв у гепатоцитах. Одним з ліцензійного іспиту Крок-1 «Загальна
механiзмiв розвитку цього захворювання є лікарська підготовка». –2015. - № 20.
зменшення утилізації нейтрального жиру
ЛПДНЩ. Якi лiпотропнi речовини 14. Пiд час голодування м’язовi бiлки
попереджують розвиток стеатозу? розпадаються до вiльних амiнокислот. В
A. Метiонiн, BC, B12 якому процесi найбiльш iмовiрно будуть
B. Аргiнiн, B2, B3 використовуватися амiнокислоти за таких
C. Аланiн, B1, PP умов?
D. Валiн, B3, B2 A. Глюконеогенез у печiнцi.
E. Iзолейцин, B1, B2 B. Глюконеогенез у м’язах.
Збірник тестових завдань для складання C. Глiкогенолiз.
ліцензійного іспиту Крок-1 «Загальна D. Декарбоксилювання.
лікарська підготовка». –2013. - № 160. E. Синтез вищих жирних кислот.
Збірник тестових завдань для складання
11. У експериментальної тварини, що ліцензійного іспиту Крок-1 «Загальна
знаходиться на безбiлковому рацiонi, лікарська підготовка». –2010. - № 150.
розвинулася жирова інфільтрація печінки
внаслiдок дефiциту метилюючих агентiв. 15. У хворого, який проходить курс
Утворення якого метаболiту порушено у лiкувального голодування, нормальний
пiддослiдної тварини? рiвень глюкози у кровi підтримується
A. Холiн головним чином за рахунок глюконеогенезу.
B. ДОФА З якої амiнокислоти при цьому упечiнцi
C. Холестерин людини найбiльш активно синтезується
D. Ацетоацетат глюкоза?
E. Лiнолева кислота A. Аланiн
Збірник тестових завдань для складання B. Лiзин
ліцензійного іспиту Крок-1 «Стоматологія». C. Валiн
–2005. – № 115. D. Глутамiнова кислота
E. Лейцин
12. У робітника хімчистки виявлена жирова Збірник тестових завдань для складання
дистрофія печінки. Порушення синтезу якої ліцензійного іспиту Крок-1 «Загальна
речовини в печінці може привести до даної лікарська підготовка». –2011. - № 66.
патології?
A. Фосфатидилхоліну.
B. Тристеарину.
64
16. Глюконеогенез активується в печінці
після інтенсивних фізичних тренувань. Яка 20. Після введення адреналіну у пацієнтки з
речовина використовується в першу чергу: постійною гіпоглікемією аналіз крові
А. Лактат суттєво не змінився. За таких обставин
Б. Піруват ймовірні порушення в печінці. Про зміну
С. Глюкоза якої функції печінки йдеться?
D. Глутамат A. Глікогендепонуючої.
Е. Аланін B. Гліколітичну.
Збірник тестових завдань для складання C. Екскреторну.
ліцензійного іспиту Крок-1 «Загальна D. Кетогенну.
лікарська підготовка». –2005. - № 56. E. Холестериноутворюючу.
Збірник тестових завдань для складання
17. У хворих з глікогенозом, тобто хворобою ліцензійного іспиту Крок-1 «Стоматологія».
фон Гірке, інгібується перетворення – 2006. – № 92.
глюкозо-6-фосфату в глюкозу, що
супроводжується неналежним розщепленням 21. У рацiонi людини велика кiлькiсть
глікогену в печінці. Причиною цього стану є вуглеводiв. Кiлькiсть яких структур
наступний недолік ферментів: збiльшиться у цитоплазмi гепатоцитiв?
A. Глюкозо-6-фосфатаза A. Гранули глiкогену
B. Глікогенфосфорилаза B. Краплини жиру
C. Глюкозо-6-фосфатдегідрогеназа C. Лiзосоми
D. Фосфофруктокіназа D. Вiльнi рибосоми
Е. Фосфоглюкомутаза E. Включення лiпофусцину
Збірник тестових завдань для складання Збірник тестових завдань для складання
ліцензійного іспиту Крок-1 «Стоматологія». ліцензійного іспиту Крок-1 «Загальна
– 2014. – № 135. лікарська підготовка». –2005. - № 57.

18. У дитини спостерігаються гепатомегалия, 22. Основна маса азоту з органiзму


гіпоглікемія, судоми, особливо натщесерце і виводиться у виглядi сечовини. Зниження
при стресових ситуаціях. Діагноз - хвороба активностi якого ферменту в печiнцi
Гірке. Генетичний дефект якого ферменту призводить до гальмування синтезу
має місце при даної хвороби? сечовини i нагромадження амонiаку в кровi i
A. Глюкозо-6-фосфатаза тканинах?
B. Аміло-1,6-глікозідаза A. Карбамоїлфосфатсинтаза
C. Фосфоглюкомутаза B. Аспартатамiнотрансфераза
D. Глікогенфосфорилаза C. Уреаза
E. Глюкокіназа D. Амiлаза
Сборник тестовых заданий для E. Пепсин
лицензионного экзамена Крок-1 «Медицина». Збірник тестових завдань для складання
- 2014. - № 179. ліцензійного іспиту Крок-1 «Загальна
лікарська підготовка». –2009. – № 63.
19. У пацiєнтки з постiйною гiпоглiкемiєю
пiсля введення адреналiну аналiз кровi 23. При біосинтезі сечовини в печінці
суттєво не змiнився. Лiкар запiдозрив вібдувається утворення орнітину та
патологiю печiнки. Про змiну якої функцiї сечовини. Яка амінокислота є проміжним
печiнки може йти мова? продуктом цього синтезу?
A. Глiкогендепонуюча A. Аргінін.
B. Холестеринутворююча B. Лейцин.
C. Кетогенна C. Цитрат.
D. Глiколiтична D. Валін.
E. Екскреторна E. Триптофан.
Збірник тестових завдань для складання Збірник тестових завдань для складання
ліцензійного іспиту Крок-1 «Стоматологія». ліцензійного іспиту Крок-1 «Загальна
– 2006. – № 92. лікарська підготовка». –2007. – № 125
65
C.Жовчоутворююча
24. У хлопчика 4-х рокiв пiсля перенесеного D. Антитоксична
важкого вiрусного гепатиту спостерiгаються E. Глiкогенутворююча
блювання, епiзоди непритомностi, судоми. У Збірник тестових завдань для складання
кровi - гiперамонiємiя. Порушення якого ліцензійного іспиту Крок-1 «Стоматологія».
бiохiмiчного процесу в печiнцi викликало – 2016. – № 69.
такий стан хворого?
A. Знешкодження амонiаку. 28. У хворого, який знаходиться на лiкуваннi
B. Декарбоксилювання амiнокислот. з приводу вiрусного гепатиту В, з’явилися
C. Знешкодження бiогенних амiнiв. ознаки печiнкової недостатностi. Якi змiни
D. Синтезу бiлкiв. кровi, що свiдчать про порушення бiлкового
E. Глюконеогенезу. обмiну, найбiльш вiрогiдно
Збірник тестових завдань для складання спостерiгатимуться у цьому випадку?
ліцензійного іспиту Крок-1 «Стоматологія». A. Абсолютна гiпоальбумiнемiя
– 2015. – № 46. B. Абсолютна гiперальбумiнемiя
C. Абсолютна гiперфiбриногенемiя
25. При токсичному ушкодженнi клітин D. Бiлковий склад кровi не змiнений
печiнки з порушенням її функцiй у хворого E. Абсолютна гiперглобулiнемiя
з’явилися набряки. Якi змiни складу плазми Збірник тестових завдань для складання
кровi є провiдною причиною розвитку ліцензійного іспиту Крок-1 «Загальна
набрякiв? лікарська підготовка» – 2008. – № 112.
A. Зниження вмiсту альбумiнiв
B. Збiльшення вмiсту глобулiнiв 29. При обтурацiйнiй жовтяницi i жовчних
C. Зменшення вмiсту фiбриногену норицях часто спостерiгається
D. Збiльшення вмiсту альбумiнiв протромбiнова недостатнiсть. З дефiцитом в
E. Зменшення вмiсту глобулiнiв органiзмi якого вiтамiну це пов’язано?
Збірник тестових завдань для складання A. K
ліцензійного іспиту Крок-1 «Загальна B. B6
лікарська підготовка» – 2012. – № 16. C. A
D. C
26. У хворого на хронiчний гепатит E. E
видалили зуб. Кровотеча, що виникла пiсля Збірник тестових завдань для складання
цього, не припинялася впродовж 2 годин. ліцензійного іспиту Крок-1 «Загальна
Проведене дослiдження системи гемостазу лікарська підготовка». – 2014. – №12.
встановило зменшення вмiсту в плазмi кровi
декiлькох факторiв зсiдання. Порушення якої 30. В інфекційне відділення поступив хворий
функцiї печiнки призвело до порушення з ознаками жовтяниці внаслідок зараження
гемостазу у цього хворого? вірусом гепатиту. Який з перерахованих
A. Бiлоксинтезуюча показників є строго специфічним, що
B. Антитоксична відрізняє паренхіматозну жовтяницю від
C. Травна інших?
D. Гормональна A. Підвищення рівня АЛТ, АСТ
E. Захисна B. Гипербілірубінемія
Збірник тестових завдань для складання C. Білірубінурія
ліцензійного іспиту Крок-1 «Стоматологія». D. Холалемія
– 2011. – № 43. E. Уробілінурія
Test items for licensing examination Krok 1
27. У хворого 35-ти років після перенесеного «Medicine». – 2017. - № 157.
гепатиту розвинулася печiнкова
недостатнiсть. Порушення якої iз функцій 31. У 15-річного хлопчика діагностували
печінки при цьому запускає механізм гострий вірусний гепатит. Які показники
утворення набрякiв? крові вказуватимуть про підтвердження
A. Бiлковоутворююча гострого ураження печінкових клітин?
B. Бар’єрна
66
A. Активність амінотрансфераз (AСаT, 35. Пацiєнт звернувся зi скаргами на гострий
AЛаT) бiль у правому пiдребер’ї. При оглядi лiкар
B. Вміст некон'югованого і кон'югованого звернув увагу на пожовтiння склер хворого.
білірубіну Лабораторно: пiдвищена активнiсть АлАТ та
C. Швидкість осідання еритроцитів (ШОЕ) негативна реакцiя на стеркобiлiн в калi. Для
D. Вміст холестерину якого захворювання характернi такi
E. Вміст білкової фракції симптоми?
Test items for licensing examination Krok 1 A. Жовчнокам’яної хвороби
«Medicine». – 2016. - № 11. B. Гемолiтичної жовтяниці
C. Гепатиту
32. До клiнiки надiйшов хворий з яскраво D. Хронiчного гастриту
вираженою жовтяничнiстю шкiри, склер, E. Хронiчного колiту
слизових оболонок. Сеча кольору темного Збірник тестових завдань для складання
пива, аналiз показав наявнiсть у нiй прямого ліцензійного іспиту Крок -1 «Загальна
бiлiрубiну. Вмiст жовчних пiгментiв в калi лікарська підготовка». – 2016. – №183.
знижений. Який тип жовтяницi
спостерiгається в хворого? 36. Пацiєнт звернувся зi скаргами на гострий
A. Обтурацiйна бiль у правому пiдребер’ї. При оглядi лiкар
B. Паренхiматозна звернув увагу на пожовтiння склер хворого.
C. Гемолiтична Лабораторно: пiдвищена активнiсть АлАТ та
D. Кон’югацiйна негативна реакцiя на стеркобiлiн в калi. Для
E. Абсорбцiйна якого захворювання характернi такi
Збірник тестових завдань для складання симптоми?
ліцензійного іспиту Крок-1 «Стоматологія». A. Гепатит
– 2009. – № 119. B. Гемолiтична жовтяниця
C. Хронiчний гастродуоденiт
33. У дитини iз гемолітичною хворобою D. Хронiчний колiт
новонародженого розвинулась E. Хронiчний гастрит
енцефалопатiя. Збiльшення вмiсту якої Збірник тестових завдань для складання
речовини в кровi спричинило ураження ліцензійного іспиту Крок -1 «Загальна
ЦНС? лікарська підготовка». – 2015. – №187.
A. Вiльний бiлiрубiн
B. Комплекс бiлiрубiн-альбумiн 37. У юнака 20-ти рокiв дiагностовано
C. Бiлiрубiн-глюкуронiд спадковий дефiцит УДФ-
D. Вердоглобiн глюкуронiлтрансферази. Пiдвищення якого
E. Жовчнi кислоти показника кровi пiдтверджує дiагноз?
Збірник тестових завдань для складання A. Непрямий (некон’югований) бiлiрубiн
ліцензійного іспиту Крок-1 «Стоматологія». B. Прямий (кон’югований) бiлiрубiн
– 2018. – № 87. C. Уробiлiн
D. Стеркобiлiноген
34. Для лiкування жовтяниць показано E. Тваринний iндикан
призначення барбiтуратiв, якi iндукують Збірник тестових завдань для складання
синтез УДФ-глюкуронiлтрансферази. ліцензійного іспиту Крок -1 «Загальна
Лiкувальний ефект при цьому обумовлений лікарська підготовка». – 2016. – №67.
утворенням:
A. Прямого (кон’югованого) бiлiрубiну 38. У недоношеного немовляти
B. Непрямого (некон’югованого) бiлiрубiну спостерiгається жовтяниця. З нестачею у
C. Бiлiвердину нього якого ферменту це пов’язано?
D. Протопорфирину A. УДФ-трансглюкуронiдаза
E. Гему оболонки B. Лужна фосфатаза
Збірник тестових завдань для складання C. Кисла фосфатаза
ліцензійного іспиту Крок-1 «Загальна D. Каталаза
лікарська підготовка». – 2010. – №82. E. НАД+-дегiдрогеназа

67
Збірник тестових завдань для складання B. Кон’югований бiлiрубiн
ліцензійного іспиту Крок -1 «Загальна C. Мезобiлiрубiн
лікарська підготовка». – 2014. – №165. D. Вердоглобiн
E. Бiлiвердин
39. У доношеного новонародженого Збірник тестових завдань для складання
спостерiгається жовте забарвлення шкiри та ліцензійного іспиту Крок -1 «Загальна
слизових оболонок. Iмовiрною причиною лікарська підготовка». – 2017. – №31.
цього стану може бути тимчасова нестача
такого ферменту: 43. У жiнки 46-ти рокiв, що страждає на
A. УДФ-глюкуронiлтрансфераза жовчнокам’яну хворобу, розвинулася
B. Уридинтрансфераза жовтяниця. При цьому сеча стала темно-
C. Гемсинтетаза жовтого кольору, а кал - знебарвлений.
D. Гемоксигеназа Вкажiть, концентрацiя якої речовини у
E. Бiлiвердинредуктаза сироватцi кровi зросте в найбiльшiй мiрi?
Збірник тестових завдань для складання A. Кон’югований бiлiрубiн
ліцензійного іспиту Крок -1 «Загальна B. Вiльний бiлiрубiн
лікарська підготовка». – 2017. – №15. C. Бiлiвердин
D. Мезобiлiрубiн
40. У новонародженого фiзiологiчна E. Уробiлiноген
жовтяниця. Рiвень вiльного бiлiрубiну в Збірник тестових завдань для складання
кровi значно перевищує норму. Нестачею ліцензійного іспиту Крок-1 «Стоматологія».
якого ферменту це обумовлено? – 2007. – № 91.
A. УДФ-глюкуронiлтрансфераза
B. Трансамiназа 44. При ферментативних жовтяницях має
C. Ксантиноксидаза мiсце порушення активностi ферменту УДФ-
D. Аденозиндезамiназа глюкуронiлтрасферази. Яка сполука
E. Гем-оксигеназа накопичується в сироватцi кровi при цих
Збірник тестових завдань для складання патологiях?
ліцензійного іспиту Крок -1 «Загальна A. Непрямий бiлiрубiн
лікарська підготовка». – 2006. – №123. B. Прямий бiлiрубiн
C. Бiлiвердин
41. Внаслiдок переливання несумісної кровi D. Мезобiлiрубiн
за антигеном Rh у хворої виникла E. Вердоглобiн
гемолiтична жовтяниця. Який лабораторний Збірник тестових завдань для складання
показник кровi пiдтверджує цей тип ліцензійного іспиту Крок-1 «Стоматологія».
жовтяницi? – 2008. – № 128.
A. Нагромадження некон’югованого
бiлiрубiну 45. У хворого 20-ти рокiв з жовтяницею
B. Зменшення вмiсту некон’югованого встановлено: пiдвищення у плазмi кровi
бiлiрубiну вмiсту загального бiлiрубiну за рахунок
C. Нагромадження уробiлiногену непрямого (вiльного), в калi та сечi –
D. Зменшення вмiсту стеркобiлiну високий вмiст стеркобiлiну, рiвень прямого
E. Зменшення вмiсту кон’югованого (зв’язаного) бiлiрубiну в плазмi кровi в
бiлiрубiну межах норми. Про який вид жовтяницi
Збірник тестових завдань для складання можна думати?
ліцензійного іспиту Крок-1 «Стоматологія». A. Гемолiтична
– 2013. – № 111. B. Паренхiматозна (печiнкова)
C. Механiчна
42. У пацiєнта, що звернувся до лiкаря, D. Жовтяниця немовлят
спостерiгається жовте забарвлення шкiри, E. Хвороба Жильбера
сеча темна, кал темно-жовтого кольору. Збірник тестових завдань для складання
Пiдвищення концентрацiї якої речовини буде ліцензійного іспиту Крок -1 «Загальна
спостерiгатися в сироватцi кровi? лікарська підготовка». – 2017. – №59.
A. Вiльний бiлiрубiн
68
46. У хворого з’явилися жовтяничність перелiчених захворювань характернi такi
шкiри, склер та слизових оболонок. У плазмi змiни?
кровi пiдвищений рiвень загального A. Паренхiматозна жовтяниця
бiлiрубiну, в калi – рiвень стеркобiлiну, в B. Гемолiтична жовтяниця
сечi – уробiлiну. Який вид жовтяницi у C. Iнфаркт нирки
хворого? D. Туберкульоз нирки
A. Гемолiтична E. Механiчна жовтяниця
B. Хвороба Жiльбера Збірник тестових завдань для складання
C. Паренхiматозна ліцензійного іспиту Крок -1 «Загальна
D. Обтурацiйна лікарська підготовка». – 2012. – №137.
E. Холестатична
Збірник тестових завдань для складання 50. Чоловiк 28-ми рокiв надiйшов зi скаргами
ліцензійного іспиту Крок -1 «Загальна на нудоту, блювання, бiль у правому
лікарська підготовка». – 2010. – №123. пiдребер’ї. Об’єктивно: жовтяничнiсть
шкiри, склер; температура тiла пiдвищена,
47. У пацієнта А. після переливання крові печiнка збiльшена, сеча темна, кал
спостерігається жовтяничність шкіри та гiпохолiчний. Гiпербiлiрубiнемiя (бiлiрубiн
слизових оболонок, у крові підвищений прямий та непрямий), бiлiрубiнурiя,
рівень загального та непрямого білірубіну, в уробiлiнурiя, гiпопротеїнемiя, зниження
сечі підвищений рівень уробіліну, у калі – зсiдання кровi. Для якого з перелiчених
стеркобіліну. Про який вид жовтяниці нижче станів найбiльш характернi цi змiни?
йдеться? A. Клiтинно-паренхiматозна жовтяниця
A. Гемолітична жовтяниця. B. Пiдпечiнкова жовтяниця
B. Жовтяниця новонароджених. C. Надпечiнкова гемолiтична жовтяниця
C. Обтураційна жовтяниця. D. Гострий холецистит
D. Паренхіматозна жовтяниця. E. Гострий панкреатит
E. Спадкова жовтяниця. Збірник тестових завдань для складання
Збірник тестових завдань для складання ліцензійного іспиту Крок -1 «Загальна
ліцензійного іспиту Крок -1 лікарська підготовка». – 2006. – №54.
«Стоматологія». – 2005. – № 121.
51. Хвора 48-ми рокiв надiйшла до клiнiки iз
48. У хворого, що страждає на жовтяницю, у скаргами на слабкiсть, дратiвливiсть,
кровi виявлено збільшення загального порушення сну. Об’єктивно: шкiра та склери
бiлiрубiну за рахунок його непрямої фракцiї. жовтого кольору. У кровi: пiдвищення рiвня
Сеча i кал iнтенсивно забарвленi. Який загального бiлiрубiну з переважанням
найвiрогiдніший механiзм розвитку вказаних прямого. Кал – ахолiчний. Сеча – темного
порушень? кольору (жовчнi пiгменти). Яка жовтяниця
A. Підвищений гемолiз еритроцитiв. має мiсце в хворої?
B. Порушення перетворення уробiлiногену в A. Механiчна
печiнцi. B. Гемолiтична
C. Порушення утворення прямого бiлiрубiну. C. Паренхiматозна
D. Пошкодження паренхiми печiнки. D. Синдром Жiльбера
E. Утруднення вiдтоку жовчi з печінки E. Синдром Крiглера-Найяра
Збірник тестових завдань для складання Збірник тестових завдань для складання
ліцензійного іспиту Крок-1 «Стоматологія». ліцензійного іспиту Крок -1 «Загальна
– 2010. – №142. лікарська підготовка». – 2009. – №74.

49. Хвора 28-ми рокiв потрапила до 52. Хворий надiйшов до клiнiки зi скаргами
iнфекцiйної лiкарнi з приводу пожовтiння на загальну слабкiсть, порушення сну. Шкiра
шкiри, склер, слизових оболонок. має жовтий колiр. У кровi: збiльшена
Лабораторно встановлене пiдвищення рiвня кiлькiсть прямого бiлiрубiну, жовчних
прямого бiлiрубiну у кровi. В сечi виявлений кислот. Кал ахолiчний. Для якого стану
уробiлiноген i бiлiрубiн. Для якого з характернi цi змiни?
A. Механiчна жовтяниця
69
B. Гемолiтична жовтяниця ультрафiолетi яскраво червоним кольором,
C. Надпечiнкова жовтяниця шкiра чутлива до свiтла, сеча забарвлена в
D. Синдром Жiльбера червоний колiр. З недостатнiстю якого
E. Хронiчний холецистит ферменту пов’язана ця хвороба?
Збірник тестових завдань для складання A. Уропорфiриноген-III-косинтази
ліцензійного іспиту Крок -1 B. Дельта-амiнолевулiнатсинтази
«Стоматологія». – 2018. – №35. C. Уропорфiриноген-I-синтази
D. Уропорфiриноген-декарбоксилази
53. У хворого на жовтяницю у кровi E. Феррохелатази
пiдвищений вмiст прямого бiлiрубiну та Збірник тестових завдань для складання
жовчних кислот; у сечi вiдсутнiй ліцензійного іспиту Крок -1
стеркобiлiноген. При якiй жовтяницi «Стоматологія». – 2010. – № 182.
можлива наявнiсть цих ознак?
A. Механiчна 57. У хворого на анемію в еритроцитах
B. Печiнкова збільшився вміст протопорфірину ІХ.
C. Паренхiматозна Нестача якого мінерального елементу
D. Гемолiтична призвела до такої патології?
E. Надпечiнкова A. Заліза
Збірник тестових завдань для складання B. Калію
ліцензійного іспиту Крок -1 «Загальна C. Магнію
лікарська підготовка». – 2016. – №67. D. Натрію
E. Фосфору
54. Хворий скаржиться на жовтяничнiсть Збірник тестових завдань для складання
шкiри, шкiрний свербiж, загальну слабкiсть. ліцензійного іспиту Крок -1 «Загальна
У сечi: вiдсутнiй уробiлiн. Яка патологiя у лікарська підготовка». –2007. – № 117
хворого?
A. Механiчна жовтяниця 58. У пацієнта відмічається підвищена
B. Паренхiматозна жовтяниця чутливість шкіри до сонячного світла, а при
C. Гостра печiнкова недостатнiсть стоянні сеча набуває темно-червоного
D. Гемолiтична жовтяниця кольору. Для якого захворювання це
E. Хронiчна печiнкова недостатнiсть характерно?
Збірник тестових завдань для складання A. Порфірії
ліцензійного іспиту Крок -1 B. Алкаптонурії
«Стоматологія». – 2013. – №134. C. Альбінізму
D. Гемолітичної жовтяниці
55. Пацієнту 43 роки. Протягом 10 років E. Пелагри
періодично звертається до лікаря зі скаргами Збірник тестових завдань для складання
на гострі болі в животі, судоми, порушення ліцензійного іспиту Крок -1
зору. У його родичів спостерігаються подібні «Стоматологія». – 2009. – № 43.
симптоми. Сеча червоного кольору.
Встановлено діагноз - гостра переміжна 59. Пацiєнту 33 роки. Хворiє 10 рокiв.
порфірія. Причиною захворювання може Перiодично звертається до лiкаря зi скаргами
бути порушення біосинтезу: на гострий бiль у животi, судоми, порушення
A. Гему зору. У його родичів спостерiгаються подiбнi
B. Жовчних кислот симптоми. Сеча червоного кольору.
C. Інсуліну Госпiталiзований з дiагнозом - гостра
D. Колагену перемiжна порфирiя. Причиною
E. Простагландинів захворювання може бути порушення
Збірник тестових завдань для складання бiосинтезу такої речовини:
ліцензійного іспиту Крок -1 «Загальна A. Гем
лікарська підготовка». –2011. – № 58. B. Iнсулiн
C.Жовчнi кислоти
56. У хворих на еритропоетичну порфiрiю D. Простагландини
(хвороба Гюнтера) зуби флюоресцують в E. Колаген
70
Збірник тестових завдань для складання Збірник тестових завдань для складання
ліцензійного іспиту Крок -1 «Загальна ліцензійного іспиту Крок-1 «Стоматологія».
лікарська підготовка». –2011. – № 58 - 2015. – №87

60. Жінка 43-х років, працівниця лако- 63. Знешкодження ксенобiотикiв (лiкарських
фарбового підприємства, скаржиться на засобiв, епоксидiв, ареноксидiв, альдегiдiв,
загальну слабкість, зниження маси тіла, нiтропохiдних тощо) та ендогенних
апатію, сонливість. Хронічна свинцева метаболiтiв (естрадiолу, простагландинiв,
інтоксикація підтверджена лабораторно - лейкотрiєнiв) вiдбувається в печiнцi шляхом
виявлено гіпохромну анемію. У крові їх кон’югацiї з:
підвищений рівень Zn-протопорфірину і A. Глутатiоном
знижений рівень альфа-амінолевулінової B. Аспарагiновою кислотою
кислоти, що свідчить про порушення C. Глiцином
синтезу: D. S-Аденозилметiонiном
A. Гему E. Фосфоаденозином
B. ДНК Збірник тестових завдань для складання
C. РНК ліцензійного іспиту Крок-1 «Загальна
D. Білка лікарська підготовка». – 2017. – №66.
E. Мевалонової кислоти
Сборник тестовых заданий для 64. Хворому, що страждає на хронiчний
лицензионного экзамена Крок-1 «Медицина». гепатит, для оцiнки знешкоджуючої функцiї
- 2011. - № 68. печiнки було проведене навантаження
бензоатом натрiю. За видiленням якої
61. Мати звернулася до лiкаря з приводу речовини з сечею судять про знешкоджуючу
того, що у дитини 5-ти рокiв пiд дією функцiю печiнки?
сонячних променiв на шкiрi з’являються A. Гiпурова кислота
еритеми, везикулярний висип, свербiж B. Фенiлоцтова кислота
шкiри. Лабораторнi дослiдження виявили C. Лимонна кислота
зменшення вмiсту залiза у сироватцi кровi, D. Валерiанова кислота
збільшення видiлення з сечею E. Щавелева кислота
уропорфiриногену I. Найбiльш Збірник тестових завдань для складання
вiрогiдноюспа дковою патологiєюу дитини є: ліцензійного іспиту Крок -1 «Загальна
A. Еритропоетична порфiрiя лікарська підготовка». – 2011. – №65.
B. Метгемоглобiнемiя
C. Печiнкова порфiрiя 65. У пацiєнта цироз печiнки. Дослiдження
D. Копропорфiрiя якої з перелiчених речовин, що
E. Iнтермiтуюча порфiрiя екскретуються з сечею, може
Збірник тестових завдань для складання характеризувати стан антитоксичної функцiї
ліцензійного іспиту Крок -1 «Загальна печiнки?
лікарська підготовка». –2009. – № 7 A. Гiпурова кислота
B. Амонiйнi солi
62. Пацієнтка, що страждає на хронiчний C. Креатинiн
гепатит, скаржиться на пiдвищення D. Сечова кислота
чутливостi до барбiтуратiв, якi ранiше E. Амiнокислоти
переносила без симптомiв iнтоксикацiї. З Збірник тестових завдань для складання
порушенням якої функцiї печiнки це ліцензійного іспиту Крок -1 «Загальна
пов’язано найбiльшою мiрою? лікарська підготовка». – 2008. – №42.
A. Детоксикаційної
B. Гемопоетичної 66. Робiтниця хiмiчного підприємства
C. Гемодинамiчної внаслiдок порушення правил безпечної
D. Утворення жовчi роботи зазнала токсичної дiї азотистої
E. Фагоцитарної кислоти та нiтритiв, якi викликають
дезамiнування цитозину в молекулi ДНК.

71
Який фермент iнiцiює ланцюг репарацiйних Збірник тестових завдань для складання
процесiв? ліцензійного іспиту Крок-1 «Стоматологія».
A. Уридин-ДНК-глiкозидаза – 2012. – №55.
B. Цитидинтрифосфатсинтетаза
C. Оротидилмонофосфат- 70. У хворого з хронічною алкогольною
декарбоксилаза хворобою печiнки порушенi процеси
D. ДНК-залежна-РНК-полiмераза бiотрансформацiї ксенобiотикiв та
E. Тимiдилатсинтаза ендогенних токсичних сполук. Зниження
Збірник тестових завдань для складання активностi якого хромопротеїну може бути
ліцензійного іспиту Крок -1 «Загальна причиною цього?
лікарська підготовка». – 2011. – №170. A. Цитохром Р-450
B. Гемоглобiн
67. У 70-тi роки вченi встановили, що C. Цитохромоксидаза
причиною важкої жовтяницi D. Цитохром b
новонароджених є порушення зв’язування E. Цитохром с1
бiлiрубiну в гепатоцитах. Яка речовина Збірник тестових завдань для складання
використовується для утворення кон’югату? ліцензійного іспиту Крок-1 «Стоматологія».
A. Глюкуронова кислота – 2015. – №193.
B. Сечова кислота
C. Сiрчана кислота 71. В печінці порушена детоксикація
D. Молочна кислота природних метаболітів і ксенобіотиків.
E. Пiровиноградна кислота Назвіть цитохром, активність якого може
Збірник тестових завдань для складання бути знижена:
ліцензійного іспиту Крок -1 «Загальна A. Цитохром Р-450
лікарська підготовка». – 2011. – №64. B. Цитохромоксидаза
C. Гемоглобін
68. Хворому 50-ти років з харчовим D. Цитохром b
отруєнням призначили крапельницю з 10% E. Цитохром а1
розчином глюкози. Вона не тільки Сборник тестовых заданий для
забезпечує енергетичні потреби організму, лицензионного экзамена Крок-1 «Медицина».
але і виконує детоксикаційну функцію за - 2011. - № 41.
рахунок утворення метаболіту, який бере
участь в реакції кон'югації: 72. При дослідженні перетворення харчового
A. Глюкуронування барвника було встановлено, що
B. Сульфування знешкодження цього ксенобиотика
C. Метилювання відбувається тільки в одну фазу
D. Глікозилювання мікросомального окислення. Назвіть
E. Гідроксилювання компонент цієї фази:
Сборник тестовых заданий для A. Цитохром P - 450
лицензионного экзамена Крок-1 «Медицина». B. Цитохром B
- 2013. - № 132. C. Цитохром C
D. Цитохром A
69. Для визначення антитоксичної функції E. Цитохромоксидаза
печінки хворому призначено бензонат Сборник тестовых заданий для
натрію, який в печінці перетворюється на лицензионного экзамена Крок-1 «Медицина».
гіпурову кислоту. Яка сполука - 2010. - № 178.
використовується для цього?
A. Гліцин 73. Універсальною біологічною системою
B. Метіонін окиснення неполярних сполук (багато
C. Цистеїн лікарських засобів, токсичних сполук),
D. ФАФС стероїдних гормонів, холестеролу є
E. УДФ - глюкуронова кислота мікросомальне окиснення. Назвіть цитохром,
який входить до складу оксигеназного
ланцюга мікросом:
72
A. Цитохром Р 450. E. Цитохром а.
B. Цитохром а3. Збірник тестових завдань для складання
C. Цитохром в. ліцензійного іспиту Крок -1 «Загальна
D. Цитохром c. лікарська підготовка». – 2004. – № 97.

73
Патологічні компоненти сечі
1. Хлопчик 13 рокiв скаржиться на загальну A. Обмiн амiнокислот
слабкiсть, запаморочення, втомлюванiсть. B. Лiпiдний обмiн
Спостерiгається вiдставання у розумовому C. Вуглеводний обмiн
розвитку. При обстеженнi виявлено високу D. Водно-сольовий обмiн
концентрацію валiну, iзолейцину, лейцину в E. Фосфорно-кальцiєвий обмiн
кровi та сечi. Сеча специфiчного запаху. Збірник тестових завдань для складання
Який найбiльш вiрогiдний дiагноз? ліцензійного іспиту Крок -1 «Загальна
A. Хвороба "кленового сиропу" лікарська підготовка». -2010. -№33.
B. Хвороба Аддiсона
C. Тирозиноз 5. У 12-ти рiчного хлопчика в сечi виявлено
D. Гiстидинемiя високий вмiст усiх амінокислот
E. Базедова хвороба алiфатичного ряду. При цьому вiдмiчена
Збірник тестових завдань для складання найбiльш висока екскрецiя цистинута
ліцензійного іспиту Крок -1 «Загальна цистеїну. Крiм того, УЗД нирок показало
лікарська підготовка». -2005. -№44. наявнiсть каменiв у них. Яка патологiя
Збірник тестових завдань для складання найбiльш вiрогiдна?
ліцензійного іспиту Крок -1 A. Цистинурiя
«Стоматологія». – 2010. – №44. B. Алкаптонурiя
C. Цистит
2. У немовляти на 6-й день життя в сечi D. Фенiлкетонурiя
виявлено надлишок фенiлпiрувату та E. Хвороба Хартнупа
фенiлацетату. Обмiн якої амінокислоти Збірник тестових завдань для складання
порушено в органiзмi дитини? ліцензійного іспиту Крок -1 «Загальна
A. Фенiлаланiн лікарська підготовка». -2010. -№42.
B. Триптофан
C. Метiонiн 6. Хвора 36-ти рокiв страждає на колагеноз.
D. Гiстидин Збiльшення вмiсту якого метаболiту
E. Аргiнiн найбiльш iмовiрно буде встановлено у сечi?
Збірник тестових завдань для складання A. Оксипролiн
ліцензійного іспиту Крок-1 «Стоматологія». B. Iндикан
– 2007. – № 93. C. Креатинiн
D. Сечовина
3. У хворої дитини виявили у сечi E. Уробiлiноген
пiдвищений рiвень фенiлпiрувату (в нормi Збірник тестових завдань для складання
практично вiдсутнiй). Вмiст фенiлаланiну в ліцензійного іспиту Крок -1 «Загальна
кровi становить 350 мг/л (норма близько 15 лікарська підготовка». -2011. -№ 80.
мг/л). Для якого захворювання характернi
перелiченi симптоми? 7. У хворого на системну склеродермiю
A. Фенiлкетонурiя посилений розпад колагену. Посилення
B. Альбiнiзм екскрецiї з сечею якої амiнокислоти буде
C. Тирозиноз вiддзеркалювати процеси деструкцiї
D. Алкаптонурiя колагену?
E. Подагра A. Оксипролiн
Збірник тестових завдань для складання B. Аланiн
ліцензійного іспиту Крок -1 C. Триптофан
«Стоматологія». – 2011. – №112. D. Серин
E. Фенiлаланiн
4. До лiкарнi доставлено дитину 2-х рокiв з Збірник тестових завдань для складання
уповiльненим розумовим i фiзичним ліцензійного іспиту Крок -1
розвитком, що страждає на частi блювання «Стоматологія». – 2008. – №50.
пiсля прийому їжi. У сечi визначена
фенiлпiровиноградна кислота. Наслiдком 8. У дитини спостерiгається затримка
якого порушення є дана патологiя? фiзичного та розумового розвитку, глибокi
74
порушення з боку сполучної тканини 12. У грудної дитини спостерiгається
внутрiшнiх органiв, у сечi виявлено забарвлення склер, слизових оболонок.
кератансульфати. Обмiн яких речовин Видiляється сеча, яка темнiє на повiтрi. В
порушений? кровi та сечi виявлено гомогентизинову
A. Глiкозамiноглiкани кислоту. Що може бути причиною даного
B. Колаген стану?
C. Еластин A. Алкаптонурiя
D. Фiбронектин B. Альбiнiзм
E. Гiалуронова кислота C. Галактоземiя
Збірник тестових завдань для складання D. Цистинурiя
ліцензійного іспиту Крок -1 «Загальна E. Гiстидинемiя
лікарська підготовка». -2015. -№ 112. Збірник тестових завдань для складання
ліцензійного іспиту Крок -1 «Загальна
9. До вiддiлення травматологiї надiйшов лікарська підготовка». -2006. -№74.
хворий iз значним пошкодженням м’язової
тканини. Який біохімічний показник сечi 13. Мати помiтила темну сечу у її 5-рiчної
буде збiльшений при цьому? дитини.Жовчних пiгментiв у сечі не
A. Креатинiн виявлено. Встановлено дiагноз:
B. Загальнi лiпiди алкаптонурiя. Дефiцит якого ферменту має
C. Глюкоза мiсце?
D. Мiнеральнi солi A. Оксидаза гомогентизинової кислоти
E. Сечова кислота B. Фенiлаланiнгiдроксилаза
Збірник тестових завдань для складання C. Тирозиназа
ліцензійного іспиту Крок -1 «Загальна D. Оксидаза оксифенiлпiрувату
лікарська підготовка». -2006. -№47. E. Декарбоксилаза фенiлпiрувату
Збірник тестових завдань для складання
10. Хворому з прогресуючою м’язовою ліцензійного іспиту Крок -1
дистрофією було проведено бiохiмiчне «Стоматологія». – 2012. – №48.
дослiдження сечi. Поява якої речовини у
великiй кiлькостi в сечі може пiдтвердити 14. При лабораторному обстеженнi дитини
захворювання м’язiв у даного хворого? виявлено пiдвищений вмiст у кровi та сечi
A. Креатин лейцину, валiну, iзолейцину та їх
B. Порфiрини кетопохiдних. Сеча мала характерний запах
C. Сечовина кленового сиропу. Недостатнiсть якого
D. Гiпуровакислота ферменту зумовила це захворювання?
E. Креатинiн A. Дегiдрогеназа розгалужених амiнокислот
Збірник тестових завдань для складання B. Амiнотрансфераза
ліцензійного іспиту Крок -1 «Загальна C. Глюкозо-6-фосфатаза
лікарська підготовка». -2016. -№44. D. Фосфофруктокiназа
E. Фосфофруктомутаза
11. Хворий госпiталiзований до клiнiки з Збірник тестових завдань для складання
попереднiм дiагнозом прогресуюча м’язова ліцензійного іспиту Крок -1
дистрофiя. Збiльшення вмiсту якої речовини «Стоматологія». – 2008. – №118.
у сечi може пiдтвердити цей дiагноз?
A. Креатин 15. При алкаптонурiї вiдбувається надмiрне
B. Пiруват видiлення гомогентизинової кислоти iз
C. Карнозин сечею. С порушенням метаболiзму якої
D. Тропонiн амiнокислоти пов’язано виникнення цього
E. Гiдроксипролiн захворювання?
Збірник тестових завдань для складання A. Тирозин
ліцензійного іспиту Крок -1 B. Фенiлаланiн
«Стоматологія». – 2006. – №101. C. Аланiн
D. Метiонiн
E. Аспарагiн
75
Збірник тестових завдань для складання Недостатнiстю якого вiтамiну обумовлене
ліцензійного іспиту Крок -1 «Загальна виникнення зазначеного
лікарська підготовка». -2013. -№128. симптомокомплексу?
A. B12
16. Чоловiк 28-ми рокiв надiйшов зi скаргами B. B2
на нудоту, блювання, бiль у правому C. B3
пiдребер’ї. Об’єктивно: жовтяничнiсть D. B5
шкiри, склер; температура тiла пiдвищена, E. B6
печiнка збiльшена, сеча темна, кал Збірник тестових завдань для складання
гiпохолiчний. Гiпербiлiрубiнемiя (бiлiрубiн ліцензійного іспиту Крок -1
прямий та непрямий), бiлiрубiнурiя, «Стоматологія». – 2005. – №127.
уробiлiнурiя, гiпопротеїнемiя, зниження
зсiдання кровi. Для якого з перелiчених 20. Спадкова хвороба: гомоцистинурiя –
нижче станів найбiльш характернi цi змiни? обумовлена порушенням перетворення
A. Клiтинно-паренхiматозна жовтяниця гомоцистеїна у метiонiн. Накопичений гомо
B. Пiдпечiнкова жовтяниця цистеїн утворює димери гомоцистеїну, який
C. Надпечiнкова гемолiтична жовтяниця знаходять у сечi. Призначення якого
D. Гострий холецистит вiтамiнного препарату зменшить утворення
E. Гострий панкреатит гомоцистеїну?
Збірник тестових завдань для складання A. Вiтамiн B12
ліцензійного іспиту Крок -1 «Загальна B. Вiтамiн C
лікарська підготовка». – 2006. – №54. C. Вiтамiн B1
D. Вiтамiн B2
17. У хворого в кровi збiльшена E. Вiтамiн PP
концентрацiя пiрувату. Значна його кiлькiсть Збірник тестових завдань для складання
екскретується з сечею. Дефiцит якого ліцензійного іспиту Крок -1 «Загальна
вiтамiну має мiсце у хворого? лікарська підготовка». -2018. -№132.
A. B1
B. E 21. У новонародженої дитини
C. B3 спостерiгаються зниження інтенсивності
D. B6 смоктання, часте блювання, гiпотонiя. У сечi
E. B2 та кровi значно пiдвищена концентрацiя
Збірник тестових завдань для складання цитрулiну. Який метаболiчний процес
ліцензійного іспиту Крок -1 «Загальна порушений?
лікарська підготовка». -2009. -№29. A. Орнiтиновий цикл
B. ЦТК
18. У хворого, що харчувався винятково C. Глiколiз
полiрованим рисом, недостатнiсть тiамiну D. Глюконеогенез
стала причиною полiневриту. Екскрецiя якої E. Цикл Корi
сполуки з сечею може бути iндикатором Збірник тестових завдань для складання
цього авiтамiнозу? ліцензійного іспиту Крок -1 «Загальна
A. Пiровиноградна кислота лікарська підготовка». -2009. -№178.
B. Малат
C. Метил-малонова кислота 22. В сечi новонародженого визначається
D. Сечова кислота цитрулiн та високий рiвень амiаку. Вкажiть,
E. Фенiлпiруват утворення якої речовини, найiмовiрнiше,
Збірник тестових завдань для складання порушене у цього малюка:
ліцензійного іспиту Крок -1 A. Сечовина
«Стоматологія». – 2011. – №108. B. Сечова кислота
C. Амiак
19. У хворого 43 рокiв з хронічним D. Креатинiн
атрофiчним гастритом i мегалобластною E. Креатин
гiперхромною анемiєю пiдвищене видiлення
метилмалонової кислоти з сечею.
76
Збірник тестових завдань для складання забарвлення. З наявнiстю яких речовин
ліцензійного іспиту Крок -1 «Загальна можуть бути пов’язанi такi змiни?
лікарська підготовка». -2011. -№67. A. Солi сечової кислоти
B. Хлориди
23. У хворого, який скаржиться на полiурiю i C. Амонiєвi солi
полiдипсiю, знайдено цукор в сечi. Вмiст D. Фосфат кальцiю
цукру в плазмi кровi у нормi. З чим E. Сульфат магнію
пов’язаний механiзм глюкозурії у хворого? Збірник тестових завдань для складання
A. Порушення реабсорбцiї глюкози в ліцензійного іспиту Крок -1 «Загальна
канальцях нефрону лікарська підготовка». -2012. -№6.
B. Порушення фiльтрацiї глюкози в
клубочковому вiддiлi нефрону 27. На основi лабораторного аналiзу у
C. Недостатня продукцiя iнсулiну хворого пiдтверджено дiагноз - подагра. Для
пiдшлунковою залозою встановлення дiагнозу було проведено
D. Iнсулiнорезистентнiсть рецепторів клiтин визначення вмiсту:
E. Гiперпродукцiя глюкокортикоїдiв A. Сечової кислоти в кровi та сечi
наднирниками B. Креатинiну в сечi
Збірник тестових завдань для складання C. Залишкового азоту в кровi
ліцензійного іспиту Крок -1 «Загальна D. Сечовини в кровi та сечi
лікарська підготовка». -2011. -№106. E.Амiаку в сечi
Збірник тестових завдань для складання
24. В ендокринологiчному вiддiленнi з ліцензійного іспиту Крок -1 «Загальна
дiагнозом цукровий дiабет лiкується жiнка лікарська підготовка». – 2013. - № 24.
42-х рокiв зi скаргами на спрагу, пiдвищений
апетит. Якi патологічні компоненти виявленi 28. У хворого, прооперованого з приводу
при лабораторному дослiдженнi сечi "гострого живота", сеча коричневого
пацiєнтки? кольору, кiлькiсть iндикану в сечі вище 93
A. Глюкоза, кетоновi тiла ммоль/добу. Про що це свiдчить?
B. Бiлок, амiнокислоти A. Збiльшення iнтенсивностi гниття бiлкiв у
C. Бiлок, креатин кишечнику
D. Бiлiрубiн, уробiлiн B. Зниження активностi ферментів
E. Кров орнiтинового циклу
Збірник тестових завдань для складання C. Збiльшення швидкостi окисного
ліцензійного іспиту Крок -1 дезамiнування ароматичних амiнокислот
«Стоматологія». – 2006. – №105. D. Порушення фiльтрацiйної здатності нирок
E. Зниження iнтенсивностi знезараження
25. На раннiй стадiї цукрового дiабету у амонiаку
хворих спостерiгається полiурiя. Чим вона Збірник тестових завдань для складання
зумовлена? ліцензійного іспиту Крок -1 «Загальна
A. Гiперглiкемiя лікарська підготовка». -2012. -№115.
B. Кетонемiя
C. Гiпохолестеринемiя 29. У хворої 43 рокiв, яка була прооперована
D. Гiперхолестеринемiя з приводу "гострого живота", сеча набула
E. Гiперкалiемiя коричневого кольору, кiлькiсть iндикану в
Збірник тестових завдань для складання кровi рiзко пiдвищилася. Про що може
ліцензійного іспиту Крок -1 свiдчити цей показник?
«Стоматологія». – 2005. – №133. A. Посилення гниття бiлкiв в кишечнику
B. Зниження iнтенсивностi орнiтинового
26. На прийом до терапевта прийшов чоловiк циклу
37-ми рокiв зi скаргами на перiодичнi C. Зниження швидкостi клуб очкової
iнтенсивнi больовi напади у суглобах фiльтрацiї у нирках
великого пальця стопи та їх припухлiсть. У D. Посилення дезамiнування амiнокислот
сечi: рiзко кисла реакцiя i рожеве E. Iнгiбування глюконеогенезу

77
Збірник тестових завдань для складання A. Порфірії
ліцензійного іспиту Крок -1 B. Алкаптонурії
«Стоматологія». – 2005. – №17. C. Альбінізму
D. Гемолітичної жовтяниці
30. У чоловiка 60-ти рокiв, який страждає на E. Пелагри
хронiчну кишкову непрохiднiсть, Збірник тестових завдань для складання
посилюється гниття бiлкiв у товстому ліцензійного іспиту Крок -1
кишечнику. Пiдтвердженням цього процесу «Стоматологія». – 2009. – № 43.
є:
A. Iндиканурiя 34. У сечi лiкаря-стоматолога, зiбранiй
B. Бiлiрубiнурiя наприкiнцi робочого дня, виявлений вмiст
C. Гiперурiкурiя бiлку 0,7 г/л. У ранковiй сечi таких змiн не
D. Креатинурiя виявлено. Як називається це явище?
E. Глюкозурiя A. Функцiональна протеїнурiя
Збірник тестових завдань для складання B. Органiчна протеїнурiя
ліцензійного іспиту Крок -1 C. Неселективна протеїнурiя
«Стоматологія». – 2012. – №111. D. Позаниркова протеїнурiя
E. Гематурiя
31. В сечі новонародженої дитини, яка Збірник тестових завдань для складання
погано набирає вагу, виявлено підвищений ліцензійного іспиту Крок -1
вміст оротової кислоти, що свідчить про «Стоматологія». – 2009. – №187.
порушення синтезу піримідинових
нуклеотидів Який метаболіт необхідно 35. 3 роки тому хворiй 34 рокiв було
використати для нормалізації метаболізму? встановлено дiагноз хронiчного
Уридин. гломерулонефриту. За останнi 6 мiсяцiв
Гістидин. з’явилися набряки. Що лежить в основi їх
Гуанозин. розвитку?
Тимідин. A. Протеїнурiя
Аденозин. B. Гiперпродукцiя вазопресину
Збірник тестових завдань для складання C. Порушення бiлковоутворюючої функцiї
ліцензійного іспиту Крок -1 печiнки
«Стоматологія». – 2008. – № 164. D. Гiперосмолярнiсть плазми
E. Гiперальдостеронiзм
32. При спадковiй оротацидурiї видiлення Збірник тестових завдань для складання
оротової кислоти в багато разів перевищує ліцензійного іспиту Крок -1 «Загальна
норму. Синтез яких речовин буде порушений лікарська підготовка». -2005. -№54.
при цiй патологiї?
A. Пiримiдиновi нуклеотиди 36. У людини вмiст глюкози в кровi 15
B. Пуриновi нуклеотиди ммоль/л (порiг реабсорбцiї – 10 ммоль/л).
C. Бiогеннi амiни Наслiдком цього буде:
D. Сечова кислота A. Глюкозурiя
E. Сечовина B. Зменшення дiурезу
Збірник тестових завдань для складання C. Зменшення реабсорбцiї глюкози
ліцензійного іспиту Крок -1 «Загальна D. Зменшення секрецiї вазопресину
лікарська підготовка». -2006. -№154. E. Зменшення секрецiї альдостерону
Збірник тестових завдань для складання Збірник тестових завдань для складання
ліцензійного іспиту Крок -1 ліцензійного іспиту Крок -1 «Загальна
«Стоматологія». – 2012. – № 56. лікарська підготовка». -2006. -№171.

33. У пацієнта відмічається підвищена 37. Хворому з ревматоїдним артритом


чутливість шкіри до сонячного світла, а при тривалий час вводили гiдрокортизон
стоянні сеча набуває темно-червоного (глюкокортикоїд). У нього з’явилися
кольору. Для якого захворювання це гiперглiкемiя, полiурiя, глюкозурiя, спрага.
характерно?
78
Цi ускладнення лiкування є наслідком оксалатний уролiтiаз, що призвело до
активації процесу: вiдкладання оксалату кальцiю в нирках.
A. Глюконеогенез Порушення обмiну якої амiнокислоти
B. Глiкогенолiз призвело до такого стану?
C. Глiкогенез A. Глiцин
D. Глiколiз B. Лiзин
E. Лiполiз C. Метiонiн
Збірник тестових завдань для складання D. Аргiнiн
ліцензійного іспиту Крок -1 «Загальна E. Гiстидин
лікарська підготовка». -2016. -№2. Збірник тестових завдань для складання
ліцензійного іспиту Крок -1
38. У пацiєнта 28-ми рокiв iз синдромом «Стоматологія». – 2011. – №114.
Iценка-Кушинга виявленi гiперглiкемiя,
глюкозурiя. Основним механiзмом 42. Основні симптоми первинного
гiперглiкемiї у даного хворого є стимуляцiя: гіперпаратиреозу - остеопороз і ураження
A. Глюконеогенезу нирокразом з розвитком сечокамʼяної
B. Глiкогенолiзу в печiнцi хвороби. Які речовини складають основу цих
C. Глiкогенолiзу в м’язах каменів при даному захворюванні?
D. Всмоктування глюкози в кишечнику A. Фосфат кальцію
E. Синтезу глiкогену B. Сечова кислота
Збірник тестових завдань для складання C. Цистин
ліцензійного іспиту Крок -1 Д. Білірубін
«Стоматологія». – 2014. – №115. E. Холестерин
Test items for licensing examination Krok 1
39. Відомо, що фруктозурія пов'язана зі «Medicine». – 2010. - № 172.
спадковим дефіцитом фруктозо-1-фосфат
альдолази. Який продукт фруктозного 43. При лабораторному обстеженнi кровi
обміну накопичується в організмі, що людини, яку вкусила змiя, виявлено гемолiз
призводить до токсичної дії? еритроцитiв, гемоглобiнурiю. Дiя зміїної
A. Фруктозо-1-фосфат отрути зумовлена наявнiстю в нiй ферменту:
B. Глюкозо-1-фосфат A. Фосфолiпаза А2
C. Глюкозо-6-фосфат B. Фосфолiпаза А1
D. Фруктозо-1,6-біфосфат C. Фосфолiпаза С
Е. Фруктозо-6-фосфат D. Фосфолiпаза Д
Збірник тестових завдань для складання E. Сфiнгомiєлiназа
ліцензійного іспиту Крок-1 «Загальна Збірник тестових завдань для складання
лікарська підготовка». – 2016. - № 173. ліцензійного іспиту Крок -1
«Стоматологія». – 2018. – №156.
40. Хворий проходив чергове обстеження, в
результатi якого у нього виявлено 44. У пацієнта вiдмiчена висока
гiперглiкемiю, кетонурiю, полiурiю, концентрація вазопресину у кровi. До яких
глюкозурiю. Яка форма КОС має мiсце за змiн дiурезу це призведе?
наявностi цих явищ? A. Олiгоурiя
A. Метаболiчний ацидоз B. Полiурiя
B. Газовий ацидоз C. Анурiя
C. Негазовий ацидоз D. Глюкозурiя
D. Газовий алкалоз E. Натрiйурiя
E. Метаболiчний алкалоз Збірник тестових завдань для складання
Збірник тестових завдань для складання ліцензійного іспиту Крок -1
ліцензійного іспиту Крок -1 «Стоматологія». – 2018. – №62.
«Стоматологія». – 2008. – №152.
45. Стресовий стан i больове вiдчуття у
41. У 2-рiчної дитини з нирковою пацієнта перед вiзитом до стоматолога
недостатнiстю виявили гiпероксалурiю, супроводжуються анурією (вiдсутнiстю
79
сечовидiлення). Це явище зумовлене Збірник тестових завдань для складання
збiльшенням: ліцензійного іспиту Крок -1
A. Секрецiї вазопресину та адреналiну «Стоматологія». – 2012. – №125.
B. Активностi парасимпатичної нервової
системи 49. Пiд час дослiдження чоловiка 24-х рокiв
C. Активностi антиноцiцептивної системи виявленi наступнi змiни сечi: добовий дiурез
D. Секрецiї вазопресину та зменшенням - 10 лiтрiв. Пацiєнт скаржиться на сильну
адреналiну спрагу, часте сечовидiлення. Що є найбiльш
E. Секрецiї адреналiну та зменшенням iмовiрною причиною даного захворювання?
вазопресину A. Гiпосекрецiя вазопресину
Збірник тестових завдань для складання B. Гiперсекрецiя глюкокортикоїдiв
ліцензійного іспиту Крок -1 «Загальна C. Гiперсекрецiя вазопресину
лікарська підготовка». -2016. -№124. D. Вiдносна iнсулiнова недостатнiсть
E. Гiперсекрецiя альдостерону
46. Пацієнт з пухлиною гіпофіза скаржиться Збірник тестових завдань для складання
на підвищений добовий діурез (поліурія). ліцензійного іспиту Крок -1
Концентрація глюкози в плазмі крові «Стоматологія». – 2010. – №168.
дорівнює 4,8 ммоль/л. Який гормон може
бути причиною цього, якщо його секреція 50. Хворий 50 рокiв скаржиться на спрагу,
порушена? п’є багато води; виражена полiурiя. Глюкоза
A. Вазопресин кровi - 4,8 ммоль/л. У сечi глюкози та
Б. Алдостерон ацетонових тiл немає, сеча безбарвна,
C. Натрійуретичний гормон питома вага- 1,002 - 1,004.
D. Інсулін Яка причина полiурiї?
E. Ангіотензин I A. Нестача вазопресину
Test items for licensing examination Krok 1 B. Гiпотиреоз
«Stomatology». – 2015. - № 133. C. Iнсулiнова недостатнiсть
D. Альдостеронiзм
47. Пiсля того, як людина випила 1,5 л води, E. Тиреотоксикоз
кiлькiсть сечi значно збiльшилась, а її Збірник тестових завдань для складання
вiдносна щiльнiсть зменшилась до 1,001. ліцензійного іспиту Крок -1 «Загальна
Зазначенi змiни є наслiдком зменшення лікарська підготовка». -2005. -№127.
реабсорбцiї води в дистальних вiддiлах
нефронiв внаслiдок зменшення секрецiї: 51. У хворої ушкодження задньої долi
A. Вазопресину гiпофiзу призвело до збiльшення добового
B. Альдостерону дiурезу до 10-15 л. Що є головним
C. Ангiотензину II механiзмом у розвитку полiурiї?
D. Ренiну A. Дефiцит вазопресину
E. Простагландинiв B. Надлишок вазопресину
Збірник тестових завдань для складання C. Надлишок альдостерону
ліцензійного іспиту Крок -1 D. Надлишок натрiйуретичного фактора
«Стоматологія». – 2013. – №123. E. Дефiцит кортикотропiну
Збірник тестових завдань для складання
48. У людини внаслiдок втрати 1,5 л кровi ліцензійного іспиту Крок -1
рiзко зменшився дiурез. Посилена секрецiя «Стоматологія». – 2005. – №140.
якого гормону, перш за все, спричинила
змiни дiурезу? 52. Хворий 20 років скаржиться на
A. Вазопресин виснажливу спрагу та рясне сечовиділення
B. Паратгормон до 10 л на добу. Рівень глюкози в крові
C. Кортизон нормальний, глюкози в сечі немає. Який
D. Окситоцин гормональний дефіцит може викликати такі
E. Натрiйуретичний зміни?
A. Вазопресин
B. Окситоцин
80
C. Інсулін зменшення реабсорбцiї іонів натрiю i води в
D. Трийодтиронін канальцях нефрона. Який гормон було
E. Кортизол введено?
Test items for licensing examination Krok 1 A. Передсердний натрiйуретичний
«Stomatology». – 2006. - № 65. B. Адреналiн
C. Тироксин
53. В нефрологiчнiй клiнiцi у юнака 19-ти D. Тестостерон
рокiв була виявлена пiдвищена кiлькiсть E. Окситоцин
калiю у вториннiй сечi. Пiдвищення секрецiї Збірник тестових завдань для складання
якого гормону, iмовiрно могло викликати ліцензійного іспиту Крок -1
такi змiни? «Стоматологія». – 2014. – № 195.
A. Альдостерон
B. Окситоцин 55. У тварини була сильна втрата крові, що
C. Адреналiн призвело до зниження реабсорбції Na+ і води
D. Глюкагон в ниркових канальцях. Це можна пояснити
E. Тестостерон впливом наступних факторів на нирки:
Збірник тестових завдань для складання А. Натрійуретичний гормон
ліцензійного іспиту Крок -1 Б. Альдостерон
«Стоматологія». – 2015. – №122. С. Ренин
D. Ангіотензин
54. Пiддослiдному собацi ввели гормон, що Е. Вазопрессин
призвело до збільшення швидкостi Test items for licensing examination Krok 1
клубочкової фiльтрацiї за рахунок «Medicine». – 2018. - № 81.
розширення приносної артерiоли i

81
Біохімія нервової тканини
1. З метою аналгезiї можуть бути 5. Хворий 84-х рокiв страждає на
використанi речовини, що iмiтують ефекти паркiнсонiзм, одним з патогенетичних ланок
морфiну, але виробляються в ЦНС. Вкажiть якого є дефiцит медiатора в окремих
таку речовину: структурах мозку. Якого медiатора
A. β-ендорфiн насамперед?
B. Окситоцин A. Дофамiн
C. Вазопресин B. Адреналiн
D. Кальцитонiн C. Норадреналiн
E. Соматолiберин D. Гiстамiн
Збірник тестових завдань для складання E. Ацетилхолiн
ліцензійного іспиту Крок -1 «Загальна Збірник тестових завдань для складання
лікарська підготовка». -2018. -№35. ліцензійного іспиту Крок -1
«Стоматологія». – 2017. – №164.
2. Пiд дiєю медiатора на постсинаптичну
мембрану нервової клiтини розвинулася 6. В експериментi збiльшили проникнiсть
гiперполяризацiя.Збiльшення проникностi мембрани збудливої клiтини для iонiв калiю.
мембрани для яких iонiв може викликати Якi змiни електричного стану мембрани при
такi змiни? цьому виникнуть?
A. Калiю A. Гiперполяризацiя
B. Натрiю B. Деполяризацiя
C. Кальцiю C. Потенцiал дiї
D. Магнiю D. Локальна вiдповiдь
E. Натрiю i калiю E. Змiн не буде
Збірник тестових завдань для складання Збірник тестових завдань для складання
ліцензійного іспиту Крок -1 «Загальна ліцензійного іспиту Крок -1 «Загальна
лікарська підготовка». -2018. -№70. лікарська підготовка». -2016. -№59.

3. Швидкiсть проведення збудження 7. До лiкарнi звернувся чоловiк 50-ти рокiв з


нервовими волокнами становить 120 м/с. розладами пам’ятi, болiсними вiдчуттями по
Який з наведених чинникiв, перш за все, ходу нервових стовбурiв, зниженням
забезпечує таку швидкiсть? iнтелектуальних функцiй, порушеннями з
A. Наявнiсть мiєлiнової оболонки боку серцево-судинної системи і явищами
B. Великий потенцiал спокою диспепсiї. В анамнезi хронiчний алкоголiзм.
C. Велика амплiтуда потенцiалу дiї Дефiцит якого вiтамiну може викликати цi
D. Малий порiг деполяризацiї симптоми?
E. Великий фактор надiйностi A. Тiамiн
Збірник тестових завдань для складання B. Нiацин
ліцензійного іспиту Крок -1 «Загальна C. Ретинол
лікарська підготовка». -2017. -№124. D. Кальциферол
E. Рибофлавiн
4. У хворого спостерiгається тремтіння рук, Збірник тестових завдань для складання
що пов’язане з хворобою Паркiнсона. ліцензійного іспиту Крок -1 «Загальна
Дефiцит якого медiатора стрiопалiдарних лікарська підготовка». -2016. -№182.
структурах призводить до таких симптомiв?
A. Дофамiн 8. При декарбоксилюванні глутамату
B. ГАМК утворюється нейромедіатор гамма-
C. Субстанцiя Р аміномасляна кислота (ГАМК). При розпаді
D. Норадреналiн ГАМК перетворюється в метаболіт циклу
E. Серотонiн лимонної кислоти, яким є:
Збірник тестових завдань для складання A. Сукцинат
ліцензійного іспиту Крок -1 «Загальна B. Лимонна кислота
лікарська підготовка». -2017. -№160. C. Малат
D. Фумарат
82
E. Оксалоацетат D. АТФ
Test items for licensing examination Krok 1 E. УДФ
«Medicine». – 2014. – № 153. Test items for licensing examination Krok 1
«Medicine». – 2011. – № 120.
9. У хворих B12-дефіцитною анемією
розвиваються дегенеративні процеси в задніх 13. У хворого спостерігається дисфункція
і бокових стовпах спинного мозку кори головного мозку, що супроводжується
(фунікулярний миелоз), пов'язані з епілептичними припадками. Йому вводили
накопиченням метилмалоновой кислоти. біогенний амін, синтезований з глутамату і
Синтез якої речовини порушується при відповідальний за центральне інгібування.
цьому? Що це за речовина?
A. Мієлін A. γ-аміномасляна кислота
B. Ацетилхолін B. Серотонін
C. Норадреналін С. Допамін
D. Дофамін D. Ацетилхолін
E. Серотонін Е. Гістамін
Test items for licensing examination Krok 1 Test items for licensing examination Krok 1
«Medicine». – 2014. – № 99. «Stomatology». – 2018. – № 103.

10. Пiсля фармакологiчної блокади iонних 14. Хворий в несвідомому стані доставлений
каналiв мембрани нервового волокна бригадою швидкої допомоги в лікарню.
потенцiал спокою зменшився з -90 до -80 мВ. Об'єктивно: рефлекси відсутні, періодично
Якi канали було заблоковано? з'являються судоми, дихання переривчасте.
A. Калiєвi Після лабораторного обстеження було
B. Натрiєвi діагностовано печінкова кома. Накопичення
C. Кальцiєвi якого метаболіту є істотним для появи
D. Магнiєвi розладів центральної нервової системи?
E. Хлорнi A. Аміак
Збірник тестових завдань для складання B. Сечовина
ліцензійного іспиту Крок -1 «Загальна C. Глутамин
лікарська підготовка». – 2013. – 112. D. Білірубін
E. Гістамін
11. Порушення процесiв мієлінізації Test items for licensing examination Krok 1
нервових волокон призводить до «Medicine». – 2013. – № 98.
неврологiчних розладiв i розумової
вiдсталостi. Такi симптоми характернi для 15. Для лікування депресії різного генезу
спадкових i набутих порушень обмiну: використовують антидепресанти, які є
A. Сфiнголiпiдiв інгібіторами моноамінооксидази. Яка
B. Нейтральних жирiв речовина є "нейромедіатором гарного
C. Вищих жирних кислот самопочуття" і його концентрація
D. Холестерину підвищується в головному мозку при дії
E. Фосфатидної кислоти антидепресантів?
Збірник тестових завдань для складання A. Серотонін
ліцензійного іспиту Крок -1 «Загальна B. Дофамін
лікарська підготовка». – 2013. – №178. C. Гліцин
D. Таурин
12. Пацієнт скаржився на запаморочення, E. Норадреналін
порушення пам'яті, періодичне судоми. Сборник тестовых заданий для
Виявилося, що це зміни були викликані лицензионного экзамена Крок-1 «Медицина».
продуктом декарбоксилювання глутамінової – 2012. – № 187
кислоти. Як називається цей продукт:
A. ГАМК 16. Клітини мозку дуже чутливі до дефіциту
B. Пиридоксальфосфат енергії, причиною якого може бути високий
C. ТДФ
83
вміст аміаку, стимулюючого відтік альфа- Збірник тестових завдань для складання
кетоглутарата з: ліцензійного іспиту Крок -1 «Загальна
A. Циклу лимонної кислоти лікарська підготовка». – 2009. – №149.
B. Орнітінового циклу
C. Гліколіз 20. Травма мозку викликала підвищене
D. Глікогеноліз утворення амiаку. Яка амiнокислота бере
E. Пентозофосфатного шляху участь у видаленнi амiаку з мозкової
Сборник тестовых заданий для тканини?
лицензионного экзамена Крок-1 «Медицина». A. Глутамiнова
– 2012. – № 195 B. Тирозин
C. Валiн
17. Депресiї та емоцiйнi розлади є наслiдком D. Триптофан
нестачi у головному мозку норадреналiну, E. Лiзин
серотонiну та iнших бiогенних амiнiв. Збірник тестових завдань для складання
Збiльшення їх вмiсту у синапсах можна ліцензійного іспиту Крок -1 «Загальна
досягти за рахунок антидепресантiв, якi лікарська підготовка». – 2008. – №65.
гальмують такий
фермент: 21. Амонiак є дуже отруйною речовиною,
A. Моноамiнооксидаза особливо для нервової системи. Яка
B. Диамiнооксидаза речовина бере особливо активну
C. Оксидаза L-амiнокислот участь у знешкодженнi амонiаку в тканинах
D. Оксидаза D-амiнокислот мозку?
E. Фенiлаланiн-4-монооксигеназа A. Глутамiнова кислота
Збірник тестових завдань для складання B. Лiзин
ліцензійного іспиту Крок -1 «Загальна C. Пролiн
лікарська підготовка». – 2009. – №140. D. Гiстидин
E. Аланiн
18. Фармакологiчнi ефекти антидепресантiв Збірник тестових завдань для складання
пов’язанi з блокуванням (iнгiбуванням) ними ліцензійного іспиту Крок -1 «Загальна
ферменту, який каталiзує розпад таких лікарська підготовка». – 2007. – №19.
бiогенних амiнiв, як норадреналiн i
серотонiн в мiтохондрiях нейронiв головного 22. В лiкарню звернувся хворий зi скаргами
мозку. Який фермент бере участь у цьому на швидку стомлюванiсть і виражену
процесi? м’язову слабкiсть. При обстеженнi виявлено
A. Моноамiнооксидаза аутоiмунне захворювання, внаслiдок якого
B. Трансамiназа порушується функцiональний стан
C. Декарбоксилаза рецепторів у нервово-м’язових синапсах. Дiя
D. Пептидаза якого
E. Лiаза медiатора буде заблокована?
Збірник тестових завдань для складання A. Ацетилхолiн
ліцензійного іспиту Крок -1 «Загальна B. Норадреналiн
лікарська підготовка». – 2009. – №194. C. Дофамiн
D. Серотонiн
19. Дитина 9-ми мiсяцiв харчується E. Глiцин
штучними сумiшами, якi не збалансованi за Збірник тестових завдань для складання
вмiстом вiтамiну B6. У дитини ліцензійного іспиту Крок -1 «Загальна
спостерiгається пелагроподiбний дерматит, лікарська підготовка». – 2008. – №175.
судоми, анемiя. Розвиток судом може бути
пов’язаний з порушенням утворення: 23. При декарбоксилуваннi глутамату в ЦНС
A. ГАМК утворюється медiатор гальмування. Назвiть
B. Гiстамiну його:
C. Серотонiну A. ГАМК
D. ДОФА B. Глутатiон
E. Дофамiну C. Гiстамiн
84
D. Серотонiн Збірник тестових завдань для складання
E. Аспарагiн ліцензійного іспиту Крок -1
Збірник тестових завдань для складання «Стоматологія». – 2017. – №106.
ліцензійного іспиту Крок -1 «Загальна
лікарська підготовка». – 2007. – №125. 26. У збудливiй клiтинi заблокували iоннi
канали. Це суттєво не змiнило рiвень
24. У медичнiй практицi для профiлактики потенцiалу спокою, але клітина втратила
алкоголiзму широко використовують здатнiсть до генерацiї ПД. Якi канали
тетурам, який є iнгiбiтором заблоковано?
альдегiддегiдрогенази. Пiдвищення в кровi A. Натрiєвi
якого метаболiту викликає вiдразу до B. Калiєвi
алкоголю? C. Натрiєвi та калiєвi
A. Ацетальдегiд D. Хлорнi
B. Етанол E. Кальцiєвi
C. Малоновий альдегiд Збірник тестових завдань для складання
D. Пропiоновий альдегiд ліцензійного іспиту Крок -1
E. Метанол «Стоматологія». – 2005. – №160.
Збірник тестових завдань для складання
ліцензійного іспиту Крок -1 «Загальна 27. У немовляти спостерiгаються
лікарська підготовка». – 2006. – №161. епiлептиформнi судоми, викликанi
дефiцитом вiтамiну В6. Це спричинено
25. Солдати, якi отримали поранення у зменшенням у нервовiй тканинi гальмiвного
розпал битви, можуть не вiдчувати болю до медiатора- γ-амiномасляної кислоти.
її завершення. Якi гормони опiатної Активнiсть якого ферменту знижена при
антиноцiцептивної системи зменшують цьому?
вiдчуття болю? A. Глутаматдекарбоксилаза
A. Ендорфiни B. Аланiнамiнотрансфераза
B. Серотонiни C. Глутаматдегiдрогеназа
C. Вазопресин D. Пiридоксалькiназа
D. Альдостерон E. Глутаматсинтетаза
E. Окситоцин Збірник тестових завдань для складання
ліцензійного іспиту Крок -1 «Загальна
лікарська підготовка». – 2016. – №16.

85
Біохімія м’язової тканини
1. В результатi виснажуючої м’язової роботи Збірник тестових завдань для складання
у робочого значно зменшилась буферна ліцензійного іспиту Крок -1 «Загальна
ємнiсть кровi. Надходженням лікарська підготовка». -2015. -№145.
якої речовини у кров можна пояснити це
явище? 5. Для пiдвищення результатів спортсмену
A. Лактат рекомендували застосовувати препарат, який
B. Пiруват мiстить у собі карнiтин. Який процес в
C. 1,3-бiсфосфоглiцерат найбiльшому ступенi активується
D. Альфа-кетоглутарат карнiтином?
E. 3-фосфоглицерат A. Транспорт жирних кислот у мiтохондрiї
Збірник тестових завдань для складання B. Синтез кетонових тiл
ліцензійного іспиту Крок -1 «Загальна C. Синтез лiпiдiв
лікарська підготовка». -2017. -№40. D. Тканинне дихання
E. Синтез стероїдних гормонiв
2. Хворому з прогресуючою м’язовою Збірник тестових завдань для складання
дистрофією було проведено бiохiмiчне ліцензійного іспиту Крок -1 «Загальна
дослiдження сечi. Поява якої речовини у лікарська підготовка». -2014. -№42.
великiй кiлькостi в сечі може пiдтвердити 6. У чоловiка 35-ти рокiв пiд час тривалого
захворювання м’язiв у даного хворого? бiгу виникла гостра серцева недостатнiсть.
A. Креатин Якi змiни iонного складу
B. Порфiрини спостерiгаються у серцевому м’язi при
C. Сечовина цьому станi?
D. Гiпуровакислота A. Накопичення в клiтинах міокарда iонiв
E. Креатинiн Na+ i Ca2+
Збірник тестових завдань для складання B. Накопичення в клiтинах міокарда iонiв K+
ліцензійного іспиту Крок -1 «Загальна i Mg2+
лікарська підготовка». -2016. -№44. C. Зменшення в клiтинах мiокарда іонів Na+
i Ca2+
3. При дослiдженнi кровi хворого виявлено D. Зменшення в позаклiтинному просторi
значне збiльшення активності МВ-форм iонiв K+ i Mg2+
КФК (креатинфосфокiнази) та ЛДГ-1. Яку E. Збiльшення в позаклiтинному просторi
патологію можна припустити? iонiв Na+ i Ca2+
A. Iнфаркт мiокарда Збірник тестових завдань для складання
B. Гепатит ліцензійного іспиту Крок -1 «Загальна
C. Ревматизм лікарська підготовка». -2012. -№126.
D. Панкреатит
E. Холецистит 7. Жiнцi 54-х рокiв поставили попереднiй
Збірник тестових завдань для складання дiагноз: iнфаркт мiокарда. Характерною
ліцензійного іспиту Крок -1 «Загальна ознакою даного захворювання є суттєве
лікарська підготовка». -2014. -№15. пiдвищення в кровi активностi такого
ферменту:
4. При напруженiй фiзичнiй роботi у A. Креатинфосфокiназа
м’язовiй тканинi накопичується молочна B. Каталаза
кислота, яка дифундує в кров i C. Г-6-ФДГ
підхоплюється печiнкою та серцем. Який D. Альфа-амiлаза
процес забезпечує вiдновлення запасiв E. Аргiназа
глiкогену в м’язах? Збірник тестових завдань для складання
A. Цикл Корi ліцензійного іспиту Крок -1 «Загальна
B. Цикл лимонної кислоти лікарська підготовка». -2013. -№14.
C. Орнiтиновий цикл
D. Цикл трикарбонових кислот 8. У вiддiлення iнтенсивної терапiї
E. Пентозофосфатний шлях доставлено жiнку 50-ти рокiв з дiагнозом:
iнфаркт мiокарда. Активнiсть якого
86
ферменту кровi буде найбiльш пiдвищена B. Лактатдегiдрогеназа
протягом перших двох дiб захворювання? C. Пiруватдегiдрогеназа
A. Аспартатамiнотрансфераза D. Глутаматдегiдрогеназа
B. Аланiнамiнотрансфераза E. Аденiлаткiназа
C. Аланiнамiнопептидаза Збірник тестових завдань для складання
D. ЛДГ4 ліцензійного іспиту Крок -1 «Загальна
E. ЛДГ5 лікарська підготовка». -2010. - № 3.
Збірник тестових завдань для складання
ліцензійного іспиту Крок -1 13. Хворий 50-ти рокiв звернувся до клiнiки
«Стоматологія». – 2013. – №91. зi скаргами на загальну слабкiсть, втрату
апетиту, аритмію серця.
9. У нетренованих людей часто виникає біль Спостерiгається гiпотонiя м’язiв, млявi
в м'язах після фізичного навантаження в паралiчi, послаблення перистальтики
результаті накопичення лактату. Це може кишечнику. Причиноют акого стану може
бути викликано інтенсифікацією наступного бути:
біохімічного процесу: A. Гiпокалiємiя
А. Гліколіз B. Гiпопротеїнемiя
Б. Глюконеогенез C. Гiперкалiємiя
C. Пентозофосфатний шлях D. Гiпофосфатемiя
D. Ліпогенез E. Гiпонатрiємiя
Е. Глікогенез Збірник тестових завдань для складання
ліцензійного іспиту Крок -1 «Загальна
10. У юнака 18-ти рокiв діагностовано лікарська підготовка». -2009. -№46.
м’язова дистрофiя. Пiдвищення в сироватцi
кровi вмiсту якої речовини найбiльш 14. Пiсля тривалого фiзичного навантаження
iмовiрне при цiй патологiї? пiд час заняття з фізичної культури у
A. Креатин студентiв розвинулась м’язова крепатура.
B. Мiоглобiн Причиноюї ї виникнення стало накопичення
C. Мiозин у скелетних м’язах молочної кислоти. Вона
D. Лактат утворилась пiсля активацiї в органiзмi
E. Аланiн студентiв:
Збірник тестових завдань для складання A. Глiколiзу
ліцензійного іспиту Крок -1 «Загальна B. Глюконеогенезу
лікарська підготовка». -2011. -№62. C. Пентозофосфатного циклу
D. Лiполiзу
11. У студента 18-ти рокiв пiд час фiзичного E. Глiкогенезу
навантаження реографiчно зареєстровано Збірник тестових завдань для складання
перерозподiл кровотоку ліцензійного іспиту Крок -1 «Загальна
органiв. У яких судинах кровотiк пiдвищився лікарська підготовка». -2010. -№155.
найбiльшою мiрою?
A. Скелетнi м’язи 15. Пiд час бiгу на короткi дистанцiї у
B. Печiнка нетренованої людини виникає м’язова
C. Головний мозок гiпоксiя. До накопичення якого метаболiту в
D. Нирки м’язах це призводить?
E. Шлунково-кишковий тракт A. Лактат
Збірник тестових завдань для складання B. Кетоновi тiла
ліцензійного іспиту Крок -1 «Загальна C. Ацетил-КоА
лікарська підготовка». -2011. -№34. D. Глюкозо-6-фосфат
E. Оксалоацетат
12. Хвора 46-ти рокiв довгий час страждає на Збірник тестових завдань для складання
прогресуючу м’язову дистрофiю( Дюшена). ліцензійного іспиту Крок -1 «Загальна
Змiни рiвня якого ферменту кровi є лікарська підготовка». -2009. -№107.
дiагностичним тестом в даному випадку?
A. Креатинфосфокiназа
87
16. Хворий 49-ти рокiв, водiй за професiєю, B. Адреналiн
скаржиться на нестерпний стискаючий бiль C. Норадреналiн
за грудниною, що "вiддає"у дiлянку шиї. D. Тироксин
Бiль виник 2 години тому. Об’єктивно: стан E. Атрiопептид
важкий, блiдiсть, тони серця послабленi. Збірник тестових завдань для складання
Лабораторне обстеження показало високу ліцензійного іспиту Крок -1 «Загальна
активнiсть креатинкiнази та ЛДГ1. Для якого лікарська підготовка». -2005. - № 40.
захворювання характернi такi симптоми?
A. Гострий iнфаркт мiокарда 20. У хворого через 12 годин після гострого
B. Гострий панкреатит нападу загруднинного болю знайдено рiзке
C. Стенокардiя пiдвищення активності АсАТ у сироватцi
D.Жовчнокам’яна хвороба кровi. Вкажiть патологiю, для якої
E. Цукровий дiабет характерне це змiщення:
Збірник тестових завдань для складання A. Iнфаркт мiокарда
ліцензійного іспиту Крок -1 «Загальна B. Вiрусний гепатит
лікарська підготовка». -2009. -№189. C. Колагеноз
D. Цукровий дiабет
17. У цитоплазмi мiоцитiв розчинена велика E. Нецукровий діабет
кiлькiсть метаболiтiв окиснення глюкози. Збірник тестових завдань для складання
Назвiть один з них, який безпосередньо ліцензійного іспиту Крок -1 «Загальна
перетворюється на лактат: лікарська підготовка». -2007. -№22.
A. Пiруват
B. Оксалоацетат 21. У хворого виявлено підвищення
C. Глiцерофосфат активностi ЛДГ 1,2, АсАТ,
D. Глюкозо-6-фосфат креатинфосфокiнази. В якому органi
E. Фруктозо-6-фосфат (органах) найбiльш вiрогiдний розвиток
Збірник тестових завдань для складання патологiчного процесу?
ліцензійного іспиту Крок -1 «Загальна A. Серцевий м’яз
лікарська підготовка». -2009. -№108. B. Скелетнi м’язи
C. Нирки та наднирковi залози
18. В лiкарню звернувся хворий зi скаргами D. Сполучна тканина
на швидку стомлюванiсть i виражену E. Печiнка та нирки
м’язову слабкiсть. При обстеженнi виявлено Збірник тестових завдань для складання
аутоiмунне захворювання, внаслiдок якого ліцензійного іспиту Крок -1 «Загальна
порушується функцiональний стан лікарська підготовка». -2007. -№11.
рецепторів у нервово-м’язових синапсах. Дiя
якого медiатора буде заблокована? 22. До вiддiлення травматологiї надiйшов
A. Ацетилхолiн хворий iз значним пошкодженням м’язової
B. Норадреналiн тканини. Який біохімічний показник сечi
C. Дофамiн буде збiльшений при цьому?
D. Серотонiн A. Креатинiн
E. Глiцин B. Загальнi лiпiди
Збірник тестових завдань для складання C. Глюкоза
ліцензійного іспиту Крок -1 «Загальна D. Мiнеральнi солi
лікарська підготовка». -2008. -№175. E. Сечова кислота
Збірник тестових завдань для складання
19. Пiд час обробки атипових кардiомiоцитiв ліцензійного іспиту Крок -1 «Загальна
синоатрiального вузла бiологiчно активною лікарська підготовка». -2006. -№47.
речовиною, зареєстровано збiльшення їх
мембранного потенцiалу через збiльшену 23. До клiнiки надiйшла дитина 1 року з
проникнiсть для iонiв калiю. Яка бiологiчно ознаками ураження м’язiв. Пiсля обстеження
активна речовина впливала на виявлений дефiцит карнітину в м’язах.
кардiомiоцити? Порушення якого процесу бiохiмiчною
A. Ацетилхолiн основою цiєї патологiї?
88
A. Транспорт жирних кислот у мiтохондрiї активується глюконеогенез, основним
B. Регуляцiя рiвня Ca2+ в мiтохондрiях субстратом якого в цьому випадку є:
C. Субстратне фосфорилювання A. Лактат
D. Утилiзацiя молочної кислоти B. Аспарагінова кислота
E. Синтез актину та мiозину C. Глутаміновая кислота
Збірник тестових завдань для складання D. Альфа-кетоглутарат
ліцензійного іспиту Крок -1 «Загальна E. Серін
лікарська підготовка». -2006. -№162. Зборник тестовых заданий для сдачи
лицензионного экзамена Крок -1 «Общая
24. Пiд час бiгу на довгi дистанцiї скелетна врачебная подготовка». -2011. -34.
мускулатура тренованої людини
використовує глюкозу з метою отримання 26. У відділення реанімації поступив чоловік
енергiї АТФ для м’язового скорочення. 47-ми років з діагнозом інфаркт міокарда.
Вкажiть основний процес утилiзацiї глюкози Яка з фракцій лактатдегідрогенази (ЛДГ)
в цих умовах: буде переважати в сироватці крові протягом
A. Аеробний глiколiз перших двох діб?
B. Анаеробний глiколiз A. ЛДГ 1
C. Глiкогенолiз B. ЛДГ 2
D. Глюконеогенез C. ЛДГ 3
E. Глiкогенез D.ЛДГ 4
Збірник тестових завдань для складання E. ЛДГ 5
ліцензійного іспиту Крок -1 «Загальна Зборник тестовых заданий для сдачи
лікарська підготовка». -2005. -№122. лицензионного экзамена Крок -1 «Общая
врачебная подготовка». -2016. -34
25. Через деякий час після інтенсивного
фізичного тренування у спортсмена

89
Біохімія сполучної тканини
1. При обстеженнi хворого виявлена Збірник тестових завдань для складання
характерна клініка колагенозу. ліцензійного іспиту Крок -1
Вкажiть,збільшення якого показника сечi «Стоматологія». – 2006. – № 104.
характерне для цієї патологiї:
A. Гiдроксипролiн 5. У п’ятирiчного хлопчика спостерi- галися
B. Аргiнiн малий зрiст, розумове вiдставання, обмеженi
C. Глюкоза рухи, грубi риси обличчя. Цi особливостi
D. Мiнеральнiсолi стали помiтними з 18-мiсячного вiку. У
E. Солi амонiю нього виявили дефіцит L-iдуронiдази. Обмiн
Збірник тестових завдань для складання яких сполук порушено?
ліцензійного іспиту Крок -1 «Загальна A. Глiкозамiноглiкани
лікарська підготовка». -2016. -№ 20. B. Бiлки
C. Нуклеотиди
2. У дитини спостерiгається затримка D. Вiтамiни
фiзичного та розумового розвитку, глибокi E. Фосфолiпiди
порушення з боку сполучної тканини Збірник тестових завдань для складання
внутрiшнiх органiв, у сечi виявлено ліцензійного іспиту Крок -1 «Загальна
кератансульфати. Обмiн яких речовин лікарська підготовка». -2015. -№ 199.
порушений? 6. У пацiєнта стоматологiчного вiддiлення
A. Глiкозамiноглiкани виявлено хворобу Педжета, що
B. Колаген супроводжується деградацiєю колагену.
C. Еластин Вирiшальним фактом для постановки
D. Фiбронектин дiагнозу було виявлення у сечi хворого
E. Гiалуронова кислота пiдвищеного рiвня:
Збірник тестових завдань для складання A. Оксипролiну
ліцензійного іспиту Крок -1 «Загальна B. Аргiнiну
лікарська підготовка». -2015. -№ 112. C. Триптофану
D. Серину
3. Спадкові захворювання, такі як - E. Аланiну
мукополісахаридози, проявляються Збірник тестових завдань для складання
порушенням обміну в сполучній тканині, ліцензійного іспиту Крок -1 «Загальна
патологією кісток і суглобів. Надлишкова лікарська підготовка». -2015. -№ 195.
екскреція в складі сечі якої речовини
свідчить про наявність такої патології? 7. Чоловiк 60-ти рокiв скаржиться на бiль у
A. Глікозаміноглікани суглобах. У сироватцi кровi пацiєнта
B. Амінокислоти виявлено пiдвищення концентрацiї С-
C. Глюкоза реактивного бiлку та оксипролiну. Для якого
D. Ліпіди захворювання характернi ці симптоми?
E. Сечовина A. Ревматизм
Test items for licensing examination Krok 1 B. Подагра
«Medicine». – 2017. – № 133. C. Гепатит
D.Жовтяниця
4. У вагiтної 28 рокiв, дослiджували E. Цукровий дiабет
ферменти в клiтинах амнiотичної рiдини. Збірник тестових завдань для складання
При цьому виявилася недостатня активнiсть ліцензійного іспиту Крок -1 «Загальна
β-глюкуронiдази. Який патологiчний процес лікарська підготовка». -2012. -№ 119.
спостерiгається?
A. Мукополiсахародоз 8. Хвора 36-ти рокiв страждає на колагеноз.
B. Глiкогеноз Збiльшення вмiсту якого метаболiту
C. Аглiкогеноз найбiльш iмовiрно буде встановлено у сечi?
D. Колагеноз A. Оксипролiн
E. Лiпiдоз B. Iндикан
C. Креатинiн
90
D. Сечовина D. Загальний холестерин
E. Уробiлiноген E. R-глiкозидаза
Збірник тестових завдань для складання Збірник тестових завдань для складання
ліцензійного іспиту Крок -1 «Загальна ліцензійного іспиту Крок -1 «Загальна
лікарська підготовка». -2011. -№ 80. лікарська підготовка». -2005. -№ 58.

9. Жiнка 30-ти рокiв хворiє близько року, 13. При остеолатеризмi зменшується
коли вперше з’явився бiль у дiлянцi суглобiв, мiцнiсть колагену, що зумовлена помiтним
їх припухлiсть, почервонiння шкiри над зменшенням утворення поперечних зшивок у
ними. Попереднiй дiагноз - ревматоїдний колагенових фiбрилах. Причиною цього
артрит. Змiна якого компоненту в структурi явища є зниження активностi такого
бiлка сполучної тканини є однiєю з причин ферменту:
цього захворювання? A. Лiзiлоксидаза
A. Колаген B. Моноамiноксидаза
B. Муцин C. Пролiлгiдроксилаза
C. Мiозин D. Лiзiлгiдроксилаза
D. Овоальбумiн E. Колагеназа
E. Тропонiн Збірник тестових завдань для складання
Збірник тестових завдань для складання ліцензійного іспиту Крок -1
ліцензійного іспиту Крок -1 «Загальна «Стоматологія». – 2012. – №146.
лікарська підготовка». -2007. -№ 24.
14. У хворого 34-х років на пародонтит
10. У хворих на колагеноз має мiсце процес внаслідок посилення розпаду коллагена
деструкцiї сполучної тканини. Це значно збільшилася екскреція з сечею
пiдтверджується збiльшенням у кровi: однією з амінокіслот.Якої кой саме?
A. Вмiсту оксипролiну та оксилiзину A. Оксипролину
B. Вмiсту креатину та креатинiну B. Валіну
C. Активностi iзоферментiв ЛДГ C. Аланіну
D. Активностi трансамiназ D. Гліцину
E. Вмiсту уратiв E. Серину
Збірник тестових завдань для складання Збірник тестових завдань для складання
ліцензійного іспиту Крок -1 «Загальна ліцензійного іспиту Крок -1
лікарська підготовка». -2006. -№ 20. «Стоматологія». – 2014. – №117.

11. У чоловіка 53-х років діагностовано 15. У процесi звапнування мiжклiтинної


хворобу Педжета. У сечі різко підвищений речовини кісткової тканини вздовж
рівень оксипроліну, що свідчить перш за все колагенових волокон відкладаються
пропосилення розпаду: кристали гідроксиапатиту. Для реалізації
A. Коллагена цього процесу необхiдна присутнiсть у
B. кератину мiжклiтиннiй речовинi лужної фосфатази.
C. Альбуміну Яка клiтина продукує цей фермент?
D. Гемоглобіну A. Остеобласт
E. Фібриногену B. Остеоцит
Test items for licensing examination Krok 1 C. Остеокласт
«Medicine». – 2014. – № 6. D. Хондробласт
E. Хондроцит
12. Жiнка 63 рокiв має ознаки ревматоїдного Збірник тестових завдань для складання
артриту. Пiдвищення рiвня якого з ліцензійного іспиту Крок -1
перерахованих нижче показникiв «Стоматологія». – 2016. – №66.
кровi буде найбiльш значущим для
пiдтвердження дiагнозу? 16. Пiсля загоєння рани на її мiсцi утворився
A. Сумарнi глiкозамiноглiкани рубець. Яка речовина є основним
B. Лiпопротеїди компонентом цього рiзновиду сполучної
C. Кисла фосфатаза тканини?
91
A. Колаген Збірник тестових завдань для складання
B. Еластин ліцензійного іспиту Крок -1
C. Кератансульфат «Стоматологія». – 2005. – №118.
D. Хондроїтинсульфат
E. Гiалуронова кислота 19. У пацiєнта навеснi з’явилися петехiальнi
Збірник тестових завдань для складання крововиливи, розхитування зубiв, вiн
ліцензійного іспиту Крок -1 вiдмiчає високу чутливiсть до простудних
«Стоматологія». – 2013. – №38. хвороб. Лiкар припустив гiповiтамiноз C.
Чим пояснюються змiни з боку зубiв?
17. У хворого на системну склеродермiю A. Порушення структури колагену
посилений розпад колагену. Посилення перiодонтальних зв’язок
екскрецiї з сечею якої амiнокислоти буде B. Змiна структури глiкозамiноглiканiв
вiддзеркалювати процеси деструкцiї C. Пiдвищення проникностi мембран
колагену? навколозубних тканин
A. Оксипролiн D. Механiчне ушкодження зубiв
B. Аланiн E. Порушення окисно-вiдновних процесiв у
C. Триптофан навколозубних тканинах
D. Серин Збірник тестових завдань для складання
E. Фенiлаланiн ліцензійного іспиту Крок -1
51. Яка речовина надає слинi в «Стоматологія». – 2010. – №193.
Збірник тестових завдань для складання
ліцензійного іспиту Крок -1 20. Пiдвищена ламкiсть судин, руйнування
«Стоматологія». – 2008. – №50. емалi i дентину у хворих на цингу
здебiльшого зумовлено порушенням
18. У хворого, що страждає на цингу, дозрiвання колагену. Який етап модифiкацiї
порушено процеси утворення сполучної проколагену порушено при цьому
тканини, що призводить до розхитування i авiтамiнозi?
випадiння зубiв. Порушення активностi A. Гiдроксилювання пролiну
якого ферменту зумовлює цi симптоми? B. Утворення полiпептидних ланцюгiв
A. Лiзилгiдроксилаза C. Глiкозилювання гiдроксилiзинових
B. Глiкозилтрансфераза залишкiв
C. Еластаза D. Видалення з проколагену С-кiнцевого
D. Проколагенпептидаза N-кiнцевого пептиду
пептиду E. Вiдщеплення N-кiнцевого пептиду
E. Проколагенпептидаза С-кiнцевого Збірник тестових завдань для складання
пептиду ліцензійного іспиту Крок -1
«Стоматологія». – 2005. – № 120.

92
Біохімія зуба та слини
1. Який фермент має демiнералiзуючу дію – 5. Захисна функція слини зумовлена
посилює розщеплення мiнеральних декількома механiзмами, в тому числі
компонентів тканин зуба? наявнiстю ферменту, який має бактерицидну
A. Кисла фосфатаза дiю, викликає лізис полісахаридного
B. Лужна фосфатаза комплексу оболонки стафiлококiв,
C. Глюкозо-6-фосфатаза стрептококiв. Вкажiть цей фермент:
D. Глiкогенфосфорилаза A. Лiзоцим
E. Фосфотрансфераза B. α-амiлаза
Збірник тестових завдань для складання C. Олiго-1,6-глюкозидаза
ліцензійного іспиту Крок -1 D. Колагеназа
«Стоматологія». – 2018. – №133. E. β-глюкуронiдаза
Збірник тестових завдань для складання
2. У процесi звапнування мiжклiтинної ліцензійного іспиту Крок -1 «Загальна
речовини кiсткової тканини вздовж лікарська підготовка». -2018. -№7.
колагенових волокон вiдкладаються
кристали гiдроксиапатиту. Для реалізації 6. Лiкарю-пародонтологу необхiдно оцiнити
цього процесу необхiдна присутнiсть у у пацiєнтки фактори неспецифiчної
мiжклiтиннiй речовинi лужної фосфатази. резистентностi слини i видільного слизової
Яка клiтина продукує цей фермент? оболонки ротової порожнини. Який фактор
A. Остеобласт неспецифiчної резистентностi слiд вивчити у
B. Остеоцит дослiджуваному матерiалi у першу чергу?
C. Остеокласт A. Лiзоцим
D. Хондробласт B. Секреторний IgA
E. Хондроцит C. Пропердин
Збірник тестових завдань для складання D. Iнтерферон
ліцензійного іспиту Крок -1 E. Комплемент
«Стоматологія». – 2013. – №96. Збірник тестових завдань для складання
ліцензійного іспиту Крок -1
3. У нормi рН слини складає 6,4 – 7,8. До «Стоматологія». – 2005. – №171.
яких змiн емалi призводить зсув рН слини у
кислий бiк (менше 6,2)? 7. Яка речовина надає слинi в’язкий,
A. Демiнералiзацiя слизовий характер, виконує захисну роль, у
B. Кальцифiкацiя тому числi від механiчного пошкодження
C. Флюороз слизової рота?
D. Мiнералiзацiя A. Муцин
E. Пiдвищення стiйкостi B. Глюкоза
Збірник тестових завдань для складання C. Калiкреїн
ліцензійного іспиту Крок -1 D. Амiлаза
«Стоматологія». – 2005. – №124. E. Лiзоцим
Збірник тестових завдань для складання
4. Катiоннi глiкопротеїни є основними ліцензійного іспиту Крок -1
компонентами слини привушних залоз. Якi «Стоматологія». – 2006. – №103.
амiнокислоти обумовлюють їх позитивний
заряд? 8. Деякi бiлки слини виконують захисну
A. Лiзин, аргiнiн, гiстидин функцiю. Який з них захищає слизову
B. Аспартат, глутамат, глiцин оболонку ротової порожнини від механiчних
C. Аспартат, аргiнiн, глутамат ушкоджень?
D. Глутамат, валiн, лейцин A. Муцин
E. Цистеїн, глiцин, пролін B. Лiзоцим
Збірник тестових завдань для складання C. Каталаза
ліцензійного іспиту Крок -1 «Загальна D. Пероксидаза
лікарська підготовка». -2013. -№90. E. Ренiн

93
Збірник тестових завдань для складання Збірник тестових завдань для складання
ліцензійного іспиту Крок -1 ліцензійного іспиту Крок -1
«Стоматологія». – 2013. – №185. «Стоматологія». – 2007. – №103.

9. У хворого на хронiчне запалення 13. При обробцi перекисом водню слизової


субмаксилярної слинної залози оболонки ротової порожнини хворого, кров
спостерiгається гiпосалiвацiя. Порушення пофарбувалась у коричневий колiр замiсть
iнкрецiї якої біологічно активної речовини пiноутворення. При зниженнi концентрацiї
спостерiгається при цьому? якого з перелiчених ферментiв це можливо?
A. Паротин A. Каталаза
B. Кальцитонiн B. Псевдохолiнестераза
C. Паратирiн C. Глюкозо-6-фосфатдегiдрогеназа
D. Глюкагон D. Ацетилтрансфераза
E. Соматостатин E. Метгемоглобiнредуктаза
Збірник тестових завдань для складання Збірник тестових завдань для складання
ліцензійного іспиту Крок -1 ліцензійного іспиту Крок -1
«Стоматологія». – 2005. – №22. «Стоматологія». – 2012. – №44.

10. У хворого iз зниженою видільною 14. При пародонтитах розвивається ліпідна


функцiєю нирок вiдзначається неприємний пероксидація в тканинах пародонта, в
запах з рота. Збiльшення екскрецiї слинними ротовій порожнині наростає вміст
залозами якої речовини є причиною цього? малонового диальдегіду, пероксиду водню.
A. Сечовина Які з перерахованих ферментів здійснюють
B. Альфа-амiлаза антиоксидантний захист?
C. Лiзоцим A. Супероксиддисмутаза, каталаза
D. Фосфатаза B. Амілаза, трипсин
E. Муцин C. Мальтаза, хімотрипсин
Збірник тестових завдань для складання D. Лактаза, лізоцим
ліцензійного іспиту Крок -1 E. Сахараза, протромбін
«Стоматологія». – 2006. – №67. Сборник тестовых заданий для
лицензионного экзамена Крок-1
11. У дитини встановлена гостра ниркова «Стоматология». – 2013. - № 113.
недостатнiсть. Якими бiохiмiчними
показниками слини це можна пiдтвердити? 15. З вiком знижується активнiсть
A. Пiдвищення рiвня залишкового азоту навколовушних слинних залоз. Активнiсть
B. Збiльшення iмуноглобулiну А якого ферменту буде зменшуватися в слинi?
C. Зниження лужної фосфатази A. Амiлаза
D. Збiльшення альфа-амiлази B. Лiзоцим
E. Зменшення рiвня фосфату C. Фосфатаза
Збірник тестових завдань для складання D. Гексокiназа
ліцензійного іспиту Крок -1 E. Мальтаза
«Стоматологія». – 2007. – №101. Збірник тестових завдань для складання
ліцензійного іспиту Крок -1
12. Пародонтит супроводжується активацiєю «Стоматологія». – 2006. – №168.
протеолiзу в тканинах пародонту.
Пiдвищення якого компоненту ротової 16. У хворої 60-ти рокiв знижена активнiсть
рiдини свiдчить про активацiю протеолiзу? основного травного ферменту слини. В
A. Амiнокислоти цьому випадку порушується первинний
B. Органiчнi кислоти гiдролiз:
C. Глюкоза A. Вуглеводiв
D. Бiогеннi амiни B. Жирiв
E. Холестерол C. Бiлкiв
D. Клiтковини
E. Молочного цукру
94
Збірник тестових завдань для складання 21. Для формування мiнерального матриксу
ліцензійного іспиту Крок -1 твердих тканин зуба необхiдна висока
«Стоматологія». – 2010. – №135. концентрацiя фосфат-iонiв, що утворюється
в процесi гiдролiзу фосфорноефiрних
17. Необхiдно оцiнити перетравлюючi зв’язкiв за участю лужної фосфатази. Iони
властивостi слини. З яким субстратом для якого металу є активаторами цього процесу:
цього її треба змiшати? A. Цинк
A. Крохмаль B. Магнiй
B. Казеїн C. Ферум
C. Жир D. Кальцiй
D. ДНК E. Натрiй
E. РНК Збірник тестових завдань для складання
Збірник тестових завдань для складання ліцензійного іспиту Крок -1
ліцензійного іспиту Крок -1 «Стоматологія». – 2013. – №136.
«Стоматологія». – 2014. – №30.
22. В економiчно розвинених країнах
18. При дослiдженнi слини людини поширеним захворюванням є карiєс зубiв. Це
необхiдно оцiнити її гідролітичні захворювання уражає бiльш 95% населення.
властивостi. Що з наведеного потрiбно при Що вiдiграє головну роль у демiнералiзацiї
цьому використати як субстрат? твердих тканин зуба при карiєсi?
A. Крохмаль A. Органiчнi кислоти
B. Бiлки B. Неповноцiнне харчування
C. Жири C. Екстремальнi впливи на органiзм
D. Клiтковина D. Розлад регуляцiї метаболiзму
E. Амiнокислоти E. Недостатнiсть вiтамiну C
Збірник тестових завдань для складання Збірник тестових завдань для складання
ліцензійного іспиту Крок -1 ліцензійного іспиту Крок -1
«Стоматологія». – 2016. – №188. «Стоматологія». – 2015. – №167.

19. При вживаннi печива, цукерок у змiшанiй 23. Надмiрна концентрацiя глюкози в
слинi тимчасово зростає рiвень лактату. ротовiй рiдинi при цукровому дiабетi
Активацiя якого бiохiмiчного процесу призводить до розвитку:
призводить до цього? A. Множинного карiєсу
A. Анаеробний глiколiз B. Гiперплазiї емалi
B. Тканинне дихання C. Гiпоплазiї емалi
C. Аеробний глiколiз D. Флюорозу
D. Глюконеогенез E. Посиленої кальцифiкацiї емалi
E. Мiкросомальне окислення Збірник тестових завдань для складання
Збірник тестових завдань для складання ліцензійного іспиту Крок -1
ліцензійного іспиту Крок -1 «Стоматологія». – 2017. – №31.
«Стоматологія». – 2005. – №119.
24. При профiлактичному оглядi дiтей в
20. У слинi пацiєнта виявлено пiдвищений закарпатському селищi у багатьох знайдено
вмiст лактату. Активацiя якого процесу є множинний карiєс. З недостатнiстю якого
основною причиною пiдвищення лактату? мiнералу в їжi можна пов’язати розвиток
A. Анаеробний розпад глюкози карiєсу?
B. Аеробний розпад глюкози A. Фтор
C. Розпад глiкогену B. Йод
D. Гiдролiз вуглеводiв C. Молiбден
E. Глюкозо-лактатний цикл D. Залiзо
Збірник тестових завдань для складання E. Кобальт
ліцензійного іспиту Крок -1 Збірник тестових завдань для складання
«Стоматологія». – 2009. – №38. ліцензійного іспиту Крок -1
«Стоматологія». – 2017. – №35.
95
29. Провiдну роль в процесi кальцифiкацiї
25. У дитини спостерiгається порушення тканин зуба вiдiграє білок остеокальцин,
процесiв окостенiння та "крапчастiсть який має високу здатнiсть зв’язувати iони
емалi". Обмiн якого мiкроелементу при кальцiю, завдяки наявностi в
цьому порушений? полiпептидному ланцюзi залишкiв
A. Фтор модифiкованої амiнокислоти:
B. Залiзо A. Гама-карбоксиглутамiнова
C. Цинк B. Аланiн
D. Хром C. Гама-амiномасляна
E. Мiдь D. Карбоксиаспарагiнова
Збірник тестових завдань для складання E. Дельта-амiнопропiонова
ліцензійного іспиту Крок -1 Збірник тестових завдань для складання
«Стоматологія». – 2007. – №102. ліцензійного іспиту Крок -1
«Стоматологія». – 2008. – №170.
26. Для ремiнералiзуючої терапiї
початкового карiєсу зубiв була призначена 30. До стоматолога звернувся чоловiк 35-ти
сiль лужного металу. Визначте препарат: рокiв зi скаргами на зменшення щiльностi
A. Натрiю фторид зубної тканини, підвищену крихкiсть зубiв
B. Натрiю бромид при вживаннi твердої їжi. Лабораторно
C. Натрiю хлорид визначили спiввiдношення Ca/P в емалi при
D. Калiю хлорид зiскоблюваннi. Яке значення цього
E. Калiю бромід показника свідчить про посилення
Збірник тестових завдань для складання демiнералiзацiї?
ліцензійного іспиту Крок -1 A. 0,9
«Стоматологія». – 2017. – №50. B. 1,67
C. 1,85
27. У хворого з нирковою недостатнiстю D. 2,5
розвинулась остеодистрофiя, що E. 1,5
супроводжується iнтенсивною Збірник тестових завдань для складання
демiнералiзацiєю кiсток. Порушення ліцензійного іспиту Крок -1
утворення активної форми якого вiтамiну є «Стоматологія». – 2011. – №187.
причиною даного ускладнення?
A. Кальциферол 31. Для лікування пародонтиту в комплекс
B. Ретинол препаратів був включений лікарський засіб із
C. Тiамiн групи водорозчинних вітамінів, похідне
D. Нафтохiнон біофлавоноїдів, який призначають разом із
E. Рибофлавiн кислотою аскорбіновою. Препарат має
Збірник тестових завдань для складання антиоксидантні властивості, зменшує
ліцензійного іспиту Крок -1 кровоточивість ясен. Який це препарат?
«Стоматологія». – 2012. – №43. A. Рутин.
B. Кальцію пантотенат.
28. Емаль характеризується високою C. Кальцію пангамат.
стiйкiстю до дiї рiзних механiчних та D. Кислота фолієва.
хiмiчних факторiв. Синтез якого компоненту E. Ціанкобаламін.
забезпечує таку резистентнiсть? Збірник тестових завдань для складання
A. Фторапатит ліцензійного іспиту Крок-1 «Стоматологія».
B. Гiдроксиапатит – 2006. – № 177
C. Хлорапатит
D. Колаген 32. Пiд час огляду дитини, що не отримувала
E. Карбонатний апатит впродовж зими свiжих овочiв і фруктiв,
Збірник тестових завдань для складання виявленi множиннi пiдшкiрнi крововиливи,
ліцензійного іспиту Крок -1 запалення ясен, карiознi порожнини в зубах.
«Стоматологія». – 2008. – №170. Комбiнацiю яких вiтамiнiв слiд призначити
дитинi?
96
A. Аскорбiнова кислота та рутин. Збірник тестових завдань для складання
B. Кальциферол та аскорбiнова кислота. ліцензійного іспиту Крок-1 «Стоматологія».
C. Рибофлавiн і нiкотинамiд. – 2009. – № 163
D. Тiамiн і пiридоксин.
E. Фолiєва кислота і кобаламiн. 36. У дитини вiдзначається затримка
Збірник тестових завдань для складання прорiзування зубiв, неправильне їх
ліцензійного іспиту Крок-1 «Стоматологія». розташування, при огляді помітна сухiсть
– 2007. – № 88 ротової порожнини, в кутиках рота –
трiщини з нагноєнням. З нестачею якого
33. У хворого спостерiгається збiльшення вiтамiну може бути пов’язаний цей стан?
проникностi стiнок кровоносних судин iз A. Вітаміну D.
розвитком пiдвищеної кровоточивостi ясен, B. Вітаміну С.
виникнення дрiбнокрапчастих крововиливiв C. Вітаміну E.
на шкiрi, випадiння зубiв. Яким порушенням D. Вітаміну K.
вiтамiнного обмiну пояснюються цi E. Вітаміну А.
симптоми? Збірник тестових завдань для складання
A. Гiповiтамiноз С ліцензійного іспиту Крок-1 «Стоматологія».
B. Гiпервiтамiноз D – 2011. – № 86.
C. Гiпервiтамiноз С
D. Гiповiтамiноз D 37. У дитини першого року життя
E. Гiповiтамiноз А спостерiгається збiльшення розмiрiв голови
Збірник тестових завдань для складання та живота, запiзнiле прорізування зубiв,
ліцензійного іспиту Крок-1 «Стоматологія». порушення структури емалi. Наслiдком
– 2005. – № 109. якого гiповiтамiнозу є цi змiни?
A. Гiповiтамiноз D
34. Обстежуючи ротову порожнину хворого, B. Гiповiтамiноз С
стоматолог звернув увагу на наявнiсть C. Гiповiтамiноз А
запально-дистрофiчного процесу в слизовiй D. Гiповiтамiноз В1
оболонцi (гунтеровський глосит, атрофiчний E. Гiповiтамiноз В2
стоматит). Аналiз кровi виявив гiперхромну Збірник тестових завдань для складання
анемiю. Який фактор є причиною цього ліцензійного іспиту Крок-1 «Стоматологія».
захворювання? – 2006. – № 90.
A. Гiповiтамiноз B12
B. Гiповiтамiноз B1 38. У дитини з нирковою недостатнiстю
C. Гiповiтамiноз B6 виявлена затримка прорізування зубiв.
D. Пiдвищення кислотностi шлункового соку Порушення утворення в нирках якої
E. Гiповiтамiноз A речовини найбiльш iмовiрно є причиною
Збірник тестових завдань для складання цього?
ліцензійного іспиту Крок-1 «Стоматологія». A. 1,25 (OH)2D3
– 2012. – № 67. B. Глiкоцiамiн
C. Глутамат
35. У дитини 10-ти мiсяцiв спостерiгається D. α-кетоглутарат
надмірна збудливiсть, порушення сну, E. Гiдроксильований лiзин
знижений тонус м’язiв, запiзнiле Збірник тестових завдань для складання
прорiзування зубiв з недостатньо ліцензійного іспиту Крок-1 «Стоматологія».
звапнованою емаллю. Дефiцит в органiзмi – 2010. – № 165.
якого вiтамiну зумовлює такi змiни?
A. Холекальциферолу. 39. Упацієнта спостерігається ерозія емалі.
B. Ретинолу. Який вітамін необхідно призначити для
C. Рибофлавiну. лікування?
D. Тiамiну. A. D3
E. Нiкотинамiду. B. C
C. K
D. B1
97
E. PP E. Фiлохiнон
Збірник тестових завдань для складання Збірник тестових завдань для складання
ліцензійного іспиту Крок-1 «Стоматологія». ліцензійного іспиту Крок-1 «Стоматологія».
– 2013. – № 33. – 2008. – № 161.

40. У 5-ти річної дитини спостерігається 44. При різноманітних захворюваннях рівень
недостатнє звапніння емалі, карієс зубів. активних форм кисню різко зростає, що
Гіповітаміноз якого вітаміну зумовлює призводить до руйнування клітинних
розвиток такого процесу? мембран. Для запобігання цьому
A. Кальциферол використовують антиоксиданти.
B. Токоферол Найпотужнішим з них є:
C. Біотин A. α-Токоферол.
D. Нікотинова кислота B. Вітамін D.
E. Фолієва кислота C. Гліцерол.
Збірник тестових завдань для складання D. Глюкоза.
ліцензійного іспиту Крок-1 «Стоматологія». E. Жирні кислоти.
– 2017. – № 32. Збірник тестових завдань для складання
ліцензійного іспиту Крок-1 «Стоматологія».
41. Гормоноподібна форма вітаміну індукує – 2009. – № 42.
на рівні геному синтез Ca-зв’вязуючих білків
у ентероцитах і регулює всмоктування іонів 45. При лiкуваннi сiалоаденiтiв (запалення
Ca2+ у кишечнику, від чого залежить слинних залоз) використовують препарати
розвиток тканин зуба. Який це вітамін? вiтамiнiв. Який iзнаведених вiтамiнiв
A. D3 вiдiграє важливу роль в антиоксидантному
B. A захистi?
C. B1 A. Токоферол
D. E B. Тiамiн
E. K C. Рибофлавiн
Збірник тестових завдань для складання D. Пантотенова кислота
ліцензійного іспиту Крок-1 «Стоматологія». E. Пiридоксин
– 2014. – № 36. Збірник тестових завдань для складання
ліцензійного іспиту Крок-1 «Стоматологія».
42. Пiд час огляду ротової порожнини – 2014. – № 119.
пацiєнта лiкар-стоматолог визначив сухiсть
слизової оболонки, численнi ерозiї.
Недостатнiсть якого вiтамiну спричинила цi
явища?
A. Вiтамiн А
B. Вiтамiн К
C. Вiтамiн Р
D. Вiтамiн Н
E. Вiтамiн РР
Збірник тестових завдань для складання
ліцензійного іспиту Крок-1 «Стоматологія».
– 2006. – № 175.

43. З метою профiлактики запалення ясен та


покращення регенерацiї епiтелiальних клiтин
пародонту до зубних паст додають один з
наступних вiтамiнiв:
A. Ретинол
B. Кальциферол
C. Тiамiн
D. Бiотин
98
Ministry of Public Health of Ukraine

O. O. BOGOMOLETS NATIONAL MEDICAL UNIVERSITY

Department of Bioorganic and Biological Chemistry

LIST OF TEST QUESTIONS

for preparation of Content module № 2

"General regularities of metabolism of


carbohydrates, lipids, amino acids"
FOR STUDENTS OF THE 2ST YEAR OF STUDY

OF MEDICAL and STOMATOLOGICAL FACULTIES

Kyiv-2018
Enzymes and coenzymes. Regulation of metabolism
1. A denaturation of proteins can befound in B. Aspartate, glutamate, glycine
some substances. Specify thesubstance that is C. Aspartate, arginine, glutamate
used for the incompletedenaturation of D. Glutamate, valine, leucine
hemoglobin: E. Cysteine, glycine, proline
A. Urea
B. Toluene 7. Parodontitis is accompanied byactivation of
C. Sulfuric acid proteolysis in parodentiumtissues. Increase of
D. Nitric acid what oral fluid’scomponent is the evidence of
E. Sodium hydroxide proteolysisactivation?
A. Aminoacids
2. Some proteins of saliva have aprotective B. Organic acids
function. Which of themprotects the oral C. Glucose
mucosa from themechanical damage? D. Biogenic amines
A. Mucin E. Cholesterol
B. Lysozyme
C. Catalase 8. The student used conserved donor blood to
D. Peroxidase determine the time of its clotting. However, any
E. Renin positive result had not been obtained. The
reason for this is the lack in this sample of
3. For the study of serum proteinsvarious blood:
physical and physicochemicalmethods can be A. Ionized calcium
used. In particular, serumalbumins and B. Factor Hageman
globulins can be separatedby this method: C. Thromboplastin
A. Electrophoresis D. Fibrinogen
B. Polarography E. Vitamin K
C. Dialysis
D. Spectrography 9. Long-term effects on the body of toxic
E. Refractometry substances led to a significant reduction in
protein synthesis in hepatocytes. What
4. Electrophoretic study of blood serumof a organelles suffered from intoxication most?
patient with pneumonia revealed anincrease in A. Granular endoplasmic reticulum
one of the protein fractions.What fraction is it? B. Mitochondria
A. γ-globulins C. Microtubules
B. Albumins D. Lysosomes
C. α1-globulins E. The Golgi’s complex
D. α2-globulins
E. β-globulins 10. Children with kwashiorkor, along with other
signs, violations of the process of dentition were
5. A woman, 36 years after surgery, received revealed. At the heart of this phenomenon is the
intravenous injection of a concentrated albumin insufficient receipt into the body:
solution. This resulted in the increased A. Lipids
movement of water in the following direction: B. Proteins
A. From intercellular fluid to capillaries C. Carbohydrates
B. From the intercellular fluid to the cells D. Vitamin C
C. From cells to the intercellular fluid E. Vitamin B1
D. From capillaries to intercellular fluid
E. There will be no change in the flow of water 11. In the daily diet of an adult healthy person,
there must be fats, proteins, carbohydrates,
6. Cationic glycoproteins are the vitamins, mineral salts and water. Specify the
majorcomponents of parotid saliva. What daily amount of protein (g) that provides normal
aminoacids are responsible for their life activity of the body:
positivecharge? A. 100-120
A. Lysine, arginine, histidine B. 50-60
C. 10-20 E. Glucose
D. 70-80
E. 40-50 17. 60-year-old man went to the doctor after the
appearance of pain in the chest. In blood serum
12. A patient with a hepatic failure has been showed a significant increase in the activity of
examined for the electrophoretic spectrum of enzymes: CPK and its MB isoform, aspartate
blood serum proteins. Which of the physico- aminotransferase. On the development of the
chemical properties of the protein molecules lie pathological process in which tissue do these
in the basis of this method? changes indicate?
A. Presence of a charge A. Heart muscle
B. Hydrophilicity B. Lungs
C. Ability to swell C. Skeletal muscle
D. Optical activity D. Liver
E. Non-dialysable E. Smooth muscles

13. The surgeon used a 70% solution of ethyl 18. There are several groups of molecular
alcohol to disinfect the hands before surgery. mechanisms that are important in the
What is the main mechanism of antiseptic action pathogenesis of cell damage, which contributes
of the drug on microorganisms? to the development of pathology. What
A. Denaturation of protoplasmic proteins. processes provide protein damage mechanisms?
B. Interaction with amino groups of A. Inhibition of enzymes
protoplasmic proteins. B. Peroxide oxidation of lipids
C. Interaction with hydroxyl groups of enzymes. C. Acidosis
D. Blockade of sulfhydryl groups of enzyme D. Osmotic stretching of membranes
systems. E. Activation of phospholipases
E. Oxidation of organic components of
protoplasm. 19. It is known that the activity of parotid
salivary glands susceptible to age-related
14. What substance makes saliva viscousand changes. The activity of which enzyme will
mucous, has protective function, protects decrease sharply in saliva?
mucous membrane of oralcavity from A. Amylase.
mechanical damage? B. Hexokinase.
A. Mucin C. Renin.
B. Glucose D. Maltase.
C. Kallikrein E. Phosphatase.
D. Amylase
E. Lysozyme 20. For biochemical diagnostics of cardiac
infarction it is necessary to determine activity of
15. Only one factor can affect the charge of a number of enzymes and their izoenzymes in
amino acid radicals in the active center of the the blood. What enzyme assay is considered to
enzyme. Name of this factor is: be optimal for confirming or ruling out cardiac
A. pH infarction at the early stage, after the patient
B. Pressure develops thoracic pain?
C. Temperature A. Creatine kinase MB isoenzyme
D. The presence of a competitive inhibitor B. Creatine kinase MM isoenzyme
E. Excessproduct C. LDH1 isoenzyme
D. LDH5 isoenzyme
16. The conjugated protein necessarily contains E. Cytoplasmic isoenzyme of
special component as a nonprotein part. Choose aspartateaminotransferase
the substance that can’t carry out this function:
A. HNO3 21. For the treatment of some infectious
B. АТP diseases caused by bacteria, sulfonamides drugs
C. Thiamine pyrophosphate are used. What is the mechanism of action of
D. AMP these drugs?
A. They are antivitamins of p-aminobenzoic the most likely development of the pathological
acid. process?
B. They are alosteric enzyme inhibitors. A. Heart
C. They are alosteric enzyme activators. B. Pancreas
D. They are participate in redox processes. C. Liver
E. They are inhibit folic acid absorption. D. Kidneys
E. Skeletal muscle
22. In the intensive care unit entered a man 47-
years old with a diagnosis of myocardial 27. An increase in the activity of LDH 4,5,
infarction. Which of the lactate dehydrogenase AlAT, carbamoyl ornithine transferase was
(LDH) fractions will prevail in the blood serum detected in the patient's blood. In what organ
during the first two days? can assume development of the pathological
A. LDH 1 process?
B. LDH 2 A. Cardiac muscle (possible myocardial
C. LDH 3 infarction)
D. LDH 4 B. Liver (possible hepatitis)
E. LDH 5 C. Skeletal muscle
D. Kidneys
23. Researchers isolated 5 isoenzymicforms of E. Connective tissue
lactate dehydrogenase from thehuman blood
serum and studied theirproperties. What 28. Albumin concentration in the patient’s blood
property indicates thattheisoenzymic forms is 2.8 g/l, increased concentration of lactate
were isolated fromthe same enzyme? dehydrogenase 5 (LDH5). On the disease of
A. Catalyzation of the same reaction what organ does that indicate?
B. The same molecular weight A. Liver
C. The same physicochemical properties B. Kidney
D. Tissue localization C. Heart
E. The same electrophoretic mobility D. Lung
E. Spleen
24. One of the means of regulating enzyme
activity in a human body is the covalent 29. There is increased activity of AST, LDH1,
modification. Glycogen phosphorylase and LDH2, and CPK in the patient’s blood.
glycogen synthetase activity is regulated by the Pathological process most likely occurs in the:
following type of covalent modification: A. Heart
A. Phosphorylation-dephosphorylation B. Skeletal muscles
B. ADP-ribosylation C. Kidneys
C. Methylation D. Liver
D. Hydrolysis E. Adrenal glands
E. Sulfonation
30. In the patient's blood plasma the activity of
25. When the patient's blood analysis revealed a LDH1 and LDH2 isoenzymes increased. About
significant increase in the activity of MB-forms which pathology of the body does this mean?
CPK (CPK) and lactate dehydrogenase-1. What A. Myocardium
pathology can be assumed? B. Liver
A. Myocardial infarction C. Kidney
B. Hepatitis D. Brain
C. Rheumatism E. Skeletal muscle
D. Pancreatitis
E. Cholecystitis 31. In a patient revealed increased the activity of
AsAT, LDH1,2, and creatine phosphokinase. In
26. An increase in the activity of LDH1, LDH2, which organ (s) is the most likely development
AsAT, and creatine kinase was detected in the of the pathological process?
patient's blood. In which organ of the patient is A. Heart muscle
B. Skeletal muscle
C. Kidney and adrenal gland D. LDH 4
D. Connective tissue E. LDH 5
E. Liver and kidneys
34. The high level of lactate dehydrogenase
32. 49-year-old patient, a driver by profession, (LDH) isozymes concentration showed the
complains of unbearable compressive pain increase of LDH-1 and LDH-2 in a patient’s
behind the sternum, "giving away" to the neck. blood plasma. Point out the most probable
Pain occurred 2 hours ago. Objectively: the diagnosis:
condition is severe, pallor, heart sounds are A. Myocardial infarction
weakened. Laboratory studies have shown high B. Skeletal muscle dystrophy
activity of creatine kinase and LDH1. What C. Diabetes mellitus
diseases are characterized by such symptoms? D. Viral hepatitis
A. Acute myocardial infarction E. Acute pancreatitis
B. Acute pancreatitis
C. Stenocardia 35. Succinate dehydrogenase catalyses the
D. Cholelithiasis dehydrogenation of succinate. Malonic acid
E. Diabetes HOOC − CH2− COOH is used to interrupt the
action of this enzyme. Choose the inhibition
33. Six hours after myocardial infarction, the type:
lactate dehydrogenase activity in the patient A. Competitive
increased in the blood. What isoenzymes should B. Allosteric
be expected in this case? C. Non-competitive
A. LDH 1 D. Limited proteolysis
B. LDH 2 E. Dephosphorylation
C. LDH 3
Water-soluble vitamins
1. Examination of a patient revealed dermatitis, C. Retinol
diarrhea, dementia. What vitamin deficiency is D. Calciferol
the cause of this condition? E. Riboflavin
A. Nicotinamide
B. Ascorbic acid 6. A patient has painfulness along bignerve
C. Folic acid trunks and excessive content of pyruvate in
D. Biotin blood. What vitamin deficit may cause such
E. Rutin changes?
A. В1
2. A patient consulted a doctor about symmetric B. Biotin
dermatitis of open skin areas. It was found out C. РР
that the patient lived mostly on cereals and ate D. Pantothenic acid
too little meat, milk and eggs. What vitamin E. В2
deficiency is the most evident?
A. Nicotinamide 7. A woman who has been keeping to a clean-
B. Calciferol rice diet for a long time was diagnosed with
C. Folic acid polyneuritis (beri-beri). What vitamin deficit
D. Biotin results in development of this disease?
E. Tocopherol A. Thiamine
B. Ascorbic acid
3. A 3 year old child with symptoms of C. Pyridoxine
stomatitis, gingivitis and dermatitis of open skin D. Folic acid
areas was delivered to a hospital. Examination E. Riboflavin
revealed inherited disturbance of neutral amino
acid transporting in the bowels. These 8. A patient has an increased pyruvate
symptoms were caused by the deficiency of the concentration in blood. A large amount of it is
following vitamin: excreted with the urine. What vitamin is lacking
A. Niacin in this patient?
B. Pantothenic acid A. B1
C. Vitamin A B. E
D. Cobalamin C. B3
E. Biotin D. B6
E. B2
4. A patient is ill with dermatitis, diarrhea,
dementia. During history taking it was revealed 9. A patient, who has been subsisting
that the main food stuff of the patient was exclusively on polished rice, has developed
maize. These disturbances are caused by polyneuritis due to thiamine deficiency. What
deficiency of the following vitamin: substance is an indicator of such avitaminosis,
A. PP when it is excreted with urine?
B. B1 A. Pyruvic acid
C. B2 B. Malate
D. B9 C. Methylmalonic acid
E. B8 D. Uric acid
E. Phenyl pyruvate
5. A 50-year-old man addressed a hospital
withcomplaints of memory disorders, painful 10. Vitamin B1 deficiency causes disturbance of
sensations along the nerve trunks, decreased oxidative decarboxylation of α-ketoglutaric
mental ability, circulatory disorders and acid. This leads to the impaired synthesis of the
dyspepsia. Anamnesis states excessive alcohol following coenzyme:
consumption. What vitamin deficiency can A. Thiamine pyrophosphate
result in such signs? B. Nicotinamide adenine dinucleotide
A. Thiamine C. Flavine adenine dinucleotide
B. Niacin D. Lipoic acid
E. Coenzyme A 16. In case of enterobiasis acrihine – the
structural analogue of vitamin B2 – is
11. To improve the trophism of the heart administered. The synthesis disorder of which
muscle, the patient is prescribed a drug that enzymes does this medicine cause in
includes cocarboxylase (thiamine diphosphate) - microorganisms?
a coenzyme form of the vitamin: A. FAD-dependent dehydrogenases
A. B1 B. Cytochromeoxidases
B. B2 C. Peptidases
C. B5 D. NAD-depended dehydrogenases
D. B6 E. Aminotransferases
E. B12
17. The preparation comlex for periodontitis
12. Pyruvic acid as an intermediate metabolite treatment includes the medicine from the group
of carbohydrate, lipid and amino acid of water soluble vitamins, bioflavonide
metabolism can undergo oxidative derivative, which is prescribed together with
decarboxylation. The cause of this process is the ascorbic acid. This preparation has anti-
lack of the following nutrient in the diet: oxidative properties, decreases gingival
A. Thiamin hemorrhage. What preparation is meant?
B. Pyridoxine A. Rutin
C. Ascorbic acid B. Calcium pantothenate
D. Citrine C. Calcium panganate
E. Pangamic acid D. Cyanocobalamin
E. Folic acid
13. A patient with chronic alcoholism has
symptoms of polyneuritis and cardiac 18. Examination of a child who hasn’t got fresh
insufficiensy. What vitamin preparation should fruit and vegetables during winter revealed
be prescribed to this patient? numerous subcutaneous hemorrhages,
A. Thiamine gingivitis, carious cavities in teeth. What
B. Ergocalciferol vitamin combination should be prescribed in
C. Retinol this case?
D. Rutin A. Ascorbic acid and rutin
E. Phylloquinone B. Thiamine and pyridoxine
C. Folic acid and cobalamin
14. A 36-year-old female patient has a history of D. Riboflavin and nicotinamide
B2-hypovitaminosis. The most likely cause of E. Calciferol and ascorbic acid
specific symptoms (epithelial, mucosal,
cutaneous, corneal lesions) is the deficiency of: 19. Examination of a patient with frequent
A. Flavin coenzymes hemorrhages from internals and mucous
B. Cytochrome A1 membranes revealed proline and lysine being a
C. Cytochrome oxidase part of collagene fibers. What vitamin absence
D. Cytochrome B caused disturbance of their hydroxylation?
E. Cytochrome C A. Vitamin С
B. Vitamin K
15. Malaria is treated with structural analogs of C. Vitamin A
vitamin B2 (riboflavin). These drugs disrupt the D. Thiamine
synthesis of the following enzymes in E. Vitamin Е
plasmodium:
A. FAD-dependent dehydrogenase 20. In spring a patient experiences petechial
B. NAD-dependent dehydrogenase haemorrhages, loosening of teeth, high liability
C. Peptidase to colds. A doctor suspects hypovitaminosis C.
D. Cytochrome oxidase In this respect loosening of teeth can be
E. Aminotransferase explained by:
A. Structural failure of collagen in the
periodontal ligaments
B. Structural change of glycosaminoglycans 25. A patient diagnosed with focal tuberculosis
C. Increased permeability of periodont of the upper lobe of the right lung had been
membranes taking isoniazid as a part of combination
D. Mechanical damage of teeth therapy. After some time, the patient reported of
E. Disturbed oxidation-reduction process in the muscle weakness, decreased skin sensitivity,
periodont blurred vision, impaired motor coordination.
Which vitamin preparation should be used to
21. Most of the members of the expedition of address these phenomena?
Magellan to America died from vitamin A. Vitamin B6
deficiency. This disease was manifested by B. Vitamin A
general weakness, subcutaneous hemorrhage, C. Vitamin D
tooth loss, bleeding from the gums. As the name D. Vitamin B12
of this vitamin deficiency? E. Vitamin C
A. scurvy (scurvy)
B. Pellagra 26. A 40-year-old male patient with pulmonary
C. Rickets tuberculosis was administered isoniazid. What
D. Polyneuritis (beriberi) vitamin deficiency can develop as a result of
E. Biermer’s anemia taking this drug for a long time?
A. Pyridoxine.
22. A 10-year-old girl has a history of repeated B. Cobalamin.
acute respiratory viral infection. After C. Biotin.
recovering she presents with multiple petechial D. Thiamine.
hemorrhages on the sites of friction from E. Folic acid.
clothing rubbing the skin. What kind of
hypovitaminosis has this girl? 27. What vitamin is a component of glutamic
A. C acid decarboxylase, participates in the
B. B6 production of GABA, and its deficiency is
C. B1 manifested by seizures?
D. A A. Pyridoxine
E. B2 B. Cobalamin
C. Tocopherol
23. The patient has an increase in the D. Folic acid
permeability of the walls of blood vessels with E. Ascorbic acid
the development of increased bleeding and the
appearance of small point hemorrhages on the 28. A 9-month-old child feeds on artificial
skin, tooth loss. How does the vitamin exchange mixtures that are not balanced in terms of
disorder explain these symptoms? vitamin B6 content. A child has pellagra similar
A. Hypovitaminosis C dermatitis, convulsions, anemia. The
B. Hypervitaminosis D development of seizures may be associated with
C. Hypervitaminosis C impaired formation:
D. Hypovitaminosis D A. GABA
E. Hypovitaminosis A B. Histamine
C. Serotonin
24. A 20-year-old male patient complains of D. DOPA
general weakness, rapid fatigability, irritability, E. Dopamine
decreased performance, bleeding gums,
petechiae on the skin. What vitamin deficiency 29. The infant has epileptiform convulsions
may be a cause of these changes? caused by vitamin B6 deficiency. This is due to
A. Ascorbic acid a decrease in the nerve tissue of the inhibitory
B. Riboflavin mediator - γ-aminobutyric acid. What enzyme
C. Thiamine activity is reduced due to this?
D. Retinol A. Glutamate decarboxylase
E. Folic acid B. Alanine aminotransferase
C. Glutamate dehydrogenase
D. Pyridoxal kinase B. Carboxybiotin
E. Glutamate synthetase C. Thiamine diphosphate
D. Pyridoxal phosphate
30. In compliance with the clinical presentations E. Nicotinamide adenine dinucleotide
a man was prescribed pyridoxalphosphate. What
processes are corrected by this preparation? 35. Pterin derivatives (aminopterin and
A. Transamination and decarboxylation of methotrexate) are the inhibitors of dihydrofolate
amino acids reductase, so that they inhibit the regeneration
B. Oxidative decarboxilation of keto acids of tetrahydrofolic acid from dihydrofolate.
C. Desamination of purine nucleotides These drugs inhibit the intermolecular tranfer of
D. Synthesis of purine and pyrimidine bases monocarbon groups, thus suppressing the
E. Protein synthesis synthesis of the following polymer:
A. DNA
31. In the clinical practice for the treatment of B. Protein
tuberculosis drug isoniazid used - antivitamin C. Homopolysaccharides
which is able to penetrate into the tubercle D. Gangliosides
bacillus. The tuberculostatic effect is due to the E. Glycosaminoglycans
violation of replication processes, redox
reactions, due to the formation of not real 36. After an extended treatment with
coenzyme from: sulfanamides a patient has developed
A. NAD+ macrocyticanemia. Production of active forms
B. FAD of the following vitamin is disrupted in such a
C. FMN condition:
D. TDP A. Folic acid
E. KoQ B. Thiamine
C. Riboflavin
32. Reactions of intermolecular transport of D. Pyridoxine
one-carbon radicals are necessary for the E. Cyanocobalamin
synthesis of proteins and nucleic acids. From
which of the following vitamins does the 37. It is known that part of carbon dioxide is
coenzyme form necessary for the above used in the body in the biosynthesis of fatty
reactions? acids, urea, gluconeogenesis, etc. Which
A. Folic acid vitamin forms the CO2-transporting form for
B. Thiamine these reactions?
C. Pantothenic acid A. Biotin
D. Ascorbic acid B. thymine
E. Riboflavin C. Riboflavin
D. Nicotinamide
33. In the synthesis of purine nucleotides E. Retinol
involved some amino acids, derivatives of
vitamins, phosphorus esters of ribose. What 38. A patient was diagnosed with seborrheic
coenzyme form of vitamin provides one-carbon dermatitis associated with vitamin H (biotin)
fragments for this synthesis? deficiency. The patient has disturbed activity of
A. Folic acid the following enzyme:
B. Pantothenic acid A. Acetyl-CoA-carboxylase
C. Nicotinic acid B. Pyruvate decarboxylase
D. Riboflavin C. Alcohol dehydrogenase
E. Pyridoxine D. Amino transferase
E. Carbomoyl phosphate synthetase
34. In the normal course of a replication process
need thymidylic nucleotides, the synthesis of 39. The patient was diagnosed megaloblastic
which occurs with the participation of anemia. Specify a vitamin deficiency which can
thymidylate synthetase, are used as a coenzyme: lead to the development of this disease.
A. Methylenetetrahydrofolate A. Cyanocobalamin.
B. Rutin. 44. In the patient with complaints about pain in
C. Nicotinamide. the stomach found a decrease in its secretory
D. Thiamine. function, which is accompanied by anemia.
E. Cholecalciferol. What substance deficiency causes the
development of B12 hypovitaminosis in a patient
40. 47. A patient 43 years old with chronic and the appearance of anemia?
atrophic gastritis and hyperchromic A. Castle factor
megaloblastic anemia increased methylmalonic B. Thiamine
acid excretion in the urine. The lack of which C. Biotin
vitamin caused the occurrence of this symptom D. Pyridoxine
complex? E. Calciferol
A. В12
B. В2 45. After the surgical removal of part of the
C. В3 stomach of the patient disrupted the absorption
D. В5 of vitamin B12, it is excreted in the feces.
E. В6 Anemia has developed. What factor is necessary
for the absorption of this vitamin?
41. After removing 2/3 of the stomach in the A. Gastromukoprotein
patient’s blood, the hemoglobin content B. Gastrin
decreased, the number of red blood cells C. Hydrochloric acid
increased, the size of these blood cells D. Pepsin
increased. What vitamin deficiency leads to E. Folic acid
such changes in the blood?
A. B12 46. In a 65-year-old patient with long-lasting
B. C complaints characteristic of chronic gastritis,
C. P megalocytes were found in peripheral blood and
D. B6 megaloblastic erythropoiesis in the bone
E. PP marrow. What is the most likely diagnosis?
A. B12-folic deficiency anemia
42. In examining the oral cavity of the patient, B. Aplastic anemia
the dentist paid attention to the presence of an C. Hypoplastic anemia
inflammatory and dystrophic process in the D. Hemolytic anemia
mucous membrane (Hunter’s glossitis, atrophic E. Iron deficiency anemia
stomatitis). A blood test revealed hyperchromic
anemia. What factor is the cause of this disease? 47. A 13-year-old girl has been prescribed a
A. Hypovitaminosis B12 certain drug for treatment of megaloblastic
B. Hypovitaminosis B1 anemia. This drugstimulates a transfer from
C. Hypovitaminosis B6 megaloblastic haemopoiesis to normoblastic,
D. Increasing pH of the gastric juice participates in synthesis of purine and
E. Hypovitaminosis A pyrimidine bases, activates proteine and
methionine synthesis.What drug does the patient
43. A year after subtotal stomach resection on take?
account of ulcer of lesser curvature the A. Cyanocobalamin
following blood changes were revealed: anemia, B. Erythropoietin
leukocytopenia and thrombocytopenia, color C. Haemostimulinum
index - 1,3, megaloblasts and megalocytes. D. Ferricsulfate
What factor deficiency caused the development E. Rosehiptea
of thos pathology?
A. Castle’s factor 48. A 50-year-old patient has been examined by
B. Hydrochloride acid a dentist and found to have crimson smooth
C. Mucin tongue. Blood analysis revealed a decrease in
D. Pepsin RBC level and hemoglobin concentration,
E. Gastrin colour index of 1,3, symptoms of megaloblastic
hematopoiesis, degenerative changes in
WBCs.What blood disorder was found in this E. Ubiquinone
patient?
A. B12-folic-acid-deficiency anemia 50. A doctor recommends a patient with
B. Iron deficiency anemia duodenal ulcer to drink cabbage and potato juice
C. Myeloid leukemia after the therapy course. Which substances
D. Aplastic anemia contained in these vegetables help to heal and
E. Hemolytic anemia prevent the ulcers?
A. Vitamin U
49. Coenzym A participates in numerous B. Pantothenic acid
important metabolic reactions. It is aderivative C. Vitamin C
of the following vitamin: D. Vitamin B1
A. Pantothenic acid E. Vitamin K
B. Thiamine
C. Niacin
D. Calciferol
Fat -soluble vitamins
1. A 2 year old child suffers fromintestinal blood - decrease in activity II, VII, X blood
dysbacteriosis that lead to thedevelopment clotting factors; lengthening of blood clotting
hemorrhagic syndrome. Themost probable cause time. What vitamin deficiency are caused by
of hemorrhage is: these changes?
A. Vitamin K deficiency A. Vitamin K
B. Activation of tissue thromboplastin B. Vitamin A
C. Hypovitaminosis PP C. Vitamin C
D. Fibrinogen deficiency D. Vitamin D
E. Hypocalcemia E. Vitamin E

2. Surgery in patients with obstructive jaundice 7. Plasma coagulation factors are subject to
and malabsorption in the intestine complicated posttranslational modification with the
by bleeding. What vitamin insufficiency has led participation of vitamin K. As a cofactor, it is
to this? needed in the γ-carboxylation enzyme system of
A. Vitamin K protein coagulation factors due to an increase in
B. Vitamin B12 the affinity of their molecules by calcium ions.
C. Vitamin C What amino acid is carboxylated in these
D. Vitamin B6 proteins?
E. Folic acid A. Glutamine
B. Valin
3. Hepatic disfunctions accompaniedby C. Serine
insufficient in flow of bile to the bowels result D. Phenylalanine
in coagulation failure. This phenomenon can be E. Arginine
explained by:
A. Vitamin K deficiency 8. A 49-year-old man is observed in the clinic
B. Leucopenia with a significant increase in blood clotting
C. Thrombocytopenia time, gastrointestinal bleeding, subcutaneous
D. Erythropenia hemorrhage. What vitamin deficiency can
E. Iron deficiency explain these symptoms?
A. K
4. The function of protein synthesis in the liver B. B1
due to a lack of vitamin K is reduced in a young C. PP
man of 16 years after suffering the disease. This D. H
can lead to a violation of: E. E
A. Blood coagulation
B. Erythrocyte sedimentation rate 9. As a result of post-translational modifications
C. Education anticoagulants of certain proteins involved in blood
D. Formation of erythropoietin coagulation, in particular prothrombin, they
E. Blood pH acquire the ability to bind calcium. In this
process participate vitamin
5. In the patient observed hemorrhage, reduced A. K
the concentration of prothrombin in the blood. B. C
What vitamin deficiency led to a violation of the C. A
synthesis of this blood clotting factor? D. B1
A. K E. B2
B. A
C. D 10. To prevent postoperative bleeding, a 6-year-
D. C old child is recommended to take vikasol, which
E. E is a synthetic analogue of vitamin K. Specify
which post-translational changes of blood
6. In the patient 37 years against the background clotting factors are activated under the influence
of long-term use of antibiotics observed of vikasol?
increasing bleeding after small injuries. In the A. Carboxylation of glutamic acid
B. Phosphorylation of serine radicals 15. In the patient with renal insufficiency has
C. Partial proteolysis developed osteodystrophy, accompanied by
D. Polymerization intense bone demineralization. Violation of the
E. Glycosylation formation of which vitamin active form was the
cause of this complication?
A. Calciferol
11. In patients with the biliary tract obstruction B. Retinol
the blood coagulation isinhibited; the patients C. Thiamine
have frequent haemorrhages caused by the D. Naphthoquinone
subnormal assimilation of the following E. Riboflavin
vitamin:
A. К 16. The child has a delayed teething, their
B. А wrong arrangement, upon examination
C. D noticeable dryness of the oral cavity, in the
D. E corners of the mouth there are cracks with
E. C suppuration. With the lack of the vitamin this
condition may be related?
12. A few days before an operation apatient A. Vitamin D.
should be administered vitamin K or its B. Vitamin C.
synthetic analogue Vicasol. Vitamin K takes C. Vitamin E.
part in the following posttranslational D. Vitamin K.
modification of the II, VII, IX, X blood clotting E. Vitamin A.
factors:
A. Carboxylation 17. In a child of the first year of life is observed
B. Decarboxylation an increase in the size of the head and abdomen,
C. Deamination late teething, violation of the enamel structure.
D. Transamination The consequence of vitamin deficiencies are
E. Glycosylation these changes?
A. Hypovitaminosis D
13. A patient, who has been suffering for a long B. Hypovitaminosis C
time from intestine disbacteriosis, has increased C. Hypovitaminosis A
hemorrhaging caused by disruptionof D. Hypovitaminosis B1
posttranslational modification of blood- E. Hypovitaminosis B2
coagulation factors II, VII, IХ, and Х in the
liver. What vitamin deficiency is thecause of 18. A 6-year-old child suffers from delayed
this condition? growth, disrupted ossification processes,
A. К decalcification of the teeth.What can be the
B. B12 cause?
C. B9 A. Vitamin D deficiency
D. С B. Decreased glucagon production
E. Р C. Insulin deficiency
D. Hyperthyroidism
14. 10 month old child has high excitability, E. Vitamin C deficiency
sleep disturbance, amyotonia, retarded dentition,
teeth erupt withinadequate enamel calcification. 19. A 35-year-old female patient with a chronic
Thesechanges are caused by deficiency of renal disease has developed osteoporosis. The
thefollowing vitamin: cause of this complication is the deficiency of
A. Cholecalciferol the following substance:
B. Riboflavin A. 1,25-dihydroxy-D3
C. Thiamine B. 25-hydroxy-D3
D. Retinol C. D3
E. Nicotinamide D. D2
E. Cholesterol
20. A child with renal insufficiency exhibits C. K
delayed teeth eruption. This is mostlikely D. B1
caused by the abnormal formationof the E. PP
following substance:
A. 1,25 (OH)2D3 26. A 5 years old child has insufficient
B. Glycocyamine calcification of enamel, tooth decay. Which
C. Glutamate vitamin hypovitaminosis leads to the
D. α-ketoglutarate development of this process?
E. Hydroxylysine A. Calciferol
B. Tocopherol
21. When examining a child, the doctor revealed C. Biotin
signs of rickets. Which compounds D. Nicotinic acid
insufficiency in the child’s body facilitates the E. Folic acid
development of this disease?
A. 1,25 [OH] -dihydroxycholecalciferol 27. On examination of 11 months’ child, the
B. Biotin pediatrician found a curvature of the bones of
C. Tocopherol the lower extremities and a delay in the
D. Naphthoquinone mineralization of the bones of the skull. What
E. Retinol vitamin deficiency leads to this pathology?
A. Cholecalciferol
22. During regular check-up a child is detected B. Thiamine
with interrupted mineralization of the bones. C. Pantothenic acid
What vitamin deficiency can bethe cause? D. Bioflavonoids
A. Calciferol E. Riboflavin
B. Riboflavin
C. Tocopherol 28. A child with signs of rickets has been
D. Folic acid prescribed a certain liposoluble vitamin drug by
E. Cobalamin pediatrician and dentist. This drug affects the
metabolism of phosphorus and calcium in the
23. In patients after gallbladder removal body and facilitates calcium accumulation in
processes are hampered Ca2+ absorption through bone tissue and dentine. If its content in the
the intestinal wall. What vitamin prescription body is insufficient, there developdisruptions of
will stimulate this process? ossification process, dental
A. D3 structureandocclusion. Name this drug:
B. RR A. Ergocalciferol
C. C B. Retinolacetate
D. B12 C. Tocopherolacetate
E. K D. Menadione (Vicasolum)
E. Thyroidin
24. A patient who suffers from chronic renal
insufficiency fell ill with osteoporosis. 29. Hormonal form of a certain vitamin induces
Disturbed synthesis of what mineral genome level synthesis of Ca binding proteins
metabolism’s regulator is the cause of and enterocytes thus regulating the intestinal
osteoporosis? absorption of Ca2+ ions required for dental
A. Formation of 1, 25(OH)2D3 tissuedevelopment. What vitamin is it?
B. Proline hydroxylation A. D3
C. Lysine hydroxylation B. A
D. Glutamate carboxylation C. B1
E. Cortisol hydroxylation D. E
E. K
25. A patient has enamel erosion. What vitamin
should be administered for itstreatment?
A. D3
B. C
30. A 4-year-old child with hereditary renal 35. In order to accelerate healing of a radiation
lesion has signs of rickets; vitamin D ulcer a vitamin drug was administered. What
concentration in blood is normal. What is the drug is it?
most probable cause of rickets development? A. Retinol acetate
A. Impaired synthesis of calcitriol B. Retabolil
B. Increased excretion of calcium C. Prednisolone
C. Hyperfunction of parathyroid glands D. Levamisole
D. Hypofunction of parathyroid glands E. Methyluracil
E. Lack of calcium in food
36. While the examination of patient’s oral
31. Vitamin A together with specific cavity the dentist found xerostomia, numerous
cytoreceptors penetrates through thenuclear erosions. What vitamin deficit caused this
membranes, induces transcriptionprocesses that effect?
stimulate growth anddifferentiation of cells. A. Vitamin A
This biologicalfunction is realized by the B. Vitamin K
following formof vitamin A: C. Vitamin P
A. Trans-retinoic acid D. Vitamin H
B. Carotin E. Vitamin PP
C. Cis-retinal
D. Retinol 37. In order to prevent gum inflammation and to
E. Trans-retinal improve regeneration of epithelial periodontium
cells manufacturers add to the tooth pastes one
32. A patient suffers from vision impairment- of the following vitamins:
hemeralopy (night blindness). What vitamin A. Retinol
preparation should beadministered the patient in B. Calciferol
order torestore his vision? C. Thiamine
A. Retinol acetate D. Biotin
B. Vicasol E. Phyloquinone
C. Pyridoxine
D. Thiamine chloride 38. A 64 year old woman has impairment of
E. Tocopherol acetate twilight vision (hemeralopy). What vitamin
should be recommended in the first place?
33. An oculist detected increased time of A. A
darkness adaptation of a patient’s eye. What B. B2
vitamin deficiency can cause such symptom? C. E
A. А D. C
B. Е E. B6
C. С
D. К 39. The patient has worsened twilight vision.
E. D Which of the vitamin preparations should be
prescribed to the patient?
34. A patient complains of photoreception A. Retinol acetate
disorder and frequent acute viral diseases. He B. Cyanocobalamin
has been prescribed a vitamin that affects C. Pyridoxine hydrochloride
photoreception processes by producing D. Ascorbic acid
rhodopsin, the photosensitive pigment. What E. Nicotinic acid
vitamin is it?
A. Retinol acetate 40. The patient with periodontal disease was
B. Tocopherol acetate prescribed a fat-soluble vitamin preparation,
C. Pyridoxine hydrochloride which is actively involved in the redox
D. Cyanocobalamin processes in the body. Antioxidant is a growth
E. Thiamine factor, antixerphthalmic, provides normal
vision. In dental practice it is used to accelerate
epithelization in case of mucosal diseases with A. Tocopherol
periodontitis. Identify this drug: B. Thiamine
A. Retinol acetate C. Gluconate
B. Ergocalciferol D. Pyridoxine
C. Tocopherol acetate E. Choline
D. Vikasol
E. Cyanocobalamin 46. When treating sialadenitis (inflammation of
the salivary glands), preparations of vitamins
41. A patient has the folowing changes: disorder are used. Which of the following vitamins plays
of twilight vision, drying out of conjunctiva and an important role in antioxidant defense?
cornea. Such disordersmay be caused by A. Tocopherol
deficiency of vitamin: B. Pantothenic acid
A. Vitamin A C. Riboflavin
B. Vitamin B D. Thiamine
C. Vitamin C E. Pyridoxine
D. Vitamin D
E. Vitamin B12 47. A woman, who had undergone mastectomy
due to breast cancer, wasprescribed a course of
42. A patient complains of photoreception radiation therapy.What vitamin preparation has
disorder and frequent acute viral diseases. He markedantiradiation effect due to its
has been prescribed a vitamin that affects antioxidantactivity?
photoreception processes by producing A. Tocopherol acetate
rhodopsin, the photosensitive pigment. B. Ergocalciferol
What vitamin is it? C. Riboflavin
A. Retinol acetate D. Cyanocobalamin
B. Tocopherol acetate E. Folic acid
C. Pyridoxine hydrochloride
D. Cyanocobalamin 48. What vitamin deficiency leads to both
E. Thiamine disorder of reproductive function and distrophy
of skeletal muscles?
43. There are various diseases that cause sharp A. Vitamin E
increase of active oxygen, leading tocell B. Vitamin A
membranes destruction. Antioxidantsare used to C. Vitamin K
prevent it from happening. Themost potent D. Vitamin D
natural antioxidant is: E. Vitamin B1
A. Alpha-tocopherol
B. Glycerol 49. Ionizing radiation or vitamin E
C. Vitamin D deficiencyaffect the cell by increasing
D. Fatty acids lysosomemembrane permeability.What are the
E. Glucose possibleconsequences of this pathology?
A. Partial or complete cell destruction
44. A pregnant woman with several B. Intensive protein synthesis
miscarriagesin anamnesis is prescribed atherapy C. Intensive energy production
that includes vitamin preparations.What vitamin D. Restoration of cytoplasmic membrane
facilitates carrying of apregnancy? E. Formation of maturation spindle
A. Alpha-tocopherol
B. Folic acid 50. Examination of a man who hadn’t been
C. Cyanocobalamin consuming fats but had been gettingenough
D. Pyridoxal phosphate carbohydrates and proteinsfor a long time
E. Rutin revealed dermatitis, poor wound healing, vision
impairment. Whatis the probable cause of
45. Parodontosis is treated by meansof metabolic disorder?
antioxidants. Which of the followingnatural A. Lack of linoleic acid, vitamins A, D, E, K
compounds is used as an antioxidant: B. Lack of palmitic acid
C. Lack of vitamins PP, H
D. Low caloric value of diet
E. Lack of oleic acid

51. For the prevention of atherosclerosis,


coronary heart disease, cerebrovascular accident,
the consumption of high-fat polyunsaturated fatty
acids is recommended. One of the following fatty
acids is:
A. Linolic
B. Oleic
C. Lauric
D. Palmitooleic
E. Stearic
Basic laws of metabolism and energy
1. The Krebs cycle plays an important role in E. They inhibit cytochrome B
the realization of the glucoplastic effect of
amino acids. This is due to the mandatory 6. A patient with poisoning with an insecticide-
transformation of a nitrogen-free residue into: rotenone was taken to the hospital. Which part
A. Oxaloacetate. of the mitochondrial electron transfer chain is
B. Malate. blocked by this substance?
C. Succinate. A. NADH coenzyme Q-reductase.
D. Fumarat. B. ATP synthetases.
E. Citrate. C. Coenzyme Q-cytochrome C-reductase.
D. Succinate-coenzyme Q-reductase.
2. At the patient of 57 years, suffering from a E. Cytochrome C-oxidase.
diabetes, the keto-acidosis has developed. The
biochemical basis of this condition is a decrease 7. Rotenone is known to inhibit respiratory
in the degree of utilization of acetyl-CoA. The chain. What complex of mitochondrial
disadvantage of which is the connection in cells respiratory chain is inhibited by this substance?
that is due? A. NADH-coenzymeQreductase
A. Oxaloacetate B. Cytochromeoxidase
B. 2-oxoglutarate C. CoenzymeQ-cytochromecreductase
C. Glutamate D. Succinate-coenzyme Qreductase
D. Aspartate E. Adenosine triphosphate synthetase
E. Succinate
8. In the intensive care unit in a serious
3. During the oxidation of carbohydrates, lipids, condition, unconscious, the patient was
a large amount of energy is formed, the bulk of admitted. Diagnosed overdose of barbiturates,
which is formed due to the oxidation of acetyl- which caused the phenomenon of tissue
CoA. How many ATP molecules are formed hypoxia. At what level was blocking the
when a single molecule of acetyl-CoA is electron transport?
completely oxidized? A. NADH-Coenzyme-Q-reductase
A. 12. B. Cytochrome oxidase
B. 8. C. Cytochrome b - cytochrome c1
C. 38. D. Ubiquinone
D. 24. E. ATP synthase
E. 36
9. Hyperthermia, bulimia, weight loss, which is
4. Prussic acid and cyanide are among the observed in patients with thyrotoxicosis,
strongest poisons. Depending on the dose, death associated with a violation:
occurs in a few seconds or minutes. The A. Conjugation of oxidation and
inhibition of the activity of which enzyme is the phosphorylation.
cause of death? B. Reactions of fat synthesis.
A. Cytochrome oxidase. C. Decay of ATP.
B. Acetyl holinesesterase. D. Reactions of the citric acid cycle
C. Catalase. E. Reactions of beta-oxidation of fatty acids.
D. Methemoglobin reductase.
E. ATP synthetases. 10. With thyrotoxicosis, the production of
thyroid hormones T3 and T4 increases, weight
5. Cyanide poisoning causes immediate death. loss develops, tachycardia, mental excitability
What is the mechanism of cyanide effect at the and so on. The mechanism of their action?
molecular level? A. separates oxidation and oxidative
A. They inhibit cytochromoxidase phosphorylation.
B. They bind substrates of tricarboxylic acid B. Activate substrate phosphorylation.
cycle C. Block the substrate by phosphorylation.
C. They block succinate dehydrogenase D. Block the respiratory chain.
D. They inactivate oxygene E. Oxidative phosphorylation is activated.
11. A woman who is receiving treatment for cytochrome C. What is its function in a normal
hyperthyroidism, there is an increase in body cell?
temperature. What is the basis of this A. Enzyme of respiratory chain of
phenomenon? electrontransport
A. Separation of oxidative phosphorylation B. Enzyme of tricarboxylic acid cycle
B. Decreased glucose utilization by tissues C. Enzyme of beta-oxidation of fatty acids
C. Reduced fat oxidation in the liver D. Component of H+ATP system
D. Violation of amino acid deamination E. Component of pyruvate-
E. Violation of glycogen synthesis dehydrogenasesystem

12. A 38-year-old woman complains of 16. Cyanide is a poison that causes instant death
increased sweating, heartbeat, and an increase in of the organism. What enzymes found in
temperature in the evening. The main exchange mitochondria are affected by cyanide?
is increased by 60%. The doctor diagnosed A. Cytochrome oxidase (aa3)
thyrotoxicosis. What properties of thyroxin lead B. Flavin enzymes
to increased heat production? C. Cytochrome 5
A. Separation of oxidative phosphorylation D. NAD+-dependent dehydrogenase
B. Increases conjugation of oxidation and E. Cytochrome P-450
phosphorylation
C. Reduces β-oxidation of fatty acids 17. Hydrocyanic acid and cyanides are the most
D. Reduces the deamination of amino acids violent poisons. According to the dose the death
E. Contributes to the accumulation of acetyl follows after a few seconds or minutes. The
CoA death is caused by the inhibited activity of the
following enzyme:
13. Cells were treated with a substance that A. Cytochrome oxidase
blocks the phosphorylation of nucleotides in the B. Acetylcholinesterase
mitochondria. The process of cell activity will C. ATP-synthetase
be violated in the first place? D. Catalase
A. Oxidative phosphorylation. E. Methemoglobin reductase
B. Glycolysis.
C. Integration of functional protein molecules. 18. Potassium cyanide that is a poison came into
D. Aerobic oxidation of glucose a patient’s organism and caused death a few
E. Synthesis of mitochondria proteins. minutes after it. The most probable cause of its
toxic effect was abnormal activity of:
14. In the presence of 2,4-dinitrophenol, the A. Cytochrome oxidase
oxidation of substrates can continue, but the B. Catalase
synthesis of ATP molecules is impossible. What C. ATP-synthetase
is the mechanism of its action? D. NADP − H-dehydrogenase
A. Separation of oxidation and phosphorylation E. Haemoglobin synthesis
in mitochondria
B. Activation of the enzyme ATPase 19. The resuscitation unit has admitted a patient
C. Transfer of substrates beyond mitochondria in grave condition. It is known that he had
D. Stimulation of hydrolysis of formed ATP mistakenly taken sodium fluoride which blocks
E. Inhibition of the enzyme cytochrome oxidase cytochrome oxidase. What type of hypoxia
developed in the patient?
15. Researches of the latest decades established A. Tissue
that immediate "executors"ofcell apoptosis are B. Hemic
special enzymes calledcaspases. Generation of C. Cardiovascular
one of themproceeds with participation of D. Hypoxic
E. Respiratory
Carbohydrate metabolism and its regulation
1. In a patient of 60 years the activity of the 6. Analysis of a patient’s saliva revealedhigh
main digestive enzyme of saliva is reduced. In concentration of lactate. This is mostprobably
this case, the primary hydrolysis of which caused by activation of thefollowing process:
organic compounds is disturbed? A. Anaerobic glucose breakdown
A. Carbohydrates. B. Aerobic glucose breakdown
B. Fats. C. Glycogen breakdown
C. Proteins. D. Carbohydrate hydrolysis
D. Cellulose. E. Glucose-lactate cycle
E. Lactose.
7. Clinical examination enabled to make a
2. With age, the activity of the parotid glands provisional diagnosis: stomach cancer. Gastric
decreases.The activity of what enzyme of juice contained lactic acid.What type of glucose
carbohydrate metabolism will decrease? catabolism turns upin the cancerous cells?
A. Amilase. A. Anaerobic glycolysis
B. Lysozyme. B. Pentose-phosphate cycle
C. Phosphatase. C. Gluconeogenesis
D. Hexokinase. D. Aerobic glycolysis
E. Maltase. E. Glucose-alanine cycle

3. A newborn develops dyspepsia after the milk 8. What process provides erythrocytes with the
feeding. When the milk is substituted by the required amount of energy in the form of ATP
glucose solution the dyspepsia symptoms for their vital activity?
disappear. The newborn has the subnormal A. Glycolysis.
activity of the following enzyme: B. Aerobic oxidation of glucose.
A. Lactase C. β-Oxidation of fatty acids.
B. Invertase D. Pentose phosphate cycle.
C. Maltase E. Tricarbonic acid cycle.
D. Amylase
E. Isomaltase 9. Human red blood cells do notcontain
mitochondria. What is themain pathway for
4. While determining power inputs of apatient’s ATP production inthese cells?
organism it was established thatthe respiratory A. Anaerobic glycolysis
coefficient equaled 1,0. This means that in the B. Aerobic glycolysis
cells of the patientthe following substances are C. Oxidative phosphorylation
mainlyoxidized: D. Creatine kinase reaction
A. Carbohydrates E. Cyclase reaction
B. Proteins
C. Fats 10. During consumption of biscuits, sweets in
D. Proteins and carbohydrates the mixed saliva temporarily increases the level
E. Carbohydrates and fats of lactate. The activation of which biochemical
process leads to this?
5. Аfter the transition to a mixed diet in a A. Anaerobic glycolysis
newborn child has arisen dyspepsia with B. Tissue breathing
diarrhea, meteorism developmental delay.The C. Aerobic glycolysis
biochemical basis of this pathology is D. Gluconeogenesis
insufficiency of: E. Microsomal oxidation
A. Saccharaseandisomaltase.
B. Lactaseandcellobiase. 11. During long-distance running, skeletal
C. Trypsinandchymotrypsin. muscle of a trained person uses glucose to
D. Lipaseandcreatinekinase. obtain the energy of ATP for muscle
E. Cellulase. contraction. Indicate the main process of
utilization of glucose in these conditions:
A. Aerobic glycolysis
B. Anaerobic glycolysis D. Enolase
C. Glycogenolysis E. Lactate dehydrogenase
D. Gluconeogenesis
E. Glycogenesis 17. Cytoplasm of the myocytes contains a lot of
dissolved metabolites resulting fromglucose
12. After restoration of blood circulationin oxidation. Name the metabolite that turns
damaged tissue accumulation oflactate comes to directly into lactate:
a stop and speed ofglucose consumption slows A. Pyruvate
down. Thesemetabolic changes are caused by B. Oxaloacetate
activationof the following process: C. Glycerophosphate
A. Aerobic glycolysis D. Glucose-6-phosphate
B. Anaerobic glycolysis E. Fructose-6-phosphate
C. Lipolysis
D. Gluconeogenesis 18. Treatment of many diseasesinvolves use of
E. Glycogen biosynthesis cocarboxylase (thiaminepyrophosphate) for
supplying cells withenergy. What metabolic
13. When blood circulation in the damaged process is activatedin this case?
tissue is restored, then lactateaccumulation A. Oxidizing decarboxylation of pyruvate
comes to a stop andglucose consumption B. Glutamate deamination
decelerates. These metabolic changes are caused C. Amino acids decarboxylation
by activation of the following process: D. Decarboxylation of biogenic amines
A. Aerobic glycolysis E. Detoxication of harmful substances inliver
B. Anaerobic glycolysis
C. Lipolysis 19. A worker of a chemical enterprise was taken
D. Gluconeogenesis to the hospital with signs of poisoning. In the
E. Glycogen biosynthesis hair of this woman found increased
concentration of arsenate, which blocks lipoic
14. A 32-year-old female patientsuffers from acid. Violation of which process is the most
gingivitis accompaniedby gum hypoxia. What likely cause of poisoning?
metabolite ofcarbohydrate metabolism is A. Oxidation decarboxylation of pyruvate.
produced inthe periodontium tissues more B. Microsomal oxidation.
actively inthis case? C. Restoration of methemoglobin.
A. Lactate D. Restoration of organic peroxides.
B. Ribose 5-phosphate E. Removal of superoxide ions.
C. Glycogen
D. Glucose 6-phosphate 20. Oxidative decarboxylation of pyruvic acid is
E. NADPH-H catalyzed by a multienzyme complex with
several functionally linked coenzymes. Name
15. Untrained people often have musclepain this complex:
after sprints as a result of lactateaccumulation. A. Thymidine diphosphate (TDP), flavin
This might be caused byintensification of the adenine dinucleotide (FAD), coenzyme A
following biochemicalprocess: (CoASH), nicotine amide adenine dinucleotide
A. Glycolysis (NAD), lipoic acid
B. Gluconeogenesis B. Flavin adenine dinucleotide (FAD),
C. Pentose phosphate pathway tetrahydrofolic acid, pyridoxal-5-phosphate,
D. Lipogenesis thymidine diphosphate (TDP), choline
E. Glycogenesis C. Nicotine amide adenine dinucleotide (NAD),
pyridoxal-5-phosphate, thymidine diphosphate
16. Anaerobic splitting of glucose to lactic acid (TDP), methylcobalamin, biotin
is regulated by the relevant enzymes. What D. Coenzyme A (CoASH), flavin adenine
enzyme is the main regulator of this process? dinucleotide (FAD), pyridoxal-5- phosphate,
A. Phosphofructokinase tetrahydrofolic acid,carnitine
B. Glucose-6-phosphate isomerase E. Lipoic acid, tetrahydrofolic acid, pyridoxal-
C. Aldolase 5-phosphate, methylcobalamin
21. It has been found out that one of pesticide consumed a small amount of alcohol, resulting
components is sodium arsenate that blocks in severe poisoning. Explain the cause of
lipoic acid. Which enzyme activityis impaired poisoning:
by this pesticide? A. Accumulation of acetaldehyde
A. Pyruvate dehydrogenase complex B. Allergic reaction
B. Microsomal oxidation C. Neuralgia disorders
C. Methemoglobin reductase D. Cardiovascular insufficiency
D. Glutathione peroxidase E. Violation of the function of the kidneys
E. Glutathione reductase
27. The biosynthesis of the purine ring occurs
22. In erythrocytes of the patient with hemolytic on ribose-5-phosphate by gradual build-up of
anemia, the activity of pyruvate kinase was nitrogen and carbon atoms and the closure of
significantly reduced. What metabolic process is the rings. The source of ribose-5-phosphate is
violated under these conditions? the following process:
A. Glycolysis A. Pentose phosphate cycle
B. Glycogenolysis B. Glycolysis
C. Gluconeogenesis C. Glyconeogenesis
D. Pentosophosphate pathway of glucose D. Gluconeogenesis
oxidation E. Glycogenolysis
E. Synthesis of glycogen
28. Due to the long-term use of sulfanilamide
23. Decreased ratio of adenylic nucleotides drugs in a young woman appeared signs of
ATP/ADP results in intensifiedglycolysis in hemolytic anemia due to hereditary violation of
parodentium tissues underhypoxia conditions. the synthesis of the enzyme pentosophosphate
What reaction is activatedin this case? pathway glucose-6-phosphate dehydrogenase,
A. Phosphofructokinase which provides formation in the body:
B. Aldolase A. NADРН.
C. Triosophosphate isomerase B. ATP.
D. Enolase C. NAD
E. Lactate dehydrogenase D. FAD
E. FMN.
24. In some anaerobic bacteria the pyruvate
produced by glycolysis is converted to the ethyl 29. 22 year old woman has been taking
alcohol (alcoholic fermentation). What is the sulfanilamides for a long time that led to
biological significance of this process? symptoms of hemolytic anaemia caused by
A. NAD+ replenishment hereditary disturbance of synthesis of glucose 6-
B. Lactate production phosphate dehydrogenase. This enzyme of
C. ADP production pentose-phosphate cycle is responsible for
D. Providing the cells with NADPH generation of:
E. ATP production A. NADP − H2
B. NAD
25. In medical practice for preventionof C. FAD
alcoholism, a drug that is an aldehyde D. FMN
dehydrogenase inhibitor is widely used. The E. ATP
increase of which metabolite in the blood causes
disgust to alcohol? 30. Sulfanilamides are applied as antimicrobal
A. Acetaldehyde agents in clinical practice. Sulfanilamide
B. Ethanol treatment, however, can result in hemolytic
C. Malone aldehyde anemia development in patients that suffer from
D. Propionic aldehyde genetic defect of the following enzyme of
E. Methanol pentose phosphatemetabolismin erythrocytes:
A. Glucose-6-phosphate dehydrogenase
26. The patient during the course of treatment B. Hexokinase
with a drug that blocks alcohol dehydrogenase, C. Transketolase
D. Transaldolase deficiency in the galactose-1-
E. Pyruvatekinase phosphate uridylyltransferase. What is the
pathological process in the child?
31. 38 year old patient takes aspirin and A. Galactosaemia
sulfanilamides. After their intake intensified B. Fructosemia
erythrocyte haemolysis is observed which is C. Hyperglycemia
caused by deficiency of glucose 6-phosphate D. Hypoglycemia
dehydrogenase. This pathology is caused by E. Hyperlactate acidemia
failure of the following coenzyme:
A. NADP − H 36. In the 2-year-old boy, an increase in the size
B. FAD − H2 of the liver and spleen, cataract is observed. The
C. Pyridoxal phosphate concentration of sugar is elevated in the blood,
D. FMN − H2 but the test of glucose tolerance is normal. An
E. Ubiquinone hereditary violation of the metabolism of which
substance is the cause of this condition?
32. The patient has a chronic inflammatory A. Galactose
process of the tonsils. Due to the what B. Fructose
biochemical process in the inflammation C. Glucose
centers, the concentration of NADPH, which is D. Maltose
necessary for the implementation of the E. Saccharose
mechanism of phagocytosis, is maintained?
A. Pentose phosphate pathway 37. Indicate, hereditary insufficiency of which
B. Cori Cycle enzyme is the cause of vomiting and diarrhea
C. Krebs cycle after taking the fruit juices in a 9-month-old
D. Ornithine cycle child whose fructose intake has led to
E. Glycolysis hypoglycemia?
A. Fructose-1-phosphataldolase.
33. A 7-year-old child has symptoms of B. Phosphofructokinase
hemolytic anemia. In the biochemical analysis C. Hexokinase.
of erythrocytes, a reduced concentration of D. Fructose-1,6-diphosphatase.
NADPH and reduced glutathione has been E. Fructokinase.
established. The deficiency of which enzyme in
this case causes biochemical changes and 38. In the 8-month-old child, vomiting and
clinical manifestations? diarrhea are observed after taking fruit juices.
A. Glucose-6-phosphate dehydrogenase Fructose intake led to hypoglycemia. Hereditary
B. Hexokinase insufficiency of which enzyme is the cause of
C. Fructokinase these disorders?
D. Pyruvate kinase A. Fructose-1-phosphataldolase
E. Lactate dehydrogenase B. Fructokinase
C. Hexokinase
34. A child’s blood presents high content of D. Phosphofructokinase
galactose, glucose concentration is low. There E. Fructose-1,6-diphosphatase
are such presentations as cataract, mental
deficiency, adipose degeneration of liver.What 39. The excess concentration of glucose in the
disease is it? oral solution in diabetes leads to development:
A. Galactosemia A. Multiple caries
B. Diabetes mellitus B. Hyperplasia of enamel
C. Lactosemia C. Hypoplasia of enamel
D. Steroid diabetes D. Fluorosis
E. Fructosemia E. Intensifiedcalcification of enamel

35. A sick child has a delayed mental 40. In the biochemical study of the patient's
development, enlarged liver, and decreased blood, hyperglycemia, hyperketonemia, glucose
vision. The physician binds these symptoms to and ketone bodies in urine were detected. On
the electrocardiogram diffuse changes in the A. Decrease of proteosynthesis
myocardium was detected. She complains of dry B. Increase of lipolysis
mouth, thirst, frequent urination, general C. Acceleration of gluconeogenesis
weakness. The patient suffers from: D. Reduction of lipolysis
A. Diabetes mellitus. E. Increase of catabolism
B. Alimentary hyperglycemia.
C. Acute pancreatitis. 45. Diabetes mellitus causes ketosis as a result
D. Ischemic heart disease. of activated oxidation of fatty acids. What
E. Non-diabetes mellitus. disorders of acid-base equilibrium may be
caused by excessive accumulation of ketone
41. A 42 year old woman diagnosed with bodies in blood?
diabetes mellitus was admitted the A. Metabolic acidosis
endocrinological department with complaints of B. Metabolic alcalosis
thirst, excessive appetite. What pathological C. Any changes woun’t happen
components are revealed in course of laboratory D. Respiratory acidosis
examination of the patient’s urine? E. Respiratory alcalosis
A. Glucose, ketone bodies
B. Protein, aminoacids 46. A patient is ill with diabetes mellitus that is
C. Protein, creatine accompanied by hyperglycemia of over 7,2
D. Bilirubin, urobilin millimole/l on an empty stomach. The level of
E. Blood what blood plasma protein allows to estimate
the glycemia rate retrospectively (4-8 weeks
42. A patient with insulin-dependent diabetes before examination)?
mellitus has been injected with insulin. After A. Glycated hemoglobin
some time at the patient has developed B. Albumin
weakness, irritability, and increased sweating. C. Fibrinogen
What is the main mechanism of development of D. C-reactive protein
hypoglycemic coma? E. Ceruloplasmin
A. Carbohydrate starvation of the brain.
B. Decrease of gluconeogenesis. 47. A 62-year-old female patient has developed
C. Increaseof glycogenolysis. a cataract (lenticular opacity) secondary to the
D. Increase of ketogenesis. diabetes mellitus. What type of protein
E. Increase of lipogenesis. modification is observed in case of diabetic
cataract?
43. Examination of a 56-year-oldfemale patient A. Glycosylation
with a history of type 1 diabetes revealed a B. Phosphorylation
disorder ofprotein metabolism that is C. ADP-ribosylation
manifestedby aminoacidemia in the laboratory D. Methylation
blood test values, and clinically by the delayed E. Limited proteolysis
wound healing and decreased synthesis of
antibodies. Which ofthe following mechanisms 48. After recovering from epidemic parotiditis a
causes the development of aminoacidemia? patient began to put off weight, he was
A. Increased proteolysis permanently thirsty, drank a lot of water, had
B. Albuminosis frequent urination, voracious appetite. Now he
C. Decrease in the concentration ofamino acids has complaints of skin itch, weakness,
in blood furunculosis. His blood contains: glucose - 16
D. Increase in the oncotic pressure inthe blood mmole/L, ketone bodies - 100 mcmole/L;
plasma glucosuria. What disease has developed?
E. Increase in low-density lipoproteinlevel A. Insulin-dependent diabetes
B. Insulin-independent diabetes
44. It is known that in patients with diabetes C. Steroid diabetes
mellitus often found inflammatory processes, D. Diabetes insipidus
reduced regeneration, and healing of wounds E. Malnutrition diabetes
decreases. The reason for this is:
49. Patient with diabetes mellitus experienced D. Itsenko-Kushing's disease
loss of consciousness and convulsions after an E. Akromegalia
injection of insulin. What might be the result of
biochemical blood analysis for concentration of 54. In a patient 15 years of age, glucose
sugar? concentration is 4,8 mmol/l, an hour after the
A. 1,5 mmol/L sugar intake - 9.0 mmol/l, after 2 hours - 7.0
B. 8,0 mmol/L mmol/l, after 3 hours - 4.8 mmol/l . These
C. 10,0 mmol/L indices are characteristic of such a disease:
D. 3,3 mmol/L A. Hidden diabetes mellitus
E. 5,5 mmol/L B. Type I diabetes
C. Type II diabetes
50. A 46-year-old patient complains of dry D. Itsenko-Kushing's disease
mouth, thirst, urination, general weakness. In E. –
the blood: hyperglycemia, hyperketonemia. In
the urine: glucose, ketone body. On the ECG: 55. A nurse accidentally injected a nearly
diffuse changes in myocardium. What is the double dose of insulin to a patient with diabetes
most probable diagnosis? mellitus. The patient lapsed into a hypoglycemic
A. Diabetes mellitus coma. What drug should be injected in order to
B. Alimentary hyperglycemia help him out of coma?
C. Acute pancreatitis A. Glucose
D. Non-diabetes mellitus B. Lidase
E. Ischaemic heart disease C. Insulin
D. Somatotropin
51. After the epidemic parotitis, the patient loses E. Noradrenaline
weight, constantly feels thirst, drinks a lot of
water, indicates frequent urination, increased 56. For type II diabetes, the characteristic
appetite, itching, weakness, furunculosis. In the features are hyperglycemia, hypochloremia.
blood: glucose - 16 mmol/l, ketone bodies - 100 Which of the following processes is activated in
μmol/l; glucosuria What disease has developed the first place?
in the patient? A. Gluconeogenesis.
A. Insulin-dependent diabetes mellitus B. Glycolysis.
B. Non-dependent diabetes mellitus C. Glycogenolysis.
C. Steroidal diabetes D. Glucose reabsorption.
D. Non-diabetes mellitus E. Transport of glucose to the cell.
E. Malnutrition-related diabetes mellitus
57. A 57 year old patient with diabetes mellitus
52. A 38-year-old patient was delivered to the was developed ketoacedosis. Biochemical base
reanimation unit with unconscious. Reflexes are of this condition is smaller extent of acetyl-
absent. Blood sugar is 2.1 mmol/L. In CoAutilization.What cell compound deficit
anamnesis - diabetes since 18 years of age. causes this effect?
What coma does a patient have? A. Oxaloacetate
A. Hypoglycemic B. 2-oxoglutarate
B. Ketoacidotic C. Glutamate
C. Lactacidotic D. Aspartate
D. Hyperosmolar E. Succinate
E. Hyperglycemic
58. A patient with diagnose of diabetes mellitus,
53. In the patient's blood, glucose is 5.6 in the morning received an assigned dose of
mmol/L, 1 hour after the sugar intake - 13.8 insulin of prolonged action. He missed the next
mmol/l, and after 3 hours - 9.2 mmol/l. What meal, and soon felt weakness, headache,
pathology is characterized by such indicators? dizziness, greediness, trembling of the body,
A. Hidden form of diabetes mellitus convulsions, hunger, and the phenomenon of
B. Healthy person hypoglycemia. The use of glucose does not
C. Thyrotoxicosis
improve the condition. What drug should be D. Low (absent) activity of glucose 6-
entered to alleviate this condition? phosphatase
A. Adrenaline E. Deficit of a gene that is responsible for
B. Triamcinolone synthesis of glucose 1-phosphaturidine
C. Noradrenaline transferase
D. Prednisolone
E. Hydrocortisone 63. After the introduction of adrenaline in a
patient with persistent hypoglycemia, blood test
59. A patient for 5 years suffering from has not changed significantly. In such
diabetes. As a result of a violation of a diet, he circumstances, there is a possibility of
developed a coma. An ambulance doctor gave disturbances in the liver. What function of the
him glucose. The condition of the patient has liver is changed?
improved. What kind of coma was in the A. Glycogendeposing.
patient? B. Glycolytic.
A. Hypoglycemic C. Excretory.
B. Acidotic D. Ketogenic.
C. Hyperglycemic E. Cholesterol-forming.
D. Hepatic
E. Hypothyroid 64. A 2-year-old child has been diagnosed with
Girke's disease, which is manifested by severe
60. Patients with Itsenko-Cushing's syndrome hypoglycemia. The reason for this condition is
are observed with a hyperglycemia and the absence of the enzyme glucose-6-
glucosuria. Synthesis and secretion of which phosphatase. With the violation of which
hormone is increased in this patient? process is this pathology involved?
A. Cortisol A. Mobilization of glycogen.
B. Adrenaline B. Gluconeogenesis.
C. Glucagone C. Glycolysis.
D. Thyroxine D. Ketogenesis.
E. Aldosterone E. Synthesis of glycogen.

61. Pancreas is known as a mixed gland. 65. The glycogen that came from the food was
Endocrine functions include production of digested in the gastrointestinal tract. Which final
insulin by beta cells. This hormone affects the product was formed as a result of this process?
metabolism of carbohydrates. What is its effect A. Glucose
upon the activity of glycogen phosphorylase B. Lactate
(GP) and glycogen synthase (GS)? C. Lactose
A. It inhibits GP and activates GS D. Galactose
B. It activates both GP and GS E. Fructose
C. It inhibits both GP and GS
D. It activates GP and inhibits GS 66. In the human diet there are a large number
E. It does not affect the activity of GP and GS of carbohydrates. The number of which
structures will increase in cytoplasm of
62. Medical ambulance delivered a 2 year old hepatocytes?
girl to the children’s department.Objectively: A. Glycogen granules
the child is languid, apathetic. Liver is enlarged B. Drops of fat
and liver biopsy revealed a significant excess of C. The lysosomes
glycogene. Glucose concentration in the blood D. Free ribosomes
streamis below normal. What is the cause of low E. Inclusion of lipofuscine
glucose concentration?
A. Low (absent) activity of glycogene 67. Andersen's disease belongs to a group of
phosphorylase in liver hereditary diseases that develop due to the
B. Low (absent) activity of hexokinase congenital malformation of the synthesis of
C. High activity of glycogen synthetase certain enzymes of glycogenolysis. The
inadequacy of which enzyme is the molecular B. Gluconeogenesis in the muscles.
basis of this glycogenose? C. Glycogenolysis.
A. Amilo (1-4 → 1-6) transglycosidases. D. Decarboxylation.
B. Glycogen synthase. E. Synthesis of higher fatty acids.
C. Glucose-6-phosphatase.
D. Lysosomal glycosidase. 73. In a patient undergoing a course of medical
E. Phosphofructokinase. starvation, the normal level of glucose in the
blood is maintained mainly due to
68. Characteristic sign of glycogenosis is gluconeogenesis. From what amino acids at the
muscle pain during physical work. Blood same time in the human liver most actively
examination reveals usually hypoglycemia. This synthesized glucose?
pathology is caused by congenital deficiency of A. Alanine
the following enzyme: B. Lysine
A. Glycogen phosphorylase C. Valine
B. Glucose 6-phosphate dehydrogenase D. Glutamic acid
C. Alpha amylase E. Leucine
D. Gamma amylase
E. Lysosomal glycosidase 74. The gluconeogenesis is activated in the liver
after intensive physical trainings. What
69. During starvation normal rate ofglucose is substance is utilized in gluconeogenesis first of
maintained by means ofactivation of all in this case:
gluconeogenesis. Whatsubstance can be used as A. Lactate
a substrate forthis process? B. Pyruvate
A. Alanine C. Glucose
B. Ammonia D. Glutamate
C. Adenine E. Alanine
D. Urea
E. Guanine 75. During intensive physical work, muscle
tissue accumulates lactic acid, which diffuses
70. It is known that human carbohydrate into the blood and is absorbed by the liver and
reserves rapidly disappear as a result of heart. What process ensures the recovery of
prolonged fasting. Which of the metabolic glycogen stores in the muscles?
processes restores glucose in the blood? A. Cori Cycle
A. Gluconeogenesis. B. Citric acid cycle
B. Aerobic oxidation of glucose. C. Ornithine cycle
C. Glycolysis. D. Cycle of tricarboxylic acids
D. Glycogenolysis. E. Pentose phosphate pathway
E. Pentose phosphate pathway.
76. The patient was diagnosed with beri-beri.
71. Hyperglycemia develops due to chronic What is the enzyme's activity affected by the
overdose of glucocorticoids in a patient. Specify patient?
the process of carbohydrate metabolism, due to A. Pyruvate dehydrogenase.
which the concentration of glucose increases: B. Citrate synthase.
A. Gluconeogenesis. C. Malate dehydrogenase.
B. Aerobic oxidation of glucose. D. Succinate dehydrogenase.
C. Glycogenolysis. E. Fumarase.
D. Glycogenenesis.
E. Pentose phosphate cycle. 77. If insufficient thiamine - vitamin B1 occurs
disease beriberi (polyneuritis) and disturbed
72. During the fasting the muscle proteins break carbohydrate metabolism. What metabolite thus
down into free amino acids. In which process accumulates in the blood?
will most likely be used amino acids under such A. Pyruvate.
conditions? B. Lactate.
A. Gluconeogenesis in the liver. C. Succinate.
D. Citrate. 82. Glycogen polysaccharide is synthesizedfrom
E. Malat. the active form of glucose. Theimmediate donor
of glucose residues duringthe glycogenesis is:
78. Some students developed myodynia after A. UDP-glucose
continuous physical activity duringphysical B. Glucose-1-phosphate
education. The reason for suchcondition was C. ADP-glucose
accumulation of lactic acidin the skeletal D. Glucose-6-phosphate
muscles. It was generatedin the students’ bodies E. Glucose-3-phosphate
after activation ofthe following process:
A. Glycolysis 83. A child with point mutation presents with
B. Gluconeogenesis absence of glucose-6-phosphatase,
C. Lipolysis hypoglycemia, and hepatomegaly. What
D. Pentose-phosphate cycle pathology are these signs characteristic of?
E. Glyconeogenesis A. Von Gierke’s disease (Glycogen storage
disease type I)
79. It is known that the pentosephosphate B. Cori’s disease (Glycogen storage disease
pathway occuring in the adipocytesof adipose type III)
tissue acts as a cycle.What is the main function C. Addison’s disease (Primary adrenal
of this cycle inthe adipose tissue? insufficiency)
A. NADPH2 generation D. Parkinson’s disease
B. Ribose-phosphate production E. McArdle’s disease (Glycogen storage disease
C. Xenobiotic detoxification type V)
D. Energy generation
E. Glucose oxidation to end products 84. A child has a history ofhepatomegaly,
hypoglycemia, seizures,especially on an empty
80. Fructosuria is known to be connected with stomach andin stressful situations. The child is
inherited deficiency of fructose-1-phosphate diagnosedwith Gierke disease. This diseaseis
aldolase.What product of fructose metabolism caused by the genetic defect ofthe following
will accumulate in the organism resulting in enzyme:
toxicaction? A. Glucose-6-phosphatase
A. Fructose-1-phosphate B. Amyloid-1,6-glycosidase
B. Glucose-1-phosphate C. Phosphoglucomutase
C. Glucose-6-phosphate D. Glycogen phosphorylase
D. Fructose-1,6-biphosphate E. Glucokinase
E. Fructose-6-phosphate
85. In patients with glycogenosis, that is von
81. The genetic defect of pyruvatecarboxylase Gierke’s disease, the conversion of glucose-6-
deficiency is the cause of delayed physical and phosphate in to glucose is inhibited, which is
mental development and early death in children. accompanied by the improper breakdown of
This defectis characterized by lacticemia, glycogen in the liver. The cause of this
lactaciduria, disorder of a number of metabolic condition is the following enzyme deficiency:
path ways. In particular, the following process is A. Glucose-6-phosphatase
inhibited: B. Glycogen phosphorylase
A. Citric acid cycle and gluconeogenesis C. Glucose-6-phosphate dehydrogenase
B. Glycolysis and glycogenolysis D. Phosphofructokinase
C. Glycogenesis and glycogenolysis E. Phosphoglucomutase
D. Lipolysis and lipogenesis
E. Pentose phosphate pathway andglycolysis
Metabolism of lipids and its regulation
1. Examination of a patient revealed that dental E. Electrolytes
hypoplasia was caused by hypovitaminosis of
vitamins A and D. These vitamins were 6. A patient was prescribed a drug with apparent
administered perorally but they didn’t have any lipophilic properties. What is the main
medicinal effect. What is the probable cause of mechanism of its absorption?
disturbed vitamin assimilation? A. Passive diffusion
A. Bile acid deficiency B. Active transporting
B. Phospholipase A2 deficiency C. Binding with transport proteins
C. Cholesterolesterase deficiency D. Pinocytosis
D. Colipase deficiency E. Filtration
E. Pancreatic lipase deficiency
7. Due to the blockage of the common bile duct
2. Examination of a man who hadn’t been (which was radiographically confirmed), the
consuming fats but had been getting enough biliary flow to the duodenum was stopped. We
carbohydrates and proteins for long time should expect the impairment of:
revealed dermatitis, poor wound healing, vision A. Fat emulsification
impairment. Whatis the probable cause of B. Protein absorption
metabolic disorder? C. Carbohydrate hydrolysis
A. Lack of linoleic acid, vitamins A, D, E, K D. Secretion of hydrochloric acid
B. Lack of palmitic acid E. Salivation inhibition
C. Lack of vitamins PP, H
D. Low caloric value of diet 8. After eating fatty foods, the patient has
E. Lack of oleic acid nausea and heartburn, steatorrhea occurs. The
reason for such a state can be:
3. In humans, the absorption of products of the A. Lack of bile
hydrolysis of fats is impaired. The reason for B. Increased lipase secretion
this may be a deficiency in the cavity of the C. Violation of trypsin synthesis
small intestine: D. Lack of amylase
A. Bile acids E. Impaired phospholipase synthesis
B. Lipolytic enzymes
C. Bile pigments 9. A coprological study found that the feces are
D. Sodium ions discolored, there are found drops of neutral fat.
E. Fat-soluble vitamins The most likely cause of this is a violation:
A. Flow of bile into the intestine
4. A patient has normally colored stool B. pH of gastric juice
including a large amount of free fatty acids. The C. Secretions of pancreatic juice
reason for this is a disturbance of the following D. Secretion of intestinal juice
process: E. Absorption processes in the intestine
A. Fat absorption
B. Fat hydrolysis 10. The respiratory ratio of the patient is 0.7.
C. Biliary excretion This indicates that in human cells prevails:
D. Bilification A. Oxidation of fats
E. Lipase secretion B. Oxidation of carbohydrates
C. Oxidation of proteins
5. A 65-year-old patient suffers from D. Mixed oxidation of fats and carbohydrates
cholelithiasis. Recently, there were signs of E. Mixed oxidation of fats and proteins
achilic syndrome due to obturation of the biliary
tract. Which components of food will be 11. A sportsman was recommended to take a
mastered the most? preparation with carnitine in order to improve
A. Fats his achievements. What process is activated by
B.Carbohydrates carnitine to the most extent?
C. Proteins A. Transporting of fatty acids to the
D. Nucleic acids mitochondria
B. Lipid synthesis B. Pancreatic amylase
C. Synthesis of ketone bodies C. Pancreatic proteases
D. Synthesis of steroid hormones D. Bile lipase
E. Tissue respiration E. Gastric proteases

12. One of the factors that cause obesity is the 17. Obesity is a common disease. The aim of its
inhibition of fatty acids oxidation due to: treatment is to lower content of neutral fats in
A. Low level of carnitine the body. What hormone-sensitive enzyme is
B. Impaired phospholipid synthesis the most important for intracellular lipolysis?
C. Excessive consumption of fatty foods A. Triacylglycerol lipase
D. Choline deficiency B. Protein kinase
E. Lack of carbohydrates in the diet C. Adenylate cyclase
D. Diacylglycerol lipase
13. A patient with a high degree of obesity in E. Monoacylglycerol lipase
the quality of a dietary supplement is
recommended carnitine to improve the 18. Disorderthe splitting of the lipids in the
"burning" of fat. What is the direct involvement small intestine is due to a disorder of lipase
of carnitine in the process of oxidation of fats? activity. Which of the following factors
A. Transport of fatty acids from cytosol to activates the lipase?
mitochondria A. Bile acids
B. Activation of fatty acids B. Hydrochloric acid
C. Involved in one of the reactions of beta- C. Enterokinase
oxidation of fatty acids D. Pepsin
D. Transport of fatty acids from fat depots to E. Solts of Na+
tissues
E. Activation of intracellular lipolysis 19. A man has 35 years of pheochromocytoma.
In the blood there is an elevated level of
14. A 1-year-old child entered the clinic with adrenaline and norepinephrine, the
signs of damage to the muscles of the concentration of free fatty acids increased in 11-
extremities and trunk. After the examination fold. Indicate which activation of any enzyme
revealed a deficit of carnitine in the muscles. under the influence of adrenaline increases
The biochemical basis of this pathology is the lipolysis?
disruption of the process: A. TAG-lipase.
A. Transport of fatty acids in mitochondria B. Lipoprotein lipase.
B. Regulation of Ca2+level in mitochondria C. Phospholipases A2.
C. Substrate phosphorylation D. Phospholipase C.
D. Utilization of lactic acid E. Cholesterol esterase
E. Oxidizing phosphorylation
20. The patient, who was in the clinic about
15. Emotional stress causes activation of pneumonia complicated by pleurisy, was
hormone-sensitive triglyceride lipase in the included in the complex therapy of
adipocytes. What secondary mediator takes part prednisolone. The anti-inflammatory action of
in this process? this synthetic glucocorticoid is associated with
A. Cyclic adenosine monophosphate blocking the release of arachidonic acid by
B. Cyclic guanosine monophosphate inhibition:
C. Adenosine monophosphate A. A2 phospholipases
D. Diacylglycerol B. Lipoxygenase
E. Ions of Ca2+ C. Phospholipases C.
D. Peroxidase
16. A significant amount of undigested fat was E. Cyclooxygenase
found in the patient's coprogram.The violation
of which enzymes secretion most likely occurs 21. In the laboratory examination of blood of a
in this person? person who was bitten by a change, hemolysis
A. Pancreatic lipase of erythrocytes, hemoglobinuria was revealed.
The action of a mixed poison is due to the C. Interferons
presence of an enzyme in it: D. Catecholamins
A. Phospholipase A2 E. Corticosteroids
B. Phospholipase D
C. Phospholipase S. 27. The drug "Geptral", which is used in liver
D. Phospholipase A1 diseases contains S-adenosylmethionine. This
E. Sphingomyelinase active amino acid is involved in the synthesis
of:
22. Patients with pain syndrome in the joints A. Phospholipids
permanently appoint aspirin. Which of the B. Fatty acids
enzymes that he enfolds? C. Triacylglycerols
A. Phospholipase A2 D. Cholesterol
B. Cyclooxygenase E. Heme
C. Phospholipase D
D. Lipoxygenase 28. It is known that a part of carbon dioxide is
E.PhospholipaseC used in the body in the biosynthesis of fatty
acids, urea, gluconeogenesis, and the like. What
23. Thromboxanes belong to the bioregulators kind of vitamin forms a CO2-transporting form
of cellular functions of lipid nature. The source for these reactions?
for the synthesis of these compounds is: A. Biotine
A. Arachidonic acid B. Timine
B. Stearic acid C. Riboflavin
C. Palmitic acid D. Nicotinamide
D. Phosphatidic acid E. Retinol
E. Palmiotoleic acid
29. A young man of 25 years consumes an
24. Methyl groups (-CH3) are used in the body excessive amount of carbohydrates (600 g per
for synthesis such important compounds as day) that exceeds its energy needs. What
creatine, choline, adrenaline, and others. The process will be activated in the human organism
source of these groups is one of the essential in this case?
amino acids, namely: A. Lypogenesis
A. Methionine B. Glycolysis
B. Valine C. Lypolis
C. Leicine D. Gluconeogenesis
D. Isoleucine E. Oxidation of fatty acids
E. Tryptophan
30. Disorders of the processes of mielinisation
25. At deficiency of biotin, there is a disorder of neuronal fibers' leads to neurological disorders
higher fatty acids synthesis. Which of these and mental retardation. Such symptoms are
metabolites can be affected by this? characteric for hereditary and acquired
A. Malonyl-CoA metabolic disturbances:
B. Succinyl-CoA A. Sphingolipids
C. Pyruvate B. Neutral fat
D. Alanine C. Higher fatty acids
E. Serotonin D. Cholesterol
E. Phosphatidic acid
26. Deficiency in the body of linoleic and
linoleic acids leads to skin damage, hair loss, 31. The mother asked the doctor about the bad
delayed healing of wounds, thrombocytopenia, health of the child - the lack of appetite, poor
and reduced probability of infection. Disorders sleep, irritability. At the biochemical
the synthesis of which substances most likely examination in the blood, the absence of the
causes these symptoms? enzyme glucocerebrosidase was detected. For
A. Eicosanoids which pathology is this characteristic?
B. Interleukins A. Gaucher’s disease
B. Tay-Sachs disease A. Choline
C. Niemann-Pika's disease B. DOPA
D. Girke's disease C. Cholesterol
E. Pompe’s disease D. Acetoacetate
E. Linolic acid
32. A patient is diagnosed with
glucocerebrosidelipidosis (Gaucher’s disease) 37. Synthesis of phospholipids is
that manifests as splenomegaly, liver disorderedunder the liver fat infiltration.
enlargement, affected bone tissue, and Indicate which of the following substances can
neuropathies. What enzyme of complex lipid enhance the process of methylation during
catabolism is deficient, thus causing this phospholipids synthesis?
disease? A. Methionine
A. Sphingomyelinase B. Ascorbic acid
B. β-galactosidase C. Glucose
C. Glucocerebrosidase D. Glycerin
D. Hexosaminidase E. Citrate
E. Hyaluronidase
38. Examination of cell culture gotfrom a
33. At utilization of arachidonic acid by patient with lysosomal pathology revealed
cyclooxygenase pathways, biologically active accumulation of great quantity of lipids in the
substances are formed. Indicate them: lysosomes. What of the following diseases is
A. Prostaglandins this disturbance typical for?
B. Thyroxine A. Tay-Sachs disease
C. Biogenic amines B. Gout
D. Somatomedin C. Phenylketonuria
E. Insulin-like growth factors D.Wilson disease
E. Galactosemia
34. The patient appointed a lipotropic drug a
donor of methyl groups, to prevent a fatty liver 39. An experimental animal has been
distrophia. This is sensible: givenexcessive amount of carbon-
A. S-Adenosylmethionine labeledglucose for a week. What compound
B. Cholesterol canthe label be found in?
C. Bilirubin A. Palmitic acid
D. Valine B. Methionine
E. Glucose C. Vitamin A
D. Choline
35. Steatosis occurs as a result of the E. Arachidonic acid
accumulation of triacylglycerols in hepatocytes.
One of the mechanisms of development of this 40. A 6 years old child was delivered to
disease is reduction of utilization neutral fat ahospital. Examination revealed that thechild
LDL. What lipotropic substances prevent the couldn’t fix his eyes, didn’t keephis eyes on
development of steatosis? toys, eye ground had thecherry-red spot sign.
A. Methionine, BC, B12 Laboratory analyses showed that brain, liver and
B. Arginine, B2, B3 spleen had highrate of ganglioside glycometide.
C. Alanine, B1, PP What congenital disease is the child ill with?
D. Valine, B3, B2 A. Tay-Sachs disease
E. Isoleucine, B1, B2 B.Wilson’s syndrome
C. Turner’s syndrome
36. In an experimental animal, receiving non- D. Niemann-Pick disease
protein diet, fatty infiltration of the liver has E. MacArdle disease
developed due to the deficiency of the
methylating agents. The formation of which
metabolite is disturbed in the experimental
animal?
41. A dry cleaner’s worker has been found to 46. The doctor gave the woman a
have hepatic steatosis. This pathology can be recommendation to continuethe low-calorie diet.
caused by the disruption of synthesis of the She decided to get the same amount of calories,
following substance: but to replace carbohydrates on fats. Which of
A. Phosphatidylcholine the following lipoprotein fractions will bee
B. Tristearin levated as a result of this diet?
C. Urea A. Chylomicrons
D. Phosphatidic acid B. VLDL
E. Cholic acid C. LDL
D. LPID
42. The key reaction of fatty acid synthesis is E. HDL
production of malonyl-CoA. What metabolite is
the source of malonyl-CoA synthesis? 47. For the prevention of atherosclerosis,
A. Acetyl-CoA coronary heart disease, cerebrovascular
B. Succinyl-CoA accident, the consumption of high-fat
C. Acyl-CoA polyunsaturated fatty acids is recommended.
D. Malonate One of the following fatty acids is:
E. Citrate A. Linolic
B. Oleic
43. A 2-year-old child presents with acute C. Lauric
psychomotor retardation, vision and hearing D. Palmitooleic
impairment, sharp enlargement of the liver and E. Stearic
spleen. The child is diagnosed with hereditary
Niemann-Pick disease. What genetic defect is 48. Hereditary hyperlipoproteinemia of type I is
the cause of this disease? duetoin sufficiency of the lipoproteinlipase.
A. Sphingomyelinase deficiency Increasing of which transport forms of lipid in
B. Glucose 6-phosphatase deficiency plasma is characteric even on an empty
C. Amylo-1,6-glucosidase deficiency stomach?
D. Acid lipase deficiency A. Lipoproteinsoflowdensity
E. Xanthine oxidase deficiency B. Chylomicrons
C. Lipoproteinsofverylowdensity
44. A 3-year-oldgirlwithmentalretardation D. High-densitylipoproteins
hasbeendiagnosedwithsphingomyelin lipidosis E. Modifiedlipoproteins
(Niemann-Pickdisease). Inthis
conditionsynthesisofthefollowing substance 49. After 4 hours taking of fatty food is found
isdisrupted: that patient blood plasma is muddy. The most
A. Sphingomyelinase probable reason for this state is increasing the
B. Glycosyltransferase concentration in plasma:
C. Sphingosine A. Chilomicrons
D. Ceramides B. VLDL
E. Gangliosides C. LDL
D. Cholesterol
45. Modern antiatherosclerotic drugs are used E. Phospholipids
for the preventions and treatment of
atherosclerosis. Such drugs as gemfibrozil and 50. In the diseased child is established
fenfibrate is inhibiting cholesterol biosynthesis hyperlipoproteinemia, inherited. Genetic defect
by enzyme digestion: of what synthesis enzyme causes this
A. β-HMG-reductase phenomenon?
B. Hexokinase A. Lipoproteinlipase
C. Glucose-6-phosphatase B. Glycosidase
D. Acyltransferase C. Proteinase
E. Acyl-CoA-cholesterolacyltransferase D. Hemesynthetase
E. Fenylanininehydroxylase
51. Rabbits were fed with cholesterol
supplementation. After 5 months, 56. During examination of a teenagerwith
atherosclerotic changes were detected in the xanthomatosis the family history
aorta. Name the main cause of atherogenesis in ofhypercholesterolemia is revealed.
this case: Whattransportable lipids are increased
A. Exogenous hypercholesterinemia inconcentration in case of such a disease?
B. Overeating A. Low-density lipoproteins
C. Hypodynamia B. Chylomicrons
D. Endogenous hypercholesterinemia C. Very low-density lipoproteins
E. Stress D. High-density lipoproteins
E. Intermediate-density lipoproteins
52. In the examination, was established the
female has insufficient activity of 57. Inthebloodofpatientswithdiabetesmellitus,
lipoproteinlipase which hydrolizes the increasesthecontentoffreefattyacids (HFAs)
cholymocrons triglycerides on the surface of the areobserved. Thereasonforthismaybe:
endothelium adipose tissue. What biochemical A.
disorders should wait? Increasingtheactivityofadipocytestriglyceridelip
A. Hyperlipoproteinemia type I ase
B. Hyperlipoproteinemia II A type B. Accumulationinthecytosolof palmitic-CoA
C. Hyperlipoproteinemia of type III C. Activationofketone bodies utilization
D. Hyperlipoproteinemia of type IV D. Activationofthesynthesisofapolipoproteins
E. Hyperlipoproteinemia II B type A-1, А-2, А-4
E. Reducedplasma phosphatidylcholine-
53. A 58-year-old patient suffers from the cholesteine acyltransferaseactivity
cerebral atherosclerosis. Examination revealed
hyperlipoidemia. What class of lipoproteins will 58. In the case of diabetes mellitus and
most probably show increase in concentration in starvation in the blood, the amount of acetone
this patient’s blood serum? bodies used in the quality of energy material
A. Low-density lipoproteins increases. Name the substance from which they
B. High-density lipoproteins are synthesized:
C. Fatty acid complexes with albumins A. Acetyl-CoA
D. Chylomicrons B. Succinyl-CoA
E. Cholesterol C. Citrate
D. Malate
54. A patient underwent a course oftreatment E. Ketoglutarate
for atherosclerosis. Laboratorytests revealed an
increase in the antiatherogeniclipoprotein 59. A 67-year-old man consumes eggs, pork fat,
fraction in theblood plasma. The treatment butter, milk and meat. Blood test results:
efficacy isconfirmed by the increase in: cholesterol - 12,3 mmol/l, total lipids - 8,2 g/l,
A. HDL increased low-density lipoprotein fraction
B. VLDL (LDL). What type of hyperlipoproteinemia is
C. IDL observed in the patient?
D. LDL A. Hyperlipoproteinemia type IIa
E. Chylomicrons B. Hyperlipoproteinemia type I
C. Hyperlipoproteinemia type IIb
55. During the examination of the patient, the D. Hyperlipoproteinemia type IV
increase in blood serum in low density E. Cholesterol, hyperlipoproteinemia
lipoproteins was detected. What disease can be
foreseen in this patient? 60. Cholesterol content in blood serum of a 12-
A. Atherosclerosis year-old boy is 25 mmol/l. Anamnesis states
B. Kidney damage hereditary familial hypercholesterolemia caused
C. Acute pancreatitis by synthesis disruption of receptor-related
D. Gastritis proteins for:
E. Lung inflammation A. Low-density lipoproteins
B. High-density lipoproteins 61. Increased HDL levels decrease the risk of
C. Chylomicrons atherosclerosis. What is the mechanism of HDL
D. Very low-density lipoproteins antiatherogenic action?
E. Middle-density lipoproteins A. They remove cholesterol from tissues
B. They supply tissues with cholesterol
C. They are involved in the breakdown of
cholesterol
D. They activate the conversion of cholesterol
to bile acids
E. They promote absorption of cholesterol in the
intestine
Amino- acid metabolism and its regulation
1. A 36-year-old female patientwho has been A. Gastrin
limiting the number offoodstuffs in her diet for B. Secretin
3 monthspresents with a decrease in body C. Cholecystokinin
weight, deterioration of physical andmental D. Somatostatin
health, face edemata. Thesechanges may be E. Neurotensin
caused by the deficiencyof the following
nutrients: 6. Young man, due to the irritation of the carotic
A. Proteins plexus by the inflammatory process (solarium),
B. Vitamins the functional activity of the glands of the
C. Fats stomach is increased, which is expressed, in
D. Carbohydrates particular, in the increase of the product of
E. Micronutrients chloride acid. Which of the following
substances causes hypochlorhydria in this case?
2. In the daily diet of an adult healthy person A. Gastrin
should be fats, proteins, carbohydrates, B. Gastrointing peptide
vitamins, mineral salts and water. Specify the C. Urologist
amount of protein per day, which provides D. Glukagon
normal functioning of the body. E. Kalikrein
A. 100-120 g
B. 50-60 g 7. The pyloric part of the stomach has been
C.10-20 g removed. Reducing the secretion of which
D. 70-80 g hormone should be seen first of all?
E. 40-50 g A. Gastrin
B. Gistamin
3. A 30 years old patient with acute C. Secretin
inflammation of the liver (pancreatitis), D. Kholetsistokinin
violations of cavity digestion of proteins were E. Stinging digestive peptide
detected. This may be due to insufficient
synthesis and secretion of the gland of such an 8. In order to determine the maximum secretion
enzyme: of hydrochloric acid in gastric juice, a solution
A. Trypsin of histaminewas given to 42-year-old patient.
B. Pepsin This led to increasing of the secretion of the
C. Lipase parietal gland of such component of juice as:
D. Dipeptidase A. Bicarbonates
E. Amilaza B. Trypsinogen
C. Lipase
4. The patient was brought to the hospital with D. Amilaza
burns of skin. To clean the wounds from dead E. Slime
tissues and mucus, the doctor prescribed an
enzyme preparation for local treatment. Name 9. In the acute experiment, a weak solution of
it: hydrochloric acid was introduced into the cavity
A. Tripsin of the duodenum of animal. Will it increase the
B. Pansinorm secrecy of the gastrointestinal hormone?
C. Asparaginase A. Sekretin
D. Pepsin B. Gastrin
E. Streptokinase C. Motilin
D. Neurotensin
5. A patient with gastric juice hypersecretion E. Gistamin
has been recommended to exclude from the
dietric broths and vegetable in fused water. A 10. Transformation of proteins in the stomach is
doctor recommended it, because these food the initial stage of protein digestion in the
products stimulate production of the following human digestive tract. What are the enzymes
hormone: involved in digestive proteins in the stomach:
A. Pepsin and gastriksin noradrenaline and serotonine in the
B. Trypsin and Cathepsin mitochondrions of cerebral neurons. What
C. Chymotropins and lysozyme enzyme participates in this process?
D. Enteropeptidase and elastase A. Monoamine oxidase
E. Carboxypeptidase and aminopeptidase B. Transaminase
C. Decarboxylase
11. A newborn child suffers from milkcurdling D. Peptidase
in stomach, this means thatsoluble milk proteins E. Lyase
(caseins) transformto insoluble proteins
(paracaseins) bymeans of calcium ions and a 16. It is known that in the metabolism of
certainenzyme. What enzyme takes part in catecholamine mediators a special role belongs
thisprocess? to the enzyme monoamine oxidase (MAO).
A. Renin How does this enzyme inactivate mediators
B. Pepsin (noradrenaline, adrenaline, dopamine)?
C. Gastrin A. The oxidative deamination
D. Secretin B. Addition of an amino group
E. Lipase C. Removal of the metal group
D. Carboxylation
12. A patient has been prescribed E. Hydrolisis
pyridoxalphosphate. What processes are
correctedwith this drug? 17. During the catabolism of histidine, a
A. Transamination and decarboxylation biogenic amine is formed, which has a powerful
ofaminoacids vasodilating effect. Name it.
B. Oxidative decarboxylation of keto acids A. Histamine.
C. Deaminization of amino acids B. DOPA.
D. Synthesis of purine and pyrimidine bases C. Dopamine.
E. Protein synthesis D. Serotonin.
E. Noradrenalin.
13. To the hospital delivered 7 years-old
childwith allergic shock, which developed after 18. When decarboxylating glutamate in the
she was stomped by bee. In the blood, the central nervous system, a mediator of inhibition
concentration of histamine is increased.The is formed. Name it.
result of which reaction formed this amine? A. GAMK.
A. Decarboxylation B. Glutathione.
B. Hydroxylation C. Histamine.
C. Degradation D. Serotonin.
D. Deamination E. Asparagine.
E. Restoration
19. Biogenic amines are used in psychiatry for
14. The allergen was administered to the patient, the treatment of a number of diseases of the
tested for hypersensitivity, under the skin,and central nervous system. Specify the drug of this
than saw reddening, edema, and even after the group, which is a mediator of inhibition:
action of histamine. As a result of which the A. γ-aminobutyric acid.
amino acid conversion of histidine forms this B. Dopamine.
biogenic amine? C. Histamine.
A. Decarboxylation D. Serotonin.
B. Methylation E. Taurin.
C. Phosphorylation
D. Isolation 20. A child 9 months old eats artificial sweeties
E. Depositing that are not balanced by the vitamin B6. A child
have a pelagic dermatitis, convulsions, anemia.
15. Pharmacological effects of The development of a cramps may be related
antidepressantsare connected with inhibition of with a violation of forming of:
an enzyme catalyzing biogenic amines A. GAMK
B. Gistamin E. Cysteine, glucine, proline
C. Serotonin
D. DOPA 26. There are several ways to dispose of
E. Dopamin ammonia in the body, but for specific organs are
specific. What is the way to neutralize this toxic
21. What neurotransmitter in the brain tissue substance is typical for brain cells?
can be synthesized from the product of A. Formation of glutamine.
reamination of alpha-ketoglutaric acid? B. Formation NH 4 +.
A. GAMK C. Formation of asparagine.
B. Tryptamine D. Formation of Creatine.
C. Dopamine E. Formation of urea.
D. Serotonin
E. Noradrenalin 27. Ammonia is a very poisonous substance,
especially for the nervous system. What
22. In a patient with a diagnosis of malignant substance is particularly active in ammonia
carcinoma, the severely increased amount of disinfection in brain tissues?
serotonin in the blood.Choose the amino acid A. Glutamic acid.
from which this compound is formed: B. Lizin.
A. Tryptophan C. Proline.
B. Allanin D. Hystidine.
C. Leicin E. Alanin.
D. Treonin
E. Methionin 28. It is known that accumulation of ammonia is
the main cause of cerebral coma in hepatic
23. Hospitalized patient with diagnosis of insufficiency. What is a free amino acid plays a
intestinal carcinoids. Analizes showed an primary role in the use of this toxic substance in
increasing secretion of serotonin. It is known the brain?
that this substance is formed from an amino acid A. Glutamic acid.
tryptophan. What biomechanical mechanism is B. Alanin.
the basis of this process? C. Histidine.
A. Decarboxylation D. Tryptophan.
B. Deamination E. Cysteine.
C. Microsomal oxidation
D. Transamination 29. After an injury to the brain, the patient has
E. Formation of paired compounds an increased ammonia formation. What amino
acid is involved in removing ammonia from this
24. Methyl groups (-CH3) are used in the body tissue?
to synthesize such important compounds as A. Glutaminov.
creatine, choline, adrenaline, and others. The B. Valin.
source of these groups is one of the essential C. Lizin.
amino acids, namely: D. Tyrosin
A. Methionin E. Tryptophan
B. Valin
C. Leicin 30. The patient entered the clinic with a
D. Isoleucine concussion of the brain. On the background of
E. Tryptophan neurological symptoms, the concentration of
ammonia in the blood increases. What substance
25. Cationic glycoproteins are the main should be used to neutralize this substance in
components of salivation of parotid glands. the brain tissue?
What amino acids cause their positive charge? A. Glutamic acid
A. Lizin, arginine, histystine B. Gistamin
B. Aspartate, glutamate, glycine C. Ascorbic acid
C. Aspartate, arginine, glutamate D. Serotonin
D. Glutamate, valine, leucine E. Nicotinic acid
31. After the operation on the intestines, the 35. In a 2 year old child, there is a Intellectual
patient appeared symptoms of poisoning with disability, intolerance of protein foods, severe
ammonia according to the type of stomach hyperammonia as the result of reduced level of
coma. What mechanism of action of ammonia urea in the plasma of blood, which is connected
on the energy supply of the central nervous with birth deficiency of such mythohondrial
system? enzyme as:
A. Braking (slowdown) CTK due to the binding A. Carbamoyl phosphate synthase
of alpha ketoglutarate B. Citrate synthase
B. Braking of glycolysis C. Succinate dehydrogenase
C. Braking of beta-oxidation of fatty acids D. Malatedehydrogenase
D. Inactivation of the enzymes of respiratory E. Monoamine oxidase
chain
E. Dissociation of oxidative phosphorylation 36. In the biosynthesis of urea in the liver, the
formation of ornithine and urea is stimulated.
32. In a newborn child there is a decrease in What amino acid is the intermediate product of
intensity of sucking, frequent vomiting, this synthesis?
hypotension. In urine and blood, the A. Аrginine
concentration of citrulline is significantly B. Leicinc.
increased. What metabolic process is affected? C. Сitrate.
A. Ornithine cycle D. Valin.
В. Glycolysis E. Tryptophan.
C. Core's cycle
D. СТК 37. In the urine of the newborn, determined by
E. Gluconeogenesis citrulline and high levels of ammonia. Indicate
which substance is most likely to be inflicted on
33. In a 3 year old child, which outlived the a baby?
severe viral infection, the doctor has indicated A. Urea.
re-vomiting, unconsciousness, convulsions. The B. Bilirubin.
hyperammonemia was detected during the C. Creatine.
analysis. What can be connected with changing D. Creatinine.
of the biochemical indicators of blood in this E. Uric acid.
child?
A. Violation of ammonia neutralization in the 38. In a patient with reduced singular function
ornithine cycle of the kidneys, an unpleasant odor is noted from
B. Activation of the processes of the mouth. Increased excretion of the salivary
decarboxylation of amino acids glands of what substance is the cause of this?
C. Violation of neutralizing of biogenic amines A. Urea
D. Strengthening of decay of proteins in the B. α-amylase.
intestines C. Mutsina.
E. Depressing the activity of transamination D. Lysozyme.
enzymes E. Fosfatasi.

34. The main part of nitrogen is taking out of 39. A patient in an unconscious state is
organism as element of urea. Reducing the delivered by an ambulance brigade to the
activity of what liver enzyme leads to inhibition hospital. Objectively: reflexes are absent,
of the synthesis of urea and increasing the seizures occur periodically, breathing is
accumulation of ammonia in blood and tissues? unequal. After a laboratory examination, the
A. Carbamoyl phosphate synthase liver was diagnosed with a coma. The
B. Aspartate aminotransferase accumulation of which metabolites in the blood
C. Urease is essential for the appearance of central nervous
D. Amilaza system disorders?
E. Pepsin A. Ammonia.
B. Bilirubin.
C. Gistamin.
D. Glutamin. D. Methionine
E. Urea. E. Glutamine

40. In a boy of 4 years after suffering from 45. A 46-year-old patient suffers from
severe viral hepatitis, vomiting, episodes of progressing Duchenne muscular dystrophy.
nephropathy, seizures are observed. In the blood Changes of the following blood enzyme level
there is hyperammonia. Violation of which of may diagnose this condition:
the biochemical process in the liver caused such A. Creatine phosphokinase
a condition of the patient? B. Lactate dehydrogenase
A. Disposal of ammonia. C. Pyruvate dehydrogenase
B. Decarboxylation of amino acids. D. Glutamate dehydrogenase
C. Disposal of Biogenic Amines. E. Adenylate kinase
D. Synthesis of alphabets.
E. Gluconeogenesis. 46. It was found that the cause of dizziness,
memory impairment, and periodic judgment in
41. Examination of a 2 year-old child with renal the patient is a violation of the decarboxylation
insufficiency revealed hyperoxaluria that caused of glutamic acid. What is the product of this
depositing of calcium oxalate stones in the reaction:
kidneys. The reason for this condition is a A. GAMK.
disturbance of metabolism of the following B. PAUL
aminoacid: C. TDP
A. Glycine D. ATP.
B. Lysine E. TGFK.
C. Methionine
D. Arginine 47. A 24 year-old patient that suffers from
E. Histidine epilepsy was administered glutamic acid. What
product of its decarboxylation caused the
42. Antioxidant enzymes inhibit lipid therapeutic effect?
peroxidation processes. Decrease of glutathione A. GABA
peroxidase activity is caused by deficiency of B. Histamine monooxygenase
the following microelement: C. Serotonin
A. Selenium D. Dopamine
B. Molybdenum E. Taurine
C. Cobalt
D. Manganese 48. A baby has epileptical convulsions that are
E. Copper caused by vitamin B6 defficiency. It is due to
the decrease of GABA in the neural tissue.
43. Blood vessels endothelium has a What enzyme activity is decrased in this case?
significantly high metabolic activity as it A. Glutamate decarboxylase
synthesizes a variety of vasoactive substances. B. Alanine aminotransferase
What powerful vasodilatator is synthesized from C. Glutamate degydrogenase
L-arginine? D. Pyridoxal kinase
A. Nitric oxide E. Glutamate synthetase
B. Histamine
C. Bradykinin 49. In a patient with a sharp increase in
D. Acetylcholine serotonin level, the diagnosis of "malignant
E. Epinephrine carcinoid" is observed in the blood. Choose the
amino acid from which the given biogenic
44. Nitric oxide is a powerful vasodilatator and amine can be formed.
blood pressure regulator. In human organism it A. Tryptophan.
is synthesized from: B. Alanine.
A. Arginine C. Leicin.
B. Proline D. Methionine
C. Lysine E. Threonine.
50. А patient suffering from pellagra was 55. A patient has the initial stage of gingivitis.
examined. It was found that his diet included Dilatation of the microcirculatory vessels
almost no meat and consisted mostly of corn. caused gum hyperaemia. What substance
His condition was caused by the deficiency of produced by mast cells caused the changes
the following substance in corn: A. Tryptophan mentioned above?
B. Tyrosine A. Histamine
C. Proline B. Epinephrine
D. Alanine C. Substance P
E. Histidine D. Endorphins
E. Actylcholine
51. A patient that is suffering from congenital
Hartnup disease has pellagra-like dermatosis 56. A cook burned his hand incautiously. What
and mental retardation due to the deficiency of substance has caused the reddening, swelling
nicotinic acid. The reason for this condition is and pain of the damaged area?
the violation of the following process: A. Histamine
A. Tryptophan absorption and reabsorption in B. Thiamine
the kidneys C. Glutamine
B. Phenylalanine transamination D. Lysine
C. Tryptophan decarboxylation E. Galactosamine
D. Methionine absorption and reabsorption in
the kidneys 57. L-DOPA is the precursor chemical of
E. Cysteine absorption and reabsorption dopamine. It is prescribed for Parkinson’s
disease treatment. What aminoacid is used for
52. A woman was bitten by a wasp. Shortly its biosynthesis?
after the bite a painful and itching blister was A. Tyrosine
formed. After some more time hives and B. Alanine
dyspnea developed. What factor was the reason C. Cysteine
for it? D. Histidine
A. Histamine E. Tryptophan
B. Hagemann factor
C. Lysosomal enzymes 58. A patient suffering from Parkinson’s disease
D. Norepinephrine has tremor of both hands. This symptom is
E. Epinephrine caused by the deficiency of the following
neurotransmitter in the striatopallidal structures:
53. Shortly after the dental treatment the patient A. Dopamine
has developed red itching blemishes on the B. GABA
facial skin and oral mucosa. Urticaria was C. Substance P
diagnosed. Vasodilatation and itching were D. Epinephrine
caused by the following bioactive substance: E. Serotonin
A. Histamine
B. Prostaglandin Е2 59. An 84 year-old patient suffers from the
C. Leucotriene В4 Parkinson’s disease. One of its pathogenetic
D. Interleukin-1 causes is the defficiency of a neurotransmitter in
E. Bradykinin certain neural structures. What neurotransmitter
is it?
54. The patient has an allergic reaction, which is A. Dopamine
accompanied by itching, swelling and reddening B. Epinephrine
of the skin. Concentration of which biogenic C. Norepinephrine
amine has increased in tissues? D. Histamine
A. Histamine. E. Acetylcholine
B. Gamma-aminobutyric acid.
C. Dopamine. 60. At the examination in the clinic, the man
D. Serotonin. was diagnosed with acute radiation sickness.
E. Triptamin. Laboratory has established a sharp decrease in
serotonin in platelets. The metabolism of a D. Gout.
substance is a possible cause of this condition? E. Fenilketonuria.
A. 5-oxytriptophane.
B. Tyrosine. 65. What is the most probable diagnosis in a
C. Histidine. child of infancy, in which there is darkening of
D. Phenylalanine. sclera, mucous membranes, auricles, separated
E. Serine. urine darkens in the air, in the blood and urine
found homogensic acid?
61. A 20-year-old woman came to the doctor A. Alkaptonuria.
with complaints of general weight loss, loss of B. Albinism.
appetite, weakness, skin discoloration C. Hemolytic anemia.
resembling bronze tan. In addtition to D. Porphyria.
hyperpigmentation, examination in the hospital E. Cystinuria.
revealed bilateral adrenal tuberculosis. What
substance leads to skin hyperpigmentation, 66. A baby has coloured sclera and mucouses
when accumulated excessively? and her urine is darkening when exposed to air.
A. Melanin Homogentisic acid was found in urine and blood
B. Bilirubin samples. What is the reason for this condition?
C. Hemozoin A Alcaptonuria
D. Lipofuscin B. Albinism
E. Adrenochrome C. Galactosemia
D. Cystinuria
62. A person consumed methionine-poor E. Histidinemia
products for a long period of time. As a result
neural and endocrine disorders have developed. 67. Dark stains on baby’s diapers indicate
They may be caused by the disturbance of the homogentisic acid synthesis. What substance’s
following substance synthesis: metabolism is disturbed?
A. Adrenaline A. Tyrosine
B. Pyruvate B. Galactose
C. Thyronine C. Methionine
D. Fatty acids D. Cholesterol
E. Glucagon E. Tryptophan

63. A 12-year-old boy in the urine revealed high 68. Excretion of homogentisic acid with the
levels of all amino acids in the aliphatic row. In urine is the reason for the condition called
this case, the highest excretion of cysteine and alcaptonuria. Disturbance of which amino acid
cysteine was noted. In addition, ultrasound of metabolism causes this condition?
the kidneys showed the presence of stones in A. Tyrosine
them. Choose a possible pathology. B. Phenylalanine
A. Cystinuria. C. Alanine
B. Alpathururia. D. Methionine
C. Cystitis E. Asparagine
D. Phenylketonuria.
E. Hartnup's disease. 69. A 5-year old child has darkened urine but no
bile enzymes were found in it. The child is
64. Parents of the 3-year-old child drew diagnosed with alcaptonuria. Which enzyme
attention to the fact that the child's urine deficiency is the reason for it?
becomes dark when restrained. Objectively: the A. Homogentisic acid oxydase
body temperature is normal, the skin is pink, B. Phenylalanine hydroxylase
clean, the stove is not elevated. What disease is C. Tyrosinase
accompanied by such manifestations. D. Oxyphenylpyruvate oxydase
A. Alkaptonuria. E. Phenylpyruvate decarboxylase
B. Hemolytic anemia.
C. Itsenko-Cushing syndrome.
70. What kind of treatment is needed in a child 75. Excess of phenylpyruvate and phenylacetate
with phenylketonuria, in whose blood there is was detected in the urine sample of a 6 day-old
an increased amount of phenylpyruvic acid? baby. Which amino acid metabolism is
A. Diet therapy. disturbed?
B. Antibacterial therapy. A. Phenylalanine
C. Vitamin therapy. B. Tryptophan
D. Hormonotherapy. C. Methionine
E. Fermentotherapy. D. Histidine
E. Arginine
71. A 9-year-old child has a mental and physical
lag. In the biochemical analysis of blood, an 76. A 1.5 year-old child is mentally and
increased amount of phenylalanine has been physically retarded, hair and skin are whitened,
detected. Blocking of an enzyme can lead to this the level of catecholamines in the blood is
condition? decreased. After adding few drops of 5%
A. Phenylalanine-4-monooxygenase. solution of ferrum trichloracetate to the urine it
B. Aspartate aminotransferases. changes its colour to olive green. This
C. Glutamintransaminase. symptoms are typical for the following
D. Glutamate-carboxylase. condition:
E. Oxidazes of homogenetic acetic acid. A. Phenylketonuria
B. Alcaptonuria
72. In a sick child, a raised level of C. Tyrosinosis
phenylpyruvate was detected in the urine (in the D. Albinism
norm practically absent). The amount of E. Xanthinuria
phenylalanine in the blood is 350 mg/l (about 15
mg/l). Which disease is characterized by the 77. A child with mental and physical
above symptoms? development retardation was delivered to the
A. Fenilketonuria. hospital. Phenylpyruvate was found in the urine.
B. Albin. Disturbance of which process is the reason for
C. Alpathonuria. this pathology?
D. Gout A. Amino acid metabolism
E. Tyrosinosis. B. Lipid metabolism
C. Carbohydrate metabolism
73. A certain type of congenital pathology is D. Water-salt balance regulation
accompanied with the inhibition of tyrosine E. Calcium and phosphate homeostasis
formation from phenylalanine. Biochemical
indicator of this condition is depositing of the 78. A patient with complaints about intolerance
following acid in the organism: to solar radiation was contacted by a doctor.
A. Phenylpyruvate There is burn skin and visual impairment.
B. Citric acid Preliminary diagnosis of albinism. Disturbance
C. Pyruvate of the exchange of amino acids observed in this
D. Lactic acid patient?
E. Glutamine A. Tyrosine.
B. Proline
74. A 6 month-old baby has mental and physical C. Lizin.
development retardation, seizures, pale D. Alanin.
eczematous skin, light hair, blue eyes. What E. Tryptophan
substance presence in blood and urine can help
to make a diagnosis in this case? 79. Albinoses has increased sensitivity to
A. Phenylpyruvate sunlight therefore they get burns instead of tan.
B. Tryptophan Which amino acid metabolism disturbance is
C. Histidine the reason for the condition?
D. Leucine A. Phenylalanine
E. Valine B. Methionine
C. Tryptophan
D. Glutamine a specific smell of maple syrup. Which enzyme
E. Histidine deficiency is typical for this condition?
A. Amino acid dehydrogenase
80. The patient is 12 years old, he has a general B. Aminotranspherase
weakness, dizziness, fatigability, lag in mental C. Glucose-6-phosphatase
development. In the laboratory examination, D. Phosphofructokinase
high concentrations of valine, isoleucine, E. Phosphofructomutase
leucine in blood and urine were detected. Urine
specific odor. What disease does this happen? 85. A hospital admitted a patient with
A. Disease of maple syrup. complaints about abdominal swelling, diarrhea,
B. Basetic disease. meteorism after consumption of food rich in
C. Hystidominemia. proteins. It is indicative of disturbed protein
D. Tyrosinosis. digestion and their intensified decaying. What
E. Addison's Illness. substance is the product of this process in the
bowels?
81. A 13 year-old boy is complaining about A. Bilirubin
general weakness, dizziness, fatigue. Mental B. Indole
retardation is detected. The examination C. Cadaverine
revealed high concentrations of valine, D. Agmatine
isoleucine, and leucine in blood and urine. The E. Putrescine
urine has a specific smell. What is the most
possible diagnosis? 86. A 60 year-old man suffers from chronic
A. Maple syrup urine disease intestinal obstruction. The processes of protein
B. Addison’s disease decomposition in the large intestine are
C. Tyrosinosis intensified. What is the evidence of these
D. Histidinemia processes?
E. Grave’s disease A. Indicanuria
B. Bilirubinuria
82. The child's urine had a characteristic smell C. Hyperuricosuria
of maple syrup. In the laboratory study, high D. Creatinuria
levels of leucine, valine, isoleucine and their E. Glucosuria
keto-derivatives were found in blood and urine.
What kind of enzyme is inadequate for this 87. A patient has been operated due to acute
disease? abdomen. His urine is brown, concentration of
A. Dehydrogenases of branched-chain amino indican is higher than 93 mmol/day. What does
acids. it indicate?
B. Aminotransferases. A. High intensity of protein decomposition
C. Glucose-6-phosphatase. processes in the intestine
D. Phosphorutokinase. B. Decreased activity of the urea cycle enzymes
E. Phosphofructomatase. C. Acceleration of aromatic amino acids
desamination
83. A baby refuses breastfeeding, it is nervous, D. Disturbance of the kidney absorption
its respiration is arrhythmic and urine has a E. Decrease of ammonia detoxification
specific smell of maple syrup. What congenital
enzymopathy is the reason for this condition? 88. A 43 year-old woman has been operated due
A. Keto acid dehydrogenase to acute abdomen. Her urine is brown and the
B. Glucose-6-phospate dehydrogenase concentration of indican in the blood has
C. Glycerol kinase increased rapidly. What does it indicate?
D. Aspartate aminotranspherase A.High intensity of protein decomposition
E. UDP-glucoronosyltranspherase B. Acceleration of amino acids desamination
C. Supression of glomerular filtration
84. A lab test revealed increased concentration D. Decreased intensity of the urea cycle
of leucine, valine, isoleucine and ketone E. Inhibition of gluconeogenesis
derivatives in the blood and urine. The urine has
89. Indican is a byproduct of certain amino acid A. Tryptophan
decomposition in the large intestine. Its B. Valine
excretion with the urine is used to indicate the C. Glycine
functional condition of the liver. What is this D. Serine
amino acid? E. Cysteine
Ministry of Public Health of Ukraine

O. O. BOGOMOLETS NATIONAL MEDICAL UNIVERSITY

Department of Bioorganic and Biological Chemistry

LIST OF TEST QUESTIONS

for preparation of Content module № 3

" Biochemistry of tissues and physiological


functions "
FOR STUDENTS OF THE 2ST YEAR OF STUDY
OF MEDICAL and STOMATOLOGICAL FACULTIES

Kyiv-2019
1
Content
Fundamentals of Molecular Biology and Genetics ................................... 3

Biochemistry of intercellular communications:

hormones of protein-peptide nature .................................... 14

hormones of steroid and thyroid nature .............................. 22

Functional and clinical biochemistry of organs and tissues:

Biochemistry and pathobiochemistry of blood ......................................... 29

Biochemistry of immune system................................................................ 46

Biochemistry of liver .................................................................................. 51

Pathologic urine components .................................................................... 59

Biochemistry of nervous tissue .................................................................. 65

Biochemistry of muscle tissue ................................................................... 68

Biochemistry of connective tissue ............................................................. 71

Biochemistry of tooth and saliva (for stomat. faculty) ............................. 74

2
Fundamentals of Molecular Biology and Genetics
1. Gastroenterologists were assigned allopurinol, C. Adenine phosphoribosiltransferase
which inhibits the synthesis of uric acid by D. Hypoxanthine phosphoribosiltransferase
inactivating such an enzyme: E. Guanine deaminase
A. Xanthine oxidase
B. Deaminase 6. Allopurinol is a competitor of xanthine
C. Hyaluronidase
oxidase, who was prescribed for urolithiasis
D. Lactate dehydrogenase
E. Transaminase after the examination. The basis for this was the
chemical analysis of kidney stones, which
2. On the basis of laboratory analysis, the consisted mainly of:
patient was diagnosed with gout. Based on A. Urat sodium
laboratory analysis, the patient confirmed the B. Dihydrate calcium oxalate
diagnosis of gout. What was the analysis for the C. Monohydrate calcium oxalate
diagnosis? D. Phosphate calcium
A. Determination of uric acid in the blood and E. Calcium Sulphate
urine
B. Determination of urinary creatinine 7. A 48 year old patient complained about
C. Determination of residual nitrogen in the intense pain, slight swelling and reddening of
blood skin over the joints, temperature rise up to
D. The determination of urea in the blood and 38oC. Blood analysis revealed high
urine concentration of urates. This condition might be
E. Determination of urine ammonia caused by disturbed metabolism of:
A. Purines
3. A patient suffering from gout was prescribed B. Collagen
allopurinol.What pharmacological property of C. Cholesterol
allopurinol provides the rapeutic effect in this D. Pyrimidines
case? E. Carbohydrates
A. Competitive inhibition of xanthine oxidase
B. Acceleration of nitrogen-containing 8. A 46 year old patient applied to a doctor
substances excretion complaining about joint pain that becomes
C. Acceleration of pyrimidine nucleotides stronger the day before weather changes. Blood
catabolism examination revealed strengthened
D. Deceleration of pyrimidine nucleotides concentration of uric acid. The most probable
salvage cause of the disease is the intensified
E. Acceleration of nucleic acids synthesis disintegration of the following substance:
A. Adenosine monophosphate
4. Blood of a 12 year old boy presents low B. Cytidine monophosphate
concentration of uric acid and accumulation of C. Uridine triphosphate
xanthine and hypoxanthine. This child has D. Uridine monophosphate
genetic defect of the following enzyme: E. Thymidine monophosphate
A. Xanthine oxidase
B. Arginase 9. A patient has increased content of uric acid in
C. Urease his blood that is clinically presented by pain
D. Ornithine carbamoyltransferase syndrome as a result of urate deposition in the
E. Glycerylkinase joints. What process does this acid result from?
A. Lysis of purine nucleotides
5. A 42-year-old man suffering from gout has B. Lysis of pyrimidine nucleotides
increased level of urinary acid in blood. C. Heme catabolism
Allopurinol was prescribed to decrease the level D. Proteolysis
of urinary acid. Competitive inhibitor of what E. Reutilization of purine bases
enzyme is allopurinol?
A. Xanthine oxidase 10. A 65 year old man suffering from gout
B. Adenosine deaminase complains of kidney pain. Ultrasound
3
examination revealed renal calculi. The most the formation of tophi, urate calculi in the
probable cause of calculi formation urinary tracts, as well as serious neuro-
is the strengthened concentration of the psychiatric disorders. The cause of this disease
following substance: is the reduced activity of the following enzyme:
A. Uric acid A.Hypoxanthine-guanine
B. Cholesterol phosphoribosyltransferase
C. Bilirubin B. Xanthine oxidase
D. Urea C. Dihydrofolate reductase
E. Cystine D. Thymidylate synthase
E. Karbamoyl phosphate synthetase
11. In the synthesis of purine nucleotides, some
amino acids, indigenous vitamins, phosphoric 16. A 1,7-year-old child with a developmental
ribozymes are involved. Name the most delay and manifestations of self-agression has
important vitamin needed for purine nucleotide the concentration of uric acid in blood at the rate
synthesis de novo. of 1,96 millimole/l. What metabolic disoder is
A. Folic acid this typical for?
B. Pantothenic acid A. Lesch-Nyhan syndrome
C. Nicotinic acid B. Podagra
D. Ryboflavin C. Acquired immunodeficiency syndrome
E. Pyridoxine D. Gierke’s disease
E. Cushing’s basophilism
12. The patient has a hypovitaminosis of folate,
which may lead to a violation of the synthesis: 17. A newborn child gains weight very slowly,
A. Purin and Tymidyl nucleotides his urine contains too much orotic acid that is
B. Purine nucleotides and cholesterol indicative of disturbed synthesis of pyrimidine
C. Tymidyl nucleotides and fatty acids nucleotides. What metabolite should be used in
D. Heme and creatine order to normalize metabolism?
E. Citrate and ketone bodies A. Uridine
B. Adenosine
13. Purine ring biosynthesis occurs in ribose-5- C. Guanosine
phosphate through gradual accumulation of D. Thymidine
nitrogen and carbon atoms and closing of the E. Histidine
rings. The source of ribose phosphate is the
process of: 18. In orоtaciduria the release of Orotic acid is
A. Pentose phosphate cycle many times higher than normal. Synthesis of
B. Glycolysis what substances will be disturbed in this
C. Glyconeogenesis pathology?
D. Gluconeogenesis A. Pyrimidine nucleotides.
E. Glycogenolysis B. Biogenic Amines.
C. Purine nucleotides.
14. A 8 year old boy suffering from the Lesh- D. Urea
Nichan disease.He has increased content of uric E. Uric acid
acid in his blood. Indicate which violation of
the process is the cause of this hereditary 19. Pterin derivatives (aminopterin and
disease? methotrexate) are the inhibitors of dihydrofolate
A. Lysis of purine nucleotides reductase, so that they inhibit the regeneration
B. Synthesis of purine nucleotides of tetrahydrofolic acid from dihydrofolate.
C. Synthesis of pyrimidine nucleotides These drugs inhibit the intermolecular tranfer of
D. Lysis of pyrimidine nucleotides monocarbon groups, thus suppressing the
E. Formation of deoxyribonucleotides synthesis of the following polymer:
A. DNA
15. Children with Lesch-Nyhan syndrome have B. Protein
a severe form of hyperuricemia accompanied by C. Homopolysaccharides

4
D. Gangliosides D. Pyridoxalphosphate
E. Glycosaminoglycans E. Nicotinamidadenindynucleotide

20. Methotrexate (structural analogue of the 25. radiation and chemotherapy. The complex
folic acid which is competitive inhibitor of the of medicinal products included 5-fluoro-
dihydrofolatreductase) is prescribed for deoxyuridine - thymidylate synthetase inhibitor.
treatment of the malignant tumour. On which Synthesis of what substance is blocked by this
level does methotrexate hinder synthesis of the drug?
nucleic acids? A. DNA
A. Mononucleotide synthesis B. i-RNA
B. Replication C. p-RNA
C. Transcription D. t-RNA
D. Reparation E. -
E. Processing
26. According to the model of double DNA
21. Methotrexate (structural analogue of the helix that was suggested by Watson and Creek,
folic acid which is competitive inhibitor of the it was established that one of chains would not
dihydrofolatreductase) is prescribed for be lost during replication and the second chain
treatment of the malignant tumour. On which would be synthesized complementary to the first
level does methotrexate hinder synthesis of the one. What way of replication is it?
nucleic acids? A. Semiconservative
A. Mononucleotide synthesis B. Analogous
B. Replication C. Identical
C. Transcription D. Dispersed
D. Reparation E. Conservative
E. Processing
27. Among organic substances of a cell there is
22. An oncological patient was administered a polymer composed of dozens, hundreds, and
methotrexate. With the lapse of time the target thousands of monomers. This molecule is
cells of the tumour lost sensitivity to this capable of self-reproduction and can be an
preparation. We can observe changes in the information carrier. X-ray structure analysis
gene expression of the following enzyme: shows this molecule to consist of two
A. Dihydrofolate reductase complementary spiral threads. Name this
B. Thiminase compound:
C. Desaminase A. DNA
D. Folate oxidase B. RNA
E. Folate decarboxylase C. Cellulose
D. Carbohydrate
23. Leukoses are treated with antimetabolite E. Hormone
methotrexate. What vitamin is its antagonist?
A. Folic acid 28. Ability to divide is characteristic of
B. Cyanocobalamin procariotic and eukaryotic cells. Procariotic cell
C. Phyllochinone division is different from that of eukaryotic, but
D. Piridoxine there is one molecular process that is the basis
E. Rutin of both types of division. Name this process.
A. DNA replication
24. For the normal of the replication process B. Transcription
necessary Thimidyl nucleotides that are C. Reparation
synthesized by the enzyme Thymidylate D. Translation
synthase are , as coenzyme is used: E. Gene amplification
A. Methylenetetrahydrofolate
B. Carboxybiotin 29. Epithelium regeneration of mucous
C. Thiamindyfosphate membrane of oral cavity (cell reproduction) was

5
accompanied by semiconservative DNA 34. You are studying functioning of a bacteria
replication (selfreproduction). Nucleotides of a operon. The operator gene has been released
new DNA chain are complementary to: from the repressor gene. Immediately after this
A. Maternal chain the following process will start in the cell:
B. Sense codons A. Transcription
C. DNA-polymerase enzyme B. Translation
D. Introns C. Replication
E. RNA-polymerase enzyme D. Processing
E. Repression
30. During cell division DNA replication occurs
after a signal is received from the cytoplasm, 35. It was proved that a molecule of immature
then a certain portion of the DNA helix unwinds mRNA (precursor mRNA) contained more
and splits into two individual strains. What triplets than amino acids found in the
enzyme facilitates this process? synthesized protein. The reason for that is that
A. Helicase translation is normally preceded by:
B. RNA polymerase A. Processing
C. Ligase B. Initiation
D. Restrictase C. Reparation
E. DNA polymerase D. Mutation
E. Replication
31. A group of researchers set an experiment
and obtained anucleatemutant cells. In the first 36. Nowadays about 50 minor bases have been
place they will have disturbed synthesis of the found in the t-RNA structure besides the main
following compounds: four nitrogenous bases. Choose the minor
A. Ribosomal RNA nitrogenous base:
B. Transfer RNA A. Dihydrouracil
C. Lipids B. Uracil
D. Monosaccharides C. Cysteine
E. Polysaccharides D. Adenine
E. Cytosine
32. Nucleolus organizers of human
chromosomes 13-15, 21, 22 include about 200 37. Inside a human cell the informational RNA
gene clusters that synthesize RNA. These containing both exons and introns was delivered
chromosomal regions contain the information to the granular endoplasmic reticulum to the
on the following type of RNA: ribosomes. What process does NOT take place?
A. rRNA A. Processing
B. tRNA B. Replication
C. mRNA C. Transcription
D. snRNA D. Translation
E. tRNA + rRNA E. Prolongation

33. Amino acids join to each other in ribosomes 38. Synthesis of i-RNA passes on the DNA matrix,
of granular endoplasmic reticulum. Knowing taking into account the principle of
the sequence of amino acids and applying complementarity. If the triplets in the DNA are the
genetic code, it is possible to determine the following - ATG-CGT, then the corresponding i-
RNA codons will be:
sequence of nucleoids in:
A. UAC-GCA
A. mRNA B. AUG-CGU
B. Introns C. ATG-CGT
C. Proteins D. UAG-CGU
D. Carbohydrates E. TAG-UGU
E. rRNA
39. General structure of eukaryotic genes is as
follows: exon-intron-exon. Such functional

6
structure of a gene leads to certain specifics of E. Regulator
the transcription process. What sequence will
correspond with precursor mRNA (immature)? 44. A patient with pulmonary tuberculosis is
A. Exon-intron-exon prescribed the most effective antituberculous
B. Exon-exon-intron antibiotic. Name this drug:
C. Exon-exon A. Rifampicin
D. Intron-exon B. Tetracycline
E. Exon-intron C. Streptocide
D. Furasolidone
40. In the nucleus of eukaryote’s cells the first E. Bactrim (Co-trimoxazole)
pro-RNA molecule is synthesized that is
complementary to the exons and introns of the 45. Tuberculosis can be treated by means of
structural gene. But the ribosome receives such combined chemotherapy that includes
i-RNA, which is complementary to exons only. substances with different mechanisms of
This indicates that the nucleus takes place: action.What antituberculous medication inhibits
A. Processing transcription of RNA into DNA in
B. Transcription mycobacteria?
C. Reparation A. Rifampicin
D. Replication B. Isoniazid
E. Reverse transcription C. Streptomycin
D. Ethionamide
41. As a result of intoxication in the epithelial E. Para-aminosalicylic acid
cell of the mucous membrane of the oral cavity,
enzymes that provide splicing are not 46. RNA that contains AIDS virus penetrated
synthesized. What is the reason for the into a leukocyte and by means of reverse
termination of protein biosynthesis in this case? transcriptase forced a cell to synthetize a viral
A. The i-RNA is not synthesised DNA. This process is based upon:
B. ATP is not synthesised A. Reverse transcription
C. The p-RNA is not synthesised B. Operon repression
D. Amino acids are not activated C. Reverse translation
E. Transport of amino acids has been disturbed D. Operon depression
E. Convariant replication
42. During reproduction of some RNA-
containing viruses that cause tumors in animals, 47. T-lymphocytes are determined to be affected
genetic information can be transmitted in the with HIV. In this case viral enzyme reverse
opposite direction from the RNA to the DNA transcriptase (RNA-dependent DNA-
via a specific enzyme. The enzyme of reverse polymerase) catalyzes the synthesis of:
transcription is called: A. DNA based on the viral RNA matrix
A. Reverse transcriptase B. Viral RNA based on the DNA matrix
B. DNA polymerase C. Viral protein based on the viral RNA matrix
C. Ligase D. Viral DNA based on the DNA matrix
D. Primase E. Informational RNA based on the viral protein
E. Topoisomerase matrix

43. In the body of a person transcription occurs. 48. For the treatment of urogenital infections
The RNA polymerase, moving along the DNA use hinolones - inhibitors of the enzyme DNA
molecule, has reached a certain sequence of gyrase. What process is disrupted by hinolones
nucleotides. After this transcription stopped. in the first place?
This section of the DNA is called: A. Replication of DNA
A. Terminator B. Reparation of DNA
B. Promoter C. Amplification of the genes
C. Repressor D. Recombination of the genes
D. Operator E. Reverse transcription

7
by different quantity of triplets ranging from
49. It was found out that some compounds, for one to six. Name this property of genetic code:
instance fungi toxins and some antibiotics can A. Degeneracy
inhibit activity of RNA-polymerase. What B. Universality
process will be disturbed in a cell in case of C. Disjointness
inhibition of this enzyme? D. Triplety
A. Transcription E. Specificity
B. Processing
C. Replication 54. Formation of ribosome subunits in a cell
D. Translation was disturbed in course of an experiment
E. Reparation (bymeans of . activated mutagenic factors). This
will have an effect on the following metabolic
50. At the stage of translation in the rough process:
endoplasmic reticulum, the ribosome moves A. Protein biosynthesis
along the mRNA. Amino acids are joined B. Carbohydrate biosynthesis
together by peptide bonds in a specific C. ATP synthesis
sequence, and thus polypeptide synthesis takes D. Photosynthesis
place. The sequence of amino acids in a E. Biological oxidation
polypeptide corresponds to the sequence of:
A. mRNA codons 55. Amino acids join to each other in ribosomes
B. tRNA nucleotides of granular endoplasmic reticulum. Knowing
C. tRNA anticodons the sequence of amino acids and applying
D. rRNA nucleotides genetic code, it is possible to determine the
E. rRNA anticodons sequence of nucleoids in:
A. mRNA
51. Students studying the features of the genetic B. Introns
code found that there are amino acids that are C. Proteins
answered by 6 codons, 5 amino acids - 4 D. Carbohydrates
different codons. Other amino acids are encoded E. rRNA
by three or two codons and only two amino
acids by one codon. What feature of the genetic 56. A tissue sample of benign tumor was studied
code did students discover? under the electron microscope. A lot of small
A. Degeneracy (Redundancy) (15-20 nm) spherical bodies, consisting of 2
B. Universality unequal subunits were
C. Collinearity detected. These are:
D. Unidirectional A. Ribosomes
E. Compose of three nucleotide B. Golgi complex
C. Smooth endoplasmic reticulum
52. The mutation of the structural gene did not D. Microtubules
lead to the substitution of amino acids in protein E. Mitochondria
molecules. This revealed the following property
of the genetic code: 57. A 36-year-old patient under went tooth
A. Degeneracy (Redundancy) extraction at a dental clinic. After two weeks the
B. Mutable stratified squamous epithelium regenerate dat
C. Collinearity the site of extraction.What organelles were
D. Insufficiency involved in there storation of the mucous
E. Universality membrane?
A. Ribosomes
53. It is known that information about amino B. Centrosomes
acid sequence in a protein molecule is stored as C. Postlysosomes
a sequence of four nucleotide types in a DNA D. Smooth EPR
molecule, and different amino acids are encoded E. Mitochondria

8
58. Genetic information is stored in DNA but
does not participate directly in protein synthesis 63. One of the protein synthesis stages is
with in DNA cells. What process ensures recognition. The first iRNA triplet starts with
transfer of genetic information into polypeptide UAU triplet. What complementary triplet is
chain? found in tRNA?
A. Translation A. AUA
B. FormationofrRNA B. AAA
C. FormationoftRNA C. GUG
D. FormationofiRNA D. UGU
E. Replication E. CUC

59. A patient has decreased concentration of 64. Cytological studies revealed a large number
magnesium ions that are required for ribosomes of different t-RNA molecules that deliver amino
connection to granular endoplasmic reticulum. acids to the ribosome. The number of different
This condition is known to disrupt the process types of t-RNA in a cell will be equal to the
of protein biosynthesis. Disruption occurs at the number of:
following stage: A. Triplets encoding amino acids
A. Translation B. Nucleotides
B. Transcription C. Amino acid
C. Replication D. Proteins synthesised in the cell
D. Amino acids activation E. Different types of RNA
E. Processing
65. Labelled amino acids alanine and
60. For the formation of a transport form of tryptophane were injected to a mouse in order to
amino acids for the synthesis of a protein, it is study localization of protein synthesis in its
necessary: cells. The labelled amino acids will be
A. Aminoacyl-tRNA synthetase accumulated near the following organellas:
B. GTP A. Ribosomes
C. m-RNA B. Smooth endoplasmic reticulum
D. Ribosome C. Cell centre
E. Revertase D. Lysosomes
E. Golgi apparatus
61. During the study of cells, a high content of
the aminoacyl-tRNA synthetase enzyme was 66. The patient was prescribed antibiotic
established in their cytoplasm. This enzyme chloramphenicol (levometsitin), which disrupts
provides the following process in a cell: protein synthesis in the body by inhibiting the
A. Activation of amino acids process:
B. Reparation A. Translation elongation
C. Elongation B. Formation of polyribosomes
D. Transcription C. Transcription
E. Replication D. Processing
E. Gene amplification
62. In the cell there is a process of translation.
When the ribosome reaches the codons of UAA, 67. Infectious diseases are treated with
UAG, or UGA, the polypeptide chain synthesis antibiotics (streptomycin, erythromycin,
is terminated. These codons are not recognized chloramphenicol). They inhibit the following
by any t-RNA in the biosynthesis of the stage of protein synthesis:
polypeptide and this is a signal: A. Translation
A. Termination B. Transcription
B. Post-translational modification C. Replication
C. Begin transcription D. Processing
D. Elongation E. Splicing
E. Initiation

9
68. A 28-year-old patient with bacterial E.coli. What portion of the lactose operon will
pneumonia was prescribed a course of treatment be unlocked from the repressor in these
with erythromycin. Its antibacterial properties conditions?
are known to be due to the ability of this A. Operator
substance to combine with the free 50S-subunit B. Promoter
of the ribosome. What substances synthesis does C. Structural gene
this antibiotic block in bacterial cells? D. Regulatory gene
A. Proteins E. Primer
B. RNA
C. DNA 73. It is known that the gene responsible for the
D. Fat development of the MN blood groups has two
E. Polysaccharides allelic states. If the gene M is considered as the
initial gene, the allelic gene N appeared due to:
69. Streptomycin and other aminoglycosides A. Mutations
prevent the joining of formylmethionyl tRNA B. Gene combinations
by bonding with the 30S ribosomal subunit. C. DNA repair
This effect leads to disruption of the following D. DNA replication
process: E. Crossing over
A. Translation initiation in procaryotes
B. Translation initiation in eucaryotes 74. As a result of treatment of viral RNA with
C. Transcription initiation in procaryotes nitrous acid, UCA triplet mutated to UGA
D. Transcription initiation in eucaryotes triplet. What kind of mutation occurred?
E. Replication initiation in procaryotes A. Transition
B. Nucleotide deletion
70. In a genetical laboratory in course of work C. Missense
with DNA molecules of white rats of Wistar’s D. Nucleotide insertion
line a nucleotide was substituted for another E. Inversion
one. At that only one amino acid was
substituded in the peptide. This 75. A mutation has occurred in a cell in the first
result is caused by the following mutation: exon of the structural gene. The number of
A. Transversion nucleotide pairs changed from 290 to 250.
B. Deletion Name this type of mutation:
C. Duplication A. Deletion
D. Displacement of reading frame B. Inversion
E. Translocation C. Duplication
D. Translocation
71. Hurtnup’s disease is caused by point E. Nullisomy
mutation of only one gene. This results in
abnormal absorption of tryptophane in the 76. When examining a 2-month-old child, the
intestine as well as its abnormal pediatrician noticed that the crying of the child
reabsorption in renal tubules. This causes resembles a cat's cry. Diagnosed microcephaly
synchronous disorders in digestive and urinary and heart disease. Using the cytogenetic
excretion systems. What genetic phenomenon is method, the child's cartiotype: 46, XX, 5p. This
observed in this case? disease is a consequence of this process:
A. Pleiotropy A. Deletion
B. Complementary interaction B. Duplication
C. Polymery C. Inversion
D. Codominance D. Translocation
E. Semidominance E. Pleiotropy

72. During the experiment, an increase in β- 77. Parents of a sick 5-year-old girl visited a
galactosidase activity was demonstrated after genetic consultation. Karyotype investigation
lactose was added to the culture medium with revealed 46 chromosomes. One chromosome of

10
the 15th pair was abnormally long, having a part D. Translocation
of the chromosome belonging to the21st pair E. Replication
attached to it. What mutation occurred in this
girl? 82. An experiment proved that UV irradiated
A. Translocation skin cells of patients with xeroderma
B. Deletion pigmentosum restore the native structure of
C. Inversion DNA slower than the cells of healthy people
D. Deficiency due to the defect in repair enzyme. What
E. Duplication enzyme takes part in this process?
A. Endonuclease
78. In some regions of South Africa there is a B. RNA ligase
spread sickle-shaped cell anemia, in which C. Primase
erythrocytes have shape of a sickle as a result of D. DNA polymerase
substitution of glutamin by valine in the E. DNA gyrase
hemoglobin molecule. What is the cause of this
disease? 83. In the course of evolution there developed
A. Gene mutation molecular mechanisms for correction of
B. Disturbance of mechanisms of genetic damaged DNA molecules. This process is
information realization called:
C. Crossingover A. Reparation
D. Genomic mutations B. Transcription
E. Transduction C. Translation
D. Replication
79. Sickle-shaped e cell anemia in humans is E. Processing
accompanied by the appearance of abnormal
hemoglobin in the blood, a change in the form 84. In the human cells under the action of
of red blood cells and the development of ultraviolet radiation the DNA molecule has been
anemia. This disease is the result of: damaged. The system of repairing the damaged
A. Gene mutations area of the DNA molecule to the intact chain
B. Polythene through a specific enzyme was revised. What is
C. Chromosomal aberration the name of this phenomenon?
D. Polyplody A. Reparation
E. Mitochondrial mutation B. Duplication
C. Replication
80. A 15-year-old man is complaining of D. Initiation
general weakness, dizziness, rapid fatigability. E. Termination
During the examination, erythrocytes of a
modified form were detected, their number was 85. Cells of a person working in the Chornobyl
reduced. Preliminary diagnosis: sickle cell Exclusion Zone have under gone a mutation in
anemia. What type of mutation causes the DNA molecule. However, with time the
development of this pathological condition? damaged interval of DNA molecule has been
A. Point mutation restored to its initial structure with a specific
B. Reading frame shift mutation enzyme. In this case the following occurred:
C. Deletion A. Repair
D. Inversion B. Replication
E. Chromosomal aberration C. Transcription
D. Reverse transcription
81. Part of the DNA chain turned 180 degree as E. Translation
a result of gamma radiation. What type of
mutation took place in the DNA chain? 86. Under the influence of physical factors there
A. Inversion can develop defect sin a DNA molecule.
B. Deletion Ultraviolet irradiation, for instance, can cause
C. Doubling development of dimers. Dimers are two adjacent

11
pyrimidine bases joined together. Name these 91. Fetal malformations may result from such
bases: maternal diseases as rubella, syphilis,
A. Thymine and cytosine toxoplasmosis, cytomegaly, herpes, chlamydia.
B. Adenine and thymine What is the form of variability relating to such
C. Guanine and cytosine malformations?
D. Adenine and guanine A. Modifiation
E. Guanine and thymine B. Mutational
C. Combinative
87. Patients suffering from xeroderma D. Genomic imprinting
pigmentosum have extremely photosensitive E. Epimutational
skin due to disrupted excision repair. Specify
the process that is affected in such patients: 92. As a result of iodine deficiency in
A. Repair of DNA molecule foodstuffs Transcarpathian people often have
B. Synthesis of iRNA endemic goiter. This disease is caused by the
C. Maturation of iRNA following type of variability:
D. Synthesis of protein primary structure A. Modification
E. Intron extraction and exon connection B. Mutational
Test items for licensing examination Krok 1 C. Combinative
«Stomatology». – 2016. - № 192. D. Ontogenetical
88. An employee of a chemical enterprise was E. Correlative
exposed to nitric acid and nitrite, which cause
cytosine deamination in the DNA molecule. 93. Lymphocyte is affected by HIV retrovirus
What enzyme initiates a chain of reparation (AIDS). In this case, the direction of
processes? information flow in the cell will be:
A. Uridine DNA glycosidase A. RNA → DNA → i-RNA → polypeptide
B. Citidine triphosphate synthetase B. DNA → i-RNA → polypeptide → DNA
C. Orotidyl monophosphate decarboxylase C. DNA → Polypeptide → i-RNA
D. DNA-dependent RNA polymerase D. i-RNA → polypeptide → DNA
E. Thymidylat synthase E. Polypeptide → RNA → DNA → i-RNA

89. Tetracycline taking in the first half of 94. A young family came for a genetic
pregnancy causes abnormalities of fetus organs counseling to identify the father of their child.
and systems, including tooth hypoplasia and The husband insists that the child does not
alteration of their colour. What type of resemble him at all and cannot possibly be his.
variability is the child’s disease related to? Polymerase chain reaction method
A. Modification for person identification is based on the
B. Combinative following:
C. Mutational A. Gene amplification
D. Hereditary B. Nucleotide deletion
E. Recombinan C. Genetic recombination
D. Missense mutation
90. The woman took antibiotics in the first half E. Transduction
of pregnancy. This led to the hypoplasia of the
teeth and the change in their color in the child. 95. A doctor was addressed by a 30-year old
The genotype has not changed. Set the type of man. There is a probability of the patient being
variability that underlies the disease: HIV-positive. To clarify the diagnosis the
A. Modification doctor proposed to perform polymerase chain
B. Combinative reaction. The basic process in this kind
C. Mutational of investigation is:
D. Correlative A. Gene amplification
E. Recombinative B. Transcription
C. Genetic recombination
D. Genomic mutation

12
E. Chromosome mutation A. Phenocopy
B. Mutation
96. It is known that the gene responsible for C. Recombination
development of blood groups according to AB0 D. Heterosis
system has three allele variants. Existence of the E. Replication
IV blood group can be explained by the
following variability form: 98. The doctor discovered a kid's disease due to
A. Combinative a lack of vitamin D, but in his manifestation
B. Mutational similar to hereditary vitamin-resistant rickets
C. Phenotypic (curvature of tubular bones, deformity of joints
D. Genocopy of lower extremities, dental abscesses). How
E. Phenocopy called defects of development, which resemble
hereditary, but not inherited?
97. A mother had taken synthetic hormones A. Phenocopy
during pregnancy. Her daughter was born with B. Genocopy
hirsutismformally resembling of adrenal C. Monosomies
syndrome. Such D. Trisomy
manifestation of variability is called: E. Gene diseases

13
Biochemistry of intercellular communications: hormones of protein-
peptide nature
1. Degeneration of glycogen in liver is B. Triamcinolone
stimulated by glucagon. What secondary C. Norepinephrine
messenger (mediator) is thus formed in the cell? D. Prenisolone
A. c-AMP E. Hydrocortisone
B. c-GMP
C. CO 6. Condition of a patient with diabetes mellitus
D. NO sharply deteriorated after a regular injection of
E. Triacylglycerol insulin. The patient became anxious and broke
out in cold sweat; tremor of the extremities,
2. The ions of metals take part in the regulation general weakness, and dizziness appeared. What
of physiological functions. One of them was medicine can remove these symptoms?
named the “king of messengers”. Such a A. Adrenaline
bioelement mediator is: B. Tolbutamide
A. Ca ++ C. Caffeine
B. Na + D. Noradrenaline
C. K + E. Glibutid(Buformin)
D. Fe +++
E. Zn ++ 7. Dentists widely apply local anaesthesia
adding adrenalin to an anaesthetic solution.
3. Tissue inosytol triphosphates are generated as What is the purpose of this method?
a result of the phosphatidyl inositol diphosphate A. Local vasoconstriction
hydrolysis and act as secondary agents B. Local vasodilatation
(mediators) in the mechanism of hormone C. Lowering of arterial pressure
action. Their effect in cells is directed at: D. Local reduction of vascular resistance
A. Calcium ion liberation from cellular depot E. Microcirculation improvement
B. Adenylate cyclase activation
C. Protein kinase A activation 8. A 50-year-old man declined anaesthesia
D. Phosphodiesterase inhibition during dental manipulations. Due to severe pain
E. Protein kinase A inhibition he developed anuria caused by acute increase in
production of:
4. The formation of a secondary mediatoris A. Adrenaline
obligatory in membrane-intracellular B. Renin
mechanism of hormone action. Point out the C. Thymosin
substance that is unable to be a secondary D. Thyroxin
mediator: E. Glucagon
A. Glycerol
B. Diacylglycerol 9. A sick woman after parenteral administration
C. Inositol-3,4,5-triphosphate of the hormone had an increase in blood
D. CAMP pressure, as well as increased levels of glucose
E. Ca2+ and lipids in the blood. What hormone was
introduced?
5. A patient with a diagnosis of diabetes, in the A. Adrenaline
morning on an empty stomach received a B. Glucagon
prescribed dose of insulin with prolonged C. Insulin
action. He missed another meal and soon felt D. Progesterone
weakness, pain, dizziness, sweating, trembling E. Folliculin
of the body, convulsions, feeling of hunger,
hypoglycemia. The use of glucose did not 10. A patient suffering from pheochromocytoma
alleviate the condition. What hormonal drug is complains of thirst, dry mouth, hunger. Blood
necessary to enter to stop this condition? test for sugar revealed hyperglycemia. What
A. Adrenaline type of hyperglycemia is it?
14
A. Adrenal
B. Hypercorticoid 15. Before the cells can utilize the glucoze, it is
C. Alimentary first transported from the extracellular space
D. Somatotropic through the plasmatic membrane inside theml.
E. Hypoinsulinemic This process is stimulated by the following
hormone:
11. Patient with diabetes mellitus experienced A. Insulin
loss of consciousness and convulsions after an B. Glucagon
injection of insulin. What might be the result of C. Thyroxin
biochemical blood analysis for concentration of D. Aldosterone
sugar? E. Adrenalin
A. 1,5 mmol/L
B. 8,0 mmol/L 16. After insulin injection, glucose levels fall
C. 10,0 mmol/L within a few seconds. This is due to the
D. 3,3 mmol/L activation of this process: A. Transport of
E. 5,5 mmol/L glucose into cells
B. Glycolysis
12. A patient with diabetes mellitus had an C. Synthesis of glycogen
insuline injection. It caused loss of D. Synthesis of lipids
consciousness and convulsions. What was the E. Pentose phosphate cycle
result of biochemic blood analysis on glucose
content? 17. Prior to glucose utilization in cells it is
A. 2,5 mmоle/l transported inside cells from extracellular space
B. 3,3 mmоle/l through plasmatic membrane. This process is
C. 8,0 mmоle/l stimulated by the following hormone:
D. 10 mmоle/l A. Insulin
E. 5,5 mmоle/l B. Glucagon
C. Thyroxin
13. A 16-year-old young man was taken to D. Aldosterone
hospital, the patient had insulin-dependent E. Adrenalin
diabetes mellitus. The patient's blood glucose
level was 1.8 mmol/l. The patient was given 18. A patient is in the state of hypoglycemic
insulin. Two hours later, the glucose level coma. What hormone can cause this condition if
decreased to 8.2 mmol/l, since insulin: overdosed?
A. Stimulates the conversion of glucose to A. Insulin
glycogen and TAG in the liver B. Progesterone
B. Stimulates glucose transport through plasma C. Cortisol
membranes in the brain and liver D. Somatotropin
C. It inhibits the synthesis of ketone bodies from E. Corticotropin
glucose
D. Stimulates the breakdown of glycogen in the 19. A patient during fasting developed
liver E. Stimulates the breakdown of glycogen ketoacidosis as a result of increased fatty acids
in the muscles decomposition. This decomposition can be
inhibited with:
14. A patient with insulin-dependent diabetes A. Insulin
had an insulin injection. Some time later he felt B. Glucagon
weakness, irritability, excessive sweating. What C. Adrenaline
is the main reason of these disorders? D. Thyroxin
A. Carbohydrate starvation of brain E. Cortisol
B. Intensified glycogenolysis
C. Intensified ketogenesis 20. Examination of a patient revealed
D. Intensified lypogenesis overgrowth of facial bones and soft tissues,
E. Reduced glyconeogenesis tongue enlargement, wide interdental spaces in

15
the enlarged dental arch. What changes of the 25. A student who passes an exam has a plasma
hormonal secretion are the most likely? glucose content of 8 mmol/l. Increased secretion
A. Hypersecretion of the somatotropic hormone which of hormones contributes to the
B. Hyposecretion of the somatotropic hormone development of hyperglycemia?
C. Hypersecretion of insulin A. Glucagon
D. Hyposecretion of thyroxin B. Insulin
E. Hyposecretion of insulin C. Thyroxine
D. Triiodothyronine
21. A 49-year-old patient was found to have a E. Aldosterone
disproportionate enlargement of hands, feet,
nose, ears, superciliary arches and cheek bones. 26. A patient with tress and painful sensation
Blood test revealed hyperglycemia, impaired before a visit to the dentist is accompanied by
glucose tolerance. What is the most likely cause anuria (lack of urination). This phenomenon is
of this pathology development? due to an increase in:
A. Hypersecretion of growth hormone A. Secretion of vasopressin and adrenaline
B. Posterior pituitary hormone hypersecretion B. Activity of the parasympathetic nervous
C. Insulin hyposecretion system
D. Vasopressin hyposecretion C. Activity antinociceptive system
E. Glucocorticoid hypersecretion D. Secretion of vasopressin and a decrease in
adrenaline
22. A woman after labor lost 20 kg of body E. Secretion of adrenaline and a decrease in
weight, her hair and teeth fall out, she has vasopressin
muscle atrophy (hypophysial cachexia).
Synthesis of what hypophysis hormone is 27. As a result of a home injury, a patient
disturbed? suffered a significant blood loss, which led to a
A. Somatotropic fall in blood pressure. Rapid blood pressure
B. Corticotrophic recovery after the blood loss is provided by the
C. Thyreotropic following hormones:
D. Gonadotropic A. Adrenaline, vasopressin
E. Prolactin B. Cortisol
C. Sex hormones
23. Adult height 100 cm with a proportional D. Oxytocin
body structure and normal mental development. E. Aldosterone
It might be caused by deficiency secretion of the
following hormone: 28. A severe injury in a 36-year-old patient
A. Somatotropic resulted in a significant blood loss which was
B. Gonadotropic accompanied by a blood pressure drop. What
C. Adrenocorticotropic hormones provide rapid recovery of blood
D. Thyroid-stimulating pressure after the blood loss?
E. Prolactin A. Adrenalin, vasopressin
B. Cortisol
24. Examination of a patient revealed C. Sex hormones
enlargement of some body parts (jaw, nose, D. Oxytocin
ears, feet, hands), but body proportions were E. Aldosterone
conserved. It might be caused by intensified
secretion of the following hormone: 29. A man has the increased volume of
A. Somatotropin circulating blood and reduced osmotic pressure
B. Somatostatin of plasma considerable. He has decrease in
C. Tetraiodothyronine diuresis. The primary cause of such diuresis
D. Triiodothyronine disorder is the hypersecretion of the following
E. Cortisol hormone:
A. Vasopressin
B. Aldosterone

16
C. Adrenalin C. Vasopressin
D. Renin D. Thyrocalcitonin
E. Natriuretic E. Glucagon

30. A 20 year old patient complains of excessive 35. A 25-year-old woman one month after
thirst and urinary excretion upto 10 L a day. The giving birth consulted a doctor about a decrease
level of glucose in blood is normal, there is no in milk production. What hormone deficiency
glucose in urine. What hormone deficit can led to this condition?
cause such changes? A. Prolactin
A. Vasopressin B. Somatostatin
B. Oxytocin C. Adrenocorticotropic hormone
C. Insulin D. Insulin
D. Triiodothyronine E. Glucagon
E. Cortisol
36. Products of some proteins hydrolysis and
31. A patient with pituitary tumor complains of modification are the biologically active
increased daily diuresis (polyuria). Glucose substances called hormones. Lipotropin,
concentration in blood plasma equals 4,8 corticotropin, melanotropin and endorphins are
mmol/l. What hormone can be the cause of this synthesized in the hypophysis of the following
if its secretion is disturbed? protein:
A. Vasopressin A. Proopiomelanocortin (POMC)
B. Aldosterone B. Neuroalbumin
C. Natriuretic hormone C. Neurostromin
D. Insulin D. Neuroglobulin
E. Angiotensin I E. Thyreoglobulin

32. Limiting water intake has led to dehydration 37. After a case of sepsis a 27-year-old woman
of the organism. What mechanism is activated developed ”bronzed” skin discoloration
under these conditions to preserve water in the characteristic of Addison’s disease.
body? Hyperpigmentation mechanismin this case is
A. Increased vasopressin secretion based on increased secretion of:
B. Increased somatostatin secretion A. Melanocyte-stimulating hormone
C. Reduced vasopressin secretion B. Somatotropin
D. Increased aldosterone secretion C. Gonadotropin
E. Reduced aldosterone secretion D. β-lipotropin
E. Thyroid-stimulating hormone
33. At ria of a test animal were super distended
with blood,which resulted in decreased 38. A 20-year-old woman came to the doctor
reabsorption of Na+ and water in renal with complaints of general weight loss, loss of
tubules.This can be explained by the effect of appetite, weakness, skin discoloration
the following factor on the kidneys: resembling bronze tan. In addtition to
A. Natriuretic hormone hyperpigmentation, examination in the hospital
B. Aldosterone revealed bilateral adrenal tuberculosis. What
C. Renin substance leads to skin hyperpigmentation,
D. Angiotensin when accumulated excessively?
E. Vasopressin A. Melanin
B. Bilirubin
34. A 32-year-old patient consulted a doctor C. Hemozoin
about the absence of lactation after parturition. D. Lipofuscin
Such disorder might be explained by the deficit E. Adrenochrome
of the following hormone:
A. Prolactin 39. A patient with gastric juice hypersecretion
B. Somatotropin has been recommended to exclude from the diet

17
rich broths and vegetable infused water. A
doctor recommended it, because these food 44. A 40-year-old patient complains of intensive
products stimulate production of the following heartbeats, sweating, nausea, visual impairment,
hormone: arm tremor, hypertension. From his anamnesis:
A. Production of gastrin 2 years ago he was diagnosed with
B. Taste Receptors pheochromocytoma. Hyperproduction of what
C. Mechano-receptors of the oral cavity hormones causes the given pathology?
D. Mechanoreceptors of the stomach A. Catecholamines
E. Formation of secretin B. Aldosterone
C. Glucocorticoids
40. During an acute experiment some of diluted D. ACTH
solution of hydrochloric acid was injected into E. Thyroidal hormones
the duodenal cavity of an experimental animal.
This will result in hypersecretion of the 45. A special diet has led to a decrease in Ca2+
following hormone: ions in the blood. What hormone will increase
A. Secretin the secretion?
B. Gastrin A. Parathormone
C. Motilin B. Thyrocalcitonin
D. Neurotensin C. Vasopressin
E. Histamine D. Somatotropin
E. Thyroxine
41. A pregnant woman with a low labor activity
entered the maternity ward. Assign a hormonal 46. A patient has the sudden decrease of Ca2+
remedy to enhance labor activity: content in blood. What hormone secretion will
A. Oxytocin increase?
B. Progesterone A. Parathormone
C. Methandrostenolone B. Thyrocalcitonin
D. Hydrocortisone C. Aldosterone
E. ACTH D. Vasopressin
E. Somatotropin
42. A 26-year-old woman at 40 weeks pregnant
has been delivered to the maternity ward. 47. The calcium ions in the blood plasma of a
Objectively: the uterine cervix is opened, but patient is increased, reduced - in the bones. The
the contractions are absent. The doctor has excessive secretion of which hormone may
administered her a hormonal drug to stimulate cause such changes?
the labor. Name this drug: A. Parathormone
A. Oxytocin B. Thyroxine
B. Hydrocortisone C. Triiodothyronine
C. Estrone D. Thyreocalcitonin
D. Testosterone E. Aldosteron
E. ACTH
48. Kidneys of a man under examination show
43. To stimulate the laboractivity aparturient increased resorbtion of calcium ions and
woman was prescribed a drug a posterior decreased resorbtion of phosphate ions. What
pituitary hormone that does not affect the blood hormone causes this phenomenon?
pressure. As the pregnancy progresses, the A. Parathormone
sensitivity to this hormone increases.Name the B. Thyrocalcitonin
prescribed drug: C. Hormonal form D3
A. Oxytocin D. Aldosterone
B. Dinoprostone E. Vasopressin
C. Dinoprost
D. Pituitrin 49. Following thyroid surgery, a 47-year old
E. Ergotal female patient had fibrillary twitching of

18
muscles in the arms, legs and face. These
disorders can be treated by the introduction of 54. A patient has hypocalcemia. What hormone
the following hormone: deficiency may be it’s cause?
A. Parathyroid hormone A. Parathormone
B. Triiodothyronine B. Thyrocalcitonin
C. Thyrotropin C. Aldosterone
D. Thyroxine D. Corticotropin
E. Thyroid-stimulating hormone E. Corticoliberin

50. During removal of the hyperplastic thyroid 55. A patient presents with osteoporosis;
gland of a 47-year-old woman, the parathyroid hypercalcemia and hypophosphatemia are
gland was damaged.One month after the surgery observed in the patient’s blood. What is the
the patient developed signs of cause of this condition?
hypoparathyroidism: frequent convulsions, A. Increased parathormone secretion
hyperreflexia, laryngospasm. What is the most B. Increased thyroxin secretion
likely cause of the patient’s condition? C. Inhibited parathormone secretion
A. Hypocalcemia D. Increased corticosteroid secretion
B. Hyponatremia E. Inhibited corticosteroid secretion
C. Hyperchlorhydria
D. Hypophosphatemia 56. A 5-month-old boy was hospitalized for
E. Hyperkalemia tonic convulsions. He has a lifetime history of
this disease. Examination revealed coarse hair,
51. A child has abnormal formation of tooth thinned and fragilenails, pale and dry skin. In
enamel and dentin as a result of low blood: calcium - 1,5 millimole/l, phosphor - 1,9
concentration of calcium ions in blood. Such millimole/l. These changes are associated with:
abnormalities might be caused by deficiency of A. Hypoparathyroidism
the following hormone: B. Hyperparathyroidism
A. Parathormone C. Hyperaldosteronism
B. Thyrocalcitonin D. Hypoaldosteronism
C. Thyroxin E. Hypothyroidism
D. Somatotropic hormone
E. Triiodothyronine 57. A 5-month-old boy was hospitalized for
tonic convulsions. Sick since birth. On
52. Owing to a pronounced decrease in plasma examination, the hair is hard, the nails are
calcium concentration, a 2-year-old child had thinned and brittle, the skin is pale and dry. In
tetanic reductions in the respiratory and the blood: calcium - 1.5 mmol / l, phosphorus -
pharyngeal muscles. Decreased secretion of 1.9 mmol / l. What are these changes related to?
what hormone can be the cause of this? A. Hypoparathyroidism
A. hormone B. Hyperparathyroidism
B. Calcitonin C. Hyperaldosteronism
C. Aldosterone D. Hypoaldosteronism
D. Somatotropin E. Hypothyroidism
E. Cortisol
58. Periodic renal colics attacks areobserved in
53. A 2-year-old child experienced convulsions a woman with primery hyperparathyroidizm.
because of lowering calciumions concentration Ultrasonic examination revealed small stones in
in the blood plasma. Function of what structure the kidneys. What is the most plausible reason
is decreased? of the stones’s formation?
A. Parathyroid glands A. Hypercalcemia
B. Hypophysis B. Hyperphosphatemia
C. Adrenal cortex C. Hypercholesterinemia
D. Pineal gland D. Hyperuricemia
E. Thymus E. Hyperkalemia

19
B. Uric acid
59. A 56-year-old man presents with C. Cystine
parathyroid tumor. The following is observed: D. Bilirubin
muscle weakness, osteoporosis, bone E. Cholesterol
deformation, nephroliths consisting of oxalates
and phosphates. The patient’s condition is 64. Parodontitis is treated with calcium
caused by: preparations and a hormone that stimulates
A. Increased secretion of parathyroid hormone tooth mineralization and inhibits tissue
B. Decreased secretion of parathyroid hormone resorption. What hormone is it?
C. Increased secretion of calcitonin A. Calcitonin
D. Decreased secretion of calcitriol B. Parathormone
E. Increased secretion of thyroxin C. Adrenalin
D. Aldosterone
60. Due to trauma the patient’s parathyroid E. Thyroxine
glands have been removed, which resulted in
inertness, thirst, sharp increase of 65. A patient with hypoparathyreosis has
neuromuscular excitability. Metabolism of the multiple carious lesions of teeth. This pathology
following substance is disturbed: is caused by insufficiency of the following
A. Calcium hormone:
B. Manganese A. Calcitonin
C. Chlorine B. Thyroxin
D. Molybdenum C. Triiodothyronine
E. Zinc D. Thyroid-stimulating hormone
E. Somatotropin
61. There is a 9 year old boy in endocrinological
department, who has already had a few fractures 66. A 46-year-old patient suffering from the
of extremeties caused by fragility of bones. diffuse toxic goiter underwent resection of the
Malfunction of what endocrinous glands (gland) thyroid gland. After the surgery the patient
takes place? presents with appetite loss, dyspepsia, increased
A. Parathyroid glands neuromuscular excitement. The body weight
B. Thyroid gland remained unchanged. Body temperature is
C. Thymus normal. Which of the following has caused such
D. Adrenal glands a condition in this patient?
E. Epiphysis A. Reduced production of parathormone
B. Increased production of thyroxin
62. A patient with signs of osteoporosis and C. Increased production of calcitonin
urolithiasis has been admitted to the D. Increased production of thyroliberin
endocrinology department. Blood test revealed E. Reduced production of thyroxin
hypercalcemia and hypophosphatemia. These
changes are associated with abnormal synthesis 67. During surgery on the thyroid gland due to
of the following hormone: the disease of the Basedow's disease, the
A. Parathyroid hormone parathyroid glands were mistakenly removed.
B. Calcitonin There were convulsions, tetany. Exchange of
C. Cortisol which item was broken?
D. Aldosterone A. Calcium
E. Calcitriol B. Magnesium
C. Potassium
63. Cardinal symptoms of primary D. Iron
hyperparathyroidism are osteoporosis andrenal E. Sodium
lesion along with development of urolithiasis.
What substance makes up the basis of these 68. Soldiers injured in the midst of a battle may
calculi in this disease? not feel pain before it is completed. What
A. Calcium phosphate

20
hormones of the opiate antinociceptive system C. Vasopresin
reduce the sensation of pain? D. Aldosterone
A. Endorphins E. Oxytocin
B. Serotonin

21
Biochemistry of intercellular communications: hormones of steroid and
thyroid nature
1. A girl is diagnosed with adrenogenital 6. Parents of a 10 y.o. boy consulted a doctor
syndrome (pseudohermaphroditism). This about extension of hair-covering, growth of
pathology was caused by hypersecretion of the beard and moustache, low voice. Intensified
following adrenal hormone: secretion of which hormone must be assumed?
A. Androgen A. Of testosterone
B. Estrogen B. Of somatotropin
C. Aldosterone C. Of oestrogen
D. Cortisol D. Of progesterone
E. Adrenalin E. Of cortisol

2. The secretion of which hypophysial 7. A female patient presents with endocrine


hormones will be inhibited after taking the oral dysfunction of follicular cells of the ovarian
contraceptives containing sexhormones? follicles resulting from an inflammation. The
A. Gonadotropic hormone synthesis of the following hormone will be
B. Vasopressin inhibited:
C. Thyrotrophic hormone A. Estrogen
D. Somatotropic hormone B. Progesterone
E. Ocytocin C. Lutropin
D. Follicle stimulating hormone
3. The woman was threatened with premature E. Follistatine
termination of pregnancy. This is most likely
due to insufficient secretion of such a hormone: 8. Intake of oral contraceptives containing sex
A. Progesterone hormones inhibits secretion of the hypophysiae
B. Estradiol hormones. Secretion of which of the indicated
C. Oxytocin hormones is inhibited while taking oral
D. Testosterone contraceptives with sexhormones?
E. Aldosterone A. Follicle-stimulating
B. Vasopressin
4. Testosterone and it’s analogs increase the C. Thyrotropic
mass of skeletal muscles that allows to use them D. Somatotropic
for treatment of dystrophy. Dueto interaction of E. Oxytocin
the hormone with what cell substrate is this
action caused? 9. Corticosteroid hormones regulate the
A. Nuclear receptors adaptation processes of the body as a whole to
B. Membrane receptors environmental changes and ensure the
C. Ribosomes maintenance of internal homeostasis. What
D. Chromatin hormone activates the hypothalamo-pituitary-
E. Proteins-activators of transcription adrenal axis?
A. Corticoliberin
5. A 30-year-old female exhibits signs of B. Somatoliberin
virilism (growth of body hair, balding temples, C. Somatostatin
menstrual disorders). This condition can be D. Corticostatin
caused by the over production of the following E. Thyroliberin
hormone:
A. Testosterone 10. A man prescribed by a doctor for a long
B. Oestriol time took the preparation of a group of
C. Relaxin glucocorticoid hormones. Which of the
D. Oxytocin following hormones will be depressed due to
E. Prolactin this?
A. Corticotropic
B. Somatotropic
22
C. Tyrotropic C. Muscle glycogenolysis
D. Sexual D. Intestinal glucose absorption
E. Mineralocorticoid E. Synthesis of glycogen

11. The release of adrenal hormones is regulated 16. A patient with Itsenko-Cushing syndrome
by ACTH of the adenohypophysis. What has persistent hyperglycemia and glycosuria,
hormones are secreted by the adrenal glands hypertension, osteoporosis, obesity. What
under the action of the latter? hormon’s synthesis and secretion are intensified
A. Glucocorticoids in this case?
B. Androgenes A. Cortisol
C. Catecholamines B. Adrenaline
D. Mineralocorticoid C. Glucagon
E. Prostaglandins D. Thyroxin
E. Aldosterone
12. A patient has been taking glucocorticoids
for a long time. Drug withdrowal caused acute 17. On examination of the patient, the doctor
attack of his disease, blood pressure reduction, suspected Itsenko-Cushing's syndrome.
weakness. What are these occurences connected Determination of what substance in the patient's
with? blood will confirm the doctor's presumption
A. Adrenal glands insufficiency A. Cortisol
B. Drug habituation B. Tocopherol
C. Sensibilisation C. Retinol
D. Hyperproduction of corticotroph hormone D. Adrenaline
E. Cumulation E. Cholesterol

13. Chronic overdosage of glucocorticoids leads 18. A patient with a diagnosis of Itsenko-
to the development of hyperglycemia. What Cushing's disease (hyperproduction of adrenal
process of carbohydrate metabolism is hormones) has an increased concentration of
responsible for this effect? glucose, ketone bodies, sodium in the blood .
A. Gluconeogenesis What is the biochemical mechanism leading to
B. Glycogenolysis the occurrence of hyperglycemia?
C. Aerobic glycolisis A. Gluconeogenesis
D. Pentose-phosphate cycle B. Glycogenesis
E. Glycogenesis C. Glycogenolysis
D. Glycolysis
14. A 40-year-old woman with Cushing’s E. Aerobic glycolysis
disease presents with steroid diabetes. On
biochemical examination she has hyperglycemia 19. The patient was found to have obesity,
and hypochloremia. What process activates in hirsutism, a “moon-shaped” face, scarring of a
the first place in such patients? purple color on the skin of the thighs. Blood
A. Gluconeogenesis pressure is 180/110 mm Hg, blood glucose is
B. Glycogenolysis 17.2 mmol / l. What change in the production of
C. Glucose reabsorption adrenal hormones is possible such a picture?
D. Glucose transportation into a cell A. Hyperproduction of glucocorticoids
E. Glycolysis B. Hypoproducts of glucocorticoids
C. Hyperproduction of mineralocorticoids
15. A 28-year-old patient with Itsenko-Cushing D. Hypoproducts of mineralocorticoids
syndrome Hyperglycemia, glycosuria were E. Hypoproduction of adrenaline
detected hyperglycemia, glycosuria. The main
mechanism of hyperglycemia in this patient is 20. A 44 year old woman complains of general
stimulation: weakness, heart pain, significant increase of
A. Gluconeogenesis body weight. Objectively: moon face, hirsutism,
B. Liver glycogenolysis AP is 165/100 mm Hg, height - 164 cm, weight

23
- 103 kg; the fat is mostly accumulated on her D. Diabetes mellitus
neck, thoracic girdle, belly. What is the main E. Essential hypertension
pathogenetic mechanism of obesity?
A. Increased production of glucocorticoids 25. A 38-year-old female patient complains of
B. Reduced production of thyroid hormones general weakness, cardiac pain, increased
C. Increased insulin production appetite, no menstruation. Objectively: the
D. Reduced glucagon production height is 166 cm, weight 108 kg, the patient has
E. Increased mineralocorticoid production moon-shaped face, subcutaneous fat is
deposited mainly in the upper body, torso and
21. A 16-year-old patient suffering from hips. There are also blood-red streaks. Ps-
Itsenko-Cushing's disease was counseled for 62/min, AP-160/105 mm Hg. Which of the
over weight. During the survey, it turned out following diseases is the described pattern of
that the energy value of the food consumed is obesity most typical for?
1700-1900 kcal / day. What is the leading cause A. Cushing pituitary basophilism
of obesity in this case? B. Alimentary obesity
A. Excess glucocorticoid C. Myxedema
B. Lack of insulin D. Insulinoma
C. Excess insulin E. Babinski-Frohlich syndrome
D. Lack of glucocorticoids
E. Hypodynamia 26. Examination of a patient with high blood
pressure revealed secondary arterial
22. A 29-year-old female patient has moon - hypertension. It was found that the cause of this
shaped face, upper body obesity, stretch marks condition of the patient is a hormonally active
on the abdomen, hirsutism; urine shows an tumor of the adrenal cortex. Hyperproduction of
increased rate of 17-oxy ketosteroids. What what hormone is the cause of secondary arterial
disease are these presentations typical for? hypertension in a patient?
A. Itsenko-Cushing syndrome A. Cortisol
B. Pheochromocytoma B. Adrenaline
C. Conn’s syndrome C. Thyroxine
D. Primary aldosteronism D. Insulin
E. Secondary aldosteronism E. Glucagon

23. The patient with complaints of permanent 27. Acceptance of corticosteroid analogues
thirst applied to the doctor. Hyperglycemia, causes the breakdown of muscle proteins to free
polyuria and increased concentration of 17- amino acids. In which process will amino acids
ketosteroids in the urine were revealed. What be involved in such conditions?
disease is the most likely? A. Gluconeogenesis in the liver
A. Steroid diabetes B. Muscle glycolysis
B. Insulin-dependent diabetes mellitus C. Synthesis of higher fatty acids
C. Myxoedema D. Glycogenolysis
D. Type I glycogenosis E. Decarboxylation
E. Addison’s disease
28. A 19-year-old male was found to have an
24. Examination of a 42 year old patient elevated level of potassium in the secondary
revealed a tumour of adenohypophysis. urine. These changes might have been caused
Objectively: the patient’s weight is 117 kg, he by the increase in the following hormone level:
has moon-like hyperemic face, redblue striae of A. Aldosterone
skin distension on his belly. Osteoporosis and B. Oxytocin
muscle dystrophy are present. AP is 210/140 C. Adrenaline
mm Hg. What is the most probable diagnosis? D. Glucagon
A. Cushing’s disease E. Testosterone
B. Cushing’s syndrome
C. Conn’s disease

24
29. A concentrated solution of sodium chloride 34. A humans are reduced diuresis,
was intravenously injected to ananimal. This hypernatremia and hypokalemia. What hormone
caused decreased reabsorption of sodium ions in hypersecretion can cause such changes?
the renal tubules. It is the result of the following A. Aldosterone
changes of hormonal secretion: B. Vasopressin
A. Aldosterone reduction C. Atrial Natriuretic Factor
B. Aldosterone increase D. Adrenaline
C. Vasopressin reduction E. Parathormone
D. Vasopressin increase
E. Reduction of atrial natriuretic factor 35. Atria of an experimental animal were
superdistended by blood that resulted in
30. A patient with adenoma of glomeral zone of decreased reabsorption of Na+ and water in
adrenal cortex (Conn’s disease) has arterial renal tubules. This can be explained by the
hypertension, convulsions, polyuria. What is the influence of the following factor upon kidneys:
main link in pathogenesis of these disorders? A. Natriuretic hormone
A. Aldosterone hypersecretion B. Aldosterone
B. Aldosterone hyposecretion C. Renin
C. Catecholamine hypersecretion D. Angiotensin
D. Glucocorticoid hypersecretion E. Vasopressin
E. Glucocorticoid hyposecretion
36. A patient has insufficient blood supply to
31. After a traffic accident a man presents with the kidneys, which has caused the development
severe blood loss, consciousness disturbance, of pressor effect due to the constriction of
low blood pressure, as well as compensatory arterial resistance vessels. This is the result of
activation of the reninangiotensin system, which the vessels being greately affected by the
results in: following substance:
A. Hyperproduction of aldosterone A. Angiotensin II
B. Increased blood coagulation B. Angiotensinogen
C. Intensification of erythropoiesis C. Renin
D. Hyperproduction of vasopressin D. Catecholamines
E. Intensification of heart contractions E. Norepinephrine

32. A patient has hyperkaliemia and 37. Examination of a patient with high blood
hyponatremia. Reduced secretion of what pressure showed him secondary arterial
hormone may cause such changes? hypertension. The cause of this condition is a
A. Aldosterone renin-producing tumor of the kidney. What is
B. Vasopressin the main link in the pathogenesis of secondary
C. Cortosol arterial hypertension in a patient?
D. Parathormone A. Angiotensin 2, aldosterone hyperproduction
E. Natriuretic hormone B. Cortisol hyperproduction
C. Insulin hyperproduction
33. People adapted to high external D. Insufficient vasopressin production
temperatures have such pecularity: profuse E. Insufficient catecholamines production
sweating isn’t accompanied by loss of large
volumes of sodium chloride.This is caused by 38. A patient with cirrhosis of the liver has
the effect of the following hormone upon the sustained arterial hypotension (BP is 90/50 mm
perspiratory glands: Hg). What is the reason for lowering blood
A. Aldosterone pressure in this pathology of the liver?
B. Vasopressin A. Reduced angiotensin synthesis
C. Cortisol B. Increased synthesis of Na-uretic hormone
D. Tgyroxin C. Excessive inactivation of vasopressin
E. Natriuretic D. Increasing the reflex influence from the
receptor zone of the aortic arch

25
E. Activation of the kallikrein-kinin system
44. Indirect calorimetry allowed to establish that
39. A patient with kidney disease has high blood a 30-year-old male patient had a 30% decrease
pressure, espesially the diastolic one. in basal metabolic rate. This might be caused by
Hypersecretion of what biologically active the reduced concentration of the following
substance causes blood pressure rise? hormones in blood plasma:
A. Renin A. Triiodothyronine, tetraiodothyronine
B. Adrenaline B. Thyrocalcitonin, parathormone
C. Noradrenaline C. Glucocorticoids
D. Vasopressin D. Catecholamines
E. Catecholamines E. Somatoliberin, somatostatin

40. Under the experimental conditions, a rabbit 45. A 37-year-old patient has has lost 5 kg in
was tied up the renal artery, it was resulted in a weight over the past three months, he complains
significant increase in blood pressure after 2 of hand tremor, excessive sweating,
weeks. As a result of an increase in the secretion exophthalmos, tachycardia. These changes
of a biologically active substance, this might have been caused by the increased
happened? secretion of the following hormone:
A. Renin A. Thyroxine
B. Adrenaline B. Cortisol
C. Vasopressin C. Insulin
D. Norepinephrine D. Glucagon
E. Natriuretic hormone E. Thyrocalcitonin

41. A patient who had been continuously taking 46. A 38-year-old woman complains of
drugs blocking the production of angiotensin II excessive sweating, palpitations, fever in the
developed bradycardia and arrhythmia. A likely evening hours. The main exchange is increased
cause of these disorders is: by 60%. The doctor decided the diagnosis -
A. Hyperkalemia thyrotoxicosis. What properties of thyroxin lead
B. Hypokalemia to increased heat production?
C. Hypernatremia A. Separates oxidative phosphorylation
D. Hypocalcemia B. Increases conjugation of oxidation and
E. Hypercalcemia phosphorylation
C. Reduces β-oxidation of fatty acids
42. A patient with kidney disease has high blood D. Reduces amino acid deamination
pressure, espesially the diastolic one. E. Promotes acetyl-coA accumulation
Hypersecretion of what biologically active
substance causes blood pressure rise? 47. A 40-year-old woman suffering from diffuse
A. Renin toxic goiter presents with constant increase of
B. Adrenaline her body temperature. What mechanism results
C. Noradrenaline in such clinical presentation?
D. Vasopressin A. Separation of oxidation and phosphorization
E. Catecholamines in cell mitochondria
B. Increased breakdown of glycogen inhepatic
43. A 40-year-old woman on examination cells
presents with intensified basal metabolic rate. C. Increased catabolism of protein incells
What hormone present in excess leads to such D. Increased excitability of nerve cells
condition? E. Increased cell sensitivity to catecholamines
A. Triiodothyronine
B. Thyrocalcitonin 48. A patient is followed up in an
C. Glucagon endocrinological dispensary on account of
D. Aldosterone hyperthyreosis. Weight loss, tachycardia, finger
E. Somatostatin tremor are accompanied by hypoxia symptoms -

26
headache, fatigue, eye flicker. What mechanism 53. A doctor has established significant growth
of thyroid hormones action underlies the retardation, disproportional body build, and
development of hypoxia? mental deficiency of a child. What is the most
A. Disjunction, oxydation and phosphorilation likely cause of this pathology?
B. Inhibition of respiratory ferment synthesis A. Hypothyroidism
C. Competitive inhibition of respiratory B. Insufficient nutrition
ferments C. Hyperthyroidism
D. Intensification of respiratory ferment D. Genetic defects
synthesis E. Hypopituitarism
E. Specific binding of active centres of
respiratory ferments 54. A 12-year-old child is of short stature, has
disproportionate body structure and mental
49. A 56 y.o. patient has been suffering from retardation. These characteristics might be
thyreotoxicosis for a long time.What type of caused by the hyposecretion of the following
hypoxia can be developed? hormone:
A. Tissue A. Thyroxine
B. Hemic B. Insulin
C. Circulatory C. Cortisol
D. Respiratory D. Somatotropin
E. Mixed E. Glucagon

50. The patient mistakenly took an excess dose 55. Inhabitants of territories with cold climate
of thyroxine. What changes in secretion of have high content of an adaptive
thyroliberin and thyrotropin will it lead to? thermoregulatory hormone. What hormone is
A. Secretion of hormones will decrease meant?
B. Secretion of hormones will increase. A. Thyroxin
C. Secretion of hormones will not change B. Insulin
D. Secretion of thyroliberin will increase, C. Glucagon
thyrotropin will decrease D. Somatotropin
E. Secretion of thyrotropin will increase, E. Cortisol
thyroliberin will decrease
56. The clinical examination of the patient
51. A 19-year-old female suffers from revealed an increase in the thyroid gland
tachycardia in rest condition, weight loss, (goiter), an increase in basal metabolism, loss of
excessive sweating, exophtalmos and body weight, an imbalance of heat, an increase
irritability. What hormone would you expect to in appetite, an increase in excitability and
find elevated in her serum? irritability, exophthalmos and tachycardia were
A. Thyroxine detected. What endocrine disruption leads to
B. Cortisol these symptoms?
C. Mineralocorticoids A. Hyperfunction of the thyroid gland
D. ACTH B. Hypofunction of the parathyroid glands
E. Insulin C. Pituitary Hyperfunction
D. Epiphysis hypofunction
52. A child presents with symptoms of psychic E. Hypofunction of the thyroid gland
and physical retardation (cretinism). It is usually
associated with the following hormone 57. It is known that steroid anti-inflammatory
deficiency: drugs inhibit the activity of phospholipase A2,
A. Thyroxin which is necessary for the synthesis of
B. Somatotropic prostaglandins. What substance is the precursor
C. Calcitonin of these inflammatory mediators?
D. Insulin A. Arachidonic acid
E. Testosterone B. Cholesterol
C. Tyrosine

27
D. Proopiomelanocortin 59. Utilization of arachidonic acid via
E. Palmitic acid cyclooxigenase pathway results in formation of
some bioactive substances. Name them:
58. A patient is followed up in the clinic on A. Prostaglandins
account of for pneumonia complicated by B. Thyroxine
pleurisy. He was given prednisolon as part of a C. Biogenic amins
combination therapy. The anti-inflammatory D. Somatomedins
effect of this synthetic glucocorticoid is E. Insulin-like growth factors
associated with blocking the release of
arachidonic acid by inhibiting: 60. Experimental studies revealed steroid
A. Phospholipase A2 hormones to have an effect on proteosynthesis.
B. Cyclo-oxygenase They influence synthesis of the following
C. Phospholipase C substances:
D. Lipoxygenase A. Specific messenger RNA
E. Peroxidase B. Adenosine triphosphate
C. Specific transfer RNA
D. Guanosine triphosphate
E. Specific ribosomal RNA

28
Biochemistry and pathobiochemistry of blood
1. A 7-year-old girl has signsof anemia. E. -
Laboratory examination revealed pyruvate
kinase deficiency in erythrocytes. What process 6. Sulfanilamides are applied as antimicrobal
disturbance plays the main role in agents in clinical practice.Sulfanilamide
anemiadevelopment? treatment, however, canresult in hemolytic
A. Anaerobic glycolysis anemia developmentin patients that suffer from
B. Oxidative phosphorylation geneticdefect of the following enzyme
C. Tissue respiration ofpentose phosphate metabolism inerythrocytes:
D. Peroxide decomposition A. Glucose-6-phosphate dehydrogenase
E. Aminoacids desamination B. Hexokinase
C. Transketolase
2. Human red blood cells contain no D. Transaldolase
mitochondria. What is the main pathway E. Pyruvate kinase
forATP production in these cells?
A. Anaerobic glycolysis 7. Biochemical analysis of an
B. Aerobic glycolysis infant’serythrocytes revealed evident
C. Oxidative phosphorylation glutathioneperoxidase deficiency and low
D. Creatine kinase reaction concentration of reduced glutathione. What
E. Cyclase reaction pathological condition can develop in this
infant?
3. Erythrocytes of the patient withhemolytic A. Hemolytic anemia
anemia present with significantdecrease of B. Pernicious anemia
pyruvate kinase activity. Whatmethabolic C. Megaloblastic anemia
process is disturbed in thiscase? D. Sicklemia
A. Glycolysis E. Iron-deficiency anemia
B. Glycogenolysis
C. Gluconeogenesis 8. A 20 year old patient complains ofgeneral
D. Pentose-phosphate pathway of weakness, dizziness, quick fatigability. Blood
glucoseoxidation analysis results: Hb-80g/l. Microscopical
E. Glycogen synthesis examination results:erythrocytes are of modified
form. Thiscondition might be caused by:
4. A 3 year old child with fever wasgiven A. Sickle-cell anemia
aspirin. It resulted in intensified erythrocyte B. Hepatocellular jaundice
haemolysis. Hemolyticanemia might have been C. Acute intermittent porphyria
caused bycongenital insufficiency of the D. Obturative jaundice
followingenzyme: E. Addison’s disease
A. Glucose 6-phosphate dehydrogenase
B. Glucose 6-phosphatase 9. With a number of hemoglobinopathies, amino
C. Glycogen phosphorylase acid substitutions occur in the α- and β-chains of
D. Glycerol phosphate dehydrogenase hemoglobin. Which of them is characteristic of
E. γ-glutamiltransferase Hb S (sickle cell anemia)?
A. Glutamate-valine
5. There is an increased tendency of B. Aspartate-lysine
erythrocytes to hemolysis in patients with a C. Alanin-serine
hereditary defect of glucose-6-phosphate D. Methionine-histidine
dehydrogenase. What metabolic process is E. Glycine-serine
impaired under these conditions?
A. Pentose phosphate pathway of glucose 10. Substitution of the glutamic acid onvaline
oxidation was revealed while examining initial molecular
B. Gluconeogenesis structure. For what inheritedpathology is this
C. Aerobic glucose oxidation symptom typical?
D. Synthesis of glycogen A. Sickle-cell anemia
29
B. Thalassemia vitamin deficiency leads to such changes in
C. Minkowsky-Shauffard disease blood?
D. Favism A. B12
E. Hemoglobinosis B. C
C. P
11. A patient suffers from mutation of a gene D. B6
that corresponds with hemoglobin synthesis. E. PP
This condition led to development of sicklecell
disease. Name the pathologicalhemoglobin 16. A 43-year-old patient with chronic atrophic
characteristic of this disease: gastritis and megaloblastic hyperchromic
A. HbS anemia excreted methylmalonic acid in the
B. HbA urine. What vitamin deficiency is caused by the
C. HbF occurrence of the specified symptom complex?
D. HbA1 A. В12
E. Bart-Hb B. В2
C. В3
12. Along with normal hemoglobin typesthere D. В5
can be pathological ones in the organism of an E. В6
adult. Name one of them:
A. HbS 17. A pregnant woman turned to the
B. HbF obstetrician-gynecologist, who was diagnosed
C. HbA1 with megaloblastic anemia. Which of the
D. HbA2 following drugs should be prescribed?
E. HbO2 A. Cyanocobalamin
B. Pentoxyl
13. After an extended treatment with C. Methyluracil
sulfanamides a patient has developedmacrocytic D. Glaucin
anemia. Production of activeforms of the E. Streptokinase
following vitamin is disruptedin such a
condition: 18. The patient was diagnosed with
A. Folic acid megaloblastic anemia. Specify a vitamin
B. Thiamine deficiency which can lead to the development of
C. Riboflavin this disease.
D. Pyridoxine A. Cyanocobalamin.
E. Cyanocobalamin B. Rutin.
C. Nicotinamide.
14. A 50-year-old patient has beenexamined by D. Thiamine.
a dentist and found tohave crimson smooth E. Cholecalciferol
tongue. Bloodanalysis revealed a decrease in
RBC leveland hemoglobin concentration, 19. In a 65-year-old patient with prolonged
symptoms of megaloblastichematopoiesis, complaints characteristic of chronic gastritis,
degenerative changes inWBCs. What blood megalocytes were found in peripheral blood,
disorder was found inthis patient? and megaloblastic erythropoiesis was found in
A. B12-folic-acid-deficiency anemia the bone marrow. What is the most likely
B. Iron deficiency anemia diagnosis?
C. Myeloid leukemia A. B12-folic deficiency anemia
D. Aplastic anemia B. Aplastic anemia
E. Hemolytic anemi C. Hypoplastic anemia
D. Hemolytic anemia
15. After removal in the patient 2/3 of the E. Iron deficiency anemia
stomach in the blood decreased the amount of
hemoglobin, the number of erythrocytes, A patient who had subtotal gastrectomy 5 years
increased the size of these blood counts. What ago developed B12 folic acid deficiency

30
anemia. What is the leading mechanism in the hemolysis of red blood cells and
development of such anemia? hemoglobinuria were detected. The action of
A. The absence of an internal factor Castle snake venom due to the presence of the enzyme
B. The absence of an external factor Castle in it:
C. Impaired absorption of vitamin B12 in the A. Phospholipase A2
small intestine B. Phospholipase A1
D. Folic acid deficiency C. Phospholipase C
E. Transcobalamin deficiency. D. Phospholipase D
E. Sphingomyelinase
20. Examination of a patient, suffering from
atrophic gastritis, revealedmegaloblastic 25. For people who permanently reside in highlands,
anemia. The anemia is likely to be caused by the adaptation to "oxygen starvation" is carried out by
deficiency of the following substance: facilitating the release of hemoglobin oxygen due to:
A. Gastromucoproteid A. Increased formation of 2,3-diphosphoglycerate in
B. Vitamin B6 erythrocytes
B. Reduced formation of 2,3-diphosphoglycerate in
C. Vitamin B1
erythrocytes
D. Iron C. The increase in CO2 partial pressure
E. Erythropoietins D. Increasing blood pH
E. Decreased blood temperature
21. Surgical removal of a part of
stomachresulted in disturbed absorption of 26. After the accident in the chemical industry,
vitamin B12, it is excreted with feces. the environment was polluted with nitro
Thepatient was diagnosed with anemia. compounds. The people living in this area, there
Whatfactor is necessary for absorption of was a sharp weakness, headache, shortness of
thisvitamin? breath, dizziness. What is the cause of hypoxia?
A. Gastromucoprotein A. Methemoglobin formation
B. Gastrin B. Inhibition of dehydrogenase
C. Hydrochloric acid C. Formation of carboxyhemoglobin
D. Pepsin D. Reduced function of flavin enzymes
E. Folic acid E. Inactivation of cytochrome oxidase

22. Examination of a 52-year-oldfemale patient 27. A 7-year-old child presents with marked
has revealed a decreasein the amount of red signs of hemolytic anemia. Biochemical
blood cells andan increase in free hemoglobin analysis of erythrocytes determined low
inthe blood plasma (hemoglobinemia).What concentration of NADPH and reduced
type ofanemia is being observed in the patient? glutathione. What enzyme is deficient in this
A. Acquired hemolytic case leading to the biochemical changes and
B. Hereditary hemolytic their clinical manifestations?
C. Acute hemorrhagic A. Glucose-6-phosphate dehydrogenase
D. Chronic hemorrhagic B. Hexokinase
E. Anemia due to diminishederythropoiesis C. Fructokinase
D. Pyruvate kinase
23. A patient with hemolytic anemia showed E. Lactate dehydrogenase
pyruvate kinase deficiency in erythrocytes. In these
conditions, the cause of hemolysis of red blood cells 28. A 38 year old patient takes aspirin and
are:
sulfanilamides. After their intake intensified
A. Reduced Na+, K+ -ATPases
B. Na+ deficiency in red blood cells erythrocyte haemolysis is observed which is
C. Excess K+ in red blood cells caused by deficiency of glucose 6-phosphate
D. Genetic defects of glycophorin A dehydrogenase. This pathologyis caused by
E. Spectrin deficiency failure of the following coenzyme:
A. NADP – H
24. During laboratory examination of the blood B. FAD - H2
of a person who was bitten by a snake, C. Pyridoxal phosphate

31
D. FMN - H2 B. Cytochrome oxidase
E. Ubiquinone C. Succinate dehydrogenase
D. Ketoglutarate dehydrogenase
29. A 22 year old woman has been taking E. Aconitase
sulfanilamides for a long time that led to
symptoms of hemolytic anaemia caused by 34. When periodontitis develops lipid
hereditary disturbance of synthesis of glucose 6- peroxidation in periodontal tissues, the content
phosphate dehydrogenase. This enzyme of of malondialdehyde, hydrogen peroxide
pentose-phosphate cycleis responsible for increases in the oral cavity. Which of the
generation of: following enzymes provide antioxidant
A. NADP - H2 protection?
B. NAD A. Superoxide dismutase, catalase
C. FAD B. Amylase, trypsin
D. FMN C. Maltase, chymotrypsin
E. ATP D. Lactase, lysozyme
E. Sucrase, prothrombin
30. It is known that pentose-phosphate pathway
actively functions in the erythrocytes. What is 35. Periodontitis induces the development of
the main function of this metabolic pathway in lipid peroxidation in the periodontal tissues, as
the erythrocytes? well as an increase in malondialdehyde and
A. Counteraction to lipid peroxidation hydrogen peroxide concentration in the oral
B. Activation of microsomal oxidation cavity. Which of the following enzymes
C. Neutralization of xenobiotics provides antioxidant protection?
D. Oxidation of glucose into lactate A. Catalase
E. Increase of lipid peroxidation B. Amylase
C. Maltase
31. In course of metabolic process active forms D. Lactase
of oxygen including superoxide anion radical E. Invertase
are formed in the human body. By means of
what enzyme is this anion inactivated? 36. Those organisms which in the process of
A. Superoxide dismutase evolution failed to develop protection from
B. Catalase H2O2 can exist only in anaerobic conditions.
C. Peroxidase Which of the following enzymes can break
D. Glutathioneperoxidase hydrogen peroxide down?
E. Glutathionereductase A. Peroxidase and catalase
B. Oxygenase and hydroxylase
32. Reduced activity of antioxidant enzymes C. Cytochrome oxidase, cytochrome B5
enhances peroxidation of cell membrane lipids. D. Oxygenase and catalase
The reduction of glutathione peroxidase activity E. Flavin-dependent oxidase
is caused by the following microelement
deficiency: 37. Patient with abscess of the cut wound
A. Selenium applied to the traumatological department. In
B. Molybdenum order to clean the wound from the pus doctor
C. Cobalt washed it with 3% hydrogen peroxide. Foam
D. Manganese was absent. What caused the absence of the
E. Copper drug activity?
A. Inherited insufficiency of catalase
33. In pathological processes accompanied by B. Low concentration H2O2
hypoxia, an incomplete reduction of the oxygen C. Inherited insufficiency of erythrocyte
molecule in the respiratory chain and phosphatdehydrogenase
accumulation of hydrogen peroxide occurs. D. Shallow wound
Specify the enzyme that ensures its destruction: E. Pus in the wound
A. Catalase

32
38. A worker has decreased buffer capacity of B. Ketone bodies
blood due to exhausting muscular work. The C. Acetyl CoA
influx of what acid substance in the blood can D. Glucose 6-phosphate
cause this symptom? E. Oxaloacetate
A. Lactate
B. Pyruvate 43. A 32-year-old female patient suffers from
C. 1,3-bisphosphoglycerate gingivitis accompanied by gum hypoxia. What
D. α-ketoglutarate metabolite of carbohydrate metabolism is
E. 3-phosphoglycerate produced inthe periodontium tissues more
actively in this case?
39. A child during the first 3 months after birth A. Lactate
developed a severe form of hypoxia, which was B. Ribose 5-phosphate
manifested by asphyxia and cyanosis of skin. C. Glycogen
The reason for this is a violation of the D. Glucose 6-phosphate
replacement of fetal hemoglobin on: E. NADPH-H
A. Hemoglobin A
B. Hemoglobin S 44. A 29-year-old patient was delivered to a
C. Glycated hemoglobin hospital because of intoxication with carbon
D. Methemoglobin monoxide. Objectively: the patient presents with
E. Hemoglobin M symptoms of severe hypoxia- evident dyspnea,
cyanosis, tachycardia. What compound is
40. Diseases of the respiratory system and produced as a result of intoxication with carbon
circulatory disorders impair the transport of monooxide?
oxygen, thus leading to hypoxia. Under these A. Carboxyhemoglobin
conditions the energy metabolism is carried out B. Methemoglobin
byanaerobic glycolysis. As a result, the C. Carbhemoglobin
following substance is generated and D. Sulfhemoglobin
accumulated in blood: E. Oxyhemoglobin
A. Lactic acid
B. Pyruvic acid 45. A man lost consciousness in a car with
C. Glutamic acid running engine where he had been waiting for a
D. Citric acid friend for a long time. What hemoglobin
E. Fumaric acid compaund can be found in the blood of the
patient?
41. A patient is followed up in an A. Carboxyhemoglobin
endocrinological dispensary on account of B. Deoxyhemoglobin
hyperthyreosis. Weight loss, tachycardia, finger C. Carbhemoglobin
tremor are accompanied by hypoxia symptoms - D. Methemoglobin
headache, fatigue, eye flicker. What mechanism E. Oxyhemoglobin
of thyroid hormones action underlies the
development of hypoxia? 46. A patient with respiratory failure has blood
A. Disjunction oxydation and phosphorilation pH of 7,35. pCO2 test revealed hypercapnia.
B. Inhibition of respiratory fermentsynthesis Urine pH test revealed an increase in the urine
C. Competitive inhibition of respiratoryferments acidity. What form of acid-base imbalance is the
D. Intensification of respiratory case?
fermentsynthesis A. Compensated respiratory acidosis
E. Specific binding of active centres B. Compensated metabolic acidosis
ofrespiratory ferments C. Decompensated metabolic acidosis
D. Compensated respiratory alkalosis
42. After a sprint an untrained person develops E. Decompensated respiratory alkalosis
muscle hypoxia. This leads to the accumulation
of the following metabolite in muscles: 47. An infant has apparent diarrhea resulting
A. Lactate from improper feeding. One of the main

33
diarrhea effects is plentiful excretion of sodium 52. Prophylactic examination of a patient
bicarbonate. What form of acid-base balance revealed hyperglycemia, ketonuria, polyuria,
disorder is the case? glycosuria. What form of acid-base balance
A. Metabolic acidosis disorder is the case?
B. Metabolic alkalosis A. Metabolic acidosis
C. Respiratory acidosis B. Gaseous acidosis
D. Respiratory alkalosis C. Nongaseous acidosis
E. No disorders of acid-base balance will be D. Gaseous alkalosis
observed E. Metabolic alkalosis

48. A 30-year-old man with diabetes mellitus 53. A patient with diabetes developed a diabetic
type I was hospitalised. The patient is comatose. coma due to the acid-base imbalance. Specify
Laboratory tests revealed hyperglycemia and the kind of this imbalance:
ketonemia. What metabolic disorder can be A. Metabolic acidosis
detected in this patient? B. Metabolic alkalosis
A. Metabolic acidosis C. Respiratory acidosis
B. Metabolic alkalosis D. Gaseous alkalosis
C. Respiratory acidosis E. Non-gaseous alkalosis
D. Respiratory alkalosis
E. Normal acid-base balance 54. Ketoacidosis that develops due to
accumulation of ketone bodies in blood serum is
49. A patient suffers from disrupted patency of a primary complication of diabetes mellitus.
the airways at the level of small and medium- What acid-base disbalance develops during this
sized bronchial tubes. What changes of acid- condition?
base balance can occur in the patient? A. Metabolic acidosis
A. Respiratory acidosis B. Metabolic alkalosis
B. Respiratory alkalosis C. Respiratory acidosis
C. Metabolic acidosis D. Respiratory alkalosis
D. Metabolic alkalosis E. –
E. Acid-base balance remains unchanged
55. Due to recurring vomiting a patient has lost
50. Diabetes mellitus causes ketosis as a result significant amount of gastric juice, which led to
of activated oxidation of fatty acids. What development of acidbase dysbalance. What type
disorders of acid-base equilibrium may be of acid-base dysbalance has developed?
caused by excessive accumulation of ketone A. Nongaseous alkalosis
bodies in blood? B. Gaseous acidosis
A. Metabolic acidosis C. Nongaseous acidosis
B. Metabolic alcalosis D. Gaseous alkalosis
C. Any changes woun’t happen E. Metabolic acidosis
D. Respiratory acidosis
E. Respiratory alcalosis 56. A mountaineer who rose to a height of
5200m developed gas alkalosis. What is the
51. Ketosis develops in the patients with cause of its development?
diabetes mellitus, as the result of activation of A. Hyperventilation
fatty acids oxidation processes. What acidbase B. Hypoventilation of the lungs
imbalance can result from accumulation of C. Hyperoxemia
excessive ketone bodies in the blood? D. Hypoxemia
A. Metabolic acidosis E. Reduced ambient temperature
B. Metabolic alkalosis
C. No imbalance occurs 57. In a laboratory study of the respiratory
D. Respiratory acidosis function of the blood, it has been established
E. Respiratory alkalosis that there is a deterioration of CO2 transport.

34
What enzyme deficiency in red blood cells can examination revealed anaemia and presence of
this be due to? the paraprotein in the zone of gamma-globulins.
A. Carboanhydrase To confirm the myeloma diagnosis it is
B. 2,3-diphosphoglycerate necessary to determine the following index in
C. Adenylate cyclase the patient’s urine:
D. Protein kinase A. Bence Jones protein
E. Phosphorylase B. Bilirubin
C. Haemoglobin
58. In the process of hemoglobin catabolismiron D. Ceruloplasmin
is released and then as a part of special transport E. Antitrypsin
protein is returned to the bonemarrow, to be
used again for hemoglobinsynthesis. Name this 63. A 16-year-old girl, who has been starving
transport protein: herself for a long time to lose weight, developed
A. Transferrin an edema. This phenomenon is mainly caused
B. Transcobalamin by:
C. Haptoglobin A. Hypoproteinemia due to protein synthesis
D. Ceruloplasmin disturbance
E. Albumin B. Hypoglycemia due to glycogen synthesis
disturbance
59. Hemoglobin catabolism results in release of C. Venous congestion and increased venous
iron which is transported to the bone marrow by pressure
a certain transfer protein and used again for the D. Deceleration of glomerular filtration rate
synthesis of hemoglobin. Specify this transfer E. Decreased production of vasopressin in the
protein: hypothalamus
A. Transferrin (siderophilin)
B. Transcobalamin 64. A 36-year-old female patient who has been
C. Haptoglobin limiting the number of food stuffs in her diet for
D. Ceruloplasmin 3 months presents with a decrease in body
E. Albumin weight, deterioration of physical and mental
health, face edemata. These changes may be
60. In the liver of a patient suffering from iron caused by the deficiency of the following
deficiency anemia, a violation of the synthesis nutrients:
of iron-containing protein, which is a source of A. Proteins
iron for heme synthesis, was found. What is the B. Vitamins
name of this protein? C. Fats
A. Ferritin D. Carbohydrates
B. Transferrin E. Micronutrients
C. Hemosiderin
D. Ceruloplasmin 65. Upon toxic damage of hepatic cells resulting
E. Hemoglobin in disruption of liver function the patient
developed edemas. What changes of blood
61. In men of 40 years as a result of enhanced plasma are the main cause of edema
hemolysis of erythrocytes, the iron content in development?
the blood plasma increased. What protein A. Decrease of albumin content
provides its deposition in the tissues? B. Increase of globulin content
A. Ferritin C. Decrease of fibrinogen content
B. Haptoglobin D. Increase of albumin content
C. Transferrin E. Decrease of globulin content
D. Transcortin
E. Albumin 66. In case of toxic damage to hepatocytes with
a violation of their protein synthesis function,
62. A patient complains about dyspnea the patient's content of albumin in the blood
provoked by the physical activity. Clinical plasma and oncotic pressure of the plasma

35
sharply decreased. What will be the result of A. γ-globulins
these changes? B. Albumins
A. The appearance of edema C. α1-globulins
B. Reduced diuresis D. α2-globulins
C. Reduced ESR E. β-globulins
D. Increased circulating blood volume
E. Increased blood viscosity 72. When the inflammatory process is activated,
some autoimmune and infectious diseases in the
67. Toxic affection of liver results in blood plasma sharply increase the level of
dysfunction of protein synthesis. It is usually proteins of the acute phase. Which of the
accompanied by the following kind of following proteins can form a gel when the
dysproteinemia: whey cools?
A. Absolute hypoproteinemia A. Cryoglobulin
B. Relative hypoproteinemia B. Haptoglobin
C. Absolute hyperproteinemia C. Ceruloplasmin
D. Relative hyperproteinemia D. C-reactive protein
E. Paraproteinemia E. α2 macroglobin

68. The concentration of albumins in human 73. A 49-year-old male patient with acute
blood sample is lower than normal. This leads to pancreatitis was likely to develop pancreatic
edema of tissues. What blood function is necrosis, while active pancreatic proteases were
damaged? absorbed into the blood stream and tissue
A. Maintaining the oncotic blood pressure proteins broke up. What protective factors of the
B. Maintaining the Ph level body can inhibit these processes?
C. Maintaining the body temperature A. α2-macroglobulin, α1-antitrypsin
D. Maintaining the blood sedimentation system B. Immunoglobulin
E. All answers are correct C. Cryoglobulin, interferon
D. Ceruloplasmin, transferrin
69. A 4 y.o. child with signs of durative E. Hemoplexin, haptoglobin
proteinic starvation was admitted to the hospital.
The signs were as follows: growth inhibition, 74. Wilson’s disease is a disorder of copper
anemia, edemata, mental deficiency. Choose a transport which leads to the accumulation of this
cause of edemata development: metal in brain and liver cells. It is associated
A. Reduced synthesis of albumins with a disturbance in the synthesis of the
B. Reduced synthesis of globulins following protein:
C. Reduced synthesis of hemoglobin A. Ceruloplasmin
D. Reduced synthesis of lipoproteins B. Metallothionein
E. Reduced synthesis of glycoproteins C. Transcobalamin
D. Haptoglobin
70. The prolonged action of a number of E. Siderophilin
antibiotics and sulfonamides is due to the fact
that they circulate in the blood for a long time in 75. Examination of a 27-year-old patient
combination with: revealed pathological changes in liver and brain.
A. Albumin Blood plasma analysis revealed an abrupt
B. Transferrin decrease in the copper concentration, urine
C. Hemoglobin analysis revealed an increased copper
D. Haptoglobin concentration. The patient was diagnosed with
E. Hemopexin Wilson’s degeneration. To confirm the
diagnosis it is necessary to study the activity of
71. Electrophoretic study of a bloodserum the following enzyme in blood serum:
sample, taken from the patient with pneumonia, A. Ceruloplasmin
revealed an increase in one of the protein B. Carbonic anhydrase
fractions. Specify this fraction: C. Xanthine oxidase

36
D. Leucine aminopeptidase B. Prothrombin
E. Alcohol dehydrogenase C. Fibrinogen
D. G immunoglobulin
76. Biochemical analysis of the blood serum of E. A immunoglobulin
a patient with hepatolenticular degeneration
(Wilson-Konovalov disease) revealed a 81. A 60-year-old man complains of joint pain.
decrease in the content of ceruloplasmin. In this An increase in the concentration of C-reactive
patient, the concentration of such ions in the protein and hydroxyproline was detected in the
serum will be increased: patient's serum. What disease are these
A. Copper symptoms characteristic of?
B. Calcium A. Rheumatism
C. Phosphorus B. Gout
D. Potassium C. Hepatitis
E. Sodium D. Jaundice
E. Diabetes
77. A patient suffering from hepatocerebral
degeneration has low concentration of 82. A 6-month-old child experienced frequent
ceruloplasmin in blood serum. What element and intense subcutaneous hemorrhages. The
accumulation will be observed in liver, appointment of a synthetic analogue of vitamin
cerebrum and kidneys of the patient? K (vikasol) gave a positive effect. In the γ-
A. Cuprum carboxylation of glutamic acid, what protein of
B. Calcium the blood coagulation system is this vitamin
C. Sodium taking part in?
D. Potassium A. Prothrombin
E. Ferrum B. Fibrinogen
C. Hageman factor
78. A 33-year-old woman suffers from D. Antihemophilic globulin A
hepatocerebral dystrophy (Wilson's disease). In E. Rosenthal factor
the blood - low content of ceruloplasmin. In the
urine - sharply elevated amino acids. What 83. After implantation of a cardiac valve a
process has increased these changes? young man constantly takes indirect
A. Complexation of amino acids with copper anticoagulants. His state was complicated by
B. Urea synthesis hemorrhage. What substance content has
C. Transamination of amino acids decreased in blood?
D. Disintegration of tissue proteins A. Prothrombin
E. Gluconeogenesis B. Haptoglobin
C. Heparin
79. A 38 year old patient suffers from D. Creatin
rheumatism in its active phase. What laboratory E. Ceruloplasmin
characteristic of blood serumis of diagnostic
importance in case of this pathology? 84. The patient complains of frequent bleeding
A. C-reactive protein from the gums. A deficiency of coagulation
B. Uric acid factor II (prothrombin) was found in the blood.
C. Urea What phase of blood coagulation is impaired in
D. Creatinine a person, above all?
E. Transferrin A. Thrombin formation
B. The formation of prothrombinase
80. Blood plasma of a healthy man contains C. Formation of fibrin
several dozens of proteins. During an illness D. Fibrinolysis
new proteins can originate, namely the protein E. Clot retraction
of "acute phase". Select such protein from the
listed below:
A. C-reactive protein

37
85. Inflammatory processes cause synthesis of 90. The student used canned donor blood to
protein of acute phase in an organism. What determine the time it was collected. However,
substances stumulate their synthesis? he could not get any positive result. The reason
A. Interleukin-1 for this is the lack of blood:
B. Immunoglobulins A. Ionized calcium
C. Interferons B. Hageman factor
D. Biogenic amins C. Thromboplastin
E. Angiotensin D. Fibrinogen
E. Vitamin K.
86. Pyrogenal administered to a rabbit, in the
course of an experiment, resulted in increase of 91. A 16 year old boy after an illness has
its body temperature. What substance of those diminished function of protein synthesis in liver
named below acts as a secondary pyrogen that is as a result of vitamin K deficiency. It will cause
a part of fever inducing mechanism? disturbance of:
A. Interleukin 1 A. Blood coagulation
B. Pseudomonas polysaccharide (Piromen) B. Erythrocyte sedimentation rate
C. Histamine C. Anticoagulant generation
D. Bradykinin D. Erythropoietin secretion
E. Immunoglobulin E. Osmotic blood pressure

87. A patient who suffers from pneumonia has 92. The patient has hemorrhages, the
high body temperature. What biologically active concentration of prothrombin is reduced in the
substance plays the leading part in origin of this blood. What vitamin deficiency led to a
phenomenon? violation of the synthesis of this clotting factor?
A. Interleukin-I A. K
B. Histamine B. A
C. Bradykinin C. D
D. Serotonin D. C
E. Leukotrienes E. E

88. After transfusion of 200 ml of blood apatient 93. A 37-year-old patient, with long-term use of
presented with body temperature rise up to antibiotics, has increased bleeding with minor
37,9oC. Which of the following substances is injuries. In the blood - a decrease in activity II,
the most likely cause of temperature rise? VII, X blood clotting factors; lengthening of
A. Interleukin-1 blood clotting time. What vitamin deficiency
B. Interleukin-2 caused these changes?
C. Tumour necrosis factor A. Vitamin K
D. Interleukin-3 B. Vitamin A
E. Interleukin-4 C. Vitamin C
D. Vitamin D
89. For the development of febrile conditions, E. Vitamin E
an increase in the level of the “acute phase”
proteins of ceruloplasmin, fibrinogen, and C- 94. Plasmic factors of blood coagulationare
reactive protein is characteristic. Specify the exposed to post-translational modification with
possible mechanism of this phenomenon: the participation of vitamin K. It is necessary as
A. Stimulating effect of IL-1 on hepatocytes a cofactor in the enzyme system of γ-
B. The destructive effect of temperature on the carboxylation of protein factors of blood
cells of the body coagulation due to the increased affinity of their
C. Proliferative effect of IL-2 on T-lymphocytes molecules with calcium ions. What amino acid
D. Degranulation of tissue basophils is carboxylated in these proteins?
E. - A. Glutamic
B. Valine
C. Serine

38
D. Phenylalanine likely to have a disorder of the following
E. Arginine biochemical process:
A. Production of gammacarboxyglutamate
95. As a result of posttranslative modifications B. Conversion of homocysteine to methionine
some proteins taking part in blood coagulation, C. Conversion of methylmalonyl CoA to
particularly prothrombin, become capable of succinyl CoA
calcium binding. The following vitamin takes D. Degradation of glutathione
part in this process: E. Hydroxylation of proline
A. K
B. C 100. Activation of a number of hemostatic
C. A factors occurs through their joining with
D. B1 calcium ions. What structural component allows
E. B2 for adjoining of calcium ions?
A. Gamma-carboxyglutamic acid
96. To prevent postoperative bleeding a 6 y.o. B. Gamma-aminobutyric acid
child was administered vicasol that is a C. Gamma-oxybutyric acid
synthetic analogue of vitamin K. Name post- D. Hydroxyproline
translational changes of blood coagulation E. Monoamine-dicarboxylic acids
factors that will be activated by vicasol:
A. Carboxylation of glutamin acid 101. A 46-year-old female patient hasa
B. Phosphorylation of serine radicals continuous history of progressive muscular
C. Partial proteolysis (Duchenne’s) dystrophy. Which blood enzyme
D. Polymerization changes will be of diagnostic value in this case?
E. Glycosylation A. Creatine phosphokinase
B. Lactate dehydrogenase
97. A few days before an operation apatient C. Pyruvate dehydrogenase
should be administered vitamin K or its D. Glutamate dehydrogenase
synthetic analogue Vicasol. Vitamin K takes E. Adenylate cyclase
part in the following posttranslational
modification of the II, VII, IX, X blood clotting 102. A 15-year-old boy has been diagnosedwith
factors: acute viral hepatitis. What blood values hould
A. Carboxylation be determined to confirm acute affection of
B. Decarboxylation hepatic cells?
C. Deamination A. Aminotransferase activity (AST, ALT)
D. Transamination B. Unconjugated and conjugated bilirubin
E. Glycosylation content
C. Erythrocytes sedimentation rate (ESR)
98. A patient, who has been suffering for a long D. Cholesterol content
time from intestine disbacteriosis, has increased E. Protein fraction content
hemorrhaging caused by disruption of
posttranslational modification of blood- 103. 12 hours after an accute attack of
coagulation factors II, VII, IХ, and Х in the retrosternal pain a patient presented a jump of
liver. What vitamin deficiency is the cause of aspartate aminotransferase activity in blood
this condition? serum. What pathology is this deviation typical
A. К for?
B. 12 A. Myocardium infarction
C. 9 B. Viral hepatitis
D. С C. Collagenosis
E. Р D. Diabetes mellitus
E. Diabetes insipidus
99. A newborn baby has numerous
hemorrhages. Blood coagulation tests reveal 104. A 43-year-old man after eating fatty foods
increased prothrombin time. The child is most and alcohol complains of severe abdominal

39
pain. The serum content of trypsin is 850 mmol D. Lungs
/ (h • l) (normal 60-240 mmol / (h • l)). What is E. Spleen
the most characteristic pathology of the
digestive system? 109. In the serum of the patient increased
A. Acute pancreatitis activity of hyaluronidase. Determining what
B. Dynamic intestinal obstruction biochemical index of serum will allow to
C. Mechanical intestinal obstruction confirm the assumption of connective tissue
D. Gastric ulcer pathology?
E. Hepatitis A. Sialic acids
B. Bilirubin
105. For biochemical diagnostics of myocardial C. Uric acid
infarction it is necessary to measure activity of a D. Glucose
number of enzymes and their isoenzymes. What E. Galactose
enzymatic test is considered to be the best to
prove or disprove the diagnosis of infarction in 110. Marked increase of activity of МВ forms
the early period after the chest pain is detected? of CPK (creatinephosphokinase) and LDH-1
A. Creatine kinase isoenzyme CK-MB was revealed by examination of the patient’s
B. Creatine kinase isoenzyme CK-MM blood. What is the most probable pathology?
C. LDH1 lactate dehydrogenaseisoenzyme A. Miocardial infarction
D. LDH2 lactate dehydrogenaseisoenzyme B. Hepatitis
E. Aspartate aminotransferasecytoplasmic C. Rheumatism
isoenzyme D. Pancreatitis
E. Cholecystitis
106. An increase in the activity of LDH4,5,
ALT, carbamoylornitene transferase was 111. An increase in the activity of LDH 1, LDH
detected in the patient's blood. In which body 2, AsAT, and creatine kinase was found in the
can you predict the development of the patient's blood. In which organ of the patient is
pathological process? the most likely development of the pathological
A. Liver (possible hepatitis) process?
B. Heart muscle (possible myocardial A. Heart
infarction) B. Pancreas
C. Skeletal muscle C. Liver
D. Kidneys D. Kidneys
E. Connective tissue E. Skeletal muscle

107. Albumin concentration in the patient’s 112. There is increased activity of AST, LDH1,
blood is 2.8 g / l, increased concentration of LDH2, and CPK in the patient’s blood.
lactate dehydrogenase 5 (LDH 5). What kind of Pathological process most likely occurs in the:
organ disease does this indicate? A. Heart
A. Liver B. Skeletal muscles
B. Kidney C. Kidneys
C. Heart D. Liver
D. Lung E. Adrenal glands
E. Spleen
113. The activity of the isoenzymes LDG1 and
108. Blood test of the patient revealed albumine LDG2 appeared in the patient's blood plasma.
content of 20 g/l and increased activity of On the pathology of a body that indicates?
lactate dehydrogenase isoenzyme 5 (LDH5). A. Myocardium
These results indicate disorder of the following B. Liver
organ: C. Kidney
A. Liver D. Brain
B. Kidneys E. Skeletal muscle
C. Heart

40
114. A patient presents high activity of LDH1,2, B. Skeletal muscle dystrophy
aspartate aminotransferase, creatine C. Diabetes mellitus
phosphokinase. In what organ (organs) is the D. Viral hepatitis
development of apathological process the most E. Acute pancreatitis
probable?
A. In the heart muscle (initial stage of 119. A 60-year-old man consulted a doctor
myocardium infarction) about an onset of chest pain. In blood serum
B. In skeletal muscles (dystrophy, atrophy) analysis showed a significant increase in the
C. In kidneys and adrenals activity of the following enzymes: creatine
D. In connective tissue kinase and its MB-isoform, aspartate
E. In liver and kidneys aminotransferase. These changes indicate the
development of the pathological process in the
115. A 49-year-old driver complains about following tissues:
unbearable constricting pain behind the A. Cardiac muscle
breastbone irradiating to the neck. The pain B. Lungs
arose 2 hours ago. Objectively: the patient’s C. Skeletal muscles
condition is grave, he is pale, heart tones are D. Liver
decreased. Laboratory studies revealed high E. Smooth muscles
activity of creatinekinase and LDH1. What
disease are these symptoms typical for? 120. Hereditary hyperlipoproteinemia type I is
A. Acute myocardial infarction caused by lipoprotein lipase deficiency.
B. Acute pancreatitis Increasing the level of some transport forms of
C. Stenocardia lipids in plasma even on an empty stomach is
D. Cholelithiasis characteristic?
E. Diabetes mellitus A. Chylomicrons
B. Low density lipoproteins
116. 6 hours after the myocardial infarction a C. Very low density lipoproteins
patient was found to have elevated level of D. High density lipoproteins
lactate dehydrogenase in blood. What E. Modified lipoproteins
isoenzyme should be expected in this case?
A. LDH1 121. Cholesterol content in blood serum of a 12-
B. LDH2 year-old boy is 25 mmol/l. Anamnesis states
C. LDH3 hereditary familial hypercholesterolemia caused
D. LDH4 by synthesis disruption of receptor-related
E. LDH5 proteins for:
A. Low-density lipoproteins
117. A 47-year-old male patient was diagnosed B. High-density lipoproteins
with myocardial infarction in the intensive care C. Chylomicrons
unit. Which of the lactate dehydrogenase (LDH) D. Very low-density lipoproteins
fractions will prevail in the blood serum during E. Middle-density lipoproteins
the first two days? М-а-2016-176
A. LDH 1
B. LDH 2 122. Examination of the patient revealed an
C. LDG 3 increase in the content of low-density
D. LDH 4 lipoproteins in the blood serum. What disease
E. LDG 5 can be assumed in this patient?
A. Atherosclerosis
118. The high level of Lactate Dehydrogenase B. Kidney damage
(LDH) isozymes concentration showed the C. Acute pancreatitis
increase of LDH-1 and LDH-2 in a patient’s D. Gastritis
blood plasma. Point out the most probable E. Inflammation of the lungs
diagnosis:
A. Myocardial infarction

41
123. A 58-year-old patient suffers from the
cerebral atherosclerosis. Examination revealed 128. During examination of a teenager with
hyperlipoidemia. What class of lipoproteins will xanthomatosis the family history of
most probably show increase in concentration in hypercholesterolemia is revealed. What
this patient’s blood serum? transportable lipids are increased in
A. Low-density lipoproteins concentration in case of such a disease?
B. High-density lipoproteins A. Low-density lipoproteins
C. Fatty acid complexes with albumins B. Chylomicrons
D. Chylomicrons C. Very low-density lipoproteins
E. Cholesterol D. High-density lipoproteins
E. Intermediate-density lipoproteins
124. Rabbits lived on food with addition of
cholesterol. Five months later the 129. The level of which plasma protein allows
atherosclerotic aorta changes were revealed. retrospectively (for the previous 4-8 weeks
Name the main cause of atherogenesis in this before the examination) to assess the level of
case: glycemia, if the patient has diabetes mellitus,
A. Exogenous hypercholesterolemia which is accompanied by fasting hyperglycemia
B. Overeating more than 7.2 mmol / l?
C. Hypodynamia A. Glycosylated hemoglobin.
D. Endogenous hypercholesterolemia B. Albumin.
E. – C. C-reactive protein.
D. Ceruloplasmin.
125. Examination of an ill child’s blood E. Fibrinogen.
revealed inherited hyperlipoproteinemia.
Genetic defect of what enzyme synthesis causes 130. A patient is diagnosed with pancreatic
this phenomenon? diabetes with associated hyperglycemia.
A. Lipoprotein lipase Glycemia rate can be assessed retrospectively
B. Glycosidase (4-8 weeks prior to examination) by measuring
C. Proteinase concentration of the following blood plasma
D. Hemsynthetase protein:
E. Phenylalanine hydroxylase A. Glycated hemoglobin
B. Albumin
126. In the study of the patient's blood plasma 4 C. Fibrinogen
hours after ingestion of fatty foods, it was found D. C-reactive protein
that it is cloudy. The most likely cause of this E. Ceruloplasmin
condition is an increase in plasma
concentration: 131. A patient is ill with diabetes mellitus that is
A. Chylomicrons accompanied by hyperglycemia of over 7,2
B. HDL millimole/l on an empty stomach. The level of
C. LDL what blood plasma protein allows to estimate
D. Cholesterol the glycemia rate retrospectively (4-8 weeks
E. Phospholipids before examination)?
A. Glycated hemoglobin
127. Blood serum of the patient has milky B. Albumin
appearance. Biochemical analysis revealed high C. Fibrinogen
content of triacylglycerols and chylomicrons. D. C-reactive protein
This condition is caused by hereditary defect of E. Ceruloplasmin
the following enzyme:
A. Lipoprotein lipase 132. A 42-year-old woman, who has been
B. Phospholipase keeping to a vegetarian diet for a long period of
C. Pancreatic lipase time, consulted a doctor. Examination revealed
D. Adipose tissue hormone-sensitive lipase negative nitrogen balance in the patient. What
E. Phosphodiesterase

42
factor is the most likely cause of such a D. Nitrogen balance unchanged
condition? E. Ketonemia
A. Insufficient amount of proteins in the diet
B. Insufficient amount of dietary fiber 138. A patient with primary nephrotic syndrome
C. Excessive amount of fats in the diet has the following content of whole protein: 40
D. Insufficient amount of fats in the diet g/l. What factor caused hypoproteinemia?
E. Decreased rate of metabolic processses A. Proteinuria
B. Transition of protein from vessels to tissues
133. Examination of a 45-year-old man whohad C. Reduced protein synthesis in liver
kept to a vegeterian diet for a long time revealed D. Increased proteolysis
negative nitrogen balance.Which peculiarity of E. Disturbance of intestinal protein absorption
his diet is the causeof this phenomenon?
A. Lack of proteins 139. Examination of a patient with chronicrenal
B. Lack of fats insufficiency revealed an increase in residual
C. Excess of water nitrogen concentration in blood up to 35
D. Excess of carbohydrates millimole/l, more than half of which is urea.
E. Lack of vitamins What type of hyperazotemia is it?
A. Retentional
134. A 14 year old child was found to have a B. Hepatic
positive nitrogen balance. Which of the C. Productional
following could be the reason for this? D. Residual
A. Body growth E. Combined
B. Fasting
C. Reduced protein in food 140. On the basis of laboratory analysis, the
D. Significant physical loads patient confirmed the diagnosis of gout. To
E. Emotional stress establish the diagnosis was carried out
determination of the content:
135. A month after a serious operation a 38- A. Urinary acid in the blood and urine
year-old patient has recovered and has now B. Creatinine in urine
positive nitrogen balance. Urine of this patient C. Residual nitrogen in the blood
may be found to have low concentration of the D. Urea in the blood and urine
following nitrogencontaining substance: E. Ammonia in the urine
A. Urea
B. Lactate 141. A young man of 18 years old was
C. Stercobilinogen diagnosed with muscular dystrophy. Is the
D. Galactose increase in the serum content of the substance
E. 17-ketosteroids most likely in this pathology?
A. Creatine
136. A chiled was diagnosed with acuterenal B. Myoglobin
failure. What biochemic saliva indices can C. Myosin
confirm this diagnosis? D. Lactate
A. Increased level of rest nitrogen E. Alanin
B. Increase of immunoglobuline A
C. Reduction of alkaline phosphatase 142. A 46-year-old patient complains of dry
D. Increase of alpha amylase mouth, thirst, frequent urination, and general
E. Decreased level of phosphate weakness. In the blood: hyperglycemia,
hyperketonemia. In the urine: glucose, ketone
137. A patient who goes out of a state of bodies. On ECG: diffuse changes in the
prolonged fasting has determined nitrogen myocardium. What is the most likely diagnosis?
metabolism. What result can you expect? A. Diabetes
A. Reducing nitrogen excretion B. Alimentary hyperglycemia
B. Increased nitrogen excretion C. Acute pancreatitis
C. Nitrogen equilibrium D. Diabetes mellitus

43
E. Coronary heart disease 147. A 48 year old patient complained about
intense pain, slight swelling and reddening of
143. The patient after suffering parotitis lost skin over the joints, temperature rise up to
weight, constantly feeling thirsty, drinks a lot of 38oC. Bloodanalysis revealed high
water, notes frequent urination, increased concentration ofurates. This condition might be
appetite, pruritus, weakness, furunculosis. In the caused by disturbed metabolism of:
blood: glucose - 16 mmol / l, ketone bodies 100 A. Purines
mkmol / l. What disease has the patient B. Collagen
developed? C. Cholesterol
A. Insulin-dependent diabetes mellitus D. Pyrimidines
B. Insulin independent diabetes E. Carbohydrates
C. Steroid diabetes
D. Diabetes mellitus 148. A patient has increased content of uric acid
E. Diabetes mellitus malnutrition in his blood that is clinically presented by pain
syndrome as a result of urate deposition in the
144. A patient was delivered to the hospital by joints. What process does this acid result from?
an emergency team. Objectively: grave A. Lysis of purine nucleotides
condition, unconscious, adynamy. Cutaneous B. Lysis of pyrimidine nucleotides
surfaces are dry, eyes are sunken, face is C. Heme catabolism
cyanotic. There is tachycardia and smell of D. Proteolysis
acetone from the mouth. Analysis results: blood E. Reutilization of purine bases
glucose -20,1 micromole/l (standard is 3,3-5,5
micromole/l), urine glucose - 3,5% (standardis - 149. A 1,7-year-old child with a developmental
0). What is the most probable diagnosis? delay and manifestations of self-agression has
A. Hyperglycemic coma the concentration of uric acid in blood at the rate
B. Hypoglycemic coma of 1,96 millimole/l. What metabolic disoder is
C. Acute heart failure this typical for?
D. Acute alcoholic intoxication A. Lesch-Nyhan syndrome
E. Anaphylactic shock B. Podagra
C. Acquired immunodeficiency syndrome
145. A 38-year-old patient was admitted to the D. Gierke’s disease
intensive care unit in an unconscious state. E. Cushing’s basophilism
Reflexes are absent. Blood sugar - 2.1 mmol / l.
In history - diabetes mellitus from 18 years. 150. M-r S presents all signs of the hepatic
What kind of coma does the patient have? coma: loss of consciousness, absence of
A. Hypoglycemic reflexes, cramps, convulsion, disorder of heart
B. Ketoacidotic activity, recurrent (periodical) respiration. What
C. Lacticidemic are cerebrotoxical substances which accumulate
D. Hyperosmolar in blood under hepar insufficiency?
E. Hyperglycemic A. Ammonia
B. IL-1
146. A patient with a diagnosis of Itsenko- C. Autoantibody
Cushing's disease (hyperproduction of adrenal D. Necrosogenic substances
hormones) in the blood has an increased E. Ketonic body
concentration of glucose, ketone bodies,
sodium. What is the biochemical mechanism 151. After a serious viral infection a 3-year-old
leading to the occurrence of hyperglycemia? child has repeated vomiting, loss of
A. Gluconeogenesis consciousness, convulsions. Examination
B. Glycogenesis revealed hyperammoniemia. What may have
C. Glycogenolysis caused changes of biochemical blood indices of
D. Glycolysis this child?
E. Aerobic glycolysis A. Disorder of ammonia neutralization in
ornithinic cycle

44
B. Activated processes of aminoacids E. Pancreas
decarboxylation
C. Disorder of biogenic amines neutralization 156. Laboratory examination of a child revealed
D. Increased purtefaction of proteins in increased concentration of leucine, valine,
intestines isoleucine and their ketoderivatives in blood and
E. Inhibited activity of transamination enzymes urine. Urine smelt of maple syrup. This disease
is characterized by the deficit of the following
152. A 2-year-old child presents with mental enzyme:
development retardation, intolerance of A. Dehydrogenase of branched amino acids
proteins, severe hyperammonemia against the B. Aminotransferase
background of low blood urea content. This C. Glucose-6-phosphatase
condition is caused by the congenital deficiency D. Phosphofructokinase
of the following mitochondrial enzyme: E. Phosphofructomutase
A. Carbamoyl phosphate synthetase
B. Citrate synthase 157. A sick child presents with high content of
C. Succinate dehydrogenase phenyl pyruvate in urine (normally it is
D. Malate dehydrogenase practically absent). Blood phenylalanine level is
E. Monoamine oxidase 350 mg/L (norm - 15 mg/L). What disease are
these symptoms characteristic of?
153. A newborn presents with weak suckling, A. Phenylketonuria
frequent vomiting, and hypotonia. Blood and B. Albinism
urine citrulline are very high. What metabolic C. Tyrosinosis
process is disturbed? D. Alkaptonuria
A. Ornithine cycle E. Gout
B. Tricarboxylic acid cycle
C. Glycolysis 158. A hospital admitted a 9 y.o. boy with
D. Gluconeogenesis mental and physical retardation. Biochemical
E. Cori cycle blood analysis revealed high content of
phenylalanine. Such condition may be caused
154. Nitrogen is being excreted from thebody by blocking of the following enzyme:
mainly as urea. When activity of acertain A. Phenylalanine-4-monooxigenase
enzyme in the liver is low, it resultsin inhibition B. Oxidase of homogentisic acid
of urea synthesis and nitrogenaccumulation in C. Glutamine transaminase
blood and tissues. Namethis enzyme: D. Aspartate aminotransferase
A. Carbamoyl phosphate synthetase E. Glutamate decarboxylase
B. Aspartate aminotransferase
C. Urease 159. Blood of the patients with diabetes mellitus
D. Amylase shows increased content of free fatty acids.
E. Pepsin Name the most likely cause of this:
A. Increased activity of adipose triglyceride
155. The patient has a reduced content of lipase
indican in the blood serum, and also his daily B. Accumulation of palmitoyl-CoA in cytosol
excretion with urine is reduced. Impairment of C. Activation of ketone bodies utilization
which organ is the cause of this? D. Activation of apoА1, apoА2, and apoА4
A. Liver apolipoprotein synthesis
B. Kidney E. Decreased activity of plasma
C. Heart phosphatidylcholine-cholesterolacyltransferase
D. Lungs

45
Biochemistry of immune system
1. Differentiation of B-lymphocytes into plasma 5. Bone marrow has been transplanted to the
cells leads to synthesis of immunoglobulins that liquidator of Chernobyl atomic power station
ensure specific immune response of the body. accident which was irradiated. The reaction “a
Differentiation of B-lymphocytes takes place in transplant against a host” development was
the following organ of immune system: diagnosed at the patient after operation. Which
A. Tonsils antigens are the reason of this reaction?
B. Red bone marrow A. Antigens of HLA system in the cells of
C. Liver liquidator’s organism
D. Thymus B. Antigens of Rh system in the erythrocytes of
E. Thyroid gland liquidator
C. Antigens HBs, HBc, HBe
2. Among lymphocytes, there are populations of D. Antigens of ABO system in the erythrocytes
cells with membrane receptors for IgM, they are of liquidator
activated by the action of specific antigens, E. Antigens of HLA system in the cells of
mitotically multiply, and differentiate into donor’s spinal cord
plasma cells that produce antibodies
(immunoglobulins). What are these cells called? 6. A male patient has been diagnosed with acute
A. B-lymphocytes poststreptococcal glomerulonephritis. It is most
B. Memory T lymphocytes likely that the lesion of the basement membrane
C. T-killer lymphocytes of renal corpuscles was caused by the following
D. T-suppressor lymphocytes allergic reaction:
E. - A. Immune complex
B. Anaphylactic
3. A 2-year-old boy often became ill with C. Cytotoxic
respiratory diseases, stomatitis, pustular skin D. Delayed
lesions. Even small damage to the gums and E. Stimulating
mucous membranes is complicated by
prolonged inflammation. It is established that 7. A 5-year-old child is diagnosed with Bruton
the blood of the child is practically absent syndrome (X-linked agammaglobulinemia) that
immunoglobulins of all classes. Reduction manifests itself in severe clinical course of
the functional activity of a cell population is the bacterial infections and absence of B
basis of the syndrome described? lymphocytes and plasma cells. What changes of
A. B-lymphocytes immunoglobulin content can be observed in
B. T lymphocytes blood serum of the child with
C. Neutrophils immunodeficiency?
D. Macrophages A. Decreased IgA, IgM
E. NK lymphocytes B. Increased IgA, IgM
C. Decreased IgD, IgE
4. A 13-year-old boy presents with eczematous D. Increased IgD, IgE
rashes on his shins and torso. Anamnesis states E. No changes
cases of otitis, pneumonia, and furuncles in the
patient. Blood test: platelets - 70 • 109/l, low 8. Examination of a child who frequently suffers
activity of T helper and T suppressor cells, low from infectious diseases revealed that IgG
IgM, with normal IgA and IgG. What concentration in blood serum was 10 times less
immunodeficient disease does this boy have? than normal, IgA and IgM concentration was
A.Wiskott-Aldrich syndrome also significantly reduced. Analysis showed also
B. Louis-Bar syndrome (Ataxiatelangiectasia) lack of B-lymphocytes and plasmocytes. What
C. Severe combined immunodeficiency (Swiss disease are these symptoms typical for?
type) A. Bruton’s disease
D. DiGeorge syndrome B. Swiss-type agammaglobulinemia
E. Chediak-Higashi syndrome C. Dysimmunoglobulinemia
D. Louis-Bar syndrome
46
E. Di George syndrome B. Combined Immunodeficiency Syndrome
C. Secondary immunodeficiency syndrome
9. Parents of 5-year-old child report him o have D. Acute lymphocytic leukemia
frequent colds that develop into pneumonias, E. Syndrome of chronic intoxication
presence of purulent rashes on the skin.
Laboratory tests have revealed the following: 13. During the examination of the patient a
absence of immunoglobulins of any type, and insufficient quantity of immunoglobulins was
naked cells are absent from the lymph nodes detected. What kind of immune cells do they
punctate. What kind of immune disorder is it? produce?
A. X-linked hypogammaglobulinemia A. Plasma cells
(Bruton type agammaglobulinemia) B. T-helper
B. Autosomal recessive agammaglobulinaemia C. T-killers
(Swiss type) D. T-suppressors
C. Hypoplastic anemia E. Plasmoblasty
D. Agranulocytosis
E. Louis-Barr syndrome 14. Following exposure to radiation a lot of
mutant cells appeared in a patient. Some time
10. A child with suspected tuberculosis was later most of them were detected and destroyed
given Mantoux test. After 24 hours the site of by the following cells of the immune system:
the allergen injection got swollen, hyperemic A. T-lymphocytes-killers
and painful.What are the main components that B. Plasmoblasts
determine such response of the body? C. T-lymphocytes-supressors
A. Mononuclear cells, T-lymphocytes and D. B-lymphocyte
lymphokines E. Stem cells
B. Granulocytes, T-lymphocytes and IgG
C. Plasma cells, T-lymphocytes and 15. Numerous plasma cells were found in the
lymphokines blood girl 16 years old, suffering from an
D. B-lymphocytes, IgM autoimmune inflammation of the thyroid gland.
E. Macrophages, B-lymphocytes and monocytes With the proliferation and differentiation of
what blood cells cause increase of plasmocyte?
11. A 27-year-old woman has dropped penicillin A B-lymphocytes
containing eye drops. In a few minutes there B T-helper
appeared feeling of itching, burning of the skin, C Mast cells
lips and eyelids D T-killer
edema, whistling cough, decrease of BP. What E T-suppressor
imunoglobulins take part in the development of
this allergic reaction? 16. The patient is diagnosed with ARVI. Class
A. IgE and IgG M immunoglobulins have been found in the
B. IgM and IgG serum. What is the period of the infection
C. IgA and IgM process in this case?
D. IgM and IgD A. Acute
E. IgG and IgD B. Prodromally
C. Incubation
12. A 3 year old child with multiple impaired D. Reconvalescence
development of the bones of the facial skull E. Micro carrier
died. The cause of death is sepsis, which
developed on the background of 17. A 34-year-old patient, after suffering an
bronchopneumonia. Blood content intestinal infection caused by Salmonella,
immunoglobulins within the physiological symptoms of the disease began to fade. What
norm. At the autopsy revealed the absence of class of immunoglobulins will be detected in the
the thymus. Name the main cause of the child’s patient's blood during the recovery period?
illness: A. Ig G
A. Syndrome of cellular immunity deficiency B. Ig A

47
C. Ig D hypotension. When this reaction is developed
D. Ig E and the allergen achieves tissue basophils, it
E. Ig M reacts with:
A. IgE
18. Preventive vaccination against polyomyelitis B. IgА
is made with inactivated vaccine introduced C. IgD
parenterally. What immunoglobulins create the D. IgМ
postvaccinal immunity in the case? E. T-lymphocytes
A. IgM, IgG
B. IgG, secretory IgA 23. A 34-year-old patient, after suffering an
C. IgM, secretory IgA intestinal infection caused by Salmonella,
D. Serum IgA, IgM symptoms of the disease began to fade. What
E. Ig E, IgM class of immunoglobulins will be detected in the
patient's blood during the recovery period?
19. A 37-year-old man was injected with A. Ig G
novocaine solution in the treatment of acute B. Ig A
pulpitis. A few minutes later the patient C. Ig D
developed an anaphylactic shock.What D. Ig E
immunoglobulin does the antigen mainly E. Ig M
interact with in the body during a given allergic
reaction? 24. One of the functions of saliva is protective,
A. IgE which is realized by the formation of local
B. IgM immunity of the mucous membrane due to the
C. IgA secretion of the parotid glands of such a protein:
D. IgD A. Secretory Immunoglobulin A
E. IgG B. Collagen
C. Elastin
20. After the introduction of lidocaine, a 25- D. Fibrinogen
year-old patient developed shortness of breath, E. Albumin
bronchospasm, and his blood pressure dropped
sharply, which required the use of immediate 25. Various cells of the oral mucous membrane
relief from a dentist doctor. What mechanism is and antimicrobial substances synthesized by
the basis of such phenomena? these cells play an important part in the local
A. Allergic reactions involving IgE immunity of the oral cavity. Specify the key
B. Allergic cytotoxic reactions factors for the local immunity:
C. The phenomenon of idiosyncrasy A. Secretory IgA
D. Hypersensitivity due to T-lymphocytes B. B-lymphocytes
E. Effects caused by T-killers C. IgG
D. Macrophages
21. Skin samples of a patient with bronchial E. Eosinophils
asthma revealed allergen sensitization of poplar
fuzz. What factor of immune system plays the 26. In our country, routine preventive
main part in vaccinations against poliomyelitis ivolve using
development of this immunopathological state? live vaccine that is administered orally. What
A. IgE immunoglobulins are responsible for the
B. IgD development of local post-vaccination immunity
C. IgM in this case?
D. Sensitized Т-lymphocytes A. Secretory IgA
E. – B. IgM
C. IgG
22. A teenger had his tooth extracted under D. Serum IgA
novocain anaesthesia. 10 minutes later he E. IgE
presented with skin pallor, dyspnea,

48
27. Throughout a year a 37-year-old woman B. DNA → i-RNA → polypeptide → DNA
periodically got infectious diseases of bacterial C. DNA → polypeptide → i-RNA
origin, their course was extremely lingering, D. i-RNA → polypeptide → DNA
remissions were short. Examination revealed E. Polypeptide → RNA → DNA → i-RNA
low level of major classes of immunoglobulins.
The direct cause of this phenomenon may be the 32. A patient with acquired immunodeficiency
following cell dysfunction: syndrome (СНІД) the immunological reactivity,
A. Plasmocytes manifested by the development of chronic
B. Phagocytes inflammatory processes, infectious diseases,
C. Neutrophils tumor growth, is significantly reduced. What
D. Macrophages types of blood cells damages HIV-infections,
E. Lymphocytes which results in reduced immune protection?
A. T4 helper
28. A patient has been hospitalized with B. Natural Kileri (NK)
provisional diagnosis of virus B hepatitis. C. T-suppressors
Serological reaction based on complementation D. T8-Effects
of antigen with antibody chemically bound to E. B-lymphocytes
peroxidase or alkaline phosphatase has been
used for disease diagnostics. What is the name 33. A 20-year-old patient, an AIDS diagnosis
of the applied serological reaction? was established. What cell populations are most
A. Immune-enzyme analysis sensitive to human immunodeficiency virus?
B. Radioimmunoassay technique A. T-helpers
C. Immunofluorescence test B. Hepatocytes
D. Bordet-Gengou test C. Endotheliocytes
E. Antigen-binding assay D. Epithelial cells
E. B lymphocytes
29. A doctor examined a patient with recurrent
aphthous stomatitis with concominant 34. A 1-year-old child often suffers from viral
candidosis and decided to eliminate a possibility and bacterial infections that are difficult to treat.
of HIV-infection. What examination can help to During the investigation the immunological
clear the situation up and make a provisional status revealed the absence of lymphocytes in
diagnosis? the blood providing cellular immunity. What
A. Immune-enzyme analysis immunodeficiency and what system is detected
B. Gel precipitation reaction in this child?
C. Reaction of hemagglutination inhibition A. In a T cell primary response
D. Reaction of hemagglutination B. In a T-cell secondary response
E. Phase-contrasr microscopy C. In a B cell primary response
D. In a macrophage system primary response
30. T-lymphocytes are determined to be affected E. In a microphages primary response
with HIV. In this case viral enzyme reverse
transcriptase (RNA-dependent DNA- 35. Blood serum of a newborn contains
polymerase) catalyzes the synthesis of: antibodies to measles virus. What kind of
A. DNA based on the viral RNA matrix immunity is this indicative of?
B. Viral RNA based on the DNA matrix A. Natural passive
C. Viral protein based on the viral RNA matrix B. Natural active
D. Viral DNA based on the DNA matrix C. Artificial passive
E. Informational RNA based on the viral protein D. Artificial active
matrix E. Heredoimmunity

31. Lymphocytes are affected by HIV retrovirus 36. A child was born with cleft palate.
(AIDS). In this case, the direction of Examination revealed aorta defects and reduced
information flow in the cell will be: number of T-lymphocytes in blood. What
A. RNA → DNA → i-RNA → Polypeptide immunodeficient syndrome is it?

49
A. Di George A. Interleukin-1
B.Wiskott-Aldrich B. Immunoglobulins
C. Chediak-Higashi C. Interferons
D. Louis-Bar D. Biogenic amins
E. Swiss-type E. Angiotensin

37. A female patient underwent liver 42. Donor skin transplantation was performed to
transplantation. 1,5 month after it her condition a patient with extensive burns. On the 8-th day
became worse because of reaction of transplant the graft became swollen and changed colour;
rejection. What factor of on the 11-th day graft rejection started.What
immune system plays the leading part in this cells take part in this process?
reaction? A. T-lymphocytes
A. T-killers B. Erythrocytes
B. Interleukin-1 C. Basophils
C. Natural killers D. Eosinophils
D. B-lymphocytes E. B-lymphocytes
E. T-helpers
43. The patient contacted the dermatologist with
38. A patient consulted an immunologist about complaints about eczematous lesions of the skin
diarrhea, weight loss within several months, that appeared after contact with the detergent
low-grade fever, enlarged lymph nodes. The "Lotus". The use of rubber gloves prevents this.
doctor suspected HIV infection. What The pathological reaction of the skin is caused
immunocompetent cells must be studied in the by activation:
first place? A. T-lymphocytes
A. Helper T-lymphocytes B. B-lymphocytes
B. Suppressor T-lymphocytes C. Monocytes
C. B-lymphocytes D. Neutrophils
D. Monocytes E. Basohiles
E. Plasma cells
44. The development of febrile states are
39. After transfusion of 200 ml of blood a characterized by an increase in the level “the
patient presented with body temperature rise up acute phase” proteins. There are ceruloplasmin,
to 37,9°C. Which of the following substances is fibrinogen, C-reactive protein. Enter the
the most likely cause of temperature rise? Possible mechanism of this phenomenon:
A. Interleukin-1 A. Stimulating effect of IL-1 on hepatocytes
B. Interleukin-2 B. The destructive effect of temperature on the
C. Tumour necrosis factor cells of the body
D. Interleukin-3 C. The proliferative effect of IL-2 on
E. Interleukin-4 T lymphocytes
D. Degranulation of tissue basophils
40. A patient with skin mycosis has disorder of E. -
cellular immunity. The most typical
characteristic of it is reduction of the following 45. A pregnant woman with the previous
index: diagnosis of toxoplasmosis has been
A. T-lymphocytes hospitalized. What serological reaction should
B. Immunoglobulin G be used to determine toxoplasmosis in test
C. Immunoglobulin E specimens?
D. B-lymphocytes A. Complement fixation test.
E. Plasmocytes B. Neutralization
C. Haemadsorption
41. Inflammatory processes cause synthesis of D. Agglutination
protein of acute phase in an organism. What E. Inhibition of hemagglutination
substances simulate their synthesis?

50
Biochemistry of liver
1. Disorderthe splitting of the lipids in the small achilic syndrome due to obturation of the biliary
intestine is due to a disorder of lipase activity. tract. Which components of food will be
Which of the following factors activates the mastered the most?
lipase? A. Fats
A. Bile acids B.Carbohydrates
B. Hydrochloric acid C. Proteins
C. Enterokinase D. Nucleic acids
D. Pepsin E. Electrolytes
E. Solts of Na+
7. The drug "Geptral", which is used in liver
2. Examination of a patient revealed that dental diseases contains S-adenosylmethionine. This
hypoplasia was caused by hypovitaminosis of active amino acid is involved in the synthesis
vitamins A and D. These vitamins were of:
administered perorally but they didn’t have any A. Phospholipids
medicinal effect. What is the probable cause of B. Fatty acids
disturbed vitamin assimilation? C. Triacylglycerols
A. Bile acid deficiency D. Cholesterol
B. Phospholipase A2 deficiency E. Heme
C. Cholesterolesterase deficiency
D. Colipase deficiency 8. The patient appointed a lipotropic drug a
E. Pancreatic lipase deficiency donor of methyl groups, to prevent a fatty liver
distrophia. This is sensible:
3. A coprological study found that the feces are A. S-Adenosylmethionine
discolored, there are found drops of neutral fat. B. Cholesterol
The most likely cause of this is a violation: C. Bilirubin
A. Flow of bile into the intestine D. Valine
B. pH of gastric juice E. Glucose
C. Secretions of pancreatic juice
D. Secretion of intestinal juice 9. Examination of cell culture gotfrom a patient
E. Absorption processes in the intestine with lysosomal pathology revealed
accumulation of great quantity of lipids in the
4. After eating fatty foods, the patient has lysosomes. What of the following diseases is
nausea and heartburn, steatorrhea occurs. The this disturbance typical for?
reason for such a state can be: A. Tay-Sachs disease
A. Lack of bile B. Gout
B. Increased lipase secretion C. Phenylketonuria
C. Violation of trypsin synthesis D.Wilson disease
D. Lack of amylase E. Galactosemia
E. Impaired phospholipase synthesis
10. Steatosis occurs as a result of the
5. Due to the blockage of the common bile duct accumulation of triacylglycerols in hepatocytes.
(which was radiographically confirmed), the One of the mechanisms of development of this
biliary flow to the duodenum was stopped. We disease is reduction of utilization neutral fat
should expect the impairment of: LDL. What lipotropic substances prevent the
A. Fat emulsification development of steatosis? A. Methionine, BC,
B. Protein absorption B12
C. Carbohydrate hydrolysis B. Arginine, B2, B3
D. Secretion of hydrochloric acid C. Alanine, B1, PP
E. Salivation inhibition D. Valine, B3, B2
E. Isoleucine, B1, B2
6. A 65-year-old patient suffers from
cholelithiasis. Recently, there were signs of
51
11. In an experimental animal, receiving non- E. Leucine
protein diet, fatty infiltration of the liver has
developed due to the deficiency of the 16. The gluconeogenesis is activated in the liver
methylating agents. The formation of which after intensive physical trainings. What substance is
metabolite is disturbed in the experimental utilized in gluconeogenesis first of all in this case:
animal? A. Lactate
A. Choline B. Pyruvate
C. Glucose
B. DOPA
D. Glutamate
C. Cholesterol E. Alanine
D. Acetoacetate
E. Linolic acid 17. In patients with glycogenosis, that is von
Gierke’s disease, the conversion of glucose-6-
12. A dry cleaner’s worker has been found to phosphate in to glucose is inhibited, which is
have hepatic steatosis. This pathology can be accompanied by the improper breakdown of
caused by the disruption of synthesis of the glycogen in the liver. The cause of this
following substance: condition is the following enzyme deficiency:
A. Phosphatidylcholine A. Glucose-6-phosphatase
B. Tristearin B. Glycogen phosphorylase
C. Urea C. Glucose-6-phosphate dehydrogenase
D. Phosphatidic acid D. Phosphofructokinase
E. Cholic acid E. Phosphoglucomutase
13. In the 2-year-old boy, an increase in the size 18. A child has a history of hepatomegaly,
of the liver and spleen, cataract is observed. The hypoglycemia, seizures, especially on an empty
concentration of sugar is elevated in the blood, stomach and in stressful situations. The child is
but the test of glucose tolerance is normal. An diagnosed with Gierke disease. This disease is
hereditary violation of the metabolism of which caused by the genetic defect of the following
substance is the cause of this condition? enzyme:
A. Galactose A. Glucose-6-phosphatase
B. Fructose B. Amyloid-1,6-glycosidase
C. Glucose C. Phosphoglucomutase
D. Maltose D. Glycogen phosphorylase
E. Saccharose E. Glucokinase
14. During starvation normal rate ofglucose is
maintained by means ofactivation of 19. A patient with chronic hypoglycemia had
gluconeogenesis. Whatsubstance can be used as adrenaline introduction. After introduction
a substrate forthis process? blood test hasn’t changed essentially. Doctor
A. Alanine assumed liver pathology. What liver function
B. Ammonia may have been changed?
C. Adenine A. Function of glycogen depositing
D. Urea B. Function of cholesterin production
E. Guanine C. Ketogenic function
D. Glycolytic function
15. In a patient undergoing a course of medical E. Excretory function
starvation, the normal level of glucose in the
blood is maintained mainly due to 20. After the introduction of adrenaline in a
gluconeogenesis. From what amino acids at the patient with persistent hypoglycemia, blood test
same time in the human liver most actively has not changed significantly. In such
synthesized glucose? circumstances, there is a possibility of
A. Alanine disturbances in the liver. What function of the
B. Lysine liver is changed?
C. Valine A. Glycogendeposing.
D. Glutamic acid B. Glycolytic.
52
C. Excretory. B. Increase of globulin conten
D. Ketogenic. C. Decrease of fibrinogen conten
E. Cholesterol-forming. D. Increase of albumin conten
E. Decrease of globulin conten
21. In the human diet there are a large number
of carbohydrates. The number of which 26. A 25-year-old patient has been diagnosed
structures will increase in cytoplasm of with chronic hepatitis. The patient complains of
hepatocytes? 10 kg weight loss within 2 months. Objectively:
A. Glycogen granules the patient has dry peeling skin, pale with
B. Drops of fat yellow shade, petechial haemorrhages,
C. The lysosomes stomatorrhagia. Petechial haemorrhages and
D. Free ribosomes stomatorrhagia are caused by the disturbance of
E. Inclusion of lipofuscine the following hepatic function:
A. Protein synthesizing
22. The main part of nitrogen is taking out of B. Chromogenic
organism as element of urea. Reducing the C. Glycogen synthesizing
activity of what liver enzyme leads to inhibition D. Detoxication
of the synthesis of urea and increasing the E. Depositing
accumulation of ammonia in blood and tissues?
A. Carbamoyl phosphate synthase 27. Hepatitis has led to the development of
B. Aspartate aminotransferase hepatic failure. Mechanism of edemata
C. Urease formation is activated by the impairment of the
D. Amilaza following liver function:
E. Pepsin A. Protein-synthetic
B. Barrier
23. In the biosynthesis of urea in the liver, the C. Chologenetic
formation of ornithine and urea is stimulated. D. Antitoxic
What amino acid is the intermediate product of E. Glycogen-synthetic
this synthesis?
A. Аrginine 28. A patient being treated for viral hepatitis
B. Leicinc. type B got symptoms of hepatic insufficiency.
C. Сitrate. What blood changes indicative of protein
D. Valin. metabolism disorder will be observed in this
E. Tryptophan. case?
A. Absolute hypoalbuminemia
24. In a boy of 4 years after suffering from B. Absolute hyperalbuminemia
severe viral hepatitis, vomiting, episodes of C. Absolute hyperfibrinogenemia
nephropathy, seizures are observed. In the blood D. Proteinic blood composition is unchanged
there is hyperammonia. Violation of which of E. Absolute hyperglobulinemia
the biochemical process in the liver caused such
a condition of the patient? 29. In obstructive jaundice and bile ducts,
A. Disposal of ammonia. prothrombin insufficiency is often observed.
B. Decarboxylation of amino acids. With a deficiency in the organism of which
C. Disposal of Biogenic Amines. vitamin it is connected?
D. Synthesis of alphabets. A. K
E. Gluconeogenesis. B. B6
C. A
25. Upon toxic damage of hepatic cells resulting D. C
in disruption of liver function the patient E. E
developed edemas. What changes of blood
plasma are the main cause of edema 30. A patient has been admitted to the
development? contagious isolation ward with signs of jaundice
A. Decrease of albumin content caused by hepatitis virus.Which of the

53
symptoms given below is strictly specific for syndrome. Which type of jaundice is developed
hepatocellular jaundice? in this patient?
A. Increase of ALT, AST level A. Gallstone
B. Hyperbilirubinemia B. Hemolytic jaundice
C. Bilirubinuria C. Hepatitis
D. Cholemia D. Chronic gastritis
E. Urobilinuria E. Chronic colitis

31. A 15-year-old boy has been diagnosed with 36. The patient turned to doctor with
acute viral hepatitis. What blood value should disturbances in the right side of the subarea.
be determined to confirm acute affection of After examination of the patient, the doctor was
hepatic cells? found yellow sclera. Laboratory diagnostic
A. Aminotransferase activity (AST, ALT) revealed increased ALT, negative stercobilin in
B. Unconjugated and conjugated bilirubin feces. Which type of disease is developed these
content symptoms?
C. Erythrocytes sedimentation rate (ESR) A. Hepatitis
D. Cholesterol content B. Haemolytic jaundice
E. Protein fraction content C. Chronic gastroduodenitis
D. Chronic colitis
32. Blood analysis of a patient with jaundice E. Chronic gastritis
reveals conjugated bilirubinemia, increased
concentration of bile acids. There is no 37. The 20-years-old man is diagnosed with
stercobilinogen in urine. What type of jaundice heredity deficiency of UDP-
is it? glucuronosyltransferase. Change in which blood
A. Obstructive jaundice parameter can prove this diagnosis?
B. Hepatocellular jaundice A. Indirect (unconjugated) bilirubin
C. Parenchymatous jaundice B. Direct (conjugated) bilirubin
D. Hemolytic jaundice C. Urobilin
E. Cythemolytic jaundice D. Stercobilinogen
E. Animal indican
33. Encephalopathy has developed in a child
with hemolytic disease of the newborn. What 38. The preterm newborn has a jaundice. Which
substance had increased in the child’s blood, enzyme is probably deficient in this baby?
resulting in damage to the CNS? A. UDP-glucosyltransferase
A. Unconjugated bilirubin B. alkaline phosphatase
B. Bilirubin-albumin complex C. acidic phosphatase
C. Bilirubin glucuronide D. catalase
D. Verdohemoglobin E. NAD+-dehydrogenase
E. Bileacids
39. The born in time newborn has yellow skin
34. Barbiturates prescribed for jaundice and mucus color. The most probable reason of
treatment because it induces UDP- this condition is temporary insufissiency of the
glucuronosyltransferase synthesis. Which following enzyme:
substance formation provide therapeutic effect? A. UDP-glucosyltransferase
A. Direct (conjugated) bilirubin B. Glucuronosyltransferase
B. Indirect (unconjugated) bilirubin C. Heme synthase
C. Biliverdin D. Heme oxygenase
D. Protoporphyrin E. Biliverdin reductase
E. Heme
40. The newborn has a physiological jaundice.
35. The patient came to doctor with compliance The level of blood free bilirubin considerably
about yellow sclera and skin. No higher than normal. What enzyme deficiency
encephalopathy, cholestasis or acholic can cause this

54
A. UDP-glucosyltransferase bilirubin is normal. Which type of jaundice this
B. Transaminase patient is most probably develop?
C. Xanthine oxidase A. Hemolytic
D. Adenosine deaminase B. Parenchymatous (hepatic)
E. Heme oxygenase C. Mechanical
D. Newborn jaundice
41. Transfusion of Rh-incompatible blood E. Gilbert's syndrome
resulted in hemolytic jaundice development in
the patient. What laboratory blood value 46. The patient develops yellowish of skin and
confirms this type of jaundice? mucus membranes. Analysis of blood plasma
A. Accumulation of unconjugated bilirubin revealed increased total bilirubin, feces –
B. Reduction of unconjugated bilirubin increased stercobilin, urine – increased urobilin.
C. Accumulation of urobilinogen What type of jaundice this patient has?
D. Reduction of stercobilin A. Hemolytic
E. Reduction of conjugated bilirubin B. Gilbert's syndrome
C. Parenchymatous
42. The patient has a following symptoms: D. Obstructive
yellow skin, dark urine, yellow-dark feces. E. Cholestatic
Which substance you will predict to be
increased in blood serum? 47. After blood transfusion patient has
A. Free bilirubin yellowish of skin and mucus membrane,
B. Conjugated bilirubin increased total and indirect bilirubin, increased
C. Mesobilirubin urine urobilin and feces stercobilin. What kind
D. Verdoglobin of jaundice can be the reason?
E. Biliverdin A. Hemolytic jaundice.
B. Newborn jaundice.
43. A 46 year old woman suffering from C. Obstructive jaundice.
chololithiasis developed jaundice. Her urine D. Parenchymatous jaundice.
became dark-yellow and feces became E. Genetic jaundice.
colourless. Blood serum will have the highest
concentration of the following substance: 48. The patient who suffers from jaundice
A. Conjugated bilirubin because of increased bilirubin as part of its
B. Unconjugated bilirubin indirect fraction. Urine and feces have intensive
C. Biliverdin color. What is the most probable explanation to
D. Mesobilirubin these disruptions?
E. Urobilinogen A. Increased erythrocyte hemolysis
B. Disrupted liver urobilinogen transformation.
44. Enzymatic jaundices are accompanied by C. Disruption of direct bilirubin formation.
abnormal activity of UDPglucuronyl D. Liver parenchyma damage.
transferase. What compound is accumulated in E. Disruption of bile flow from liver
blood serum in case of these pathologies?
A. Unconjugated bilirubin 49. The 28-years-old woman came to infectious
B. Conjugated bilrubin department with yellow skin, sclera, mucus
C. Dehydrobilirubin membrane. Laboratory diagnosis revealed
D. Hydrobilirubin increased blood direct bilirubin; urobilinogen
E. Choleglobin and bilirubin in urine. Which of following
condition can lead to those symptoms?
45. The 20-years-old patient has a jaundice and A. Parenchymatous jaundice
the following laboratory parameter: increase B. Hemolytic jaundice
general blood bilirubin, predominantly by C. Kidney infarction
indirect (free) fraction; urine and feces has a D. Renal tuberculosis
high stercobilin; blood direct (conjugated) E. Mechanical jaundice

55
50. The 28-years old man came to doctor to 54. The patient has yellow skin, skin rash,
complaints about acute disturbances in the right general weakness. In the urine: there is no
side of the subarea, nausea, vomiting. urobilin. Which type of pathology this patient is
Objectively: jaundice of the skin and sclera; the most probably develop?
temperature of the body is elevated, the urine is A. Mechanical jaundice
dark color. Feces is hypochloric. The patient B. Parenchymal jaundice
have hyperbilirubinemia (bilateral and indirect C. Acute liver insufficiency
biliary), bilirubinuria, urobilinuria, D. Haemolytic jaundice
gipoproteinemiya, reduction of blood E. Chronic liver insufficiency
transfusion. Which of following condition can
lead to those symptoms? 55. The 43-years-old patient during 10 years has
A. Hemolitic jaundice a repetitive compliance about acute bellyache,
B. Subacute jaundice cramps, vision problems. His relatives have
C. Suprachondrial hemolytic jaundice Patient urine have red color. He was diagnosed
D. Acute cholecystitis with acute intermittent porphyria. The reason of
E. Acute pancreatitis he his disease is not being a vampire but
disrupted biosynthesis of:
51. The 48-years-old woman was delivered to A. Heme
hospital with compliance of general weakness, B. Bile acids
sleep disturbance. Objectively: skin and sclera C. Insulin
are yellow. Blood analysis has increased total D. Collagen
bilirubin with direct bilirubin predominance. E. Prostaglandin
Feces is acholic. Urine has dark color, because
of bile pigments. Which type of jaundice this 56. The patients with erythropoietic porphyria
patient has? (Gunther disease) have ultraviolet red-
A. Mechanical fluorescent tooth, sun-sensitive skin, red-
B. Hemolytic colored urine. Which enzyme deficiency cause
C. Parenchymatous this disease?
D. Gilbert's syndrome A. Uroporphyrinogen III synthase
E. Crigler–Najjar syndrome B. δ- aminolevulinic acid synthase
52. The patient was delivered to hospital with C. Uroporphyrinogen I synthase
complaints about general weakness and sleep D. Uroporphyrinogen decarboxylase
disturbance. Skin has yellow color. There is E. Ferrochelatase
increased blood direct bilirubin and bile acids.
Feces is acholic. Which type of disease can 57. The patient who suffers from anemia has
provoke those changes? increased protoporphyrin IX. Which mineral
A. Mechanical jaundice deficiency can lead to such pathology?
B. Hemolytic jaundice A. Iron
C. Pre-hepatic jaundice B. Potassium
D. Gilbert's syndrome C. Magnesium
E. Chronic cholecestitis D. Sodium
E. Phosphorus
53. The patient with jaundice has increased
direct bilirubin and bile acids in blood; no 58. The patient has increased photosensitivity,
stercobilinogen was revealed in urine. What after prolong sun exposure urine starts to get
type of jaundice can be the reason of these dark-red color. Which type of disease has this
symptoms? characteristic?
A. Mechanical A. Porphyria
B. Hepatic B. Alkaptonuria
C. Parenchymatous C. Albinism
D. Hemolytic D. Hemolytic jaundice
E. Post-hepatic E. Pellagra

56
59. The 33 years old patient worries 10 years.
Periodically, he addresses the doctor with 63. Detoxification of xenobiotics (drugs,
complaints of acute abdominal pain, epoxides, the arena oxides, aldehydes, nitro
convulsions, visual impairment. His relatives derivatives, etc.) and endogenous metabolites
are witnessing similar symptoms. Urine red. (estradiol, prostaglandins, leukotrienes) takes
Hospitalized with diagnoses - acute diarrhea place in the liver by conjugation with:
porphyria. The cause of the disease may be a A. Glutathione
violation of the biosynthesis of such a B. Aspartic acid
substance: C. Glycine
A. Gem D. S-Adenozylmethionin
B. Insulin E. Fosfoadenozyn
C.Chicken acids 64. For a patient suffering from chronic
D. Prostaglandins hepatitis, a load of sodium benzoate was carried
E. Collagen out to evaluate the degreasing function of the
liver. For the discharge of what substances with
60. A 43-year-old woman complains of general urine judge the neutralizing function of the
weakness, weight loss, apathy, and drowsiness. stomach?
Chronic lead intoxication confirmed laboratory - A. Hippuric acid
revealed hypochromic anemia. In the blood, an B. Phenylacetic acid
increased level of Zn-protoporphyrin and a C. Citric acid
reduced level of alpha-aminolevulinic acid, D. Valerian acid
indicating a violation of the synthesis: E. Oxalic acid
A. Heme
B. DNA 65. Patient has cirrhosis of the liver. The study
C. RNA of which of the listed substances that are
D. Protein excreted in the urine can characterize the state
E. Mevalonic acid of the antitoxic function of the liver?
A. Hippuric acid
61. A mother consulted a doctor about her 5- B. Ammonium salts
year-old child who develops erythemas, C. Creatinine
vesicular rash and skin itch under the influence D. Uric acid
of sun. Laboratory studies revealed decreased E. Amino acids
iron concentration in the blood serum, increased
uroporphyrinogen I excretion with the urine. 66. A woman of a chemical enterprise due to
What is the most likely inherited pathology in violation of the rules of safe work has suffered a
this child? toxic action of nitric acid and nitrites, which
A. Erythropoietic porphyria cause deamination of cytosine in the DNA.
B. Methemoglobinemia What enzyme initiates a chain of reparative
C. Hepatic porphyria processes?
D. Coproporphyria A. Uridine-DNA-glucosidase
E. Intermittent porphyria B. Cytidine triphosphate synthetase
C. Orthotidyl monophosphate-decarboxylase
62. A patient suffering from chronic hepatitis D. DNA-Dependent-RNA Polymerase
complains of increasing the sensitivity to E. Timidilatsintaza
barbiturates, which were earlier tolerated
without symptoms of intoxication. With the 67. In the 70's, scientists found that the cause of
disturbance of which of the liquor functions, severe jaundice of newborns is a disturbance of
this is connected at the highest possible level? the binding of bilirubin in hepatocytes. What
A. Detoxification substance is used to form a conjugate?
B. Hemopoietic A. Glucuronic acid
C. Hemodynamic B. Uric acid
D. Formation of bile C. Sulfuric acid
E. Phagocytic D. Lactic acid

57
E. Pyruvic acid
71. In the liver, detoxification of natural
68. A 50-year-old patient with food poisoning metabolites and xenobiotics is impaired. Name
was prescribed a dropper with a 10% glucose cytochrome, the activity of which can be
solution. It not only provides the energy needs reduced:
of the body, but also performs a detoxification A. Cytochrome P-450
function due to the formation of a metabolite B. Cytochrome oxidase
that participates in the conjugation reaction: C. Hemoglobin
A. Gluuronation D. Cytochrome b
B. Sulfonation E. Cytochrome c1
C. Methylation
D. Glycosylation 72. Study of conversion of a food colouring
E. Hydroxylation agent revealed that neutralization of this
xenobiotic takes place only in one phase -
69. To determine the antitoxic function of the microsomal oxydation. Name a component of
liver, the patient is assigned sodium benzonate, this phase:
which in the liver turns into hippuric acid. What A. Cytochrome Р-450
compound is used for this? B. Cytochrome B
A. Glycine C. Cytochrome C
B. Methionine D. Cytochrome A
C. Cysteine E. Cytochrome oxidase
D. FAFS
E. UDF-glucuronic acid 73. The universal biological system of oxidation
of nonpolar compounds (drugs, toxic
70. In a patient with chronic alcoholic liver compounds), steroid hormones, cholesterol is
disease, processes of xenobiotics and microsomal oxidation. What is the name of the
endogenous toxic compounds biotransformation cytochrome, which is part of the oxygenase
are disturbed. Reducing the activity of which chain of the microsome.
chromoprotein can be the cause of this? A. Cytochrome P 450.
A. Cytochrome P-450 B. Cytochrome a3.
B. Hemoglobin C. Cytochrome in.
C. Cytochrome Oxidase D. Cytochrome c.
D. Cytochrome b E. Cytochrome a.
E. Cytochrome c1

58
Pathologic urine components
1. A 13 year-old boy is complaining about A. Cystinuria.
general weakness, dizziness, fatigue. Mental B. Alpathururia.
retardation is detected. The examination C. Cystitis
revealed high concentrations of valine, D. Phenylketonuria.
isoleucine, and leucine in blood and urine. The E. Hartnup's disease.
urine has a specific smell. What is the most
possible diagnosis? 6. A 36-year-old female patient has a history of
A. Maple syrup urine disease collagen disease. Urine analysis is likely to
B. Addison’s disease reveal an increased concentration of the
C. Tyrosinosis following metabolite:
D. Histidinemia A. Oxyproline
E. Grave’s disease B. Indican
C. Creatinine
2. Excess of phenylpyruvate and phenylacetate D. Urea
was detected in the urine sample of a 6 day-old E. Urobilinogen
baby. Which amino acid metabolism is
disturbed? 7. A patient with systemic scleroderma has an
A. Phenylalanine intensified collagen destruction. Collagen
B. Tryptophan destruction will be reflected by intensified
C. Methionine urinary excretion of the
D. Histidine following amino acid:
E. Arginine A. Oxyproline
B. Alanine
3. In a sick child, a raised level of C. Tryptophane
phenylpyruvate was detected in the urine (in the D. Serine
norm practically absent). The amount of E. Phenylalanine
phenylalanine in the blood is 350 mg/l (about 15
mg/l). Which disease is characterized by the 8. A child has physical and mental retardation,
above symptoms? serious abnormalities in connective tissue of
A. Fenilketonuria. internal organs; urine contains keratan sulfates.
B. Albin. This is caused by metabolic disorder of the
C. Alpathonuria. following substance:
D. Gout A. Glycosaminoglycan
E. Tyrosinosis. B. Collagen
C. Elastin
4. A child with mental and physical D. Fibronectin
development retardation was delivered to the E. Hyaluronic acid
hospital. Phenylpyruvate was found in the urine.
Disturbance of which process is the reason for 9. A traumatology unit received a patient with
this pathology? crushed muscular tissue. What biochemical
A. Amino acid metabolism indicator of urine will be raised in this case?
B. Lipid metabolism A. Creatinine
C. Carbohydrate metabolism B. Total lipids
D. Water-salt balance regulation C. Glucose
E. Calcium and phosphate homeostasis D. Mineral salts
E. Uric acid
5. A 12-year-old boy in the urine revealed high
levels of all amino acids in the aliphatic row. In 10. A biochemical urine analysis has been
this case, the highest excretion of cysteine and performed for a patient with progressive
cysteine was noted. In addition, ultrasound of muscular dystrophy. In the given case muscle
the kidneys showed the presence of stones in disease can be confirmed by the high content of
them. Choose a possible pathology. the following substance in urine:
59
A. Creatine B. Phenylalanine
B. Porphyrin C. Alanine
C. Urea D. Methionine
D. Hippuric acid E. Asparagine
E. Creatinine
16. The 28-years old man came to doctor to
11. A biochemical urine analysis has been complaints about acute disturbances in the right
performed for a patient with progressive side of the subarea, nausea, vomiting.
muscular dystrophy. In the given case muscle Objectively: jaundice of the skin and sclera; the
disease can be confirmed by the high content of temperature of the body is elevated, the urine is
the following substance in urine: dark color. Feces is hypochloric. The patient
A. Creatine have hyperbilirubinemia (bilateral and indirect
B. Porphyrin biliary), bilirubinuria, urobilinuria,
C. Urea gipoproteinemiya, reduction of blood
D. Hippuric acid transfusion. Which of following condition can
E. Creatinine lead to those symptoms?
A. Hemolitic jaundice
12. A baby has coloured sclera and mucouses B. Subacute jaundice
and her urine is darkening when exposed to air. C. Suprachondrial hemolytic jaundice
Homogentisic acid was found in urine and blood D. Acute cholecystitis
samples. What is the reason for this condition? E. Acute pancreatitis
A Alcaptonuria
B. Albinism 17. A patient has an increased pyruvate
C. Galactosemia concentration in blood. A large amount of it is
D. Cystinuria excreted with the urine. What vitamin is lacking
E. Histidinemia in this patient?
A. B1
13. A 5-year old child has darkened urine but no B. E
bile enzymes were found in it. The child is C. B3
diagnosed with alcaptonuria. Which enzyme D. B6
deficiency is the reason for it? E. B2
A. Homogentisic acid oxydase
B. Phenylalanine hydroxylase 18. A patient, who has been subsisting
C. Tyrosinase exclusively on polished rice, has developed
D. Oxyphenylpyruvate oxydase polyneuritis due to thiamine deficiency. What
E. Phenylpyruvate decarboxylase substance is an indicator of such avitaminosis,
when it is excreted with urine?
14. A lab test revealed increased concentration A. Pyruvic acid
of leucine, valine, isoleucine and ketone B. Malate
derivatives in the blood and urine. The urine has C. Methylmalonic acid
a specific smell of maple syrup. Which enzyme D. Uric acid
deficiency is typical for this condition? E. Phenyl pyruvate
A. Amino acid dehydrogenase
B. Aminotranspherase 19. A patient 43 years old with chronic atrophic
C. Glucose-6-phosphatase gastritis and hyperchromic megaloblastic
D. Phosphofructokinase anemia increased methylmalonic acid excretion
E. Phosphofructomutase in the urine. The lack of which vitamin caused
the occurrence of this symptom complex?
15. Excretion of homogentisic acid with the A. В12
urine is the reason for the condition called B. В2
alcaptonuria. Disturbance of which amino acid C. В3
metabolism causes this condition? D. В5
A. Tyrosine E. В6

60
components are revealed in course of laboratory
20. A hereditary disease - homocystinuria - is examination of the patient’s urine?
caused by disturbed transformation of A. Glucose, ketone bodies
homocysteine in to methionine. Accumulated B. Protein, aminoacids
homocysteine forms its dimer (homocystine) C. Protein, creatine
that can be found in urine. What vitamin D. Bilirubin, urobilin
preparation can decrease homocysteine E. Blood
production?
A. Vitamin B12 25. A patient at the early stage of diabetes
B. Vitamin C mellitus was found to have polyuria.What is its
C. Vitamin B1 cause?
D. Vitamin B2 A. Hyperglycemia
E. Vitamin PP B. Ketonemia
C. Hypocholesterolemia
21. In a newborn child there is a decrease in D. Hypercholesterolemia
intensity of sucking, frequent vomiting, E. Hyperkaliemia
hypotension. In urine and blood, the
concentration of citrulline is significantly 26. At the reception to the therapist came a man
increased. What metabolic process is affected? of 37 years of age with complaints of periodic
A. Ornithine cycle intense pain in the joints of the thumb and their
В. Glycolysis swelling. In the urine: slightly acidic reaction
C. Core's cycle and pink color. With the presence of which
D. СТК substances can these changes be associated?
E. Gluconeogenesis A. Salt of uric acid
B. Chlorides
22. In the urine of the newborn, determined by C. Amoniev's salt
citrulline and high levels of ammonia. Indicate D. Phosphate calcium
which substance is most likely to be inflicted on E. Magnesium sulphate
a baby?
A. Urea. 27. On the basis of laboratory analysis, the
B. Bilirubin. patient was diagnosed with gout. Based on
C. Creatine. laboratory analysis, the patient confirmed the
D. Creatinine. diagnosis of gout. What was the analysis for the
E. Uric acid. diagnosis?
A. Determination of uric acid in the blood and
23. According to the results of glucose tolerance urine
test a patient has no disorder of carbohydrate B. Determination of urinary creatinine
tolerance. Despite that glucose is detected in the C. Determination of residual nitrogen in the
patients’s urine (5 mmol/l). The patient has been blood
diagnosed with renal diabetes. What renal D. The determination of urea in the blood and
changes cause glucosuria in this case? urine
A. Decreased activity of glucose reabsorption E. Determination of urine ammonia
enzymes
B. Increased activity of glucose reabsorption 28. A patient has been operated due to acute
enzymes abdomen. His urine is brown, concentration of
C. Exceeded glucose reabsorption threshold indican is higher than 93 mmol/day. What does
D. Increased glucose secretion it indicate?
E. Increased glucose filtration A. High intensity of protein decomposition
processes in the intestine
24. A 42 year old woman diagnosed with B. Decreased activity of the urea cycle enzymes
diabetes mellitus was admitted the C. Acceleration of aromatic amino acids
endocrinological department with complaints of desamination
thirst, excessive appetite. What pathological D. Disturbance of the kidney absorption

61
E. Decrease of ammonia detoxification
34. Analysis of a dentist’s urine obtained at the
29. A 43 year-old woman has been operated due end of his working day revealed protein
to acute abdomen. Her urine is brown and the concentration at the rate of 0,7 g/l. His morning
concentration of indican in the blood has urine hadn’t such changes. What is this
increased rapidly. What does it indicate? phenomenon called?
A.High intensity of protein decomposition A. Functional proteinuria
B. Acceleration of amino acids desamination B. Organic proteinuria
C. Supression of glomerular filtration C. Nonselective proteinuria
D. Decreased intensity of the urea cycle D. Extrarenal proteinuria
E. Inhibition of gluconeogenesis E. Hematuria

30. A 60 year-old man suffers from chronic 35. Chronic glomerulonephritis was diagnosed
intestinal obstruction. The processes of protein in a 34-year-old patient 3 years ago. Edema has
decomposition in the large intestine are developed within the last 6 monthes. What
intensified. What is the evidence of these caused the edema?
processes? A. Proteinuria
A. Indicanuria B. Hyperproduction of vasopressin
B. Bilirubinuria C. Liver disfunction of protein formation
C. Hyperuricosuria D. Hyperosmolarity of plasma
D. Creatinuria E. Hyperaldosteronism
E. Glucosuria
36. Glucose concentration in a patient’s blood is
31. A newborn child gains weight very slowly, 15 millimole/l (reabsorption threshold is 10
his urine contains too much orotic acid that is millimole/l). What effect can be expected?
indicative of disturbed synthesis of pyrimidine A. Glucosuria
nucleotides. What metabolite should be used in B. Diuresis reduction
order to normalize metabolism? C. Reduced glucose reabsorption
A. Uridine D. Reduced vasopressin secretion
B. Adenosine E. Reduced aldosterone secretion
C. Guanosine
D. Thymidine 37. A patient with rheumatoid arthritis has been
E. Histidine given hydrocortisone for a long time. He has
developed hyperglycemia, polyuria, glycosuria,
32. In orоtaciduria the release of Orotic acid is thirst. These complications of treatment result
many times higher than normal. Synthesis of from the activation of the following process:
what substances will be disturbed in this A. Gluconeogenesis
pathology? B. Glycogenolysis
A. Pyrimidine nucleotides. C. Glycogenesis
B. Biogenic Amines. D. Glycolysis
C. Purine nucleotides. E. Lipolysis
D. Urea
E. Uric acid 38. A 28-year-old patient with Itsenko-Cushing
syndrome Hyperglycemia, glycosuria were
33. The patient has increased photosensitivity, detected hyperglycemia, glycosuria. The main
after prolong sun exposure urine starts to get mechanism of hyperglycemia in this patient is
dark-red color. Which type of disease has this stimulation:
characteristic? A. Gluconeogenesis
A. Porphyria B. Liver glycogenolysis
B. Alkaptonuria C. Muscle glycogenolysis
C. Albinism D. Intestinal glucose absorption
D. Hemolytic jaundice E. Synthesis of glycogen
E. Pellagra

62
39. Fructosuria is known to be connected with E. Sphingomyelinase
inherited deficiency of fructose-1-phosphate
aldolase.What product of fructose metabolism 44. A patient presents with high content of
will accumulate in the organism resulting in vasopressin (antidiuretic hormone) in the blood.
toxicaction? What changes in the patient’s diuresis will
A. Fructose-1-phosphate occur?
B. Glucose-1-phosphate A. Oliguria
C. Glucose-6-phosphate B. Polyuria
D. Fructose-1,6-biphosphate C. Anuria
E. Fructose-6-phosphate D. Glycosuria
E. Natriuria
40. Prophylactic examination of a patient
revealed hyperglycemia, ketonuria, polyuria, 45. A patient with tress and painful sensation
glycosuria. What form of acidbase balance before a visit to the dentist is accompanied by
disorder is the case? anuria (lack of urination). This phenomenon is
A. Metabolic acidosis due to an increase in:
B. Gaseous acidosis A. Secretion of vasopressin and adrenaline
C. Nongaseous acidosis B. Activity of the parasympathetic nervous
D. Gaseous alkalosis system
E. Metabolic alkalosis C. Activity antinociceptive system
D. Secretion of vasopressin and a decrease in
41. Examination of a 2 year-old child with renal adrenaline
insufficiency revealed hyperoxaluria that caused E. Secretion of adrenaline and a decrease in
depositing of calcium oxalate stones in the vasopressin
kidneys. The reason for this condition is a
disturbance of metabolism of the following 46. A patient with pituitary tumor complains of
aminoacid: increased daily diuresis (polyuria). Glucose
A. Glycine concentration in blood plasma equals 4,8
B. Lysine mmol/l. What hormone can be the cause of this
C. Methionine if its secretion is disturbed?
D. Arginine A. Vasopressin
E. Histidine B. Aldosterone
C. Natriuretic hormone
42. Cardinal symptoms of primary D. Insulin
hyperparathyroidism are osteoporosis andrenal E. Angiotensin I
lesion along with development of urolithiasis.
What substance makes up the basis of these 47. After a person had drunk 1,5 liters of water,
calculi in this disease? the amount of urine increased significantly, and
A. Calcium phosphate its relative density decreased to 1,001. These
B. Uric acid changes are a result of decreased water
C. Cystine reabsorption in the distal nephron portion due to
D. Bilirubin reduced secretion of:
E. Cholesterol A. Vasopressin
B. Aldosterone
43. In the laboratory examination of blood of a C. Angiotensin II
person who was bitten by a change, hemolysis D. Renin
of erythrocytes, hemoglobinuria was revealed. E. Prostaglandins
The action of a mixed poison is due to the
presence of an enzyme in it: 48. A man has a considerable decrease in
A. Phospholipase A2 diuresis as a result of 1,5 l blood loss. The
B. Phospholipase D primary cause of such diuresis disorder is the
C. Phospholipase S. hypersecretion of the following hormone:
D. Phospholipase A1 A. Vasopressin

63
B. Corticotropin level of glucose in blood is normal, there is no
C. Natriuretic glucose in urine. What hormone deficit can
D. Cortisol cause such changes?
E. Parathormone A. Vasopressin
B. Oxytocin
49. Analysis of urine from a 24-year-old man C. Insulin
revealed the following changes: daily diuresis - D. Triiodothyronine
10 l, relative density - 1,001, qualitative E. Cortisol
alterations are absent. A patient complains of
excessive thirst, frequent urination. What is the 53. A 19-year-old male was found to have an
most likely cause of this disease? elevated level of potassium in the secondary urine.
A. Vasopressin hyposecretion These changes might have been caused by the
B. Glucocorticoid hypersecretion increase in the following hormone level:
C. Vasopressin hypersecretion A. Aldosterone
D. Relative insulin insufficiency B. Oxytocin
C. Adrenaline
E. Aldosteron hypersecretion D. Glucagon
E. Testosterone
50. A 50 year old patient complains of excessive
thirst, drinks a lot of water; expressed polyuria. 54. A concentrated solution of sodium chloride
Blood glucose - 4.8 mmol / l. There is no was intravenously injected to an animal. This
glucose and acetone bodies in urine, colorless caused decreased reabsorption of sodium ions in
urine, specific gravity - 1.002 - 1.004. What is the renal tubules. It is the result of the following
the cause of polyuria? changes of hormonal secretion:
A. Lack of vasopressin A. Aldosterone reduction
B. Hypothyroidism B. Aldosterone increase
C. Insulin Deficiency C. Vasopressin reduction
D. Aldosteronism D. Vasopressin increase
E. Thyrotoxicosis E. Reduction of atrial natriuretic factor
51. A patient with damage to the posterior lobe 55. At ria of a test animal were super distended
of the pituitary gland has increase in daily with blood,which resulted in decreased
diuresis to 10-15 liters What is the main reabsorption of Na+ and water in renal
mechanism in the development of polyuria? tubules.This can be explained by the effect of
A. Vasopressin deficiency the following factor on the kidneys:
B. Excess vasopressin A. Natriuretic hormone
C. Excess aldosterone B. Aldosterone
D. Excess natriuretic factor C. Renin
E. Corticotropin deficiency D. Angiotensin
E. Vasopressin
52. A 20 year old patient complains of excessive
thirst and urinary excretion upto 10 L a day. The

64
Biochemistry of nervous tissue
1. A substance that is synthesized in the central A. Hyperpolarization
nervous system and simulates the effects of B. Depolarization
morphine can be used for analgesia. Specify the C. Action potential
following substance: D. Local response
A. β-endorphin E. There will be no changes
B. Oxytocin
C. Vasopressin 7. A 50-year-old man came to a hospital with
D. Calcitonin complaints of memory disorders, painful
E. Somatoliberin sensations along the nerve trunks, decreased
mental ability, circulatory disorders and
2. Cell membrane restpotential changed from-85 dyspepsia. Anamnesis states excessive alcohol
to-90mV. It can becaused by activation of the consumption. What vitamin deficiency can
following cell membrane channels: result in such symptoms?
A. Potassium A. Thiamine
B. Sodium B. Niacin
C. Potassium and sodium C. Retinol
D. Calcium E. Potassium and calcium D. Calciferol
E. Riboflavin
3. The speed of the excitation nerve fibers is
120 m / s. Which of the above factors, above all, 8. Decarboxylation of glutamate induces
provides such speed? production of gamma aminobutyric acid
A. Presence of myelin sheath (GABA) neurotransmitter. After breakdown,
B. Great resting potential GABA is converted into a metabolite of the
C. Large amplitude potential effects citric acid cycle, that is:
D. Low threshold of depolarization A. Succinate
E. A big factor of reliability B. Citric acid
C. Malate
4. The patient is trembling hands, which is D. Fumarate
associated with Parkinson's disease. What E. Oxaloacetate
mediator deficiency in striopalidar structures
leads to such symptoms? 9. Degenerative changes in posterior and lateral
A. Dopamine columns of spinal cord (funicular myelosis)
B. GABA caused by methylmalonic acid accumulation
C. Substance P occur in patients with B12-deficiency anemia.
D. Norepinephrine This results in synthesis disruption of the
E. Serotonin following substance:
A. Myelin
5. An 84-year-old patient suffers from B. Acetylcholine
parkinsonism. One of the pathogenetic C. Norepinephrine
development elements of this disease is D. Dopamine
deficiency of a certain mediator in some of the E. Serotonin
brain structures. Name this mediator:
A. Dopamine 10. After pharmacological blockade ion
B. Adrenaline channels of the nerve fiber membrane resting
C. Noradrenaline potential decreased from -90 to -80 mV. What
D. Histamine channels were blocked?
E. Acetylcholine A. Potassium
B. Sodium
6. In the experiment, the permeability of the C. Calcium
excitable cell membrane to potassium ions was D. Magnesium
magnified. What changes in the electrical state E. Chloric
of the membrane will occur?
65
11. Disruption of nerve fiber myelinogenesis concentration increases in the brain under the
causes neurological disorders and mental action of antidepressants?
retardation. These symptoms are typical for A. Serotonin
hereditary and acquired alterations in the B. Dopamine
metabolism of: C. Glycine
A. Sphingolipids D. Taurine
B. Neutral fats E. Norepinephrine
C. Higher fatty acids
D. Cholesterol 16. Brain cells are very sensitive to energy
E. Phosphatidic acid deficiency, which can be caused by a high
content of ammonia, which stimulates the
12. A patient complained about dizziness, outflow of alpha-ketoglutarate from:
memory impairment, periodical convulsions. It A. Citric acid cycle
was revealed that these changes were caused by B. Ornithine cycle
a product of decarboxylation of glutamic acid. C. Glycolysis
Name this product: D. Glycogenolysis
A. GABA E. Pentose phosphate pathway
B. Pyridoxal phosphate
C. TDP 17. Depressions and emotional insanities result
D. ATP from the deficit of noradrenalin, serotonin and
E. UDP other biogenic amines in the brain. Their
concentration in the synapses can be increased
13. A patient presents with dysfunction of the by means of the antidepressants that inhibit the
cerebral cortex accompanied by epileptic following enzyme:
seizures. He has been administered a biogenic A. Monoamine oxidase
amine synthetized from glutamate and B. Diamine oxidase
responsible for central inhibition. What C. L-amino-acid oxidase
substance is it? D. D-amino-acid oxidase
A. γ-aminobutyric acid E. Phenylalanine-4-monooxygenase
B. Serotonin C.
С. Dopamine 18. Pharmacological effects of antidepressants
D. Acetylcholine are based upon blocking (inhibiting) the enzyme
E. Histamine that acts as a catalyst for the breakdown of
biogenic amines noradrenalin and serotonin in
14. An unconscious patient was taken by the mitochondria of cephalic neurons. What
ambulance to the hospital. On objective enzyme takes part in this process?
examination the patient was found to have no A. Monoamine oxidase
reflexes, periodical convulsions, irregular B. Transaminase
breathing. After laboratory examination the C. Decarboxylase
patient was diagnosed with hepatic coma. D. Peptidase
Disorders of the central nervous system develop E. Lyase
due to the accumulation of the following
metabolite: 19. A 9-month-old infant is fed with artificial
A. Ammonia formulas with unbalanced vitamin B6
B. Urea concentration. The infant presents with pellagral
C. Glutamine dermatitis, convulsions, anaemia. Convulsion
D. Bilirubin development might be caused by the disturbed
E. Histamine formation of:
A. GABA
15. For the treatment of depression of different B. Histamine
genesis, antidepressants are used, which are C. Serotonin
monoamine oxidase inhibitors. What substance D. DOPA
is a "neurotransmitter of well-being" and its E. Dopamine

66
inhibitor aldehyde dehydro- nazy The increase
20. Cerebral trauma caused increase of in blood of what tabolita aversion to al to
ammonia formation. What aminoacid takes part someone?
in removal of ammonia from A. Acetaldehyde
cerebral tissue? B. Ethanol
A. Glutamic C. Malonic aldehyde
B. Tyrosine D. Propionic aldehyde
C. Valine E. Methanol
D. Tryptophan
E. Lisine 25. Soldiers who were injured in the midst of
the battle may not be aware of the pain until it is
21. Ammonia is a very toxic substance, completed. What hormones are opiate
especially for nervous system. What substance antinociceptive system reduce the sensation of
takes the most active part in ammonia pain?
detoxication in brain tissues? A. Endorfines
A. Glutamic acid B. Serotonines
B. Lysine C. Vasopressin
C. Proline D. Aldosterone
D. Histidine E. Oxytocine
E. Alanine
26. In an excitable cell the ion channels were
22. A patient presented to a hospital with blocked. It hasn’t changed essentially the value
complaints about quick fatigability and of rest potential, but the cell lost its ability to
significant muscle weakness. Examination generate AP (action potential). What channels
revealed an autoimmune disease that were blocked?
causes functional disorder of receptors in the A. Natrium
neuromuscular synapses. This will result in the B. Potassium
disturbed activity of the following mediator: C. Natrium and potassium
A. Acetylcholine D. Chloric
B. Noradrenaline E. Calcium
C. Dopamine
D. Serotonin 27. The infant has epileptiform convulsions
E. Glycine caused by vitamin B6 deficiency. This is due to
a decrease in the nervous tissue of the inhibitory
23. Glutamate decarboxylation results in mediator, γ-aminobutyric acid. What is the
formation of inhibitory transmitter in CNS. enzyme's activity reduced in this case?
Name it: A. Glutamate decarboxylase
A. GABA B. Alanine aminotransferase
B. Glutathione C. Glutamate dehydrogenase
C. Histamine D. Pyridoxal kinase
D. Serotonin E. Glutamate Synthetase
E. Asparagine

24. In medical practice for alcoholism


prevention widely used teturam, which is

67
Biochemistry of muscle tissue
1. Buffer capacity of a worker’s blood was
decreased due to exhausting muscular work. By 6. A 35-year-old man developed acute heart
coming of what acid substance in the blood can failure while running for a long time. What
this state be explained? changes in ionic composition can be observed in
A. Lactate the cardiac muscle?
B. Pyruvate A. Accumulation of Na+ and Ca2+ ions in the
C. 1,3-bisphosphoglycerate myocardium cells
D. α-ketoglutarate B. Accumulation of K+ and Mg2+ ions in the
E. 3-phosphoglycerate myocardium cells
C. Reduction of Na+ and Ca2+ ions in the
2. A biochemical urine analysis has been myocardium cells
performed for a patient with progressive D. Reduction of K+ and Mg2+ ions in the
muscular dystrophy. In the given case muscle extracellular space
disease can be confirmed by the high content of E. Reduction of Na+ and Ca2+ ions in the
the following substance in urine: extracellular space
A. Creatine
B. Porphyrin 7. A patient is diagnosed with cardiac infarction.
C. Urea Blood test for cardiospecific enzymes activity
D. Hippuric acid was performed. Which of the enzymes has three
E. Creatinine isoforms?
A. Creatine kinase
3. A considerable increase of activity of МВ- B. Lactate dehydrogenase
forms of CPK (creatinephosphokinase) and C. Aspartate transaminase
LDH-1 was revealed on the examination of D. Alanine transaminase
patient’s blood. What is the most likely E. Pyruvate kinase
pathology?
A. Miocardial infarction 8. A 50-year-old woman diagnosed with cardiac
B. Hepatitis infarction has been delivered into an intensive
C. Rheumatism care ward. What enzyme will be the most active
D. Pancreatitis during the first two days?
E. Cholecystitis A. Aspartate aminotransferase
B. Alanineaminotransferase
4. With intense physical work, muscle tissue C. Alanineaminopeptidase
accumulates lactic acid, which diffuses into the D. LDH4
blood and is absorbed by the liver and the heart. E. LDH5
What process ensures the recovery of glycogen
stores in the muscles? 9. Untrained people often have muscle pain
A. The Cori Cycle after sprints as a result of lactate accumulation.
B. Citric Acid Cycle This might be caused by intensification of the
C. Urea Cycle following biochemical process:
D. A Cycle of Tricarboxylic Acids A. Glycolysis
E. Hexose Monophosphate Pathway B. Gluconeogenesis
C. Pentose phosphate pathway
5. A sportsman needs to improve his sporting D. Lipogenesis
results. He was recommended to take a E. Glycogenesis
preparation that contains carnitine. What
process is activated the most by this compound? 10. A patient with suspected diagnosis
A. Fatty acids transporting "progressing muscular dystrophy"got his urine
B. Amino acids transporting tested.What compound will confirm this
C. Calcium ions transporting diagnosis if found in urine?
D. Glucose transporting A. Kreatine
E. Vitamin K transporting B. Collagen
68
C. Porphyrin C. Acetyl CoA
D. Myoglobin D. Glucose 6-phosphate
E. Calmodulin E. Oxaloacetate

11. Rheography of an 18 year old student during 16. A 49-year-old driver complains about
exercise showed redistribution of blood flow unbearable constricting pain behind the
between organs. The peak blood flow will be breastbone irradiating to the neck. The pain
observed in the following vessels: arose 2 hours ago. Objectively: the patient’s
A. Skeletal muscles condition is grave, he is pale, heart tones are
B. Liver decreased. Laboratory studies revealed high
C. Cerebrum activity of creatine kinase and LDH1. What
D. Kidneys disease are these symptoms typical for?
E. Gastrointestinal tract A. Acute myocardial infarction
B. Acute pancreatitis
12. A 46-year-old female patient has continuous C. Stenocardia
history of progressive muscular (Duchenne’s) D. Cholelithiasis
dystrophy. Which blood enzyme changes will E. Diabetes mellitus
be of diagnostic value in this case?
A. Creatine phosphokinase 17. Cytoplasm of the myocytes contains a lot of
B. Lactate dehydrogenase dissolved metabolites resulting from glucose
C. Pyruvate dehydrogenase oxidation. Name the metabolite that turns
D. Glutamate dehydrogenase directly into lactate:
E. Adenylate cyclase A. Pyruvate
B. Oxaloacetate
13. A 50-year-old patient complains about C. Glycerophosphate
general weakness, appetite loss and cardiac D. Glucose-6-phosphate
arrhythmia. The patient presents with muscle E. Fructose-6-phosphate
hypotonia, flaccid paralyses, weakened
peristaltic activity of the bowels. Such condition 18. A patient came to the hospital complaining
might be caused by: about quick fatigability and apparent muscle
A. Hypokaliemia weakness. Examination revealed an
B. Hypoproteinemia autoimmune disease that causes disorder of
C. Hyperkaliemia functional receptor condition in neuromuscular
D. Hypophosphatemia synapses. What transmitter will be blocked?
E. Hyponatremia A. Acetylcholine
B. Noradrenalin
14. Some students developed myodynia after C. Dopamine
continuous physical activity during physical D. Serotonin
education. The reason for such condition was E. Glycine
accumulation of lactic acid in the skeletal
muscles. It was generated in the students’ 19. During processing of the atypical
bodies after activation of the following process: cardiomyocytes of the synaptic node with a
A. Glycolysis biologically active substance, an increase in
B. Gluconeogenesis their membrane potential was recorded due to
C. Pentose-phosphate cycle increased permeability for potassium ions. What
D. Lipolysis biologically active substance affects
E. Glycogeny cardiomyocytes?
A. Acetylcholine
15. After a sprint an untrained person develops B. Adrenalin
muscle hypoxia. This leads to the accumulation C. Noradrenaline
of the following metabolite in muscles: D. Thyroxine
A. Lactate E. Atriopeptide
B. Ketone bodies

69
20. 12 hours after an accute attack of A. Transporting of fatty acids to mitochondrions
retrosternal pain a patient presented a jump of B. Regulation of Ca2+ rate in mitochondrions
aspartate aminotransferase activity in blood C. Substrate phosphorylation
serum. What pathology is this deviation typical D. Lactic acid utilization
for? E. Actin and myosin synthesis
A. Myocardium infarction
B. Viral hepatitis 24. After a sprint an long distantion the skeletal
C. Collagenosis muscle training person uses glucose for the
D. Diabetes mellitus purpose of obtaining energy ATP for muscle
E. Diabetes insipidus contraction. Specify basic glucose utilization
process in these conditions:
21. A patient presents high activity of LDH 1,2, A. Aerobic glycolysis
aspartate aminotransferase, creatine B. Anaerobic glycolysis
phosphokinase. In what organ (organs) is the C. Glycogenolysis
development of a pathological process the most D. Gluconeogenesis
probable? E. Glycogenesis
A. In the heart muscle (initial stage of
myocardium infarction) 25. The gluconeogenesis is activated in the liver
B. In skeletal muscles (dystrophy, atrophy) after intensive physical trainings.What
C. In kidneys and adrenals substance is utilized in gluconeogenesis first of
D. In connective tissue all in this case:
E. In liver and kidneys A. Lactate
B. Pyruvate
22. A traumatology unit received a patient with C. Glucose
crushed muscular tissue. What biochemical D. Glutamate
indicator of urine will be raised in this case? E. Alanine
A. Creatinine
B. Total lipids 26. 6 hours after the myocardial infarction a
C. Glucose patient was found to have elevated level of
D. Mineral salts lactate dehydrogenase in blood. What is enzyme
E. Uric acid should be expected in this case?
A. LDH1
23. A 1 y.o. child with symptoms of muscle B. LDH2
affection was admitted to the hospital. C. LDH3
Examination revealed carnitine deficit D. LDH4
inmuscles. Biochemical base of this pathology E. LDH5
is disturbed process of:

70
Biochemistry of connective tissue
1. Examination of a patient revealed typical C. Nucleotides
presentations of collagenosis. This pathology is D. Vitamins
characterized by increase of the following urine E. Phospholipids
index:
A. Hydroxyproline 6. A patient in the dental department has been
B. Arginine diagnosed with Paget's disease, which is
C. Glucose accompanied by collagen degradation. The
D. Mineral salts decisive fact for the diagnosis was the detection
E. Ammonium salts in the patient's urine of an elevated level:
A. Hydroxproline
2. A child has physical and mental retardation, B. Arginine
serious abnormalities in connective tissue of C. Tryptophan
internal organs; urine contains keratan sulfates. D. Serina
This is caused by metabolic disorder of the E. Alanina
following substance:
A. Glycosaminoglycan 7. A 60 year old man complains about in the
B. Collagen joints. In the serum of the patient, the
C. Elastin concentration of C-reactive protein and
D. Fibronectin oxyproline was found to increase. what disease
E. Hyaluronic acid these symptoms can be associated with:
A. Rheumatism
3. Inherited diseases, such as B. Gout
mucopolysaccharidoses, are manifested in C. Hepatitis
metabolic disorders of connective tissue, bone D. Jaundice
and joint pathologies. The sign of this disease is E. Diabetes
the excessive urinary excretion of the following
substance: 8. A 36-year-old female patient has a history of
A. Glycosaminoglycans collagen disease. Urine analysis is likely to
B. Amino acids reveal an increased concentration of the
C. Glucose following metabolite:
D. Lipids A. Oxyproline
E. Urea B. Indican
C. Creatinine
4. A 28 year old pregnant woman had the D. Urea
enzymes in the cells of amniotic fluid analyzed. E. Urobilinogen
The analysis revealed insufficient activity of β-
glucuronidase. What pathological process is it? 9. A 30-year-old woman first developed pain,
A. Mucopolysaccharidosis swelling, and skin redness in the area of joints
B. Glycogenosis about a year ago. Provisional diagnosis is
C. Aglycogenosis rheumatoid arthritis. One of the likely causes of
D. Collagenosis this disease is change in the structure of the
E. Lipidosis following connective tissue protein:
A. Collagen
5. The five year old boy was observed small B. Mucin
stature, mental retardation, limited movement, C. Myosin
rough facial features. These features have D. Ovalbumin
become noticeable from 18 months of age. He E. Troponin
was diagnosed with a deficiency of L-
iduronidase. The exchange of which compounds 10. Collagenosis patients typically present with
is broken? connective tissue destruction processes.The
A. Glycosminoglycans presence of the seprocesses can beconfirmed by
B. Proteins the increasein:
71
A. Blood oxyproline and oxylysine alkaline phosphatase in the intercellular
B. Blood creatine and creatinine substance. What cell produces this enzyme?
C. LDH-isoenzyme activity in the blood A. Osteoblast
D. Transaminase activity in the blood B. Osteocyte
E. Blood urates C. Osteoclast
D. Chondroblast
11. A 53-year-old male patient is diagnosed E. Chondrocyte
with Paget’s disease. The concentration of
oxyproline in daily urine is sharply increased, 16. When a wound heals, a scar takes its place.
which primarily means intensified disintegration What substance is the main component of its
of: connective tissue?
A. Collagen A. Collagen
B. Keratin B. Elastin
C. Albumin C. Keratan sulfate
D. Hemoglobin D. Chondroitin sulfate
E. Fibrinogen E. Hyaluronic acid

12. A 63-year-old woman developed symptoms 17. A patient with systemic scleroderma has an
of rheumatoid arthritis. Their increase of which intensified collagen destruction. Collagen
blood values indicators could be the most destruction will be reflected by intensified
significant in proving the diagnosis? urinary excretion of the following amino acid:
A. Additive glycosaminoglycans A. Oxyproline
B. Lipoproteids B. Alanine
C. Acid phosphatase C. Tryptophane
D. General cholesterol D. Serine
E. R-glycosidase E. Phenylalanine

13. Osteolaterism is charcterized by a decrease 18. A patient who is ill with scurvy displays
in collagen strength caused by much less disturbed processes of connective tissue
intensive formation of crosslinks in collagen formation that leads to loosening and falling of
fibrils. This phenomenon is caused by the low teeth. Disturbed activity of what enzyme causes
activity of the following enzyme: these symptomps?
A. Lysyl oxidase A. Lisilhydroxylase
B. Monoamino-oxidase B. Glycosiltransferase
C. Prolyl hydroxylase C. Elastase
D. Lysyl hydroxylase D. Procollagenpeptidase of N-terminal peptide
E. Collagenase E. Procollagenpeptidase of C-terminal peptide

14. A 34-year-old patient has a history of 19. In spring a patient experiences petechial
periodontitis. As a result of increased collagen haemorrhages, loosening of teeth, high liability
degradation, there is a significantly increased to colds. A doctor supposes hypobitaminosis C.
urinary excretion of one of the amino acids. In this respect loosening of teeth can be
Which one? explained by:
A. Hydroxyproline A. Structural failure of collagen in the
B. Valine periodontal ligaments
C. Alanine B. Structural change of glycosaminoglycan
D. Glycine C. Increased permeability of periodont
E. Serine membranes
D. Mechanical damage of teeth
15. Calcification of the intercellular substance E. Disturbed oxidation-reduction process in the
of bone tissue is accompanied by the deposition periodont
of hydroxyapatite crystals along the collagen
fibers. This process requires the presence of

72
20. Increased fragility of vessels, enamel and B. Formation of polypeptide chains
dentine destruction resulting from scurvy are C. Glycosylation of hydroxylysine residues
caused by disorder of collagen maturation. What D. Removal of C-ended peptide from
stage of procollagen modification is disturbed procollagen
under this avitaminosis? E. Detaching of N-ended peptide
A. Hydroxylation of proline

73
Biochemistry of tooth and saliva
1. What enzyme has a demineralizing effect - 6. The periodontal dentist must be assessed in
enhances the cleavage of the mineral the patient by factors of non-specific resistance
components of the tissues in the tooth? of saliva and excretory mucous membrane of
A. Sulfate phosphatase the oral cavity. What factor of non-specific
B. Alkaline phosphatase resistance should first be studied in the
C. Glucose-6-phosphatase investigated material?
D. Glucogenphosphorylase A. Lysozyme
E. Phosphotransferase B. Secretory IgA
C. Properdine
2. The hydroxyapatite crystals are deposited D. Interferon
along the collagen fibers in the process of E. Complement
grafting the intercellular substance of the tissue.
In order to realize this process, the presence of 7. What substance gives slime a viscous mucous
intercellular alkaline phosphatase must be character, performs a protective role and
present. What kind of creatine produces this including from mechanical damage to the oral
enzyme? mucosa?
A. Osteoblast A. Mutsin
B. Osteocyte B. Glucose
C. Osteoclasts C. Kalikrein
D. Hondroblaste D. Amilaza
E. Chondrocyte E. Lysozyme

3. The pH of saliva is 6.4 - 7.8 it is in the norm. 8. Some beards of saliva perform a protective
What changes in the enamel leads to a shift in function. Which one protects the oral mucosa
the pH of saliva in the sour beet (less than 6.2)? from mechanical damage?
A. Demineralization A. Mutsin
B. Calcification B. Lysozyme
C. Fluorosis C. Catalase
D. Mineralization D. Peroxidase
E. Strengthening of stability E. Renin

4. Cationic glycoproteins are the main 9. The patient with chronic inflammation of
components of salivation of the parotid glands. submaxilar salivary glands have observed
What Amino Acids cause their positive charge? giposalivation. What violation of the increment
A. Lizin, arginine, hististine of which biologically active substance is
B. Aspartate, glutamate, glucine observed at this process?
C. Aspartate, arginine, glutamate A. Parotin
D. Glutamate, valine, leucine B. Calcitonin
E. Cysteine, glucine, proline C. Paratyrin
D. Gluckagon
5. The protective function of saliva is due to E. Somatostatin
several mechanisms, including the presence of
an enzyme that has bactericidal action, causes 10. Patient with symptoms of reduced excretory
lysis of the polysaccharide complex of the function of the kidneys, an unpleasant smell
staphylococcal shell, streptococci. Indicate this from the mouth is indicated. What substance are
enzyme: increased excretion of the salivary glands is the
A. Lysozyme cause of this?
B. α-amylase A. Urea
C. Oligo-1,6-glucosidase B. Alpha amylase
D. Collagenase C. Lysozyme
E. β-glucuronidase D. Phosphatase
E. Mutsin
74
16. A 60 year old patient was found to have a
11. The child has acute renal insufficiency. dysfunction of main digestive enzyme of saliva.
What biochemical indicators of saliva can be This causes the disturbance of primary
confirmed? hydrolysis of:
A. Increasing the level of residual nitrogen A. Carbohydrates
B. Increase in immunoglobulin A. B. Fats
C. Reduction of alkaline phosphatase C. Proteins
D. Increase in alpha amylase D. Cellulose
E. Reducing the phosphate level E. Lactose

12. Periodontitis is accompanied by activation 17. It is necessary to evaluate the digestible


of proteolysis in periodontal tissues. The properties of saliva. With what substrate for this
increase of which oral fluid component testifies it needs to be shifted?
to the activation of proteolysis? A. Starch
A. Amino acids B. Casein
B. Organic acids C. Fat
C. Glucose D. DNA
D. Biogenic Amines E. RNA
E. Cholesterol
18. When investigating human saliva it is
13. When treated with hydrogen peroxide in the necessary to assessits hydrolytic properties.
mucous membrane of the patient's oral cavity, What substance should be used as a substrate in
the blood was painted in a brown color instead the process?
of pene formation. When the concentration of A. Starch
any of the enzymes listed below is reduced, is it B. Proteins
possible? C. Fats
A. Catalase D. Fiber
B. Pseudocholinesterase E. Amino acids
C. Glucose-6-phosphate dehydrogenase
D. Acetyltransferase 19. Afther using cookies, sweets in mixed
E. Methemoglobin reductase saltines temporarily increases the level of
lactate. The activation of which biomic process
14. In periodontites, lipid peroxidation develops leads to this?
in the periodontal tissues, and the content of A. Anaerobic digestion
malonic dialdehyde, hydrogen peroxide B. Tissue breathing
increases in the oral cavity. Which of the C. Aerobic Glucose
enzymes is antioxidant protection? D. Gluconeogenesis
A. Superoxide dismutase, catalase E. Microsomal oxidation
B. Amilase, trypsin
C. Maltase, Chymotrypsin 20. In the saliva of patient is increased content
D. Lactase, lysozyme of lactate was detected. Activation of a process
E. Saharaza, prothrombin. is the main reason for the increase of lactate?
A. Anaerobic decomposition of glucose
15. With age, the activity of the peripheral B. Aerobic decomposition of glucose
salivary glands decreases. Which does an C. Decomposition of glycogen
enzyme's activity decrease in slime? D. Hydrolysis of carbohydrates
A. Amilaza E. Glucose-lactate cycle
B. Lysozyme
C. Phosphatase 21. To form a mineral matrix of hard tooth
D. Geksokinase tissues, a high concentration of phosphate ions
E. Malthus is required, which is formed during the process
of hydrolysis of phosphorous-bonded bonds
with the participation of alkaline phosphatase.

75
What ions of the metal are the triggers of this D. Calcium chloride
process: E. Kalia bromide
A. Zinc
B. Magnium 27. A patient with renal insufficiency developed
C. Iron osteodystrophy, which is accompanied by
D. Calcium intensive demineralization of the hips. Which
E. Natrium active form of vitamin violation’s of the
formation is the cause of this complication?
22. In economically developed countries, A. Calcipherol
common caries is a cureus of teeth. This disease B. Retinol
affects more than 95% of the population. What C. Thiamin
role plays in the demineralization of hard tooth D. Naftohinon
tissues in caries? E. Ryboflavin
A. Organic acids
B. Malnutrition 28. The enamel is characterized by high stability
C. Extreme effects on the organism to the action of various mechanical and
D. Disturbance of regulation of metabolism chemical factors. Which component is synthesis
E. Insufficient vitamin C of provides such resistance?
A. Fluorapatite
23. The high excess concentration of glucose in B. Hydroxyapatite
oral solution in diabetes leads to development: C. Chlorapatite
A. Multiple caries D. Collagen
B. Hyperplasia enamel E. Carbonate apatite
C. Hypoplasia of enamel
D. Fluorosis 29. Calcification of dental tissues is
E. Enhanced calcification of enamel significantly influenced by osteocalcin protein
which has an ability to bind calcium ions due to
24. In the Transcarpathian settlement, after the presence of the following modified amino
prophylactic examination of many children have acid residues in the polypeptide chain:
found a multiple of caries. With mineral of the A. γ-carbon glutamine
insufficiency in food you can link development B. Alanine
of careers? C. γ-aminobutyric
A. Fluoride D. Carboxy aspargine
B. Iodine E. δ-aminopropionic
C. Molibden
D. Iron 30. A 35-year-old patient consulted a dentist
E. Cobalt about low density of dental tissues, increased
fragility of teeth on eating solid food. In order to
25. A child has disturbed processes of determine Ca/P relation a scrape of enamel was
ossification and "punctate"enamel. What sent to the laboratory. What value of this index
microelement metabolism is disturbed? is suggestive of intensified demineralization?
A. Fluorine A. 0,9
B. Iron B. 1,67
C. Zinc C. 1,85
D. Chromium D. 2,5
E. Copper E. 1,5

26. A saline metal was assigned for salvage 31. The preparation comlex for periodontitis
therapy at the initial caries of the teeth. Which treatment includes the medicine from the group
drug is determinated of this process? of water soluble vitamins, bioflavonide
A. Sodium fluoride derivative, which is prescribed together with
B. Sodium bromide ascorbic acid. This preparation has anti-
C. Sodium chloride

76
oxidative properties, decreases gingival E. Nicotinamide
hemorrhage. What preparation is meant?
A. Rutin 36. The child has a delayed teething, their
B. Calcium pantothenate wrong arrangement, upon examination
C. Calcium panganate noticeable dryness of the oral cavity, in the
D. Cyanocobalamin corners of the mouth there are cracks with
E. Folic acid suppuration. With the lack of the vitamin this
condition may be related?
32. Examination of a child who hasn’t got fresh A. Vitamin D.
fruit and vegetables during winter revealed B. Vitamin C.
numerous subcutaneous hemorrhages, C. Vitamin E.
gingivitis, carious cavities in teeth. What D. Vitamin K.
vitamin combination should be prescribed in E. Vitamin A.
this case?
A. Ascorbic acid and rutin 37. In a child of the first year of life is observed
B. Thiamine and pyridoxine an increase in the size of the head and abdomen,
C. Folic acid and cobalamin late teething, violation of the enamel structure.
D. Riboflavin and nicotinamide The consequence of vitamin deficiencies are
E. Calciferol and ascorbic acid these changes?
A. Hypovitaminosis D
33. The patient has an increase in the B. Hypovitaminosis C
permeability of the walls of blood vessels with C. Hypovitaminosis A
the development of increased bleeding and the D. Hypovitaminosis B1
appearance of small point hemorrhages on the E. Hypovitaminosis B2
skin, tooth loss. How does the vitamin exchange
disorder explain these symptoms? 38. A child with renal insufficiency exhibits
A. Hypovitaminosis C delayed teeth eruption. This is mostlikely
B. Hypervitaminosis D caused by the abnormal formationof the
C. Hypervitaminosis C following substance:
D. Hypovitaminosis D A. 1,25 (OH)2D3
E. Hypovitaminosis A B. Glycocyamine
C. Glutamate
34. In examining the oral cavity of the patient, D. α-ketoglutarate
the dentist paid attention to the presence of an E. Hydroxylysine
inflammatory and dystrophic process in the
mucous membrane (Hunter’s glossitis, atrophic 39. A patient has enamel erosion. What vitamin
stomatitis). A blood test revealed hyperchromic should be administered for itstreatment?
anemia. What factor is the cause of this disease? A. D3
A. Hypovitaminosis B12 B. C
B. Hypovitaminosis B1 C. K
C. Hypovitaminosis B6 D. B1
D. Increasing pH of the gastric juice E. PP
E. Hypovitaminosis A
40. A 5 years old child has insufficient
35. 10 month old child has high excitability, calcification of enamel, tooth decay. Which
sleep disturbance, amyotonia, retarded dentition, vitamin hypovitaminosis leads to the
teeth erupt withinadequate enamel calcification. development of this process?
Thesechanges are caused by deficiency of A. Calciferol
thefollowing vitamin: B. Tocopherol
A. Cholecalciferol C. Biotin
B. Riboflavin D. Nicotinic acid
C. Thiamine E. Folic acid
D. Retinol

77
41. Hormonal form of a certain vitamin induces B. Calciferol
genome level synthesis of Ca binding proteins C. Thiamine
and enterocytes thus regulating the intestinal D. Biotin
absorption of Ca2+ ions required for dental E. Phyloquinone
tissuedevelopment. What vitamin is it?
A. D3 44. There are various diseases that cause sharp
B. A increase of active oxygen, leading tocell
C. B1 membranes destruction. Antioxidantsare used to
D. E prevent it from happening. Themost potent
E. K natural antioxidant is:
A. Alpha-tocopherol
42. While the examination of patient’s oral B. Glycerol
cavity the dentist found xerostomia, numerous C. Vitamin D
erosions. What vitamin deficit caused this D. Fatty acids
effect? E. Glucose
A. Vitamin A
B. Vitamin K 45. When treating sialadenitis (inflammation of
C. Vitamin P the salivary glands), preparations of vitamins
D. Vitamin H are used. Which of the following vitamins plays
E. Vitamin PP an important role in antioxidant defense?
A. Tocopherol
43. In order to prevent gum inflammation and to B. Pantothenic acid
improve regeneration of epithelial periodontium C. Riboflavin
cells manufacturers add to the tooth pastes one D. Thiamine
of the following vitamins: E. Pyridoxine
A. Retinol

78

You might also like